Asap 2011

479
ASAP 2011 Copyright: 2011, American Association of Clinical Endocrinologists (AACE). Marks: 1.00 Which of the following incidental finding on radiologic imaging would be most consistent with a 25-year-old man with hypertension and a cerebellar hemangioblastoma? Choose one answer. A. Unilateral adrenal mass with density of <10 Hounsfield units B. Bilateral adrenal masses C. Adrenal mass that is hyperintense on CT scan D. Adrenal mass with high signal intensity on T2-weighted magnetic resonance imaging Educational objective: Recognize radiological characteristics of adrenal incidentalomas that in clinical context suggest specific diagnoses qsid=146109&aid 0

description

 

Transcript of Asap 2011

Page 1: Asap 2011

ASAP 2011Copyright: 2011, American Association of Clinical Endocrinologists (AACE).

Marks: 1.00

Which of the following incidental finding on radiologic imaging would be most

consistent with a 25-year-old man with hypertension and a cerebellar

hemangioblastoma?

Choose one answer.

A. Unilateral adrenal mass with density of <10 Hounsfield units

B. Bilateral adrenal masses

C. Adrenal mass that is hyperintense on CT scan

D. Adrenal mass with high signal intensity on T2-weighted magnetic resonance imaging

Educational objective: Recognize radiological characteristics of adrenal

incidentalomas that in clinical context suggest specific diagnoses

Rationale: Hypertension and cerebellar hemangioblastoma in a 25-year-old man with

hypertension and cerebellar hemangioblastoma is highly suggestive of Von Hippel-

Lindau syndrome which also includes pheochromocytoma and these tumors typically

have high signal intensity on T2-weighted magnetic resonance imaging.

References

qsid=146109&aid=3834&qid=16&checksum=c2e57a9580c8d274356764acec04365e&sesskey=9GgVY0EvCf&new state=0

Page 2: Asap 2011

1. Blake MA, Jhaveri KS, Sweeney AT, et al. State of the art in adrenal imaging. Curr

Probl Diagn Radiol. 2002;31:67-78.

2. Lockhart ME, Smith JK, Kenney PJ. Imaging of adrenal masses. Eur J Radiol.

2002;41:95-112.

Which of the following statements concerning incidentally discovered adrenal

masses is correct?

Choose one answer.

A. Normal 11PM salivary cortisol rules out the diagnosis of autonomous cortisol hypersecretion by an incidentaloma

B. A morning cortisol level of 20 µg/dL following administration of 1 mg dexamethasone the previous evening establishes the diagnosis of autonomous cortisol secretion by an incidentaloma

C. A normal 24-hour urine for metanephrines excludes the diagnosis of pheochromocytoma

D. Patients with an incidentaloma and plasma metanephrine levels that are 1.5 times the upper limit of normal have a less than 40% likelihood of having a pheochromocytoma

E. A 5-cm asymptomatic adrenal lesion with a computed tomography density of 25 Hounsfield units is likely to be an adrenal carcinoma

Educational Objective: Understand the effects of test characteristics and disease

prevalence on diagnostic strategies

Rationale: Test characteristics (sensitivity and specificity) and pretest probability of disease (disease

prevalence) affect diagnostic strategies. Subclinical autonomous glucocorticoid hypersecretion is

characterized by loss of dexamethasone suppressibility and suppressed ACTH levels. However, urinary

cortisol secretion may not exceed the normal range; urinary cortisol secretion has relatively low

sensitivity for the diagnosis of subclinical autonomous glucocorticoid hypersecretion. Similarly,

although 11PM salivary cortisol is relatively sensitive, it is not 100% sensitive. Notwithstanding these

facts, patients should be observed for postoperative adrenal insufficiency following the removal of such

a lesion. Although a cortisol level of 20 µg /dL following 1 mg dexamethasone is abnormal and

Page 3: Asap 2011

indicates failure of suppression, this test has low specificity, meaning it has a high false positive rate.

This lack of suppression is consistent with pituitary Cushing's disease but in this particular

circumstance, a level of 20 µg /dL is also consistent with not having taken the dexamethasone or taking

another medication that accelerates the metabolism of dexamethasone, e.g., phenytoin. A single 1 mg

suppression is insufficient to establish the diagnosis of autonomous cortisol secretion. Thus,

confirmatory tests must be done. Similarly, tests for pheochromocytoma are neither 100% sensitive nor

100% specific. Pre-test probability of pheochromocytoma is low (5-10%) and post-test probability is

even lower. It is necessary to continue to follow the patient clinically and repeat testing as indicated

because as noted no test for pheochromocytoma is 100% sensitive.

Sensitivities of urine metanephrines average around 80%. In a study of

incidentalomas referred for surgery, only 30% who had elevated plasma

metanephrines up to 2 times the normal limit turned out to be pheochromocytomas.

Although larger adrenal incidentalomas are more likely to be malignant than smaller

lesions, most lesions of 5 cm diameter are benign. This is true even for lesions that are

>10 Hounsfield units.

A 37-year-old woman presents with increased growth of hair on her chin, upper lip

and torso for the past 8 months. Her periods have become increasingly irregular. Her

body mass index (BMI) is 25 kg /m2, blood pressure is 148/94 mmHg. She has facial

plethora, acne, abdominal obesity without any striae. Ferriman- Gallwey score is 14.

Her serum potassium is 3.2 mmol/L (normal, 3.6-5.2), testosterone is 326 ng/dl

(normal, 20-80), dehydroepiandrosterone sulfate (DHEAS) is 1223 mcg/dl (normal,

65-325) and 24 hour urinary free cortisol is 3 times above the upper limit of normal.

Which of the following is the most likely diagnosis?

Choose one answer.

Page 4: Asap 2011

A. Cortisol producing adrenal adenoma

B. Cushing's disease

C. Polycystic ovarian syndrome (PCOS)

D. Adrenocortical carcinoma

E. Hyperaldosteronsim

Educational Objective: Recognize the presenting features of adrenocortical

carcinoma

Rationale: Adrenocortical carcinoma is a rare malignancy affecting 1-2 per 1 million

population. Typically, women are affected more commonly than men, in a bimodal

distribution (first peak in childhood, second in 4th-5th decade of life). Patients may

present with or without virilization. Rapid onset of her symptoms in addition to very

high levels of dehydroepiandrosterone sulfate (DHEAS) and testosterone along with

hypercortisolemia are suggestive of an underlying adrenocortical carcinoma.

Hypertension and hypokalemia may be seen due to activation of mineralocorticoid

receptors by excess cortisol.

A 21-year -old woman comes to see you in follow up after an Emergency room (ER)

visit. She was evaluated there for an episode of acute left lower abdominal pain. Her

work up included transvaginal ovarian ultrasound that detected multiple ovarian cysts.

She never had problems with fertility and has a healthy 2 year old son. She has

regular menstrual periods.

On physical examination, her body mass index (BMI) is 24 kg /m2, and there are a

few hairs on her upper lip and lower abdominal. Ferriman-Gallwey score is 4. She

denies changes in her physical appearance over past 4 years.

Page 5: Asap 2011

Laboratory test obtained in ER shows a blood sugar of 84 mg/dl (normal, 70-100).

Which ONE of the following statements is correct relative to the findings

reported?

Choose one answer.

A. She has hirsutism, but no further tests are needed

B. She has polycystic ovary syndrome

C. She has hirsutism and needs to have her testosterone level measured

D. She has hirsutism and needs further work up including measurement of dehydroepiandrosterone sulfate (DHEAS) and 17 hydroxyprogesterone levels to exclude an associated adrenal problem and nonclassic adrenal hyperplasia

E. She does not have hirsutism and no further tests are needed

Educational Objective: Review the significance of hirsutism noted in a woman seen

for follow-up of multiple ovarian cysts on ultrasound

Rationale: Hirsutism is a common sign of underlying androgen effects that can be

present in 5% to 15% of women. It is defined as excessive male pattern terminal hair

growth. While many women consider any male pattern growth unsightly and

excessive, a Ferriman-Gallwey score up to 8 is considered "normal" (95 % of women

have a score less than 8). Serious underlying disorder is unlikely in a woman with

minimal-to-mild stable hair growth on the upper lip and lower abdomen and regular

menses and thus, measurement of androgen levels is not needed. The prevalence of

polycystic ovaries is 21.6 % in women < 35 years of age. The sole presence of ovarian

cysts has limited clinical implications and by itself is not indicative of polycystic

ovaries syndrome unless other symptoms such as hirsutism, irregular periods or

anovulation are associated with it.

Page 6: Asap 2011

A 45-year-old woman presented with a six months history of increasing abdominal

girth, 6 kg weigh gain without change in diet or activity, and new purple striae.

On physical examination, her blood pressure is 165/95 mmHg, heart rate 70 beats per

minute. She had Cushinoid features with central obesity and 2 cm wide purple striae.

Laboratory tests showed serum sodium of 142 meq/L (normal, 135-145), serum

potassium 2.6meq/L (normal, 3.6-4.5), serum cortisol 32 microgram/dl (normal, 7-

25), adrenocorticotropin hormone, <5 pg/ml (normal, 10-60) and 24-hour urine free

cortisol of 450 micrograms (normal, <45).

Body imaging revealed a 16 cm heterogeneous right adrenal mass.

The right adrenal mass was resected with negative margins and confirmed as

adrenocortical carcinoma.

Three months later, new hepatic and pulmonary lesions were noticed and confirmed

to be metastases.

Which ONE of the following is the best treatment option for this patient?

Choose one answer.

A. Mitotane alone

B. Insulin-like growth factor-1 receptor inhibitor

C. Systemic chemotherapy with etoposide, cisplatin, and doxorubicin

D. Systemic chemotherapy (etoposide, cisplatin, and doxorubicin) with mitotane

Educational Objective: Recognize the role of mitotane and chemotherapy in the

management of metastatic adrenocortical carcinoma

Page 7: Asap 2011

Rationale: Adrenocortical carcinoma has a high recurrence rate. In the presence of

multiorgan metastases, systemic chemotherapy with mitotane has been reported to

have response rate of about 55% while mitotane alone is reported to have 25% alone

as a single agent. Insulin-like growth factor-1 receptor inhibitors are still in clinical

trials and efficacy data are still lacking. Systemic chemotherapy with mitotane is the

preferred option in this case of rapidly progressive disease with distant metastases.

Which statement is correct regarding the genetic factors that contribute to the pathophysiology of type 1 diabetes mellitus (T1DM)?Choose one answer.

A. T1DM will develop in almost all individuals who have positive antibodies against beta cells

B. Compared with other ethnic groups, African Americans have the highest risk for T1DM

C. The strongest genetic association for susceptibility to T1DM is with the insulin gene

D. HLA-DR3 and/or DR4 are susceptibility alleles for T1DM

E. Increased expression of insulin in the thymus is associated with higher risk for development of T1DM

Educational Objective: To understand the multiple genetic factors that contribute to the pathophysiology of type 1 diabetes

Rationale: HLA-DR3 and DR4 are 2 major histocompatibility complex (MHC) class II alleles associated with susceptibility for type 1 diabetes. Although at least one antibody to beta-cell antigens is found in most patients with newly diagnosed type 1 diabetes, individuals with antibodies against beta cells do not necessarily develop diabetes. These individuals display some autoimmunity but not diabetes; current research suggests that this is probably because they have regulatory immune mechanisms, such as regulatory T cells (Tregs), that prevent further destruction of beta cells. Relative to other races, Caucasians have a higher risk of type 1 diabetes. The strongest associations of genes that influence susceptibility to type 1 diabetes are within the MHC class II gene. Lower expression of insulin in the thymus is associated with a higher risk for the development of type 1 diabetes. This is because more thymic insulin expression will allow for more deletion of autoreactive cells specific for insulin.

Which statement regarding beta cells is correct?Choose one answer.

A. Regeneration of pancreatic beta cells does not occur in normal physiology

B. New pancreatic islets are formed mostly during gestation and in the first 2 decades of life

Page 8: Asap 2011

C. Pancreatic stem cells, from which new beta cells arise, can be identified by presence of the glutamic acid decarboxylase (GAD) antigen

D. Beta cells are characterized by rapid cell loss, similar to white blood cells

Educational Objective: To highlight the finding that new beta cells are generated mostly early in life, and to emphasize that the exact mechanism of new beta-cell generation is still unknown

Rationale: New beta cells are generated mostly early in life, during gestation and in the first 2 decades of life. Although it has been established that postnatal beta cells are capable of limited regeneration, the nature of the cells that give rise to these new beta cells has not yet been identified. Possible candidates include beta cells themselves, islet stem cells, and stem cells from other tissues (e.g., hematopoietic stem cells). Specific markers that characterize pancreatic stem cells have not yet been found.

Which statement is correct regarding amylin and its analogue?

Choose one answer.

A. Amylin and glucagon are co-secreted from alpha cells in response to glucose

B. Pramlintide, a synthetic analogue of the naturally occurring hormone amylin, is FDA approved for adjunctive therapy in type 1 diabetes but not in type 2 diabetes

C. Amylin deposits found in islets of patients with type 2 diabetes improve beta-cell function and protect against beta-cell apoptosis

D. Amlyn inhibits glucagon secretion

E. Amylin accelerates gastric emptying

Educational Objective: To understand the function of amylin and its synthetic analogue, pramlintide

Rationale: Amylin is co-secreted with insulin and has several "incretin-like" physiologic effects. Similar to glucagon-like peptide 1 (GLP-1) it slows gastric emptying, inhibits glucagon secretion, and induces satiety. However, in contrast to GLP-1, it inhibits insulin secretion. Amylin deposits found in pancreata of patients with type 2 diabetes have long been thought to play a role in the demise of beta cells. However, recently it was found that smaller toxic forms of this protein appear to be the cause of beta-cell death. The synthetic form of amylin, pramlintide, is a promising adjunct medication for both major forms of diabetes. It has been approved by the FDA for adjunctive therapy in type 1 diabetes and in insulin-requiring type 2 diabetes. However, the necessity to administer pramlintide via subcutaneous injection has limited its acceptance.

Which agent is a direct inhibitor of renin?Choose one answer.

A. Aliskiren

B. Telmisartan

Page 9: Asap 2011

C. Ramipril

D. Eplerenone

E. Carvedilol

Educational Objective: Recognize a new class of antihypertensive agents: renin inhibitors

Rationale: Aliskiren is a direct inhibitor of renin and represents a new class of modulating agents of the renin-angiotensin-aldosterone system for the treatment of hypertension. Telmisartan is an angiotensin-receptor blocking agent. Ramipril is an angiotensin-converting enzyme (ACE) inhibitor. Eplerenone is an aldosterone antagonist. Carvedilol is a beta-adrenergic blocking agent.

Which of the following manifestations on clinical assessment best characterizes

autonomic neuropathy?

Choose one answer.

A. Enhanced pinprick sensation

B. Enhanced vibration perception using a 128-Hz tuning fork

C. Enhanced perception of pressure using a 1-g monofilament at the distal halluces

D. Reduced heart rate variability

E. Decreased post-voiding residual volume on bladder sonography

Educational Objective: Recognize and differentiate between manifestations of

diabetic autonomic neuropathy and peripheral neuropathy

Rationale: Peripheral neuropathy may be indicated by abnormalities on physical

examination such a reduced pinprick sensation, reduced vibration perception, or

reduced monofilament perception in the lower extremities. Manifestations of

autonomic neuropathy may include reduced heart rate variability and increased post-

voiding residual volume.

A 39-year-old woman has been on treatment with metformin for 10 years for type 2 diabetes. She has recently noted, over the past 6 months, progressive symptomatic dysesthesias. Examination findings show loss of monofilament perception and vibratory perception in the

Page 10: Asap 2011

lower extremities, despite an A1C of 6.2% (normal: <6%). She has been normotensive, and serum lipid levels have been well controlled with statin therapy (3-hydroxy-3-methyl-glutaryl co-enzyme A reductase). She does not smoke. Alcohol use is reserved for holidays; she consumes one glass of wine several times per year.

Which of the following must be considered as a possible contributing factor for this patient's neuropathy?

Choose one answer.

A. Folate deficiency

B. Deficiency of alpha-lipoic acid

C. Vitamin B12 deficiency

D. Porphyria

E. Vasculitis

Educational Objective: Consider the possibility of vitamin B12 deficiency in patients with diabetes mellitus treated with metformin

Rationale: Long-term metformin therapy may cause malabsorption of vitamin B12, with consequent worsening of diabetic neuropathy. Folate deficiency is rarely seen in diabetes mellitus per se, unless other comorbid conditions are present, such as celiac disease. Therapy with alpha-lipoic acid therapy may be beneficial for treating the symptoms of diabetic peripheral polyneuropathy, but lipoic acid deficiency has not been reported as a cause or contributing factor to diabetic neuropathy. Porphyria and vasculitis are highly unlikely to be contributing factors to the neuropathy in this patient.

Diabetic retinopathy (DR) is the most common cause of visual loss in adults aged 20 to 74 years. Which statement about DR is the most accurate?Choose one answer.

A. Visual loss develops early in the disease and continues to progress

B. Neuronal changes occur in the retina before the development of clinically evident vascular lesions

C. Intensive blood glucose control reverses the visual loss caused by DR

D. Coexistent systemic conditions such as hypertension and dyslipidemia have little effect on DR

E. Vitreous hemorrhage is the most common cause of vision loss in DR

Educational Objective: Identify the most common cause of visual loss in adults with diabetes mellitus

Rationale: Visual loss may not be evident until advanced stages of the disease. Intensive blood glucose control helps to retard the progression of DR, but it does not completely

Page 11: Asap 2011

reverse the visual loss. Hypertension and dyslipidemia are important comorbid conditions. Macular edema is the most common cause of visual loss in DR

Diabetic ketoacidosis (DKA) is characterized by which of the following?

Choose one answer.

A. Hyperinsulinemia

B. Enhanced peripheral glucose utilization

C. Decreased beta-oxidation of free fatty acids

D. Increased gluconeogenesis

E. Hypoglucagonemia

Educational Objective: Recognize the metabolic disturbances that characterize

diabetic ketoacisdosis (DKA)

Rationale: The hyperglycemia in DKA results from the following three events: (1)

increased gluconeogenesis, (2) accelerated glycogenolysis, and (3) impaired glucose

utilization by peripheral tissues (e.g., liver, muscle, and fat). Also, alteration of lipid

metabolism in DKA is the result of major hormonal changes (decreased insulin and

increased glucagon, catecholamines, cortisol, and growth hormones). Increased free

FFA, as a result of excess lipolysis, provide substrate for beta-oxidation of FFA. This

beta-oxidation leads to increased ketone-body formation, as well as conversion of

FFA to very-low-density lipoprotein (VLDL) by the liver. Increased anion gap and

leukocytosis are usually present in DKA.

Which of the following can falsely lower (cause a false negative) serum insulin-like growth factor-1 (IGF)-1 in a patient with acromegaly?Choose one answer.

A. Menopause

Page 12: Asap 2011

B. Hypoglycemia

C. Uncontrolled diabetes mellitus

D. Obesity

E. Statin therapy

Educational Objective: To understand pitfalls in the diagnosis of acromegaly

Rationale: Measurement of insulin-like-growth factor-1 (IGF-1) is a favored option in the diagnosis of acromegaly. However several caveats apply and physicians must be aware of factors that could potentially affect interpretation. It should be noted that levels must be compared with age dependent and gender specific normative data. Physiologic increase in IGF-1 may be seen during puberty and pregnancy. Estrogens lower IGF-1 level due to its growth hormone antagonism. Hepatic failure, renal failure, uncontrolled diabetes mellitus and malnutrition can be associated with spuriously low IGF-1 levels. Menopause and statin therapy are associated with normal levels of IGF-1. Hypoglycemia and obesity may be associated with normal or elevated levels of IGF-1.

Which ONE of the following statements is most appropriate regarding variations

in the levels of growth hormone (GH) and insulin-like-growth factor-1 (IGF-)

which are tests used in the evaluation of a patient suspected of having

acromegaly?

Choose one answer.

A. Serum IGF-1levels are high in patients with renal failure

B. Random serum GH levels are rarely helpful in the diagnosis of acromegaly

C. Suppression of serum GH level to <1ng/mL after oral glucose tolerance test confirms the diagnosis of acromegaly

D. A serum GH level <0.4ng/mL with a normal IGF-1 level makes the diagnosis of acromegaly highly unlikely

E. Serum IGF-1 levels are rarely useful in the diagnosis of acromegaly

Educational Objective: To recognize value of laboratory tests useful in establishing

the diagnosis of acromegaly

Page 13: Asap 2011

Rationale: Despite well-described clinical features, acromegaly remains vastly

undiagnosed. It is not uncommon for clinicians to miss the diagnosis altogether.

Understanding of normal growth hormone (GH) secretion dynamics and the tests used

in making diagnosis is very crucial. Clinicians must be familiar with conditions

affecting GH and insulin-like-growth factor-1 (IGF-1) measurement. Serum IGF-1,

random GH level, and serial photographs can be used to investigate a patient for

suspected acromegaly. A GH level <0.4ng/mL with a normal IGF-1 level excludes the

diagnosis of acromegaly.

GH suppression following ingestion of glucose is considered the gold standard to

exclude acromegaly. Suppression of GH level to <1ng/mL after a glucose load

excludes the diagnosis of acromegaly. However, failure to suppress or even

paradoxical increase in GH levels may be seen in uncontrolled diabetes mellitus,

renal failure, hepatic failure, and starvation/anorexia nervosa. A failure to

suppression may also be seen during puberty as well as in some patients with obesity.

Systemic illness such as hepatic or renal failure is associated with low serum IGF-1

levels.

Which one of the following tests must be obtained on a patient with known

acromegaly in addition to growth hormone and insulin-like-growth factor-1?

Choose one answer.

A. Serum creatinine

B. Serum potassium

C. Serum prolactin

D. Ultrasound of gall bladder

Page 14: Asap 2011

E. Bone density

Educational Objective: To understand the pathophysiolgy of acromegaly related to

pituitary adenoma

Rationale: Nearly 25% of adenomas may be mixed growth hormone (GH) and

prolactin secreting adenomas. Some contain both cell types while others consist of

monomorphic cells that secrete both GH and prolactin. Syndromes associated with

acromegaly / GH hypersecretion include — McCune Albright syndrome, MEN-1,

Carney complex and familial acromegaly related to AIP (aryl hydrocarbon receptor

interacting protein) gene mutation. It must be noted that stalk compression alone can

lead to prolactin elevation as well. Serum creatinine, potassium, and ultrasound of the

gallbladder and bone density do not need to be obtained in patients with acromegaly

unless otherwise clinically indicated.

A 34-year old gravida 2, para 1, presents at 35-weeks gestation in early labor with nausea,

weakness, polyuria, and polydipsia for several weeks. The pregnancy has been complicated

by recent preeclampsia with elevated liver enzymes.

Laboratory testing reveals serum sodium 141 mEq/L (normal, 135-145 mEq/L), plasma

glucose 95 mg/dL (normal, 80-100 mg/dL), and urine osmolality 100 mOsm/kg H2O (normal,

300-800 mOsm/kg H2O).

Which of the following recommendations is the next step in identifying the etiology of this patient's polyuria and polydipsia?Choose one answer.

A. Measurement of plasma antidiuretic hormone arginine vasopressin

B. Administration of desmopressin (1 deamino 8 D arginine vasopressin)

C. Water deprivation test

D. Measurement of plasma prolactin level

Page 15: Asap 2011

E. Magnetic resonance imaging of the pituitary

Educational Objective: Recognize gestational diabetes insipidus and its initial management

This patient with preeclampsia and liver enzyme abnormalities has symptoms of polyuria and polydipsia with a low urine osmolality in her third trimester of pregnancy. This is suggestive of gestational diabetes insipidus, in which the vasopressinase enzyme produced by the placenta increases clearance of arginine vasopressin (AVP). Excess levels of vasopressinase can be associated with decreased hepatic function, such as fatty liver of pregnancy or preeclampsia with liver involvement (HELLP, Hemolytic anemia, Elevated Liver enzymes, Low Platelets). This condition can also occur in women with partial central diabetes insipidus who may be asymptomatic when not pregnant, but manifest polyuria during pregnancy due to inability to match the rate of vasopressin secretion with increased vasopressin degradation by even normal levels of vasopressinase. Both gestational diabetes insipidus and partial central diabetes insipidus that are unmasked during pregnancy usually remit in the postpartum period. The initial management for this pregnant patient should involve treatment first to correct any existing water deficits and to limit further loss of water. It is usually not worth the potential harm (osmotically-increased AVP levels can cause premature uterine contractions) and delay in treatment to perform a formal diagnostic water deprivation test.

Desmopressin (1 deamino 8 D arginine vasopressin) is effective for treatment of gestational diabetes insipidus because it is resistant to degradation by placental vasopressinase. Aqueous AVP administration is not an effective treatment for polyuria in pregnancy as AVP is readily metabolized by the placental vasopressinase.

Measurement of plasma AVP will not be helpful as it will be low or undetectable in both gestational diabetes insipidus and central diabetes insipidus and therefore, provides essentially no more information than is already known. The treatment would be the same as outlined above. Measurement of prolactin would not be helpful because of the placental hormones present.

Magnetic resonance imaging (MRI) of the pituitary would be negative and provide no further useful information in the evaluation of this patient's polyuria and polydipsia. Also, MRI would need to be done without contrast because of the pregnancy and would be of little diagnostic use without gadolinium.

Which of the following conditions is most likely to be associated with adult growth hormone deficiency?Choose one answer.

A. Hypothyroidism, post-radioiodine therapy

B. Traumatic brain injury

C. Short bowel syndrome

D. Anorexia nervosa

E. Celiac disease

Educational Objective: Understand the causes of adult growth hormone (GH) deficiency

Page 16: Asap 2011

Rationale: GH deficiency is caused by lesions in the hypothalamic-pituitary area, such as pituitary macroadenoma, craniopharyngioma, and other masses, as well as by surgery and radiation used to treat such lesions. In addition, some children with idiopathic GH deficiency remain deficient upon retesting in adulthood. Neurologic disorders, such as traumatic brain injury and subarachnoid hemorrhage can also cause GH deficiency and other pituitary hormone deficiencies. Growth hormone deficiency may also be seen in a variety of conditions in adulthood including empty sella syndrome, granulomatous disease such as neuro-sarcoidosis, Langerhans cell histiocytosis, and metastatic pituitary lesions. There are a number of disorders in which there is GH resistance but not deficiency such as HIV/AIDS, anorexia nervosa, and other malnutrition states. In such conditions, insulin-like growth factor-I (IGF-I) is low but GH levels are high. Post-radioiodine hypothyroidism is not associated with GH deficiency.

A 28-year old woman on stable levothyroxine, cortisol, and growth hormone replacement

therapy elects to begin oral contraceptive therapy.

Which of the following change in medication is most appropriate with the initiation of oral contraceptive therapy?Choose one answer.

A. Reduce growth hormone (GH) dose by approximately 25% to 50% and advise insulin-like growth factor-1 (IGF-1) measurement in 1 to 2 months

B. Decrease thyroxine dose by approximately 50% because of expected increase in thyroid-binding globulin

C. Increase GH dose by 25% to 50% and advise IGF-1 measurement in approximately 1 to 2 months

D. Double glucocorticoid dose because estrogen will increase cortisol metabolism

E. Discontinue GH for 3 months, and then resume at prior dose

Educational Objective: Recognize the blunting of hepatic IGF-1 production that occurs on oral estrogen therapy

Rationale: Oral estrogen blunts the effect of GH at the liver, so that women on oral estrogen need approximately twice the dose of GH to maintain the same level of IGF-1 as women using transdermal estrogen or no estrogen. Therefore, in this patient starting oral estrogen, the dose should be increased and IGF-1 levels followed to ensure ideal dosing. While GH may increase cortisol metabolism in some patients (thereby causing new hypoadrenalism or need to increase glucocorticoid dose) this is not an effect of estrogen. GH may increase the peripheral conversion of tetraiodothyronine (thyroxine, T4) to tri-iodothyronine (T3), but clinical significance of this is uncommon. Thyroid dose is not decreased upon initiation of estrogen.

Which of the following statements is correct regarding growth hormone (GH) therapy in adults with growth hormone deficiency?Choose one answer.

A. There is a significant increase in spinal and hip bone mineral density in most

Page 17: Asap 2011

patients after 18 to 24 months GH replacement

B. There is a significant decrease in weight 6 to 12 months after initiation of GH replacement

C. GH replacement is associated with significant increase in bone mineral density of the radius within 6 months of GH replacement

D. There is worsening of insulin sensitivity in most patients after 12 to 18 months of beginning GH replacement

E. There is a significant decrease in systolic blood pressure in older (> 65 yrs old) men and women

Educational Objective: Recognize the effects of growth hormone (GH) replacement on clinical features of GH deficiency

Rationale: Soon after beginning GH replacement, body fat decreases and lean mass increases. Bone mineral density of the spine and hip also improve, but it takes longer, typically 18-24 months. No change in radial bone mineral density or weight is seen in most patients. The effects of GH therapy on blood pressure are generally neutral, but certainly not beneficial in women over age 65 years. Fasting glucose may rise short term but there is improvement and often normalization of insulin sensitivity after 1 year of GH therapy.

What is the cell origin of the most frequently encountered type of nonfunctioning pituitary tumor?Choose one answer.

A. Somatotroph

B. Corticotroph

C. Thyrotroph

D. Gonadotroph

E. Null cell or oncocyte

Educational Objective: To appreciate the origin of nonfunctional pituitary tumors and their lack of association with a clinical syndrome of hormone hypersecretion.

Rationale: Pituitary adenomas are the most common type of tumors in the sella turcica. They are defined as non-metastasizing neoplasms comprised of endocrine adenohypophysial cells. They usually arise within the bony sella turcica at the base of the skull. Gonadotroph adenomas represent the most frequent form of nonfunctioning pituitary adenomas. A few are of somatotroph or corticotroph origin that do not secrete hormones and are referred to as "silent adenomas". Null cell or oncocyte is not a histologically or histochemically defined cell type. Earlier reports referred to non-functioning adenomas as null cell adenomas.

Skip to main content

Page 18: Asap 2011

Disorders of the Thyroid Gland

You are logged in as Mohammed Aldawish (Logout)

Page path

Home

/ ► My courses

/ ► THYROID

/ ► Assessment

/ ► Disorders of the Thyroid Gland Assessment

/ ► Review of attempt 1

Review of attempt 1

Started onTuesday, 8 October 2013, 11:54 AM

Completed onTuesday, 8 October 2013, 11:54 AM

Time taken25 secs

Grade0.00 out of a maximum of 37.00 (0%)

FeedbackBlocked

Top of Form

Directions: In this section, each item includes a lead question ans a list of options labeled with letters. Select the ONE lettered option that is BEST in each case by selecting the radio

button corresponding to the correct answer.

Question 1

Marks: 1.00

9SD9XfnshK

qsid=146315&aid=3839&qid=191&checksum=1d30c93de1e2dd9b6abea5d28f6d2e22&sesskey=9SD9XfnshK&new state=0

Page 19: Asap 2011

A 63-year-old man has been treated with amiodarone for 2 years for atrial fibrillation. The treatment was effective until a few weeks ago, when he noted palpitations and was found to

have rapid atrial fibrillation again. An intentional weight loss of 3 kg has also occurred over the last 2 months. No new medications were started over the last year. The examination

reveals irregular heart rate at 92 beats/minute, trace lower-extremity edema, and central obesity but is otherwise unremarkable, including a normal thyroid examination.

Laboratory data: thyroid-stimulating hormone (TSH) <0.001 mIU/L (normal, 0.3-4.5), free thyroxine 2.0 ng/dL (normal, 0.8-1.8) and free triiodothyronine 3.4 pg/mL (normal, 2.3-4.2).

Echocardiogram reveals a left ventricular ejection fraction of 50%, unchanged from a year earlier.

Which of the following tests is the most useful to finalize the diagnostic workup for his thyroid abnormalities?

Choose one answer.

A. Thyroid iodine uptake and scan

B. Thyroid ultrasound with Doppler investigation

C. Interleukin-6 (IL-6) level

D. Diagnostic trial 1 week off amiodarone and repeat thyroid tests

E. Thyroperoxidase antibodies

Educational objective: To recognize and conduct the evaluation of amiodarone-induced thyrotoxicosis (AIT)

Rationale: The patient has thyrotoxicosis with a clinical presentation of apathetic hyperthyroidism. Given his use of amiodarone, this thyrotoxicosis would be consistent with

AIT. The important step will be to define if this represents AIT type 1 or type 2. Thyroid ultrasound will be able to identify structural abnormalities (goiter, nodules), and the use of

Doppler will allow an assessment of the gland's vascularity. A highly vascular gland, along with a goiter and/or nodules argues for type 1 AIT but low-normal/low vascularity with

normal size gland argues for a type 2 process. Iodine studies in these patients have not been found to be useful in the majority of studies as the uptake is consistently low owing to the large content of iodine in amiodarone. IL-6 level was thought to be elevated in AIT type 2,

but the test is not very sensitive or specific and has a long turnaround time. Any short-term change in amiodarone dosing will have minimal impact on the drug levels due to its long half-life (50-100 days) and extensive adipose tissue deposition. Therefore, no significant

Page 20: Asap 2011

change in thyroid levels can be expected after discontinuing amiodarone for 1 week. Finally, the thyroperoxidase antibody status will not be able to identify the type of AIT and is more

helpful in identifying patients at risk for hypothyroidism upon initiation of amiodarone therapy.

References:

Basaria S, Cooper DS. Amiodarone and the thyroid. Am J Med. 2005; 118:706-714.

Bogazzi F, Bartalena L, Brogioni S, et al. Color flow Doppler sonography rapidly differentiates type I and type II amiodarone-induced thyrotoxicosis. Thyroid. 1997; 7:541-545.

Martino E, Safran M, Aghini-Lombardi F, et al. Environmental iodine intake and thyroid dysfunction during chronic amiodarone therapy. Ann Intern Med. 1984;

101:28-34.

Incorrect

Marks for this submission: 0.00/1.00.

Question 2

Marks: 1.00

A 63-year-old man is evaluated for recurrent episodes of atrial fibrillation. The cardiologist recommends starting amiodarone. The baseline evaluation is unremarkable for pulmonary,

eye, liver, or thyroid abnormalities, including negative thyroperoxidase antibodies with a thyroid-stimulating hormone (TSH) of 1.8 mIU/L (normal, 0.3-4.5). Amiodarone therapy is

initiated and the patient feels well until another episode of atrial fibrillation occurs 6 weeks later. The patient is evaluated in the emergency room and a set of thyroid function tests is

obtained. TSH is 5.9 mIU/L with free thyroxine 1.7 ng/dL (normal, 0.8-1.8) and free triiodothyronine 1.9 pg/mL (normal, 2.3-4.2). The atrial fibrillation resolves and the patient

feels well. He is referred for endocrine evaluation. He denies any other changes since started on amiodarone and the physical examination is unremarkable.

Which of the following is the most appropriate next step in evaluating his thyroid abnormalities?

Choose one answer.

A. Initiate levothyroxine therapy on a weight-based regimen .

qsid=146316&aid=3839&qid=193&checksum=bbee04eb850d0bc28b20c15c35f92fb5&sesskey=9SD9XfnshK&new state=0

Page 21: Asap 2011

B. Obtain a thyroid ultrasound with Doppler to assess for thyroiditis .

C. Start perchlorate to discharge inorganic iodine .

D. Repeat the thyroid tests in 2 months if the patient remains asymptomatic.

E. Discontinue amiodarone .

Educational objective: To recognize the transient nature of thyroid changes induced by amiodarone in most individuals

Rationale: The patient described has no symptoms of thyroid dysfunction, yet a mild increase of TSH is noted at 6 weeks along with a decrease in triiodothyronine values. These

changes are likely to reflect the inhibition of peripheral 5'-deiodinase that is overcome by most patients after 3 months on amiodarone. Therefore, treating the abnormalities with

levothyroxine is unnecessary in an asymptomatic patient. Same, attempting to reverse this abnormality with perchlorate or by discontinuing amiodarone exposes the patient to undue risks from perchlorate or lack of cardiovascular benefits of amiodarone that are not justified

by the transient nature of these abnormalities. Thyroid ultrasound would be useful in the workup of amiodarone-induced thyroiditis to differentiate type 1 from type 2 but is of no

use in this situation.

References:

Basaria S, Cooper DS. Amiodarone and the thyroid. Am J Med. 2005; 118: 706-714.

Martino E, Safran M, Aghini-Lombardi F, et al. Environmental iodine intake and thyroid dysfunction during chronic amiodarone therapy. Ann Intern Med. 1984 ;

101 :28-34.

Incorrect

Marks for this submission: 0.00/1.00.

Question 3

Marks: 1.00

qsid=146317&aid=3839&qid=195&checksum=c9ca13fa41153ff4bd1be860d1d96dbc&sesskey=9SD9XfnshK&new state=0

Page 22: Asap 2011

A 48-year-old woman is evaluated for epilepsy. She is also treated with levothyroxine for hypothyroidism due to Hashimoto's thyroiditis. Her hypothyroidism has been in good control over the last 6 years and her thyroid-stimulating hormone (TSH) during this evaluation is 1.9 mIU/L (normal, 0.3-4.5). At the conclusion of the evaluation she is started on phenytoin and

is advised to have a repeat TSH in 3 months. At that follow-up she has not had any more seizures but reports mild cold intolerance and fatigue, and her TSH is 9.2 mIU/L.

Which of the following is the most likely explanation for her elevated TSH?

Choose one answer.

A. Lack of medication compliance

B. Spuriously elevated TSH due to an assay problem caused by phenytoin

C. Increased levothyroxine needs due to its accelerated metabolism

D. Increased levothyroxine binding to thyroxine-binding globulin with decreased free thyroxine

E. Inhibition of 5'-deiodinases with decreased T3 levels and hypothyroidism

Educational objective: To recognize the changes in thyroid hormones induced by phenytoin

Rationale: The patient described has long-standing hypothyroidism, and therefore she is dependent on exogenous levothyroxine for preserving her thyroid hormone levels.

Phenytoin does increase the metabolism of thyroid hormones, and therefore the dose of levothyroxine will have to be increased to compensate for this. The assay problem noted

with phenytoin use has been one of low free thyroxine values, related to dilution of the sample during the processing, with preserved normal TSH values. Phenytoin actually

displaces thyroxine and triiodothyronine from their binding proteins and increases the free hormone fraction. (The increased metabolism, though, will still lead to a lower level of free

thyroid hormones in a patient with levothyroxine therapy). There is no evidence for inhibition of thyroxine to triiodothyronine conversion with phenytoin, in contrast

to amiodarone-induced thyroid effects. In the history there is no hint at medication noncompliance, and the clinical course in this case is rather typical for phenytoin-induced

changes.

Reference:

Surks MI, DeFesi CR. Normal serum free thyroid hormone concentrations in patients treated with phenytoin or carbamazepine. A paradox resolved. JAMA. 1996; 275:1495-1498.

Page 23: Asap 2011

Incorrect

Marks for this submission: 0.00/1.00.

Question 4

Marks: 1.00

You are asked to evaluate a 52-year-old woman for abnormal thyroid function tests after her recent visit to her primary care physician for a routine general examination. She reports for

the past 6 months an increased frequency of hot flashes and diaphoresis that she has had for the past 3 years and has attributed to being menopausal. She has been taking Premarin

0.625 mg for the past 3 months with little relief of the symptoms. Her weight fluctuates within 5 lb, but she denies changes in bowel habits. She has noted some difficulty sleeping at

night and has experienced increased fatigue in the past year. She has a history of hypertension, well controlled with atenolol 25 mg/day; and hyperlipidemia, for which she takes simvastatin. She also takes a multivitamin and some over-the-counter supplements.

Her family history is significant for a maternal aunt and a sister with hypothyroidism on levothyroxine replacement.

On physical examination, her blood pressure is 120/75 mmHg, pulse 80 beats/minute and regular, body mass index 26 kg/m2; thyroid is palpable, approximately 25 g, but smooth

without nodularity. No conjunctival injection noted. Skin is dry and warm.

Laboratory studies:

Thyroid-stimulating hormone (TSH)1.2 mIU/L (normal, 0.3-5.0 mIU/L)

Total thyroxine (TT4)13.5 µg/dL (normal, 8-12.5 ng/dL)

Total triiodothyronine (TT3)200 ng/dL (normal, 80-190 ng/dL)

Hemoglobin13.5 g/dL (normal, 12-16 g/dL)

Erythrocyte sedimentation rate (ESR)10 mm/hr (normal, 0-15 mm/hr)

Which of the following is the next best test to evaluate the cause of this patient's abnormal thyroid function tests?

qsid=146318&aid=3839&qid=197&checksum=b7bce1b5a12ed1e771fc925dc59d14ed&sesskey=9SD9XfnshK&new state=0

Page 24: Asap 2011

Choose one answer.

A. Repeat TT4 and TT3 in 2 weeks .

B. Obtain 24-hour thyroid uptake .

C. Repeat TSH .

D. Obtain a pituitary magnetic resonance imaging (MRI).

E. Obtain free thyroxine and free triiodothyronine .

Educational Objective: To recognize euthyroid hyperthyroxinemia due to increased thyroxine-binding globulin (TBG) levels in patients on estrogen therapy

Rationale: This is a case of euthyroid hyperthyroxinemia due to TBG excess secondary to oral estrogen. This woman, who presented for a routine medical examination, does not have

specific symptoms of hyperthyroidism but is found to have elevated levels of total thyroxine (T4) and triiodothyronine (T3) with a normal TSH value. Approximately 99% of circulating T4

and T3 are bound to one of 3 circulating binding proteins: TBG (75%), thyroxine binding prealbumin (TBPA) (15%), and albumin (10%). In women taking oral estrogen, the serum levels of TBG are significantly increased with increased binding of T3 and T4, resulting in

increased serum levels of bound TT4 and TT3. However, the free (biologically active) fraction of T4 and T3 remains normal, and therefore patients are clinically euthyroid. The

discrepancy between the abnormal elevated thyroid hormone levels with a normal TSH in a patient without overt symptoms of hyperthyroidism should prompt the physician to search

for causes of euthyroid hyperthyroxinemia. Measurement of the free fraction of thyroid hormones in serum would show normal levels, confirming that the patient is indeed

euthyroid. Repeating TT4 and TT3 measurement will not be helpful, as these levels will remain elevated as long as the patient remains on oral estrogen, and TSH will not change.

Pituitary MRI is not indicated unless free thyroid hormone levels were elevated, suggesting inappropriate secretion of TSH.

References:

Steingold KA, Matt DW, Deziegler D, et al. Comparison of transdermal to oral estradiol administration on hormonal and hepatic parameters in women with premature ovarian

failure. J Clin Endocrinol Metab. 1991; 73:275-280.

Swanson MA, Custer TR, Suey CM. Free thyroxine and free thyroxine index in women taking oral contraceptives. Clin Nucl Med. 1981; 6:168-171.

Incorrect

Page 25: Asap 2011

Marks for this submission: 0.00/1.00.

Question 5

Marks: 1.00

A 49-year-old woman is referred to you for evaluation of hyperthyroidism. She has been in good health, but in the past 6 months noted an unintentional 7-lb weight loss and difficulty

sleeping at night due to increased palpitations. She denies changes in bowel habits but reports occasional mild heat intolerance, which she attributed to being perimenopausal, as

her periods had become irregular in the past year. She denies eye symptoms. Her family history is significant for a sister with Hashimoto's thyroiditis on levothyroxine and a maternal

aunt with a goiter. She takes no medications except for a daily multivitamin and occasional Tylenol for headaches.

Physical examination shows a slightly anxious woman. Blood pressure, 106/64 mmHg; pulse, 96 beats/minute and regular; body mass index, 20 kg/m2. Eye examination shows no

conjunctival injection or chemosis, and extraocular movements are intact. Thyroid examination shows a diffusely enlarged gland, approximately 25 to 30 g with a 1.5-cm, firm,

palpable nodule in the right mid lobe, which moves freely with swallowing. There is no palpable neck lymphadenopathy and no bruit over the thyroid. There is a very subtle tremor

of her extended fingers, and deep tendon reflexes are brisk and symmetric. The rest of her examination is unremarkable.

Laboratory studies:

Thyroid-stimulating hormone0.02 mIU/L (normal, 0.3-5.0 mIU/L)

Free thyroxine1.8 ng/dL (normal, 0.8-1.8 ng/dL)

Free triiodothyronine4.9 ng/dL (normal, 2.3-4.2 ng/dL)

24-hour thyroid uptake28%) normal, 15-30%(

Complete blood countnormal

Erythrocyte sedimentation rate8 mm/hr (normal, 5-15 mm/hr)

qsid=146319&aid=3839&qid=201&checksum=e4da5d1b4a3a7343078b200b76b5a23e&sesskey=9SD9XfnshK&new state=0

Page 26: Asap 2011

What is the next best test to determine the cause of this patient's hyperthyroidism?

Choose one answer.

A. Measurement of thyrotropin receptor antibodies

B. Thyroid scintigraphy with 24 hour radioiodine uptake

C. Thyroid ultrasound with doppler

D. Fine-needle aspiration (FNA) biopsy of the nodule

E. Measurement of thyroperoxidase antibodies

Educational Objective: To understand the role and utility of different imaging studies in the evaluation of patients with hyperthyroidism and thyroid nodules

Rationale: This patient has evidence of hyperthyroidism and a thyroid nodule. She has uptake that is high normal, which may be seen with both Graves' disease and toxic nodular

goiter. It is important to determine if the hyperthyroidism is due to an autonomously functioning nodule or whether she has Graves' disease and an incidental nonfunctioning nodule. The best study to assess function of a nodule is a thyroid scintigram (scan). If the

nodule is shown by scintigraphy to be hyperfunctioning, FNA biopsy would not be necessary as functioning (so-called "hot") nodules are very seldom malignant. If, on the other hand,

the scintigram shows diffuse symmetrical uptake with a nonfunctioning nodule, this patient would be diagnosed with Graves disease', but a FNA biopsy would be needed to further evaluate the nodule and exclude malignancy, because surgery (rather than radioiodine)

would be indicated if thyroid cancer was found in a patient with Graves' disease and a coincidental nodule. Thyroperoxidase antibodies are often positive in patients with Graves'

disease but may also be found in up to 10% of the population in the absence of overt thyroid dysfunction, and so would not be helpful. Thyrotropin-receptor antibodies are usually

positive in up to 90% of patients with Graves' disease but would not help further in deciding whether the nodule needs to be biopsied. Thyroid ultrasound would not be helpful to

determine whether the nodule is functioning or not or whether the patient has Graves' disease. It could only confirm the presence of the nodule, which does not help beyond what

is already noted on examination of the thyroid.

References:

Page 27: Asap 2011

Intenzo CM, dePapp AE, Jabbour S, et al. Scintigraphic manifestations of thyrotoxicosis. RadioGraphics. 2003; 23:857-868.

Sarkar SD. Benign thyroid disease: what is the role of nuclear medicine? Semin Nucl Med. 2006; 36:185-93.

Incorrect

Marks for this submission: 0.00/1.00.

Question 6

Marks: 1.00

A 43-year-old female pharmacist is referred to you for evaluation of subclinical hyperthyroidism, noted by her primary care provider. Six weeks ago, she developed a sore throat, fatigue, and a low-grade fever. Her local physician diagnosed an upper respiratory

infection and recommended symptomatic treatment with Tylenol. Because fever persisted, a week later she returned to her primary care provider, who prescribed a course of ampicillin.

She improved but continued to complain of fatigue on follow-up 4 weeks later; a thyroid-stimulating hormone (TSH) level was ordered that was suppressed. She denies neck pain,

fever, or weight loss. There is no personal or family history of thyroid disease. She is otherwise healthy, exercises daily at the local gym, and takes no prescription medications,

but recently started taking an over-the-counter (OTC) supplement recommended by a friend to "boost her energy".

Physical examination shows: blood pressure, 105/62 mmHg, pulse, 86 beats/minute and regular; body mass index, 24 kg/m2. Neck examination reveals a 15-g thyroid, nontender, without nodularity; no palpable adenopathy. The rest of the examination is unremarkable.

Laboratory studies:

TSH>0.01 mIU/L (normal, 0.3-5.0 mIU/L)

Free thyroxine (free T4)1.8 ng/dL (normal, 0.8-1.8 ng/dL)

Free triiodothyronine (FT3)3.9 ng/dL (normal, 2.3-4.2 ng/dL)

Complete blood countnormal

qsid=146320&aid=3839&qid=208&checksum=5693a59a1f7773a37e98cae4cc0e59f2&sesskey=9SD9XfnshK&new state=0

Page 28: Asap 2011

Erythrocyte sedimentation rate14 mm/hr (normal, 5-15 mm/hr)

24-hour thyroid uptake0.8) % normal, 10%-30%(

What is the next best test in the evaluation of this patient's hyperthyroidism?

Choose one answer.

A. Measurement of thyroid peroxidase antibodies

B. Repeat TSH

C. Thyroid ultrasound

D. Measure serum thyroglobulin

E. Thyroid scintigraphy (scan)

Educational Objective: To distinguish between potential causes of hyperthyroidism with low-iodine uptake and recognize factitious thyrotoxicosis

Rationale: This patient, with a recent history of upper respiratory infection and mild subclinical thyrotoxicosis, could have painless thyroiditis, which would typically be expected

to resolve spontaneously and requires no further treatment in the absence of symptoms. However, she gives a history of recently starting an OTC supplement to "boost energy," and

the possibility of exogenous (surreptitious or unintentional) thyroid hormone ingestion needs to be considered. Thyroglobulin measurement is the best test to distinguish between

these 2 possibilities because both conditions can present with identical laboratory results and both would show low iodine uptake. Serum thyroglobulin, however, is usually elevated

in cases of destructive thyroiditis due to release into the circulation of pre-formed thyroid hormone, but will typically be suppressed or low in patients taking exogenous thyroid

hormone. Thyroid scintigraphy would not be helpful in this patient with very low uptake, and ultrasound is not indicated in the absence of abnormalities on examination. Thyroid

peroxidase antibodies will likely be negative in both conditions and would not help make this distinction.

Reference:

Page 29: Asap 2011

Ross DS. Syndromes of thyrotoxicosis with low radioactive iodine uptake. Endocrinol Metab Clin North Am. 1998; 27:169-185.

Incorrect

Marks for this submission: 0.00/1.00.

Question 7

Marks: 1.00

A 46-year-old veterinarian is referred to you for evaluation of abnormal thyroid function tests found as part of his routine medical examination. He has a history of hyperlipidemia

and hypertension, for which he takes simvastatin and hydrochlorothiazide, respectively. He smokes 1 pack of cigarettes daily. He feels well and denies any significant weight changes,

heat or cold intolerance, or change in bowel habits. Family history is negative for thyroid disease.

Physical examination shows a well-appearing man: blood pressure, 130/86 mmHg; pulse, 86 beats/minute and regular; body mass index, 30 kg/m2. Thyroid is normal to palpation

without nodules, and there is no cervical adenopathy. Eye examination shows no conjunctival injection or chemosis. There is no tremor of his fingers, and he has normal deep

tendon reflexes.

Laboratory tests:

Thyroid-stimulating hormone (TSH)1.8 mIU/L (normal, 0.3-5 mIU/L)

Free thyroxine (FT4)2.8 ng/dL (normal, 0.8-1.8 ng/dL)

Free triiodothyronine (FT3)3.8 pg/mL (normal, 2.4-4.2 pg/mL)

Complete blood countnormal

Erythrocyte sedimentation rate (ESR)8 mm/hr (normal, 0-15 mm/hr)

Alanine aminotransferase (ALT)24 U/L (normal, 7-55 U/L)

qsid=146321&aid=3839&qid=211&checksum=d1bfbb860e2e8f2bef499ee2b32cd40c&sesskey=9SD9XfnshK&new state=0

Page 30: Asap 2011

What is the next appropriate step in the evaluation of this patient’s thyroid status?

Choose one answer.

A. Obtain pituitary magnetic resonance imaging (MRI)

B. Obtain a thyroid uptake and scan

C. Repeat TSH

D. Start methimazole therapy

E. A. Repeat FT4 by equilibrium dialysis

Educational Objective: To recognize euthyroid hyperthyroxinemia and differential diagnosis of inappropriate secretion of TSH

Rationale: This patient has evidence of hyperthyroxinemia on laboratory testing but shows no physical signs or clinical symptoms to suggest thyrotoxicosis, and no evidence of goiter.

Laboratory tests, however, suggest inappropriate secretion of TSH. Although a TSH-secreting pituitary adenoma is a possibility, these tumors are very rare, and such patients generally

have signs and symptoms of hyperthyroidism and often a goiter as well. Another consideration is the possibility of thyroid hormone resistance, but the lack of a family history

of thyroid disease or abnormalities of thyroid function makes this unlikely.

This patient has euthyroid hyperthyroxinemia. Interfering heterophilic antibodies (antithyroxine immunoglobulins) are more common than pituitary TSH-producing adenomas

and need to be excluded, as these can result in spurious elevations of thyroxine levels, particularly when FT4 is measured by conventional radioimmunoassay. Measurement of FT4

by equilibrium dialysis would help avoid this problem. Repeat TSH is likely to be normal. A thyroid scan and MRI would not be helpful, as these will also likely be normal. Treatment

with methimazole would not be indicated without previous confirmation of hyperthyroidism.

References:

Stockigt JR. Dyer SA. Mohr VS. et al. Specific methods to identify plasma binding abnormalities in euthyroid hyperthyroxinemia. J Clin Endocrinol Metab.1986; 62:230-233.

Weintraub BD, Menezes-Ferreira MM, et al. Inappropriate TSH secretion. Endocr Res. 1989; 15: 601-617.

Page 31: Asap 2011

Incorrect

Marks for this submission: 0.00/1.00.

Question 8

Marks: 1.00

A 26-year-old woman presents to your office for evaluation and management of hyperthyroidism. She reports a 3-month history of palpitations, nervousness, and difficulty

sleeping, along with an unintentional 15-lb weight loss. She has difficulty climbing more than one flight of stairs due to muscle weakness and in the past month has noted increasing eye

redness and a gritty sensation with frequent tearing and marked periorbital edema in the mornings. Her mother had a thyroidectomy many years ago and is on levothyroxine and one

older sister has Hashimoto's thyroiditis and is also on thyroid hormone replacement. She is reluctant to consider surgery. She has a 5-month-old baby at home and is still breast-

feeding.

Her examination shows an anxious, thin woman with a blood pressure of 100/56 mmHg, pulse 110 beats/minute and regular, and a body mass index of 20 kg/m2. Eye examination is

notable for positive stare and lid lag, and mild conjunctival injection but no chemosis. Extraocular movements are intact. Thyroid examination reveals a 35-g, diffusely enlarged goiter without nodules. No bruit is noted over her thyroid gland. There is an obvious fine

tremor of her extended fingers. Cardiac examination shows only tachycardia but no murmurs. Deep tendon reflexes are brisk.

Laboratory tests:

Thyroid stimulating hormone (TSH)>0.001m IU/L (normal, 0.3-5 mIU/L)

Free thyroxine (FT4)3.2 ng/dL (normal, 0.8-1.8 ng/dL)

Free triiodothyronine (FT3)6.2 pg/mL (normal, 2.4-4.2 pg/mL)

Hemoglobin10 g/dL (normal, 12-15.5 g/dL)

Aspartate transaminase86 U/L (normal, 8-48 U/L)

Alkaline phosphatase180 U/L (normal, 52-144 U/L)

qsid=146322&aid=3839&qid=214&checksum=76107787c01c331caabc961979f422f5&sesskey=9SD9XfnshK&new state=0

Page 32: Asap 2011

Which of the following is the best treatment option?

Choose one answer.

A. Start a β-adrenergic blocking agent

B. Initiate propylthiouracil (PTU) 100 mg 3 times daily

C. Radioactive iodine ablation

D. A β-adrenergic blocking agent and methimazole

Educational Objective: To be able to manage Graves' disease

Rationale: This young woman clearly has Graves' disease, as she has clear evidence of hyperthyroidism, a goiter and eye symptoms consistent with this diagnosis. She is

symptomatic and has a small child whom she is still breast-feeding. To control her tachycardia and adrenergic symptoms, a β-blocker should be started and dose titrated as needed, but this will not be sufficient to control her hyperthyroidism as thyroid hormone

levels will not be significantly affected. Because she is breast-feeding, radioactive iodine would not be an option at this time. Although PTU would be effective to reduce thyroid

hormone synthesis and secretion and reduce thyroid hormone levels, there are concerns regarding serious hepatotoxicity with PTU. The US Food and Drug Administration has

recently issued a warning that PTU be considered only for second-line therapy in patients with adverse reactions to methimazole who refuse or are not good candidates for surgery or

radioiodine. For this patient, therefore, the best treatment option would be a combination of β-blocker and methimazole.

References:

Pearce EN, Braverman LE. Hyperthyroidism: advantages and disadvantages of medical therapy. Surg Clin North Am. 2004; 84:833-847.

Cooper DS, Rivkees SA. Putting PTU in perspective. J Clin Endocrinol Metab. 2009; 94:1881-1882.

Cooper DS. Antithyroid drugs. N Engl J Med. 2005; 352:905-917.

Incorrect

Page 33: Asap 2011

Marks for this submission: 0.00/1.00.

Question 9

Marks: 1.00

Which of the following drugs increases the need for levothyroxine by forming an insoluble chelate with levothyroxine, and thereby decreasing its absorption?

Choose one answer.

A. Calcium carbonate

B. Carbamazepine

C. Amiodarone

D. Metformin

E. Amlodipine

Educational Objective: To understand the mechanisms by which common drugs and supplements affect the treatment of hypothyroidism

Rationale: Calcium carbonate, like ferrous sulfate, may form complexes with levothyroxine (LT4) in the gut, decreasing LT4 absorption. Therefore, it is important to separate the

ingestion of these drugs by 4 hours. Carbamazepine, along with other anticonvulsants, stimulates hepatic enzyme activity that degrades thyroxine more rapidly, and this may result

in an increased need for LT4. Amiodarone, along with large doses of propanolol and glucocorticoids, decrease T4- 5'- deiodinase activity, thereby decreasing T4 to T3 conversion

peripherally, which may increase the need for LT4 in hypothyroidism. Metformin and amlodipine do not affect LT4 absorption or metabolism and do not increase LT4 need.

Reference:

Meier CA, Burger AG. Effects of pharmacologic agents on thyroid hormone homeostasis. In: Braverman LE, Utiger RD, eds. Werner and Ingbar's the Thyroid: A Fundamental and Clinical

Text, 7th ed. Philadelphia, PA: Lippincott-Raven; 1996: 276-286.

qsid=146323&aid=3839&qid=217&checksum=ce1e8dd884d7311a1907c44ce4da38c0&sesskey=9SD9XfnshK&new state=0

Page 34: Asap 2011

Incorrect

Marks for this submission: 0.00/1.00.

Question 10

Marks: 1.00

In reference to chronic autoimmune thyroiditis (CAT), which of the following statements is most appropriate?

Choose one answer.

A. The established hypothyroidism of CAT is always permanent .

B. The suppression goal in the treatment of a goiter due to CAT is a thyroid-stimulating hormone (TSH) level of 0.2 mU/L .

C. Rapid enlargement of a goiter associated with CAT should be reason to consider more aggressive TSH suppression or radioactive iodine therapy .

D. Nodules developing in a goiter associated with CAT are about 75% likely to be benign .

E. If thyroid-stimulating immunoglobulins develop in a goitrous hypothyroid patient on levothyroxine treatment, eventual hyperthyroidism could result .

Educational Objective: To understand the various aspects of CAT and its treatment

Rationale: The hypothyroidism of CAT is usually permanent, but there are transient cases, and even cases of transformation from hypothyroidism to hyperthyroidism over time.

Generally, if suppression of a goiter is being considered in a euthyroid patient with CAT, the TSH level should not be allowed to fall below the lower limit of normal (0.4-0.5 mU/L). More aggressive treatment is reserved for thyroid cancer cases. A rapidly enlarged thyroid gland in

the setting of CAT should raise concern for thyroid lymphoma. Nodules within a CAT goiter can certainly be malignant, but at about the same incidence as nodules developing in a

normal thyroid gland (5%-10%). It is rare for a hypothyroid CAT patient to become hyperthyroid, but it is possible, usually due to the development of thyroid-stimulating

immunoglobulins.

qsid=146324&aid=3839&qid=219&checksum=a54a45ba5684e7d880c1a6d5b967d8fe&sesskey=9SD9XfnshK&new state=0

Page 35: Asap 2011

Reference

1 .American Association of Clinical Endocrinologists. AACE medical guidelines for clinical practice for the evaluation and treatment of hyperthyroidism and hypothyroidism. Endocr

Pract. 2002; 8:457-469.

Incorrect

Marks for this submission: 0.00/1.00.

Question 11

Marks: 1.00

Which of the following statements regarding the evaluation of hypothyroidism is most appropriate?

Choose one answer.

A. Free thyroxine measurement is the most sensitive test of thyroid status .

B. Free triiodothyronine levels should be obtained as part of screening for hypothyroidism .

C. Serum levels of thyroid peroxidase antibodies are useful in monitoring response to treatment .

D. Patients with central hypothyroidism always have low serum levels of thyroid-stimulating hormone (TSH) .

E. Serum TSH levels are the best test to monitor response to treatment in patients with primary hypothyroidism .

Educational Objective: To understand the value of different thyroid function tests in the evaluation of patients with hypothyroidism

Rationale: In patients with primary hypothyroidism, measurement of levels of sensitive TSH is the most important and reliable test of thyroid function, as even mild thyroid dysfunction

can be detected before overt hypothyroidism occurs (subclinical hypothyroidism), as TSH usually will become elevated even before levels of serum thyroid hormones (thyroxine and

qsid=146325&aid=3839&qid=221&checksum=f21d4782c5d2803e5ded52be1a4fc9c2&sesskey=9SD9XfnshK&new state=0

Page 36: Asap 2011

triiodothyronine) drop below normal. Free thyroxine levels may be normal in the early stages of hypothyroidism. Thyroid peroxidase antibody levels are useful in identifying the

cause of hypothyroidism (autoimmune thyroid disease or Hashimoto’s thyroiditis), but levels are not helpful in monitoring response to treatment as they may remain elevated

even after normalization of thyroid function tests. Patients with central hypothyroidism may have low or (inappropriately) normal serum levels of TSH, which fail to rise in response to

declining levels of thyroid hormone.

Reference

1 .Helfand M, Schmittner J. Screening for thyroid dysfunction: which test is best? JAMA. 1993; 270:2297-2298.

Incorrect

Marks for this submission: 0.00/1.00.

Question 12

Marks: 1.00

Which of the following statements regarding hypothyroidism is most appropriate regarding hypothyroidism?

Choose one answer.

A. Calcium-channel blocking agents produce a transient hypothyroidism in low doses .

B. The hypothyroidism associated with subacute thyroiditis is recurrent and ultimately becomes permanent .

C. The hypothyroidism associated with Hashimoto's thyroiditis is transient and does not require therapy .

D. Therapy with interferon-α can result in hypothyroidism

Educational objective: To review the causes of transient/reversible hypothyroidism

Rationale: Certain drugs, such as interferon-α, lithium, and amiodarone or other iodine-containing medications, can result in hypothyroidism, which may resolve with

qsid=146326&aid=3839&qid=223&checksum=3e016a684c57fd1494560c48215a68b1&sesskey=9SD9XfnshK&new state=0

Page 37: Asap 2011

discontinuation of the offending agent. Subacute thyroiditis is often associated with an initial phase of thyrotoxicosis, often followed by transient hypothyroidism that will generally

resolve spontaneously and patient will become euthyroid. Hashimoto's thyroiditis, however, is an autoimmune thyroid disorder that leads to gradual but progressive cell and antibody mediated destruction of thyroid tissue, with ensuing hypothyroidism that is not reversible

and requires replacement with exogenous thyroid hormone to achieve euthyroid state. Calcium-channel blocking agents are not associated with hypothyroidism.

Reference

1 .Ross DS. Disorders that cause hypothyroidism. Up to date. Online 18.1. January 2010.

Incorrect

Marks for this submission: 0.00/1.00.

Question 13

Marks: 1.00

Subclinical hypothyroidism is most likely to be associated with which of the following?

Choose one answer.

A. Thyroid-stimulating hormone (TSH) adenoma of the pituitary

B. Hypokalemia

C. Hypercholesterolemia

D. Atrial fibrillation

E. Absence of progression to overt hypothyroidism

Educational objective: To recognize possible symptoms and risks of subclinical hypothyroidism

Rationale: Subclinical hypothyroidism, although most often asymptomatic, can sometimes be associated with subtle neuropsychiatric effects or mild hyperlipidemia and will often

qsid=146327&aid=3839&qid=226&checksum=ac0b7ee33c70b4763caaea119ba0a4c5&sesskey=9SD9XfnshK&new state=0

Page 38: Asap 2011

progress to overt hypothyroidism. Although most of these effects have been documented in patients with TSH levels above 10 mU/L, such symptoms and findings are rare and even

controversial in patients with more subtle thyroid dysfunction. Increased risk of atrial fibrillation, however, is associated with subclinical hyperthyroidism. Hypokalemia is not

associated with hypothyroidism. TSH adenomas of the pituitary are associated with hyperthyroidism.

References

Cooper DS. Clinical practice: subclinical hypothyroidism. N Engl J Med. 2001; 345:260-265.

McDermott MT, Ridgway EC. Subclinical hypothyroidism is mild thyroid failure and should be treated. J Clin Endocrinol Metab. 2001; 86:4585-4590.

Incorrect

Marks for this submission: 0.00/1.00.

Question 14

Marks: 1.00

In a patient with acute renal failure, which of the following changes in thyroid function tests would you expect to find?

Choose one answer.

A. Low thyroid-stimulating hormone (TSH), normal thyroxine (T4), low triiodothyronine (T3), and low reverse triiodothyronine (rT3)

B. Normal TSH, low T4, low T3, normal rT3

C. High TSH, high T4, high T3, normal rT3

D. Normal TSH, normal T4, low T3, high rT3

E. Low TSH, high T4, high T3, high rT3

qsid=146328&aid=3839&qid=229&checksum=7602f5ac3385cd153e47dfa5a7f46fa2&sesskey=9SD9XfnshK&new state=0

Page 39: Asap 2011

Educational Objective: To learn the changes in thyroid function tests associated with acute renal failure

Rationale: In acute renal failure, serum T4 and T3 levels are low, but free T4 and TSH levels are normal. Reverse T3 level is in the normal range, which is in contrast to the marked

elevation seen in patients with nonthyroidal illness syndrome.

Reference

Kaptein EM, Levitan D, Feinstein EI, et al. Alterations of thyroid hormone indices in acute renal failure and in acute critical illness with and without acute renal failure. Am J Nephrol.

1981; 1:138-143.

Incorrect

Marks for this submission: 0.00/1.00.

Question 15

Marks: 1.00

A 62-year-old man is started on amiodarone as treatment for atrial fibrillation .On examination, the thyroid is of normal size; blood pressure is 120/80 mmHg ;

heart rate is 72 beats/minute and regular. The patient does not have a tremor and his deep tendon reflexes are normal.

Which one of the following patterns of thyroid function tests would you expect to see for this patient 4 months after the initiation of amiodarone therapy?

Choose one answer.

A. Thyroid-stimulating hormone (TSH) 0.35 µU/mL (normal, 0.35-5.5), total thyroxine (T4) 15 µg/dL (normal, 8-12), total triiodothyronine (T3) 76 ng/dL

(normal, 80-210)

B. TSH 0.2 µU/mL, total T4 7 µg/dL, total T3 70 ng/dL, free T4 1.2 ng/dL (normal, 0.7-1.8)

C. TSH 0.2 U/mL, total T4 10 g/dL, total T3 65 ng/dL

qsid=146329&aid=3839&qid=233&checksum=d03c870fa80834296650a274ca08adf8&sesskey=9SD9XfnshK&new state=0

Page 40: Asap 2011

D. TSH 2 µU/mL, total T4 16 µg/dL, free T4 1.0 ng/dL

E. TSH 5.2 µU/mL, total T4 5 µg/dL, total T3 100 ng/dL

Educational Objective: To learn the effects of amiodarone on thyroid hormone tests

Rationale: Amiodarone causes a decrease in hepatic uptake of T4 and a decrease in the conversion of 5'-monodeiodination of T4 to T3, with resultant high-normal or high T4 levels and low to low-normal T3 concentrations. Because amiodarone contains a large amount of iodine, hyperthyroidism may be precipitated in patients with autonomous thyroid nodules

by invoking the Jod-Basedow phenomenon.

References

Surks MI. Drug interactions with thyroid hormones. In: UpToDate. Available at: www.utdol.com. Accessed on 1/31/10.

Wang R, Melson JC, Wilcox RB. Salsalate administration — a potential pharmacological model of the sick euthyroid syndrome. J Clin Endocrinol Metab. 1998; 83:3095-3099.

Incorrect

Marks for this submission: 0.00/1.00.

Question 16

Marks: 1.00

Which of the following statements is correct regarding the evaluation and management of patients with nonthyroidal illness syndrome (NTIS)?

Choose one answer.

A. In mild illness and starvation, serum triiodothyronine (T3) levels increase ,and as the severity of the illness worsens, both thyroxine (T4) and T3 levels

increase .

B. In the most severe forms of euthyroid sick syndrome, the thyroid-stimulating hormone (TSH) concentration may be at the lower end of the normal range, or

frankly suppressed, despite low total T4 and T3 concentrations .

qsid=146330&aid=3839&qid=237&checksum=7ed2c60ec0e28eabfc824ff26fb90305&sesskey=9SD9XfnshK&new state=0

Page 41: Asap 2011

C. Reverse T3 is decreased in NTIS .

D. Administration of levothyroxine has been shown to improve clinical outcomes in patients with NTIS .

E. TSH decreases in the recovery phase of NTIS .

Educational Objective: To learn the pattern of change of thyroid hormone levels in NTIS

Rationale: NTIS, often referred to as euthyroid sick syndrome, refers to the abnormalities in thyroid function tests that occur during nonthyroidal illness. These abnormalities in thyroid

hormone concentration occur in a wide variety of illnesses, including severe infection, trauma, myocardial infarction, major surgery, malignancy, inflammatory conditions, and starvation. In mild illness and starvation, T3 levels decrease. As the severity of the illness worsens, serum (T4 and T3 levels decrease. In moribund patients, serum TSH levels also decrease to subnormal levels. In the recovery phase, TSH may rise as high as 20 µU/mL.

Several studies have shown that as the levels of T4 and T3 decrease, prognosis worsens and mortality increases. Patients with low T4 and very low T3 have the greatest mortality. These

alterations in thyroid hormone levels result from changes in the hypothalamic-pituitary-thyroid axis, thyroid hormone binding to serum proteins, tissue uptake of thyroid hormones,

and thyroid hormone metabolism. Treatment with levothyroxine has not been shown to improve mortality or clinical outcomes in humans.

References

Chopra IJ. Euthyroid sick syndrome: is it a misnomer. J Clin Endocrinol Metab. 1997; 82:329-334.

De Groot LJ. Dangerous dogmas in medicine: the nonthyroidal illness syndrome .J Clin Endocrinol Metab. 1999; 84:151-164.

McIver B, Gorman CA. Euthyroid sick syndrome: an overview. Thyroid. 1997;7:125-132.

Incorrect

Marks for this submission: 0.00/1.00.

Question 17

Marks: 1.00

Page 42: Asap 2011

A 30-year-old man with hepatitis C is treated with interleukin-2 and interferon-α.

Three months into treatment he develops fatigue, insomnia, and tachycardia.

Biochemical testing revealed:

Thyroid-stimulating hormone (TSH)>0.01 µU/mL (normal, 0.3-4.0)

Free thyroxine (T4)3.2 ng/dL (normal, 0.8-1.8)

Triiodothyronine (T3)450 ng/dL (normal, 80-210)

Which of the following would be the best treatment option?

Choose one answer.

A. Propranolol LA (long-acting) 60 mg twice daily

B. Methimazole 20 mg daily

C. Thyroidectomy

D. Radioactive iodine therapy

E. Potassium iodide (SSKI)

Educational Objective: To understand that interleukin-2 (IL-2) and interferon-α therapy for hepatitis C may cause thyroiditis in individuals with thyroid autoimmunity

Rationale: Thyroiditis with hyperthyroid and hypothyroid phases have been reported with the use of IL-2 and interferon-α for the treatment of hepatitis C. This effect usually occurs in

patients with underlying thyroid autoimmunity.

The patient described has thyrotoxicosis most likely related to IL-2 and interferon-α therapy. The best treatment option is treatment of the tachycardia with a β-adrenergic blocking

qsid=146331&aid=3839&qid=241&checksum=44058ba1e75d0d33398f4175b5dd1ce5&sesskey=9SD9XfnshK&new state=0

Page 43: Asap 2011

agent such as propranolol. If the thyrotoxicosis persists, a decision regarding continuing IL-2 and interferon-α therapy will need to be made or the patient may need therapy with anti-

thyroid drugs such as methimazole to control the thyrotoxicosis. The anti-thyroid drug would be discontinued when the thyrotoxicosis remits. Methimizole may have adverse hepatic

effects, and liver function will need to be carefully monitored while the patient receives this treatment. Radioiodine therapy and thyroidectomy are not indicated until persistence of the drug-induced thyrotoxicosis occurs and the offending medication needs to be continued for

the underlying liver disease and anti-thyroid drugs are contraindicated. Potassium iodide may initially be beneficial by suppressing thyroid hormone production in the thyroid, but

escape from this effect occurs with return of the thyrotoxicosis, which may be aggravated by the excess iodine intake.

Reference

Surks MI. Drug interactions with thyroid hormones. In: UpToDate. Available at: www.utdol.com. Accessed on 1/31/10.

Incorrect

Marks for this submission: 0.00/1.00.

Question 18

Marks: 1.00

You are called to the intensive care unit to evaluate a patient with abnormal thyroid function tests. The patient was admitted with pneumonia and respiratory failure. A goiter was noted

on physical examination and thyroid function tests were obtained.

Thyroid-stimulating hormone (TSH)0.09 µU/mL (normal, 0.4-4)

Total thyroxine (T4)4 µg/dL (normal, 8-12)

Total triiodothyronine (T3)10 ng/dL (normal, 80-120)

Which of the following would help you determine if the patient has central hypothyroidism or nonthyroidal illness syndrome (NTIS)?

Choose one answer.

qsid=146332&aid=3839&qid=245&checksum=7d5dda5ad0c74098f9c0baa4c712fe67&sesskey=9SD9XfnshK&new state=0

Page 44: Asap 2011

A. Free T<SUB4< sub> by equilibrium dialysis

B. Reverse T3

C. Free T3 by equilibrium dialysis

D. Heart rate

E. Temperature

Educational Objective: To learn how to differentiate between central hypothyroidism and NTIS

Rationale: Reverse T3 will be low in hypothyroidism but elevated in NTIS. The serum concentration of reverse T3 (rT3) is increased in NTIS, except in the case of renal failure.

Decreased enzymatic activity of 5'-monodeiodinase in tissue leads to decreased conversion of T4 to T3 and metabolic clearance of rT3. The T3 level also decreases whenever caloric

intake is low and in any nonthyroidal illness, even mild illness.

References

McIver B, Gorman CA. Euthyroid sick syndrome: an overview. Thyroid. 1997; 7:125-132.

Kaptein EM, Robinson WJ, Grieb DA, et al. Peripheral serum thyroxine, tri-iodothyronine and reverse triiodothyronine kinetics in the low thyroxine state of acute nonthyroidal illnesses. J

Clin Invest. 1982; 69:526-535.

Incorrect

Marks for this submission: 0.00/1.00.

Question 19

Marks: 1.00

Which of the following sets of laboratory studies would be most consistent with an asymptomatic patient with familial dysalbuminemic hyperthyroxinemia (FDH)?

)FT4 = free thyroxine; FT3 = free triiodothyronine; TSH = thyroid-stimulating hormone(

qsid=146333&aid=3839&qid=248&checksum=5af370df390b2ff5a35e9a64da2111c8&sesskey=9SD9XfnshK&new state=0

Page 45: Asap 2011

FT4 (ng/dL))normal, 0.8-2.0(

FT3 (pg/dL))normal, 1.1-4.7(

TSH (mU/L))normal, 0.3 -4.0(

A.

1.02.86.9

B.

3.17.40.9

C.

3.33.01.3

D.

2.66.35.8

E.2.14.952.7

Choose one answer.

A.

B .

C .

D.

E .

Educational Objective: To correctly interpret thyroid function tests in both healthy patients and those with thyroid disorders

Rationale: The patient with FDH has an abnormal T4-binding protein that does not bind T3, resulting in a high free T4 level (if the assay is by radioimmunoassay) and not a high T3 level.

This patient, as is typical of FDH, is euthyroid based on the normal TSH level.

Laboratory results in choice A would be consistent with an asymptomatic patient with mild subclinical hypothyroidism, with positive thyroid autoantibodies. The patient illustrated in

Page 46: Asap 2011

choice B has antimouse antibodies (heterophile antibodies), which will bind to T4 and T3 and elevate the levels, but this individual is euthyroid based on the normal TSH level.

The laboratory results in choice D are typical of the resistance to thyroid hormone syndrome ,

or a thyroid-stimulating hormone adenoma (TSH-oma). The patient in option E has resistance to thyroid hormone (RTH) and primary hypothyroidism, with a TSH level >10 mU/L

(often much higher) but normal to high free T4 and free T3 due to the thyroid hormone resistance.

Reference

Tran HA. Difficulties in diagnosing coexisting primary hypothyroidism and resistance to thyroid hormone. Endocr Pract. 2006; 12:228-293.

Incorrect

Marks for this submission: 0.00/1.00.

Question 20

Marks: 1.00

A young woman has episodes of anxiety and intermittent palpitations. There are no significant physical findings. The free thyroxine (T4) level is 2.9 ng/dL (normal, 0.5-2.0), free triiodo-thyronine (T3) level is 3.1 pg/dL (normal, 1.1-4.7), and thyroid-stimulating hormone

(TSH) level is 2.1 mU/L (normal, 0.3-4.5). The results are unchanged when repeated at a different laboratory.

Which of the following statements best fits the presenting symptoms and laboratory values?

Choose one answer.

A. She has T4 toxicosis and the normal T3 does not suppress the TSH.

B. She has resistance to thyroid hormone and is euthyroid .

C. She is secretly taking oral contraceptives .

qsid=146334&aid=3839&qid=251&checksum=4764303d8d696ab5fb8107025b00d762&sesskey=9SD9XfnshK&new state=0

Page 47: Asap 2011

D. She has other family members with similar laboratory results .

E. This is a laboratory error or there are anti-T4 antibodies

Educational Objective: To correctly interpret abnormal thyroid function tests

Rationale: In the patient described in the clinical vignette, the free T4 level is abnormally high but TSH and free T3 levels are normal. In spite of symptoms, this patient is euthyroid as

reflected by the normal TSH. She does not have T4 toxicosis, as she is not hyperthyroid.

In resistance to thyroid hormone, both the free T4 and free T3 are elevated, which is not the case here. Oral contraceptives will not raise the free T4. Laboratory error is always possible, but it is not the best choice here given that the tests were repeated. Anti-T4 antibodies are

excluded because these antibodies bind both the T4 and T3. The patient most likely has familial dysalbuminemic hyperthyroxinemia, and other family members would likely test

similarly.

Reference

Tran HA. Difficulties in diagnosing coexisting primary hypothyroidism and resistance to thyroid hormone. Endocr Pract. 2006; 12:228-293.

Incorrect

Marks for this submission: 0.00/1.00.

Question 21

Marks: 1.00

In reference to resistance to thyroid hormone (RTH), which of the following statements is most appropriate?

Choose one answer.

A. The free thyroxine (FT4) level is always elevated .

B. Less than 50% of cases are associated with a mutation in the thyroid hormone (TH) receptor (TR) β gene .

qsid=146335&aid=3839&qid=255&checksum=30d957c9b21ac12e36747f989c696211&sesskey=9SD9XfnshK&new state=0

Page 48: Asap 2011

C. Most cases are clinically hypothyroid and require treatment .

D. RTH may be differentiated from a thyroid-stimulating hormone-adenoma (TSH-oma) by showing that in the TSH-oma the TSH level suppresses with levothyroxine

and stimulates with thyrotropin-releasing hormone .

E. Goiter and bradycardia are commonly observed .

Educational Objective: To review the clinical features of resistance to thyroid hormone (RTH)

Rationale: The hallmark of RTH is an elevated FT4 and FT3 level. RTH can be excluded by a normal FT4 in the absence of coexisting primary hypothyroidism. RTH is associated with an

abnormal TRβ gene in 90% of cases. Most cases of RTH are euthyroid, with the high thyroid-stimulating hormone (TSH) compensating for the resistance. There may be individual body

tissues that have decreased or increased thyroid hormone activity. On the contrary, the TSH is suppressible and able to be stimulated in RTH and much less so or not at all in a TSH-oma. Although goiter is the most common physical finding in RTH (up to 95%), tachycardia (up to

80%) rather than bradycardia is also common.

Reference

1 .Tran HA. Difficulties in diagnosing coexisting primary hypothyroidism and resistance to thyroid hormone. Endocr Pract. 2006; 12:228-293.

Incorrect

Marks for this submission: 0.00/1.00.

Question 22

Marks: 1.00

Which of the following findings are commonly seen in patients with resistance to thyroid hormone (RTH)?

Choose one answer.

A. Absence of a goiter

qsid=146336&aid=3839&qid=256&checksum=76031cdec7a555fc7c2c6dc750d6aaff&sesskey=9SD9XfnshK&new state=0

Page 49: Asap 2011

B. Low serum thyroid-stimulating hormone (TSH) levels

C. Elevated free thyroxine and free triiodothyronine levels

D. Elevated α subunit/TSH molar ratio

Educational Objective: To review common clinical findings in patients with RTH

Rationale: Patients with RTH are generally euthyroid, but serum (TSH) levels are often mildly elevated to compensate for the peripheral RTH. A goiter is a common finding seen in up to 95% of patients with RTH. Alpha subunit and its molar ratio to TSH are normal in RTH, but

elevated in patients with TSH-producing adenoma.

References

Refetoff S, Weiss RE, Usala SJ. The syndromes of resistance to thyroid hormone. Endocr Rev. 1993; 14:348-399.

Weiss RE, Refetoff S. Resistance to thyroid hormone. Rev Endocr Metab Disord. 2000; 1:97-108.

Incorrect

Marks for this submission: 0.00/1.00.

Question 23

Marks: 1.00

A 68-year-old woman undergoes a screening carotid ultrasound scan. Incidentally noted is a 2.5-cm hypoechoic thyroid nodule. She is asymptomatic of local compressive symptoms, and

has no symptoms to suggest either hypo- or hyperthyroidism. There is no known family history of nodular thyroid disease and no personal history of radiation exposure.

The first step in the investigation of this nodule should be which of the following?

Choose one answer.

A. Isotope scanning with technetium or I-123 to determine if the nodule is "cold"

qsid=146337&aid=3839&qid=257&checksum=c0423ae08835fb64d8fba3dce625dd9b&sesskey=9SD9XfnshK&new state=0

Page 50: Asap 2011

B. Ultrasound-guided fine needle aspiration (FNA) biopsy

C. Diagnostic ultrasound of the thyroid gland

D. Measurement of serum thyrotropin

E. Positron emission tomography (PET) scan to assess whether the nodule is metabolically active

Educational Objective: To understand the value of thyroid-stimulating hormone (TSH) measurement in the evaluation of a patient presenting with a thyroid nodule

Rationale: Measurement of serum TSH should be the first step in the evaluation of a patient with a thyroid nodule. If the patient has hyperthyroidism (suppressed thyrotropin), the

appropriate next step would be isotope scanning to determine if the nodule is "hot" with suppression of the surrounding gland. Hot nodules should not be biopsied, both because of

the very low probability of malignancy and because of the high "false-positive" rate of a biopsy in this setting, with high rates of "suspicious for follicular neoplasm" caused by the

high cellularity and scanty colloid typical in a hyperactive nodule. Cold nodules, or nodules of this size in a euthyroid patient, should be considered for ultrasound-guided FNA biopsy.

Diagnostic ultrasound of the thyroid should be considered in conjunction with ultrasound-guided FNA biopsy if the nodule is determined to be cold on the basis of isotope scanning of

the thyroid. A PET scan is not necessary, as current isotope scanning is easier and more accessible for assessment of whether the nodule is "hot" (hyperfunctioning) or "cold"

(nonfunctioning).

References

Cooper DS, Doherty GM, Haugen BR, et al. Revised American Thyroid Association management guidelines for patients with thyroid nodules and differentiated thyroid cancer.

Thyroid. 2009; 19:1167-214.

Gharib H, Papini E, Valcavi R, et al.; AACE/AME Task Force on Thyroid Nodules. American Association of Clinical Endocrinologists and Associazione Medici Endocrinologi medical

guidelines for clinical practice for the diagnosis and management of thyroid nodules. Endocr Pract. 2006; 12:63-102.

Incorrect

Marks for this submission: 0.00/1.00.

Question 24

Marks: 1.00

Page 51: Asap 2011

A 32-year-old woman finds a painless 1-cm nodule on the left side of her neck laterally, biopsy of which proves positive for papillary thyroid cancer. She undergoes thyroid-ectomy

and modified left lateral neck dissection, which reveals a 5-mm, left-sided papillary carcinoma within the thyroid gland and 3 positive lymph nodes in the low left jugular chain.

Postoperatively, she receives 100 mCi radioactive iodine "remnant ablation," and a post-therapy scan is negative. Six months later, her thyroglobulin on suppressive therapy is 2.4

ng/dL (normal, ≤33), which stimulates to 14 ng/dL following 2 injections of recombinant human thyroid-stimulating hormone (TSH) (Thyrogen™).

The next appropriate evaluation in this patient would be which of the following?

Choose one answer.

A. A computed tomography (CT) scan, without contrast, of neck and chest to identify the location of the residual thyroid cancer

B. Withdrawal from thyroid hormone, then treatment with 150 to 200 mCi of I-131, followed by a post-therapy scan to identify the location of the treated disease

C. Ultrasound scan of the neck to localize the likely source of the thyroglobulin, within central compartment lymph nodes

D. Positron emission tomography (PET) scan, to screen for metastatic thyroid cancer

E. Thyrogen-stimulated whole-body iodine scan to localize regions of uptake

Educational Objective: To understand the role of ultrasonography in the evaluation of patients with thyroid cancer

Rationale: In this node-positive but otherwise low-risk patient, the likelihood of metastatic spread is very low. Her prior negative whole-body scan makes it unlikely that the disease will

be localized by iodine scanning 6 months later. PET scans are rarely helpful in this setting, being insufficiently sensitive for small deposits of disease, and often negative in well-

differentiated thyroid cancer. The most likely location (>90%) of residual disease is one or more lymph nodes in the central neck, along the tracheo-esophageal groove, and ultrasound

qsid=146338&aid=3839&qid=260&checksum=eab5dc28d24d8224aa6eacc680514f0a&sesskey=9SD9XfnshK&new state=0

Page 52: Asap 2011

is the most sensitive tool to identify this disease. There is no indication for repeat radioiodine therapy unless evidence of recurrent cancer or metastases is found.

Reference

Stulak JM, Grant CS, Farley DR, et al. Value of preoperative ultrasonography in the surgical management of initial and reoperative papillary thyroid cancer. Arch Surg. 2006; 141:489-94;

discussion 494-496.

Incorrect

Marks for this submission: 0.00/1.00.

Question 25

Marks: 1.00

A 56-year-old man presents with a large compressive nodule on the left side of his thyroid gland, causing mild dysphagia but no hoarseness. Ultrasound confirms a 4.5-cm left-sided

thyroid nodule. The remainder of the gland appears normal and no abnormal lymph nodes are identified. His thyroid-stimulating hormone (TSH) level is normal and vocal cord check by

direct laryngoscopy is normal. He has no recognized risk factors for thyroid cancer. Fine needle aspiration (FNA) biopsy is reported to show features suggesting a "follicular lesion"/

Which of the following is the next appropriate step in the evaluation of this man?

Choose one answer.

A. Proceed to thyroidectomy, removing both side of the thyroid gland (total thyroidectomy), because the risk for malignancy is high in such large nodules .

B. Observe and follow by ultrasound, proceeding with thyroid surgery only if the nodule progresses, because the risk for malignancy in these "follicular neoplasms"

is low .

C. Initiate thyroid hormone treatment to suppress the TSH and try to reduce the size of the nodule .

D. Repeat the FNA biopsy after 3 to 6 months to try to clarify the nature of the nodule and consider the use of molecular markers (for example, BRAF mutation

qsid=146339&aid=3839&qid=261&checksum=9497421016c5b8111f05b87cb3b4fa97&sesskey=9SD9XfnshK&new state=0

Page 53: Asap 2011

analysis) if the diagnosis remains uncertain .

E. Perform thyroid lobectomy, moving to total thyroidectomy if either frozen section or final pathology demonstrates a malignancy .

Educational Objective: To understand the appropriate management of patients with thyroid nodules reported as "follicular lesions" (or "suspicious for follicular neoplasms")

Rationale: Even in a solitary, large thyroid nodule in a middle-aged man, the rate of malignancy is less than 20%. Lobectomy is a lower-risk surgery, avoiding any surgical threat

to the contralateral recurrent laryngeal nerve and parathyroid glands. Consequently, lobectomy is the recommended procedure in this setting, which also enables most such

patients to avoid the need for thyroxine replacement therapy. The rate of malignancy in a thyroid nodule reported as a "follicular lesion" ranges from 5% (in "follicular lesion of

undetermined significance") to almost 20% (in "suspicious for follicular neoplasm"), so observation with or without thyroxine therapy is not usually recommended. Repeat biopsy is

unlikely to clarify the diagnosis; molecular markers are in experimental use, but are not yet widely accepted as determinants of the need for surgery.

References

Cooper DS, Doherty GM, Haugen BR, et al. Revised American Thyroid Association management guidelines for patients with thyroid nodules and differentiated thyroid cancer.

Thyroid. 2009; 19:1167-1214.

Gharib H, Papini, E, Paschke R, et al.; AACE/AME/ETA Task Force on Thyroid Nodules. American Association of Clinical Endocrinologists, Associazione Medici Endocrinologi, and European Thyroid Association medical guidelines for clinical practice for the diagnosis and

management of thyroid nodules. Endocr Pathol. 2010; 16:468-475.

Incorrect

Marks for this submission: 0.00/1.00.

Question 26

Marks: 1.00

A 34-year-old woman finds an asymptomatic nodule in her thyroid gland. She has no recognized risk factors for thyroid cancer and is biochemically euthyroid. Ultrasound-guided

fine needle aspiration (FNA) biopsy proves positive for papillary thyroid cancer and the

qsid=146340&aid=3839&qid=263&checksum=839bd82f55dd3c76dc28bcdfb7e1057d&sesskey=9SD9XfnshK&new state=0

Page 54: Asap 2011

ultrasound reveals a 2-cm nodule. Surgery (total thyroidectomy and central compartment node dissection) is uncomplicated and confirms a 1.8-cm intrathyroidal papillary carcinoma

(PTC). A second focus of PTC is found in the same thyroid lobe, measuring 1 mm, and one central compartment lymph node is positive for metastatic disease.

Which of the following statements is correct regarding this woman?

Choose one answer.

A. Despite the lymph node involvement and multifocality, this patient has "low-risk" thyroid cancer and can be reassured that her life is not at significant risk. She

can be followed and treated with thyroid hormone alone .

B. The multifocal, node-positive nature of her disease places her in a "high-risk" group and mandates more aggressive therapy and follow-up than a solitary, node-

negative cancer of the same size .

C. The guidelines from the American Thyroid Association suggest that this patient should receive treatment with I-131 because the disease is multifocal and node-

positive, and I-131 is beneficial in reducing the rate of both recurrence and death in this setting .

D. There is convincing evidence that this patient should receive suppressive thyroxine therapy, targeting her thyroid-stimulating hormone (TSH) to below 0.1

mU/L to reduce the risk of recurrent disease .

E. Because of the lymph node involvement, she is at higher risk for distant metastatic spread. In addition to stimulated thyroglobulin level and whole-body

iodine scanning, a positron emission tomography (PET) scan should be considered as part of her metastatic survey .

Educational Objective: To discuss the postoperative management of patients with "low-risk" papillary thyroid cancer

Rationale: This patient has "low-risk" disease by all accepted criteria, as far as life expectancy is concerned. The multifocality and nodal involvement may somewhat increase her risk of

recurrent disease, arising within cervical lymph nodes, mandating careful follow-up. However, the American Thyroid Association (ATA) guidelines suggest that I-131 be used

"selectively" in this setting, with insufficient evidence to recommend in favor of treatment. No compelling evidence exists that either radioactive iodine or aggressive thyroxine therapy

improves the outcome for these patients. Intensive surveillance for metastatic disease is usually reserved for patients with high or rising thyroglobulin level, or those with other

evidence of high-risk disease.

Page 55: Asap 2011

References

Cooper DS, Doherty GM, Haugen BR, et al. Revised American Thyroid Association management guidelines for patients with thyroid nodules and differentiated thyroid cancer.

Thyroid. 2009; 19:1167-214.

Hay ID, Thompson GB, Grant CS, et al. Papillary thyroid carcinoma managed at the Mayo Clinic during six decades (1940-1999): temporal trends in initial therapy and long-term

outcome in 2444 consecutively treated patients. World J Surg. 2002; 26:879-885.

Shaha AR, Shah JP, Loree TR. Low-risk differentiated thyroid cancer: the need for selective treatment. Ann Surg Oncol. 1997; 4:328-333.

Incorrect

Marks for this submission: 0.00/1.00.

Question 27

Marks: 1.00

A 78-year-old man presents with a rapidly enlarging goiter associated with compressive symptoms of dysphagia, dyspnea, hoarseness, and stridor. He also complains of pain,

radiating to the right ear. On examination, a large hard mass is felt in the right anterior low neck, which appears fixed to underlying tissues. Ultrasound confirms a large, complex

thyroid nodule with cystic components. Fine needle aspiration (FNA) biopsy reveals only necrotic debris and is nondiagnostic.

Which of the following would be the most appropriate approach to this man's presenting medical problems?

Choose one answer.

A. Proceed with palliative care without further evaluation or intervention

B. Magnetic resonance imaging or computed tomography scan of the neck to assist the surgeon in determining operability

C. Thyroid scintography

qsid=146341&aid=3839&qid=265&checksum=ce6dd585aa2b057c3459df40019cb9a5&sesskey=9SD9XfnshK&new state=0

Page 56: Asap 2011

D. Proceed to surgery for decompression, open biopsy and resection or debulking of the tumor

Educational Objective: To discuss appropriate evaluation and management in patients with suspected anaplastic thyroid cancer

Rationale: The differential diagnosis includes anaplastic thyroid cancer and lymphoma, though other possibilities exist, including acute (bacterial) thyroiditis or hemorrhage within a

preexisting thyroid nodule, causing acute enlargement of the gland. A tissue diagnosis is highly desirable, because lymphoma is amenable to chemotherapy and radiation therapy,

avoiding the need for surgery in most cases. The high level of clinical suspicion mandates a careful metastatic survey, since widely metastatic anaplastic thyroid cancer carries a

prognosis so dismal that palliative care alone may sometimes be appropriate. However, localized disease may sometimes respond to a combination of surgery, radiation therapy and

chemotherapy, so a detailed evaluation is appropriate, despite the poor prognosis of most patients with anaplastic thyroid cancer. Thyroid scintography has no role in the assessment

of this patient's neck mass.

References

McIver B, Hay ID, Giuffrida DF, et al. Anaplastic thyroid carcinoma: a 50-year experience at a single institution. Surgery. 2001; 130:1028-1034.

Cooper DS, Doherty GM, Haugen BR, et al. Revised American Thyroid Association management guidelines for patients with thyroid nodules and differentiated thyroid cancer.

Thyroid. 2009; 19:1167-1214.

Incorrect

Marks for this submission: 0.00/1.00.

Question 28

Marks: 1.00

Maternal hypothyroidism is associated with which of the following complications?

Choose one answer.

A. Large-for-gestational-age infant

qsid=146342&aid=3839&qid=266&checksum=bb6ac8a1756e4587cebe58c5441294ff&sesskey=9SD9XfnshK&new state=0

Page 57: Asap 2011

B. Hypokalemia

C. Choanal atresia

D. Postmaturity infant

E. Pregnancy-induced hypertension

Educational Objective: To review potential risks and adverse effects of maternal hypothyroidism for mother and baby.

Rationale: Maternal hypothyroidism has been associated with increased risk of problems, including pregnancy-induced hypertension, placenta abruption, premature delivery, low

birth weight, and impaired cognitive function of the child. Choanal atresia is a rare problem that has been associated with methimazole use. Hypokalemia is not a complication of

maternal hypothyroidism.

References

Smallridge RC. Hypothyroidism and pregnancy. Endocrinologist. 2002; 12:454-464.

LaFranchi SH, Haddow JE, Hollowell JG. Is thyroid inadequacy during gestation a risk factor for adverse pregnancy and developmental outcomes? Thyroid. 2005; 15:60-71.

Haddow JE, Palomaki GE, Allan WC, et al. Maternal thyroid deficiency during pregnancy and subsequent neuropsychological development of the child. N Engl J Med. 1999; 341:549-555.

Incorrect

Marks for this submission: 0.00/1.00.

Question 29

Marks: 1.00

Which of the following statements is most appropriate regarding thyroid disease in pregnancy?

Choose one answer.

qsid=146343&aid=3839&qid=268&checksum=8f23dc72be6ffd3b0c63e574f27baee8&sesskey=9SD9XfnshK&new state=0

Page 58: Asap 2011

A. The mother is the source of thyroid hormone for the fetus during the first trimester .

B. Human chorionic gonadotropin (hCG) is capable of stimulating thyroid-stimulating hormone (TSH) secretion .

C. Thyroid hormone production is unchanged during pregnancy .

D. TSH is most likely to be low during the last trimester .

Educational Objective: To understand normal variations in thyroid function tests and the effects of hCG on thyroid function during pregnancy

Rationale: During the first trimester of pregnancy, the mother is virtually the sole source of thyroid hormone for the fetus. hCG can cross-react with the TSH receptor and produce some

thyroid stimulating activity, which can lead to a decrease in TSH. The peak level of hCG typically occurs at 10 to 12 weeks' gestation, so TSH values are most likely to be affected in

the first and second trimesters.

References

Hershman JM. Human chorionic gonadotropin and the thyroid: hyperemesis gravidarum and trophoblastic tumors. Thyroid. 1999; 9:653-657.

Hershman JM. Physiological and pathological aspects of the effect of human chorionic gonadotropin on the thyroid. Best Pract Res Clin Endocrinol Metab. 2004; 18:249-265.

Smallridge RC, Ladenson PW. Hypothyroidism in pregnancy: consequences to neonatal health. J Clin Endocrinol Metab. 2001; 86:2349-2353.

Incorrect

Marks for this submission: 0.00/1.00.

Question 30

Marks: 1.00

Which of the following statements regarding iodine requirements for a pregnant woman is most appropriate?

qsid=146344&aid=3839&qid=269&checksum=32e3a4279947e1d98de94f0813a6db89&sesskey=9SD9XfnshK&new state=0

Page 59: Asap 2011

Choose one answer.

A. Iodine requirements are met with a daily prenatal vitamin .

B. Iodine requirements do not differ between pregnant and nonpregnant women.

C. Iodine requirements are 150 µg daily .

D. Iodine requirements are higher in pregnant women than in nonpregnant women .

E. Iodine requirements are 50 µg daily .

Educational Objective: To understand what constitutes adequate iodine intake during pregnancy

Rationale: Iodine requirements increase during pregnancy. The recommended dietary allowance is 220 µg daily. Prenatal vitamins often do not contain iodine.

References

Smallridge RC, Glinoer D, Hollowell JG, et al. Thyroid function inside and outside of pregnancy: what we know and what don't we know? Thyroid. 2005; 15:54-59.

Smallridge RC, Ladenson PW. Hypothyroidism in pregnancy: consequences to neonatal health. J Clin Endocrinol Metab. 2001; 86:2349-2353.

Food and Nutrition Board, Institute of Medicine. Dietary Reference Intakes. Washington, DC: National Academy Press; 2001.

Incorrect

Marks for this submission: 0.00/1.00.

Question 31

Marks: 1.00

Which of the following is true regarding treatment of Graves' disease during pregnancy?

qsid=146345&aid=3839&qid=270&checksum=2b0f77436c56602869baa8a509f00b38&sesskey=9SD9XfnshK&new state=0

Page 60: Asap 2011

Choose one answer.

A. Methimazole is the preferred agent in the first trimester .

B. Propylthiouracil is associated with aplasia cutis .

C. Treatment goals are to maintain a thyroxine level in the upper normal to slightly elevated range .

D. Surgery can be offered as treatment in the first trimester .

Educational Objective: To review appropriate goals and management of hyperthyroidism during pregnancy

Rationale: Propylthiouracil is the preferred agent in the first trimester, during organogenesis, as methimazole has been associated with rare congenital abnormalities such as aplasia cutis.

Current guidelines recommend preferential use of methimazole for Graves' disease in pregnancy after organogenesis has been completed (second and third trimesters). Surgery,

when necessary, should be done in the second trimester. Thyroxine levels should be maintained in the upper normal to slightly elevated range.

References

Smallridge RC. Hypothyroidism and pregnancy. Endocrinologist. 2002; 12:454-464.

Abalovich M, Amino N, Barbour LA, et al. Management of thyroid dysfunction during pregnancy and postpartum: an Endocrine Society Clinical Practice Guideline. J Clin

Endocrinol Metab. 2007; 92(8 suppl):s1-s47.

Incorrect

Marks for this submission: 0.00/1.00.

Question 32

Marks: 1.00

Which of the following recommendations is appropriate if any change in the dose of levothyroxine would be needed when a woman taking levothyroxine becomes pregnant?

Choose one answer.

qsid=146346&aid=3839&qid=271&checksum=2806eaf370b47203a5a196708fdc6bdc&sesskey=9SD9XfnshK&new state=0

Page 61: Asap 2011

A. No change in dosage will be necessary .

B. The dose should be lower by the end of the pregnancy .

C. The dose should increase during the last trimester .

D. The dose should increase during the first 2 trimesters .

E. The dose should be reduced in the first and second trimester, then increased in the last trimester .

Educational Objective: To review appropriate changes in thyroid hormone replacement during pregnancy

Rationale: In most women, the dose of levothyroxine needs to be increased during pregnancy. The increased requirement typically occurs during the first 16 to 20 weeks.

Reference

Alexander EK, Marqusee E, Lawrence J, et al. Timing and magnitude of increases in levothyroxine requirements during pregnancy in women with hypothyroidism. N Engl J Med.

2004; 351:241-249.

Incorrect

Marks for this submission: 0.00/1.00.

Question 33

Marks: 1.00

A 67-year-old man with chronic atrial fibrillation presents with a 1-year history of a 10-lb weight loss and general malaise. His medications include a β-adrenergic blocking agent and

warfarin. On examination, he has a palpable nodule in the left lobe of the thyroid which is mobile and nontender. Laboratory testing shows a free thyroxine (FT4) level of 1.6 ng/dL

(normal, 0.8-1.9 ng/dL) and thyroid-stimulating hormone (TSH) level of 0.15 µU/mL (normal, 0.4-5.5µU/mL).

Which of the following is the best next test in the evaluation of his thyroid nodule?

qsid=146347&aid=3839&qid=273&checksum=bf08d25d1791e5ca07713b29816c2fbb&sesskey=9SD9XfnshK&new state=0

Page 62: Asap 2011

Choose one answer.

A. Measure serum thyroglobulin level

B. Obtain serum thyroperoxidase antibody titer

C. Measure serum calcitonin level

D. Thyroid ultrasound

E. Nuclear thyroid scan

Educational Objective: To understand the evaluation of thyroid nodules in patients presenting with suppressed serum TSH levels

Rationale: Hyperthyroidism in older patients is often caused by non-autoimmune nodular goiter. A nuclear thyroid scan is recommended when the TSH level is suppressed below the

reference range to determine if the nodule is hyperfunctioning, as the so-called "hot" or hyperfunctioning nodules are seldom malignant, and such a finding may obviate the need for fine-needle aspiration (FNA). An ultrasound is recommended after the nuclear thyroid

examination to examine the areas that do not take up the isotope (cold or cool areas in the thyroid) for location (anterior, posterior, substernal) and to determine which nodules

require biopsy based on suspicious ultrasound characteristics. Basal calcitonin levels lack the sensitivity and specificity to recommend routinely in all patients with a thyroid nodule.

Thyroperoxidase antibody testing is not indicated in the setting of a toxic nodular goiter.

References

Gharib H, Papini E, Paschke R, et al; AACE/AME/ETA Task Force on Thyroid Nodules. American Association of Clinical Endocrinologists, Associazione Medici Endocrinologi, and

European Thyroid Association Medical Guidelines for Clinical Practice for the Diagnosis and Management of Thyroid Nodules. Endocr Pathol .

2010 ;16)suppl 1:(1-43.

Cooper DS, Doherty GM, Haugen BR, et al. Revised American Thyroid Association management guidelines for patients with thyroid nodules and differentiated thyroid cancer.

Thyroid. 2009; 19:1167-1214.

Marqusee E, Benson CB, Frates MC, et al. Usefulness of ultrasonography in the management of nodular thyroid disease. Ann Intern Med. 2000; 133:696-700.

Incorrect

Page 63: Asap 2011

Marks for this submission: 0.00/1.00.

Question 34

Marks: 1.00

A 23-year-old morbidly obese woman complains of anterior neck pressure for the last 6 months that is worsening in severity. She has no family history of thyroid cancer, nor has she

had exposure to head and neck radiation.

On examination, a 3-cm nodule is found in her left thyroid lobe. The nodule is nontender and mobile. A thyroid-stimulating hormone (TSH) performed by her primary care doctor was

normal at 1.4 µU/mL in the last 12 months. An ultrasound of the thyroid shows a 3-cm, heterogeneous, hypoechoic spongiform nodule located posteriorly in the left thyroid lobe

that is wider than tall, with distinct margins and no microcalcifications. The anterior surface of the thyroid is 2.5 cm below the skin.

Which of the following should be done next?

Choose one answer.

A. Fine-needle aspiration (FNA) biopsy by palpation

B. Order a nuclear thyroid scan (I-123 or 99m-technetium)

C. Order a calcitonin

D. Assess vascular flow with Doppler analysis

E. Perform a neck survey for abnormal lymph nodes

Educational Objective: To review the American Thyroid Association and the American Association of Clinical Endocrinologists/Associazione Medici Endocrinologi/European Thyroid

Association guidelines for thyroid nodule evaluation

Rationale: There is good evidence supporting the use of FNA biopsy in the diagnosis of palpable and nonpalpable thyroid nodules. Guidelines from all of these organizations agree

in the evaluation of a thyroid nodule to measure TSH level and, if normal or high, to perform

qsid=146348&aid=3839&qid=274&checksum=8f313f52b6c18484b6abdd152798ac3c&sesskey=9SD9XfnshK&new state=0

Page 64: Asap 2011

an ultrasound evaluation, which should include an evaluation of amount and pattern of vascular flow by Doppler analysis, and FNA biopsy. Peripheral blood flow around the edge of

the nodule is not associated with malignancy whereas chaotic intranodular blood flow is associated with malignancy. A nuclear thyroid scan is not recommended unless the TSH is

suppressed. The value of a nuclear thyroid scan in a euthyroid individual is limited, as more than 80% to 85% of nodules are cold or hypofunctioning with reduced isotope accumulation

compared to the surrounding normal thyroid tissue.

According to the 2009 American Thyroid Association Revised Guidelines, spongiform nodules are rarely malignant and do not need to be biopsied unless >2 cm, but alternatively, may be

monitored without biopsy with serial ultrasound evaluation. A neck survey for enlarged lymph nodes is not required unless clinical suspicion for cancer is very high or until after

cytology is confirmed as malignant.

FNA should be performed with ultrasound guidance. There are good data showing that ultrasound improves diagnostic accuracy and decreases insufficient rates. An ultrasound

should be done to assess whether the nodule is partially cystic requiring ultrasound guidance to sample the solid portion of the nodule, to determine whether the nodule is anterior in the

gland and to determine the depth of the overlying muscle and adipose tissue. Ultrasound guidance for this patient would be required because of her morbid obesity and the posterior

location of the nodule. Palpation FNA may be considered in an easily palpable nodule that is located anteriorly in the thyroid gland.

References

Gharib H, Papini E, Paschke R, et al; AACE/AME/ETA Task Force on Thyroid Nodules. American Association of Clinical Endocrinologists, Associazione Medici Endocrinologi, and

European Thyroid Association Medical Guidelines for Clinical Practice for the Diagnosis and Management of Thyroid Nodules. Endocr Pathol .

2010 ;16)suppl 1:(1-43.

Cooper DS, Doherty GM, Haugen BR, et al. Revised American Thyroid Association management guidelines for patients with thyroid nodules and differentiated thyroid cancer.

Thyroid. 2009; 19:1167-1214.

Incorrect

Marks for this submission: 0.00/1.00.

Question 35

Marks: 1.00

qsid=146349&aid=3839&qid=276&checksum=7e14c683875b1deea8bd218b93b39c0d&sesskey=9SD9XfnshK&new state=0

Page 65: Asap 2011

A 75-year-old woman is referred to your clinic with an enlarged thyroid found during a magnetic resonance imaging scan of the neck obtained for evaluation of posterior neck pain.

She denies esophageal or tracheal compressive symptoms. She has no history of head and neck radiation. Her older sister had a thyroidectomy at age 70 years for a "huge goiter".

The patient's primary care doctor obtained a thyroid-stimulating hormone (TSH) level of 1.5 µU/mL prior to her appointment with you. Ultrasound of the thyroid revealed an enlarged

thyroid with each lobe measuring >7 cm in the sagittal dimension. The right lobe contains 3 to 4 confluent spongiform nodules ~2 to 2.5 cm in diameter. The left lobe contains similar

spongiform nodules with the largest nodule measuring 3 cm and a 1.5-cm hypoechoic solid vascular nodule.

Which of the following should be the next best step in evaluating this goiter?

Choose one answer.

A. Order a nuclear thyroid scan

B. Biopsy the largest 3-cm nodule in the left thyroid lobe

C. Biopsy the 1.5-cm hypoechoic nodule

D. Order thyroperoxidase antibody titer

E. No biopsy and continued observation with serial ultrasound examinations

Educational Objective: To review the American Thyroid Association (ATA) and the American Association of Clinical Endocrinologists (AACE)/Associazione Medici Endocrinologi

(AME)/European Thyroid Association (ETA) guidelines for thyroid nodule(s) evaluation in a multinodular goiter

Rationale: With a strong family history of large benign multinodular goiter, it is likely that the patient has a genetic inheritance for goiter, but the presence of multiple nodes does not

reduce her risk of a thyroid malignancy. A thyroid ultrasound should be performed, but if only the "dominant" or largest nodule is aspirated, the thyroid cancer may be missed.

The guidelines suggest that if there are 2 or more nodules >1 to 1.5 cm, then the nodules with suspicious ultrasound characteristics should be biopsied. If the lobe is composed of

multiple confluent spongiform nodules without suspicious ultrasound characteristics, then the largest nodule may be biopsied, or ultrasound monitoring without biopsy is a reasonable

alternative. The AACE/AME/ETA does not make fine-needle aspiration biopsy (FNAB) recommendations for confluent spongiform nodules in a multinodular goiter. But in this

case, in addition to the confluent spongiform nodules, there is a solid nodule with suspicious

Page 66: Asap 2011

ultrasound characteristics. FNAB should be performed on the suspicious nodule and not the largest spongiform nodule. Thyroperoxidase antibody study is not necessary as it will not

add to the diagnosis or management of this patient. A nuclear thyroid scan is not necessary because TSH is normal. Observation is not appropriate for a >1-cm nodule with suspicious

ultrasound characteristics.

References

Gharib H, Papini E, Paschke R, et al; AACE/AME/ETA Task Force on Thyroid Nodules. American Association of Clinical Endocrinologists, Associazione Medici Endocrinologi, and

European Thyroid Association Medical Guidelines for Clinical Practice for the Diagnosis and Management of Thyroid Nodules. Endocr Pathol. 2010; 16(suppl 1):1-43.

Cooper DS, Doherty GM, Haugen BR, et al. Revised American Thyroid Association management guidelines for patients with thyroid nodules and differentiated thyroid cancer.

Thyroid. 2009; 19:1167-1214.

Frates MC, Benson C, Doubilet PM, et al. Prevalence and distribution of carcinoma in patients with solitary and multiple thyroid nodules on sonography. J Clin Endocrinol Metab.

2006; 91:3411-3417.

Fish SA, Langer J, Mandel, SJ. Sonographic imaging of thyroid nodules and cervical lymph nodes. Endocrinol Metab Clin North Am. 2008; 37:401-417; ix.

Incorrect

Marks for this submission: 0.00/1.00.

Question 36

Marks: 1.00

A 56-year-old man is referred to you for an enlarged thyroid, found by an oto-laryngologist during an examination for a hoarse voice. Laryngoscopy showed a paralyzed right vocal cord.

The otolaryngologist ordered a computed tomography (CT) scan of the neck that demonstrated a 4-cm right thyroid nodule and enlarged lymph nodes in the right jugular

nodes (levels III and IV). There was tracheal deviation without narrowing. There was no evidence of gross extrathyroidal extension.

The patient is referred to your clinic for biopsy. He denies obstructive symptoms but notes that he had to buy new shirts in the last few months because of an increase in his neck size

without an overall weight gain. On examination, he has an enlarged right thyroid lobe. Ultrasound demonstrated a 4.3-cm isoechoic nodule in the right lobe of an otherwise normal

qsid=146350&aid=3839&qid=278&checksum=089f00e9ac9b9280a7df917a93a32ce5&sesskey=9SD9XfnshK&new state=0

Page 67: Asap 2011

thyroid. The nodule has indistinct borders, is taller than wide, with increased and disorganized intranodular blood flow. Enlarged hypoechoic lymph nodes measuring 1 to 2

cm are identified by ultrasound in the right lateral neck, levels III and IV. Thyroid-stimulating hormone (TSH) level is 4.1 µU/mL (normal, 3.5-4.4).

Which of the following should be done next?

Choose one answer.

A. Observation only because there are no obstructive symptoms or tracheal narrowing and the nodule is not hypoechoic

B. Nuclear thyroid scan to determine if the nodule is hypofunctioning

C. Refer for thyroidectomy without biopsy because the goiter is getting larger

D. Fine-needle aspiration (FNA) biopsy of the right thyroid nodule and one of the enlarged right level III lymph nodes for cytology with thyroglobulin measurement

in the needle washout

E. Obtain thyroperoxidase antibody levels

Educational Objective: To review the American Thyroid Association and the American Association of Clinical Endocrinologists/Associazione Medici Endocrinologi/European Thyroid

Association guidelines for thyroid nodule evaluation in a high-risk clinical scenario

Rationale: This is a highly suspicious clinical situation even though the nodule is isoechoic and not hypoechoic. All thyroid nodules should be evaluated by ultrasound. Although

suspicious ultrasound characteristics either singly or in combination (hypoechoic, intranodular hypervascularity, indistinct margins, microcalcifications) are very specific for

thyroid cancer, the sensitivity is <60%, which means that many thyroid cancers do not have these ultrasound features. Follicular thyroid carcinoma, in particular, may not be hypoechoic

on ultrasound. If a highly suspicious node and thyroid nodule is seen by ultrasound examination, then biopsy of both is required. In addition, a thyroglobulin level should be

measured in the washout of one of the node biopsy needles. The rapid growth of the solid nodule, the presence of extensive lymphadenopathy and evidence of injury to the right

recurrent laryngeal nerve from either invasion or pressure suggests a poorly differentiated or anaplastic thyroid carcinoma or lymphoma. New observational studies have shown that

the risk of malignancy increases with higher TSH levels, even within the reference range, but the clinical significance of this finding is not clear. Observation is not appropriate, and

nuclear thyroid scan will not be helpful to determine if this is a malignancy. If thyroglobulin is high in the node, lymphoma would be excluded. Anaplastic thyroid cancer and lymphomas

can be difficult to differentiate in a primary thyroid nodule. Biopsy is necessary, as total

Page 68: Asap 2011

thyroidectomy with compartmental nodal dissection surgery is not a good option if the tumor is an anaplastic carcinoma or lymphoma. Measurement of thyroperoxidase antibody

levels is not helpful in this euthyroid patient.

References

Gharib H, Papini E, Paschke R, et al; AACE/AME/ETA Task Force on Thyroid Nodules. American Association of Clinical Endocrinologists, Associazione Medici Endocrinologi, and

European Thyroid Association Medical Guidelines for Clinical Practice for the Diagnosis and Management of Thyroid Nodules. Endocr Pathol .

2010 ;16)suppl 1:(1-43.

Cooper DS, Doherty GM, Haugen BR, et al. Revised American Thyroid Association management guidelines for patients with thyroid nodules and differentiated thyroid cancer.

Thyroid. 2009; 19:1167-1214.

Fish SA, Langer J, Mandel, SJ. Sonographic imaging of thyroid nodules and cervical lymph nodes. Endocrinol Metab Clin North Am. 2008; 37:401-417; ix.

Incorrect

Marks for this submission: 0.00/1.00.

Question 37

Marks: 1.00

A 37-year-old woman is referred to you for the evaluation of a thyroid nodule. Her history is significant for Hodgkin's lymphoma at the age of 16 that was treated with mantle radiation.

The patient noted a slowly growing neck mass over the last year and saw her oncologist. The oncologist confirmed a neck mass and obtained 18F-fluoro-2-deoxy-d-glucose (FDG)—

positron emission tomography (PET) scan. A computed tomography (CT) scan revealed that the palpable mass was an enlarged nodular thyroid gland. 18F-FDG uptake was focally high

in one of the thyroid nodules. No other suspicious masses were seen on the PET/CT scan. Ultrasound in your office confirms a nodular thyroid, and the mass that corresponds to the

focal hypermetabolic focus is a 0.7-cm hypoechoic nodule without microcalcifications or increased vascular flow .

No evidence of cervical adenopathy is seen.

Which of the following statements is correct regarding the management of this woman's thyroid nodule?

Choose one answer.

qsid=146351&aid=3839&qid=280&checksum=307f31844d73cf51bed2cb5add75978a&sesskey=9SD9XfnshK&new state=0

Page 69: Asap 2011

A. Biopsy is not necessary in a <1-cm thyroid nodule .

B. Nearly all hypermetabolic 18F-FDG—positive nodules are cancers .

C. This patient has a recurrence of her lymphoma and should be treated with radiation .

D. If fine-needle aspiration (FNA) biopsy is positive for thyroid cancer, a lobectomy should be done for this patient .

E. Radioactive iodine ablation after thyroidectomy is not necessary in a <1-cm thyroid cancer .

Educational Objective: To review the American Thyroid Association and the American Association of Clinical Endocrinologists/Associazione Medici Endocrinologi/European Thyroid

Association guidelines for thyroid nodule evaluation with a history of head and neck radiation and 18F-FDG-PET—positive uptake

Rationale: The risk of malignancy in a thyroid nodule is ~5%, but after significant head and neck radiation, the risk of papillary thyroid carcinoma increases. Thyroid nodules with

positive 18F-FDG uptake have a risk of malignancy of approximately 35% to 45% and such nodules should always be biopsied, especially if >5 mm. In addition, any size thyroid nodule

should be biopsied in a patient with a history of head and neck radiation. Because of such history, which increases the risk of malignancy in her entire multi-nodular thyroid gland, a near-total thyroidectomy should be performed rather than lobectomy, if the biopsy of the

nodule is positive for differentiated thyroid carcinoma, despite the small size of her primary tumor. There are insufficient data at this time to determine if a FDG-PET—positive thyroid

malignancy has a more aggressive clinical course.

References

Gharib H, Papini E, Paschke R, et al; AACE/AME/ETA Task Force on Thyroid Nodules. American Association of Clinical Endocrinologists, Associazione Medici Endocrinologi, and

European Thyroid Association Medical Guidelines for Clinical Practice for the Diagnosis and Management of Thyroid Nodules. Endocr Pathol .

2010 ;16)suppl 1:(1-43.

Cooper DS, Doherty GM, Haugen BR, et al. Revised American Thyroid Association management guidelines for patients with thyroid nodules and differentiated thyroid cancer.

Thyroid. 2009; 19:1167-1214.

Frates MC, Benson C, Doubilet PM, et al. Prevalence and distribution of carcinoma in patients with solitary and multiple thyroid nodules on sonography. J Clin Endocrinol Metab.

2006; 91:3411-3417.

Page 70: Asap 2011

Choi JY, Lee K, Kim HJ, et al. Focal thyroid lesions incidentally identified by integrated 18F-FDG PET/CT: clinical significance and improved characterization. J Nucl Med. 206; 47:609-

615.

Fish SA, Langer J, Mandel, SJ. Sonographic imaging of thyroid nodules and cervical lymph nodes. Endocrinol Metab Clin North Am. 2008; 37:401-417; ix.

Incorrect

Marks for this submission: 0.00/1.00.

Reminder:

You have three (3) attempts to successfully complete this assessment with a 75% score or higher.

You have 30 days to complete each attempt, once an attempt is started.

If you need to save your answers and come back to the assessment, you must click the 'Next' button at the bottom-left of the assessement prior to logging

out of ASAP.

Only click 'Submit all and finish' once you are satisfied with all your assessment answers.

Bottom of Form

Finish review

Skip Quiz navigation

Quiz navigation

i (Closed ) 1 (Incorrect ) 2 (Incorrect ) 3 (Incorrect ) 4 (Incorrect ) 5 (Incorrect ) 6 (Incorrect ) 7 (Incorrect ) 8 (Incorrect ) 9 (Incorrect ) 10 (Incorrect ) 11 (Incorrect ) 12 (Incorrect ) 13

(Incorrect ) 14 (Incorrect ) 15 (Incorrect ) 16 (Incorrect ) 17 (Incorrect ) 18 (Incorrect ) 19 (Incorrect ) 20 (Incorrect ) 21 (Incorrect ) 22 (Incorrect ) 23 (Incorrect ) 24 (Incorrect ) 25 (Incorrect ) 26 (Incorrect ) 27 (Incorrect ) 28 (Incorrect ) 29 (Incorrect ) 30 (Incorrect ) 31 (Incorrect ) 32 (Incorrect ) 33 (Incorrect ) 34 (Incorrect ) 35 (Incorrect ) 36 (Incorrect ) 37

(Incorrect ) i (Closed )

Finish review

You are logged in as Mohammed Aldawish (Logout)

THYROID

Copyright: 2011, American Association of Clinical Endocrinologists (AACE).All material published on the Web site is the property of AACE and may not be reproduced in

any form or by any electronic means

Page 71: Asap 2011

including information, storage and retrieval systems without the written consent of AACE.

This activity is sponsored by the American Association of Clinical Endocrinologists.

Skip to main content

Hypertension

You are logged in as Mohammed Aldawish (Logout)

Page path

Home

/ ► My courses

/ ► HYPER

/ ► Assessment

/ ► Hypertention Assessment

/ ► Review of attempt 1

Review of attempt 1

Started onTuesday, 19 November 2013, 10:20 PM

Completed onTuesday, 19 November 2013, 10:21 PM

Time taken32 secs

Grade0.00 out of a maximum of 22.00 (0%)

FeedbackFailed

Top of Form

9SD9XfnshK

Page 72: Asap 2011

Directions: In this section, each item includes a lead question and a list of options labeled with letters. Select the ONE lettered option that is BEST in each case by selecting the radio

button containing the correct answer .

Question 1

Marks: 1.00

Hypertension a significant cardiovascular risk factor. The Joint National Committee (JNC) has set guidelines for the Prevention, Detection, Evaluation, and Treatment of High Blood

Pressure.

Which ONE of the following statements is consistent with the JNC guidelines?

Choose one answer .

A. In persons older than age 50 years, systolic blood pressure (BP) greater than 140 mm Hg is a much more important cardiovascular disease (CVD) risk factor than

diastolic BP

B. The risk of CVD begins at BP of 130/80 mm Hg and doubles with each increment of 20/10 mm Hg

C. Individuals with a systolic BP of 130 to 139 mm Hg or a diastolic BP of 90 to 99 mm Hg should be considered as pre-hypertensive and require health-promoting

lifestyle modifications to prevent CVD

D. Beta-adrenergic blocking agents should be used in drug treatment for most patients with uncomplicated hypertension, either alone or combined with drugs from other classes. Most patients with hypertension will achieve goal BP with a

single anti-hypertensive agent

Educational Objective: To understand the key messages of the Seventh Report of the Joint National Committee on Prevention, Detection, Evaluation, and Treatment of High Blood

Pressure (JNC-7).

Rationale: The JNC-7 report provides new guidelines for the prevention and management of hypertension. The following are the key messages: In persons older than age 50 years,

systolic blood pressure (BP) of >140 mm Hg is a much more important cardiovascular disease (CVD) risk factor than diastolic BP. The risk of CVD, beginning at 115/75 mm Hg, doubles

qsid=148124&aid=3882&qid=43&checksum=8e6a00d763886deb853b6f57e072680d&sesskey=9SD9XfnshK&new state=0

Page 73: Asap 2011

with each increment of 20/10 mm Hg; individuals who are normotensive at age 55 have a 90% lifetime risk for developing hypertension. Individuals with a systolic BP of 120 to 139 mm Hg or a diastolic BP of 80 to 89 mm Hg should be considered as prehypertensive and require health-promoting lifestyle modifications to prevent CVD. Thiazide-type diuretics

should be used in drug treatment for most patients with uncomplicated hypertension, either alone or combined with drugs from other classes. Certain high-risk conditions are compelling

indications for the initial use of other antihypertensive drug classes (angiotensin-converting enzyme inhibitors, angiotensin-receptor blocking agents, β-adrenergic blocking agents,

calcium channel blocking agents). Most patients with hypertension will require 2 or more antihypertensive medications to achieve goal BP (20/10 mm Hg above goal BP, consideration

should be given to initiating therapy with 2 agents, one of which usually should be a thiazide-type diuretic. The most effective therapy prescribed by the most careful clinician

will control hypertension only if patients are motivated.

Reference

Chobanian AV, Bakris GL, Black HR, et al., for the National Heart, Lung, and Blood Institute Joint National Committee on Prevention, Detection, Evaluation, and Treatment of High Blood

Pressure; National High Blood Pressure Education Program Coordinating Committee. The Seventh Report of the Joint National Committee on Prevention, Detection, Evaluation, and

Treatment of High Blood Pressure: the JNC 7 report. JAMA. 2003;289:2560-2572.

Incorrect

Marks for this submission: 0.00/1.00.

Question 2

Marks: 1.00

In a 50 year old woman with type 2 diabetes mellitus and hypertension (blood pressure, 152/94), which ONE of the following statements best describes the appropriate

management and blood pressure (BP) goal for this patient?

Choose one answer .

A. Treat with diet and exercise with a BP goal of 140/90

B. Treat with hydrochlorthiazide 12.5 mg daily with a BP goal of 135/90

qsid=148125&aid=3882&qid=44&checksum=e89879bc00c3cbf9edd124b1947d3676&sesskey=9SD9XfnshK&new state=0

Page 74: Asap 2011

C. Treat with hydrochlorthiazide 12.5 mg and lisinopril 10 mg daily with a BP goal of 130/80

D. Treat with amlodipine 5mg daily with a BP goal of 140/90

E. Treat with clonidine 0.1 mg daily with a BP goal of 135/85

Educational Objective: Understand the appropriate management and goal blood pressure in patients with diabetes mellitus.

Rationale: Patients with diabetes mellitus and hypertension need to be aggressively treated for both their diabetes mellitus and hypertension. Diet and exercise will not suffice.

Pharmacological therapy is indicated. The patient could have been started initially on hydroclorthiazide with the addition of lisinopril later in 3 months if goal blood pressure of

130/80 or less was not achieved with hydrochlorthiazide alone. Hydrochlorthiazide remains the best initial therapy for hypertension even in patients with diabetes mellitus. Amlodipine and clonidine can be used as third or fourth tier anti-hypertensive agents to optimize blood

pressure control to the target of 130/80. Clonidine is rarely used in the treatment of hypertension in patients with diabetes mellitus.

Reference

Chobanian AV, Bakris GL, Black HR, et al., for the National Heart, Lung, and Blood Institute Joint National Committee on Prevention, Detection, Evaluation, and Treatment of High Blood

Pressure; National High Blood Pressure Education Program Coordinating Committee. The Seventh Report of the Joint National Committee on Prevention, Detection, Evaluation, and

Treatment of High Blood Pressure: the JNC 7 report. JAMA. 2003;289:2560-2572.

Incorrect

Marks for this submission: 0.00/1.00.

Question 3

Marks: 1.00

A 63 year old man is seen by his doctor for resistant hypertension despite being on a full dose of a thiazide diuretic, lisinopril and amlodipine.

Which of the following concurrent medications may be contributing to his "resistant" hypertension?

qsid=148126&aid=3882&qid=45&checksum=61927eb018606017e16ece9c49fc72bb&sesskey=9SD9XfnshK&new state=0

Page 75: Asap 2011

Choose one answer .

A. Atorvastatin

B. Ibuprofen

C. Glipizide

D. Metformin

E. Pioglitazone

Educational Objective: Understand the appropriate management of resistant hypertension.

Rationale: Resistant hypertension is the failure to reach a blood pressure (BP) goal in patients who are adhering to full doses of an appropriate 3-drug regimen that includes a

diuretic. After excluding potential causes of identifiable hypertension, clinicians should carefully explore reasons why the patient is not at goal BP. These reasons may include

improper BP measurement, volume overload, inadequate diuretic therapy, concomitant medications such as non-steroidal anti-inflammatory drugs (NSAID), or excess alcohol intake.

NSAIDs are commonly used to relieve pain or reduce inflammation from conditions such as arthritis. However, NSAIDs can worsen hypertension. Atorvastatin has a neutral effect on BP.

Metformin and pioglitazone may have a neutral or mildly beneficial effect on BP.

Reference

Chobanian AV, Bakris GL, Black HR, et al., for the National Heart, Lung, and Blood Institute Joint National Committee on Prevention, Detection, Evaluation, and Treatment of High Blood

Pressure; National High Blood Pressure Education Program Coordinating Committee. The Seventh Report of the Joint National Committee on Prevention, Detection, Evaluation, and

Treatment of High Blood Pressure: the JNC 7 report. JAMA. 2003;289:2560-2572.

Incorrect

Marks for this submission: 0.00/1.00.

Question 4

Marks: 1.00

Page 76: Asap 2011

A 26-year old woman is currently 12 weeks pregnant and her blood pressure is 160/92.

Which of the following drugs are safe to use in pregnancy?

Choose one answer .

A. Angiotensin-Converting Enzyme Inhibitor

B. Angiotensin Receptor Blocking agent

C. Clonidine

D. Alpha Methyldopa

E. Alpha-adrenergic Blocking agent

Educational Objective: Understand the appropriate management of hypertension in pregnancy.

Rationale: Women with hypertension who become pregnant should be followed carefully because of increased risks to mother and fetus. Alpha-methyldopa, beta-adrenergic blocking

agents, and vasodilators are the preferred antihypertensive medications in pregnancy for the safety of the fetus. Angiotensin-converting enzyme inhibitors and angiotensin receptor

blocking agents should not be used during pregnancy and in women who are likely to become pregnant because of the potential for fetal defects. Pregnant women should be

monitored for preeclampsia, which may develop into a hypertensive urgency or emergency and require hospitalization, intensive monitoring, early fetal delivery, and parenteral

antihypertensive and anticonvulsant therapy .

Reference

National High Blood Pressure Education Program. Report of the National High Blood Pressure Education Program Working Group on high blood pressure in pregnancy. Am J

Obstet Gynecol. 2000;183:S1-S22.

Incorrect

Marks for this submission: 0.00/1.00.

qsid=148127&aid=3882&qid=46&checksum=6ce35f5d9d8c748439edbdc258712659&sesskey=9SD9XfnshK&new state=0

Page 77: Asap 2011

Question 5

Marks: 1.00

A 72-year old man comes to you with symptoms of almost "passing out" whenever he stands up.

Which of his concomitant medications below is most likely to be causing his symptoms?

Choose one answer .

A. Aspirin

B. Atorvastatin

C. Metformin

D. Pioglitazone

E. Nitrates

Educational Objective: Understand the appropriate management of postural hypotension.

Rationale: Postural hypotension is defined as a decrease in standing systolic blood pressure of more than 10 mm Hg. It is more frequent in older patients with systolic hypertension or

diabetes mellitus and may be associated with dizziness or fainting.and in those patients taking diuretics, venodilators (e.g., nitrates, α-adrenergic blocking agents, and sildenafil-like

drugs), and some psychotropic drugs.. Blood pressure in these individuals who may be susceptible to postural hypotension should be monitored in the upright position. Caution

should be used to avoid volume depletion and excessively rapid dose titration of antihypertensive drugs. Therapy with aspirin, atorvastatin, metformin or pioglitazone is not

associated with an increased risk of postural hypotension unless there is some other confounding factor such as bleeding (aspirin) or dehydration.

Reference

qsid=148128&aid=3882&qid=47&checksum=023f7f4dcbd7e09237493bdc14022855&sesskey=9SD9XfnshK&new state=0

Page 78: Asap 2011

Chobanian AV, Bakris GL, Black HR, et al., for the Joint National Committee on Prevention, Detection, Evaluation, and Treatment of High Blood Pressure. National Heart, Lung, and

Blood Institute. National High Blood Pressure Education Program Coordinating Committee. Seventh report of the Joint National Committee on Prevention, Detection, Evaluation, and

Treatment of High Blood Pressure. Hypertension. 2003;42:1206-52.

Incorrect

Marks for this submission: 0.00/1.00.

Question 6

Marks: 1.00

Hypertension is a significant co-morbidity in patients with diabetes mellitus.

Which ONE of the following statements is correct as to what the first priority should be in patients with hypertension and diabetes mellitus?

Choose one answer .

A. Administration of an agent acting at the renin-angiotensin-aldosterone system

B. Reducing the blood pressure to levels as close as possible to 130 and 80 mmHg systolic and diastolic, respectively

C. Avoiding use of thiazide diuretics because of their potentially adverse effect on glucose control

D. Avoiding use of beta-adrenergic blocking agents because of their potential adverse effect on recognition of hypoglycemia

E. Continued observation of blood pressure levels beyond elevated measurements of 140/90 mmHg on two separate occasions to confirm persistent hypertension

Educational Objective: To emphasize the importance of achieving blood pressure control more so than the means of doing so.

qsid=148129&aid=3882&qid=48&checksum=7c5deae0f33863adc40614bd192cd4b1&sesskey=9SD9XfnshK&new state=0

Page 79: Asap 2011

Rationale: The evidence supporting particular benefit of RAAS is modest. Addition of a low dosage of a thiazide diuretic is often a very effective first- or second-line agent without

significant effect on glucose control. Similarly, beta-blockers often play an important role in the management of cardiomyopathies, and use of vasodilating types of beta-blockers, such

as nebivolol and carvedilol, may even result in improvement in insulin sensitivity and glucose control. Two separate blood pressure measurements above 140/90 mmHg are sufficient for

the diagnosis of hypertension and for treatment.

References

Bakris GL, Williams M, Dworkin L, et al. for the National Kidney Foundation Hypertension and Diabetes Executive Committees Working Group. Preserving renal function in adults with

hypertension and diabetes: a consensus approach. Am J Kidney Dis. 2000;36:646-661.

Chobanian AV, Bakris GL, Black HR, et al. The Seventh Report of the Joint National Committee on Prevention, Detection, Evaluation, and Treatment of High Blood Pressure (The

JNC 7 Report). Hypertension. 2003;42:1206-1252.

Ferdinand KC, Ferdinand DP. Trends in hypertension treatment in diabetes. Curr Hypertens Rep. 2009;11:437-43.

Incorrect

Marks for this submission: 0.00/1.00.

Question 7

Marks: 1.00

Which ONE of the following blood pressure treatments is best for the initial therapy of patients with hypertension and diabetes mellitus on the basis of current guidelines?

Choose one answer .

A. Alpha-2 adrenergic agonist

B. Alpha-adrenergic blocking agent

C. Angiotensin-converting enzyme inhibitor

qsid=148130&aid=3882&qid=49&checksum=cb80c43af2f657576c048a4817a5cec9&sesskey=9SD9XfnshK&new state=0

Page 80: Asap 2011

D. Methyldopa agent

E. Beta-adrenergic blocking agent

Educational Objective: To guide vasculo-protective and antihypertensive therapy in persons with diabetes mellitus and/or insulin resistance.

Rationale: Based on numerous studies and meta-analyses, expert consensus and current treatment guidelines recommend initiation of antihypertensive treatment with either an

angiotensin-converting enzyme inhibitor or angiotensin-receptor blocker in persons with or at risk for developing diabetes mellitus. Calcium channel blocking agents and beta-

adrenergic blocking agents are considered as second or third tier anti-hypertensive agents for patients with hypertension and diabetes mellitus. Alpha-adrenergic blocking agents or

agonists or methyldopa are rarely, if ever, used in the treatment of hypertension in patients with diabetes mellitus.

References

AACE Hypertension Task Force. American Association of Clinical Endocrinologists Medical Guidelines for Clinical Practice for the diagnosis and treatment of hypertension. Endocr

Pract. 2006;12:193-222.

Chobanian AV, Bakris GL, Black HR, et al. Seventh report of the Joint National Committee on Prevention, Detection, Evaluation, and Treatment of High Blood Pressure (The JNC 7 Report).

Hypertension. 2003;42:1206-1252.

Incorrect

Marks for this submission: 0.00/1.00.

Question 8

Marks: 1.00

Which one of the following statements regarding agents that block the renin-angiotensin-aldosterone system (RAAS) have been shown to be true?

Choose one answer .

A. They worsen glycemic levels in persons with impaired glucose tolerance

qsid=148131&aid=3882&qid=50&checksum=43e52b84598b367f3d18b1709ab8812a&sesskey=9SD9XfnshK&new state=0

Page 81: Asap 2011

B. They decrease post-load glucose levels in persons with impaired glucose tolerance

C. They decrease the risk of hypoglycemia in insulin-treated diabetic patients

D. They have an adverse effect on glycemic control as measured by HbA1c

E. They have a neutral effect on glycemia in persons with impaired glucose tolerance

Educational Objective: To be aware of the possible benefit/risk effects on glycemia of use of agents that block the renin-angiotensin-aldosterone system (RAAS)

Rationale: A decrease in the risk of developing diabetes mellitus with use of agents that block the RAAS has not been confirmed. A decrease in postprandial glycemia in patients with

impaired glucose tolerance and an increase in risk of hypoglycemia in insulin-treated diabetic patients has been reported. RAAS blocking agents appear to have an insulin-

sensitizing effect.

References

Braga MF, Leiter LA. Role of renin-angiotensin system blockade in patients with diabetes mellitus. Am J Cardiol. 2009;104:835-9

Herings RM, de Boer A, Stricker BH, et al. Hypoglycaemia associated with use of inhibitors of angiotensin converting enzyme. Lancet. 1995;345:1195-8.

Morris AD, Boyle DI, McMahon AD, et al. ACE inhibitor use is associated with hospitalization for severe hypoglycemia in patients with diabetes. DARTS/MEMO Collaboration. Diabetes

Audit and Research in Tayside, Scotland. Medicines Monitoring Unit. Diabetes Care. 1997;20:1363-7.

Incorrect

Marks for this submission: 0.00/1.00.

Question 9

Marks: 1.00

qsid=148132&aid=3882&qid=51&checksum=f3a6fc4f6d8d80e4abc7455cfa4f94a4&sesskey=9SD9XfnshK&new state=0

Page 82: Asap 2011

Which ONE of the following classes of blood pressure-lowering agents is most likely to worsen glycemia?

Choose one answer .

A. Aldosterone antagonists

B. Thiazide diuretics

C. Calcium channel blocking agents

D. Methyldopa

E. Alpha-adrenergic blocking agents

Educational Objective: To alert the physician to a potential adverse effect on glucose control with certain drug therapy.

Rationale: There is well-accepted evidence that thiazide diuretics, particularly in higher dosage, and selective B1 receptor blockers are likely to worsen glycemic control that needs to be balanced with the potential benefits of their usage. Aldosterone antagonists, calcium channel blocking agents, methyldopa and alpha-adrenergic blocking agents have not been

reported to adversely affect glycemia.

References

The Antihypertensive and Lipid-Lowering Treatment to Prevent Heart Attack Trial (ALLHAT) Officers. Major outcomes in high-risk hypertensive patients randomized to angiotensin-

converting enzyme inhibitor or calcium channel blocker vs diuretic. JAMA. 2002;288:2981-2997.

UK Prospective Diabetes Study Group. Efficacy of atenolol and captopril in reducing risk of macrovascular and microvascular complications in type 2 diabetes: UKPDS 39. BMJ.

1998;317:713-720.

Bangalore S, Parkar S, Grossman E, et al. A meta-analysis of 94,492 patients with hypertension treated with beta blockers to determine the risk of new-onset diabetes

mellitus. Am J Cardiol. 2007;100:1254-62

Incorrect

Marks for this submission: 0.00/1.00.

Question 10

Page 83: Asap 2011

Marks: 1.00

Given the evidence of benefit of both angiotensin receptor blockers and angiotensin converting enzyme inhibitors when administered to diabetic patients with hypertension,

current research is highly suggestive that which of the following combinations will be particularly useful?

Choose one answer .

A. Angiotensin receptor blockers and angiotensin converting enzyme inhibitors

B. Angiotensin receptor blockers and aldosterone antagonists

C. Angiotensin converting enzyme inhibitors and direct renin inhibitors

D. Angiotensin converting enzyme inhibitors and aldosterone antagonists

E. There is little clinical evidence to support the theoretical benefits of any particular combinations

Educational objective: To clarify the current state of evidence-based therapy of hypertension in patients with diabetes mellitus.

Rationale: There is little clinical evidence to support the theoretical benefits of any particular combinations, although most diabetic hypertensive patients will require three or more

agents to achieve goal blood pressure levels.

Reference

Mann JF, Schmieder RE, McQueen M; ONTARGET investigators. Renal outcomes with telmisartan, ramipril, or both, in people at high vascular risk (the ONTARGET study): a

multicentre, randomised, double-blind, controlled trial. Lancet. 200;372:547-53.

Incorrect

Marks for this submission: 0.00/1.00.

Question 11

qsid=148133&aid=3882&qid=52&checksum=e040a4e715e4ee37b8853bafc198b5bb&sesskey=9SD9XfnshK&new state=0

Page 84: Asap 2011

Marks: 1.00

Which of the following statements regarding the diagnosis, treatment and cardiovascular disease risk of hypertension is TRUE?

Choose one answer .

A. Hypertension secondary to identifiable underling causes is more common than essential hypertension in individuals less than 40 years of age

B. Most patients can achieve adequate treatment of hypertension to attain and maintain the desirable blood pressure goal with a thiazide diuretic and lifestyle

intervention

C. Hypertension without associated coronary artery disease has minimal impact on the risk of congestive heart failure

D. The risk of having a cardiovascular event doubles for every 20/10 mm Hg above 115/75 mm Hg

E. Treatment of dyslipidemia in patients with hypertension should be deferred pending achievement of target blood pressure level

Educational Objective: To emphasize the appropriate diagnosis and aggressive management of hypertension.

Rationale: Essential hypertension is the most common cause of hypertension, even in people under the age of 40. Secondary hypertension accounts for approximately 5% to 10% of

hypertension. Lifestyle modification is necessary in all treatment regimens for hypertension. Due to the natural progression of the disease, most patients will require at least 2 drugs to

manage hypertension during the course of therapy, if not at the time of diagnosis. Hypertension increases the risk of heart failure by 2 to 3 fold independent of coronary artery

disease status. The risk of a cardiovascular event doubles for every 20/10 mm Hg above 115/75 mm Hg. These data provide some of the rationale for tighter control of blood

pressure as suggested by the JNC-7. Hyperlipidemia/dyslipidemia is extremely common in patients diagnosed with hypertension. Screening for hyperlipidemia with a fasting lipid

panel, including measurements of total cholesterol, triglycerides, high-density lipoprotein cholesterol and calculated low-density lipoprotein cholesterol should be performed in all

qsid=148134&aid=3882&qid=53&checksum=1fb418bebdecbe45906aeef2f2af8d3b&sesskey=9SD9XfnshK&new state=0

Page 85: Asap 2011

patients with hypertension. Treatment should occur accordingly and should not be delayed until target blood pressure level is achieved.

References

Chobanian AV, Bakris GL, Black HR, et al. The Seventh Report of the Joint National Committee on Prevention, Detection, Evaluation, and Treatment of High Blood Pressure: the

JNC 7 report. JAMA. 2003;289(19):2560-2572.

Lewington S, Clark R, Qizilbash N, et al. Age-specific relevance of usual blood pressure to vascular mortality: A meta-analysis of individual data for one million adults in 61 prospective

studies. Lancet. 2002;360:1903-1913.

Incorrect

Marks for this submission: 0.00/1.00.

Question 12

Marks: 1.00

Standard blood pressure measurement should be carried out in both arms.

Which of the following BEST describes the correct patient positioning for standard blood pressure measurement?

Choose one answer .

A. The patient should be seated in a chair with feet resting on the floor and with arms supported to the level of the heart for at least 5 minutes

B. The patient should be seated in a chair with feet resting on the floor and with the arms supported for at least 2 minutes

C. The patient should be seated in a chair, with feet resting on the floor and arms at the side with or without support for at least 5 minutes

D. The patient should be standing for at least 2 minutes with arms at the side and with the blood pressure cuff bladder encircling 50% of the arm at the level of the

heart

qsid=148135&aid=3882&qid=54&checksum=d99402b47982355a1468a8afe2857875&sesskey=9SD9XfnshK&new state=0

Page 86: Asap 2011

E. The patient should be recumbent for 2 minutes with arms by the side at the level of the heart with the blood pressure taken in both arms

Educational Objective: To emphasize the correct assessment of blood pressure.

Rationale: In order to adequately assess for hypertension, a patient should be made as comfortable as possible under the least amount of stress as possible. It is necessary to

measure the blood pressure with an appropriate sized cuff and with the patient seated with feet resting on the floor. The patient should be allowed to rest for at least 5 minutes prior to the assessment of blood pressure, with the arms supported to the level of the heart, and the cuff bladder should encircle at least 80% of the arm. If coarctation of the aorta is suspected,

then blood pressure should be measured in the lower extremities as well; however, this is not necessary for routine evaluations. In order to assess for postural hypotension, blood

pressure should be measured with the patient recumbent first, then sitting after 2 minutes have elapsed and standing after 2 minutes have elapsed.

References

Chobanian AV, Bakris GL, Black HR, et al. Seventh report of the Joint National Committee on Prevention, Detection, Evaluation, and Treatment of High Blood Pressure. Hypertension.

2003;42(6):1206-1252.

Bickley L, Hoekelman R, et al. Bates’ Guide to Physical Examination and History Taking. Philadelphia, PA: Lippincott, 1999: 294-299.

Incorrect

Marks for this submission: 0.00/1.00.

Question 13

Marks: 1.00

Which of the following statements is TRUE regarding isolated systolic hypertension?

Choose one answer .

A. Treatment of isolated systolic hypertension secondary to diabetes mellitus or chronic renal insufficiency is not effective in reducing cardiovascular risk

qsid=148136&aid=3882&qid=55&checksum=1e77a6432f25687a53ad8937cbb0a690&sesskey=9SD9XfnshK&new state=0

Page 87: Asap 2011

B. Isolated systolic hypertension is more common in elderly women compared to age-matched men

C. Isolated systolic hypertension is defined as systolic blood pressure of greater than 140 mm Hg with diastolic blood pressure less than 100 mm Hg

D. Isolated systolic hypertension has a limited adverse cardiovascular effect in the absence of diabetes mellitus and/or chronic renal insufficiency

E. Appropriate treatment of isolated systolic hypertension has not been shown to reduce morbidity or mortality

Educational Objective: To recognize and understand the significance of isolated systolic hypertension.

Rationale: Isolated systolic hypertension is 14.2% more common among elderly women than age-matched men. It is an independent risk factor for cardiovascular and cerebrovascular adverse events. The Framingham study and Systolic Hypertension in the Elderly Program

(SHEP) studies showed reduction in coronary artery disease, congestive heart failure, and stroke with the treatment of systolic hypertension in the absence of diastolic hypertension.

Isolated systolic hypertension is defined as systolic blood pressure >140 mm Hg with diastolic blood pressure <90 mm Hg.

References

Franklin SS, Larson MG, Khan SA, et al. Does the relation of blood pressure to coronary heart disease risk change with aging? The Framingham Heart Study. Circulation. 2001;103:1245-

1249.

SHEP Cooperative Research Group. Prevention of stroke by antihypertensive drug treatment in older persons with isolated systolic hypertension. Final results of the Systolic

Hypertension in the Elderly Program (SHEP). JAMA. 1991;65(24):3255-3264.

Burt VL, Whelton P, Roccella EJ, et al. Prevalence of hypertension in the US adult population: results from the Third National Health and Nutrition Examination Survey, 1988-1991.

Hypertension. 1995;25:305-313.

Incorrect

Marks for this submission: 0.00/1.00.

Question 14

Marks: 1.00

Page 88: Asap 2011

Which of the following statements regarding hypertension identification and treatment is TRUE?

Choose one answer .

A. An increased risk for stroke and heart failure has been shown to occur when pulse pressure is widened by more than 10 mm Hg from baseline with treatment

B. "White Coat" syndrome is an example of underlying isolated systolic hypertension and is associated with an increased cardiovascular risk

C. Dysautonomia, as in Shy-Drager Syndrome, is a rare cause of orthostatic hypotension in the elderly

D. Increased stiffness of the blood vessels caused by atherosclerosis may lead to a falsely low blood pressure reading

E. Treatment of systolic hypertension in patients over 80 years of age has been shown to be less effective in stroke reduction than comparable treatment in

younger patients between 60 and 70 years of age

Educational Objective: To appropriately identify and treat significant hypertension.

Rationale: The Systolic Hypertension in the Elderly Program (SHEP) trial showed a 24% increased risk for stroke and 32% increased risk of heart failure when pulse pressure

widened by more than 10 mm Hg from baseline with treatment. This increased risk is related to decreased compliance of the vascular system with treatment in the elderly. One cause of

orthostatic hypotension is dysautonomia. A primary cause of dysautonomia is Shy-Drager syndrome, which is more common in elderly patients. In elderly patients, there can be

increased stiffness of the blood vessels caused by atherosclerosis. This may lead to falsely elevated blood pressure, known as pseudohypertension. Pseudohypertension should be

suspected in an elderly patient when there is an absence of end organ damage. "White Coat" syndrome is the isolated elevation of blood pressure within a physician's office. There

is no corresponding end organ damage. Blood pressure measured outside of the office is normal. There is no increased risk of cardiovascular events associated with "White Coat" syndrome. In the SHEP trial, a greater benefit among patients ≥80 years old compared to those in their 60s was noted, with greater reduction in stroke, with a relative risk of 0.53

qsid=148137&aid=3882&qid=56&checksum=8d103a8233a5211903f8150d67dd5b1b&sesskey=9SD9XfnshK&new state=0

Page 89: Asap 2011

(95% CI, 0.32-0.88) and 0.74 (95% CI, 0.48-1.14), respectively. These benefits have been confirmed by other available data supporting treatment of systolic hypertension in patients

with stage 1 disease (systolic blood pressure of 140-159 mm Hg) as well.

References

Kjeldsen SE, Hedner T, Jamerson K, et al. Hypertension Optimal Treatment (HOT) Study, Home blood pressure in treated hypertensive subjects. Hypertension. 1998;31:1014-1020.

SHEP Cooperative Research Group. Prevention of stroke by antihypertensive drug treatment in older persons with isolated systolic hypertension. Final results of the Systolic

Hypertension in the Elderly Program (SHEP). JAMA. 1991;65(24):3255-3264.

Messerli FH. Osler’s maneuver, pseudohypertension, and true hypertension in the elderly. Am J Med. 1986;80:906-910.

Incorrect

Marks for this submission: 0.00/1.00.

Question 15

Marks: 1.00

A 39 year-old African-American man presents for establishment of primary care. His blood pressure (BP) measured in both arms is 154/92. He has been in apparent good health

without prior need for medical attention and claims to feel anxious being in clinic. He is not aware of hypertension in the family. He has been athletic all of his life. He has noted an

increasing use of non-steroidal anti-inflammatory drugs (NSAIDs) for arthralgias and myalgias to keep competing with the younger "guys." He takes no other medications on a

regular basis and has no complaints. Except for being slightly overweight with a BMI of 29, and the elevated BP reading, there are no abnormal findings on his physical examination.

The recommended approach to evaluation and management of this patient would be which of the following?

Choose one answer .

A. Defer further evaluation until blood pressure (BP) elevation is confirmed again in about one month

qsid=148138&aid=3882&qid=57&checksum=6f6616875f140360701b803057840d6f&sesskey=9SD9XfnshK&new state=0

Page 90: Asap 2011

B. Refer for lifestyle modification and return in three months for follow-up with fasting laboratory studies, including urinalysis

C. Discuss lifestyle modification, including minimal over-the-counter drug use, order fasting laboratory studies including urinalysis and a baseline

electrocardiogram (ECG) and return for repeat BP check within 2-4 weeks

D. Refer for lifestyle modification, order fasting laboratory studies, ECG screening serum aldosterone and plasma renin activity, and return for follow-up within 1-2

weeks

E. Discuss lifestyle modifcation, start a thiazide diuretic and return in 2-4 weeks with routine laboratory studies and urinalysis

Educational objective: To appropriately identify and initiate treatment for significant hypertension.

Rationale: Blood pressure must be determined during at least two different office visits in the least stressful environment possible before the diagnosis of hypertension can be made.

When 'white coat syndrome' (elevation of blood pressure in a physician office) in the absence of end-organ damage is suspected, ambulatory blood pressure recording may be

used to make the diagnosis of hypertension and to monitor treatment. Generally, readings >135/85 mm Hg measured at home are considered to be diagnostic of hypertension, and warrant therapy while readings consistently below 130/80 do not require pharmacologic

therapy. Prior to initiating medication for hypertension, laboratory testing must be performed to assess end organ involvement and to screen for secondary causes. This testing

includes complete blood count, serum creatinine and blood urea nitrogen with corresponding estimation of glomerular filtration rate, serum electrolytes, serum calcium,

lipid profile after a 12-hour fast, and urinalysis with microscopic evaluation. Prior to the initiation of appropriate therapy, a baseline electrocardiogram (ECG) is also recommended

to evaluate for prior myocardial ischemic changes or left ventricular hypertrophy. The measurements of urinary creatinine and albumin excretion to measure albumin to creatinine

ratio is considered optional by most hypertension specialists. It is important to discuss lifestyle modification, minimal usage of over-the-counter medications such as non-steroidal

anti-inflammatory agents, which may elevate blood pressure, prior to anti-hypertensive therapy. Studies for evaluation of secondary causes of hypertension should be deferred until

after initial laboratory studies of serum electrolytes, creatinine, calcium, hematology panel, lipid levels and routine urinalysis and resting ECG have been obtained and reviewed.

References

Chobanian AV, Bakris GL, Black HR, et al. Seventh Report of the Joint National Committee on Prevention, Detection, Evaluation, and Treatment of High Blood Pressure. Hypertension.

2003;42(6):1206-1252.

Page 91: Asap 2011

Kjeldsen SE, Hedner T, Jamerson K, et al. Hypertension Optimal Treatment (HOT) Study, Home blood pressure in treated hypertensive subjects. Hypertension. 1998;31:1014-1020.

Chobanian AV, Bakris GL, Black HR, et al. The Seventh Report of the Joint National Committee on Prevention, Detection, Evaluation, and Treatment of High Blood Pressure: the

JNC 7 report. JAMA. 2003;289(19):2560-2572.

Incorrect

Marks for this submission: 0.00/1.00.

Question 16

Marks: 1.00

Which ONE of the following factors contributes adversely to cardiovascular risk but is not a known risk factor for hypertension?

Choose one answer .

A. Obesity

B. High salt (NaCl) intake

C. Cigarette smoking

D. Excessive alcohol intake

E. Lack of exercise

Educational Objective: To understand which factors, when modified, lead to a reduction in blood pressure in hypertensive individuals.

Rationale: While a risk factor for the development of coronary and peripheral vascular diseases, cigarette smoking is not a contributor to the development of hypertension.

Correction of obesity, high salt intake, excessive alcohol intake, and lack of exercise leads to a healthier lifestyle in individuals who manifest these behaviors. Reductions in blood

qsid=148139&aid=3882&qid=58&checksum=0dc6ef94a0505661e9bff14d4e823cc1&sesskey=9SD9XfnshK&new state=0

Page 92: Asap 2011

pressure accompany a regular exercise program and reduction in weight, salt intake and alcohol intake across all stages of hypertension.

Reference

Chobanian AV, Bakris GL, Black HR, et al. The Seventh Report of the Joint National Committee on Prevention, Detection, Evaluation, and Treatment of High Blood Pressure.

JAMA. 2003;289:2560-2572.

Incorrect

Marks for this submission: 0.00/1.00.

Question 17

Marks: 1.00

Which the following statements regarding the non-pharmacological treatment of hypertension is TRUE?

Choose one answer .

A. High nutrient intake of fruits and vegetables has little impact on blood pressure control independent of weight loss

B. Absence of qualitative dietary changes in the non-pharmacological management of hypertension reduces the beneficial effect of weight loss and exercise on blood

pressure control in patients with hypertension

C. Reduction in sodium intake alone results in significant blood pressure reduction but the combination with qualitative dietary changes in fruits, vegetables and fiber

provides additional beneficial effect on blood pressure control

D. Reducing alcohol intake among heavy drinkers has minimal effect on blood pressure control in hypertensive patients receiving antihypertensive medication

E. In addition to limiting salt intake, supplementing the diet with potassium, magnesium, and calcium brings about additional decreases in blood pressure

qsid=148140&aid=3882&qid=59&checksum=673385d3a2963740a806be7e0d3c2832&sesskey=9SD9XfnshK&new state=0

Page 93: Asap 2011

Educational Objective: To understand which behaviors, when modified, aid in the control of hypertension.

Rationale: The JNC-7 recommends lifestyle modification as initial or coincident therapy across hypertension stages. Aerobic and limited-resistance exercise as a lifestyle change

decreases blood pressure in younger hypertensive patients, but may be of limited value in patients over age 60. Combined with dietary modification, exercise does aid in weight

reduction, which does have a beneficial effect in lowering blood pressure. Modest reduction in alcohol intake in hypertensive populations positively affects both systolic and diastolic

blood pressure. The Dietary Approaches to Stop Hypertension (DASH) diet with qualitative dietary changes (fruits, vegetables, high fiber, low saturated fat) combined with reduction in

salt intake lowers blood pressure to a greater degree than either intervention, although each is beneficial. The addition of qualitative dietary changes may assist with weight loss and add

to the beneficial effect on blood pressure reduction from weight reduction and exercise. Supplementing calcium, magnesium, and potassium offer promise as "mineral

management," but interventional trials have yet to be performed demonstrating a statistically significant effect.

References

 

Chobanian AV, Bakris GL, Black HR, et al. The Seventh Report of the Joint National Committee on Prevention, Detection, Evaluation, and Treatment of High Blood Pressure.

JAMA. 2003;289:2560-2572

Sacks FM, Svetkey LP, Vollmer WM, et al. Effects on blood pressure of reduced dietary sodium and the dietary approaches to stop hypertension (DASH) diet. N Engl J Med.

2001;344(1):3-9.

Stewart KJ, Bacher AC, Turner KL, et al. Effect of exercise on blood pressure in older persons. Arch Intern Med. 2005;165:756-762.

Incorrect

Marks for this submission: 0.00/1.00.

Question 18

Marks: 1.00

Liddle's syndrome is characterized by the onset of hypertension, hypokalemia, and metabolic acidosis in childhood.

qsid=148141&aid=3882&qid=60&checksum=985a351db6667ebfc1ba25f6cfb0de70&sesskey=9SD9XfnshK&new state=0

Page 94: Asap 2011

Which of the following statements regarding this syndrome are correct?

Choose one answer .

A. This is an autosomal dominant heritable disorder resulting from a mutation of the β-subunit of the aldosterone receptor in the distal nephron

B. The mutation defect results in decreased renal reabsorption of sodium

C. Plasma renin activity will be elevated

D. The disorder can be distinguished from primary aldosteronism by decreased urinary excretion of aldosterone

E. Treatment with spironolactone (50-100 mg in divided doses) will correct the manifestations of Liddle's syndrome

Educational Objective: To understand the etiology, evaluation, and appropriate treatment of Liddle's syndrome.

Rationale: Liddle's syndrome is a rare, autosomal dominant heritable disorder that is usually diagnosed in childhood and is characterized by hypertension, hypokalemia and metabolic

acidosis. The defect is a mutation in the β- or γ-subunits of the renal epithelial sodium channel located in the distal nephron, resulting in increased renal reabsorption of sodium. It

is a low-renin, low-aldosterone disorder with low urinary excretion of aldosterone. The kidneys will appear normal on imaging. This disorder is distinguished from primary

aldosteronism by the decreased urinary excretion of aldosterone. This syndrome does not respond to spironolactone because it is a low aldosterone disorder. Amiloride or

triamterene, which specifically close the sodium channel, are the appropriate treatment choices for this syndrome.

References

Botero-Velez M, Curtiss JJ, Warnock DG. Brief report: Liddle's syndrome revisited — a disorder of sodium reabsorption in the distal tubule. N Engl J Med. 1994;330:178-181.

Snyder PM. The epithelial Na+ channel: cell surface insertion and retrieval in Na+ homeostasis and hypertension. Endocrine Reviews. 2002;23:258-275.

Incorrect

Marks for this submission: 0.00/1.00.

Question 19

Page 95: Asap 2011

Marks: 1.00

A 47-year-old woman with a 2-year history of hypertension developed weakness and became easily fatigued approximately 2 months ago. Laboratory tests at that time showed sodium 138 mEq/L, potassium 2.8 mEq/L, carbonate 28 mg/dL, blood urea nitrogen (BUN)

20 mg/dL, and creatinine 1.2 mg/dL. Computed tomographic scan of the adrenals was negative.

After discontinuing all antihypertensive medication, which of the following would be the best initial test to determine the etiology of hypertension and hypokalemia in this patient?

Choose one answer .

A. Measure 24-hour urinary potassium excretion

B. Obtain a supine and upright plasma aldosterone concentration (PAC) and plasma renin activity (PRA)

C. Obtain a magnetic resonance imaging scan of the adrenal glands

D. Obtain bilateral adrenal scintography

E. Order bilateral adrenal vein sampling

Educational Objective: To understand the appropriate diagnostic evaluation of hypertension and hypokalemia.

Rationale: This patient may have primary or secondary aldosteronism. As a screening test, the postural stimulation test with supine and upright plasma aldosterone concentration

(PAC) and plasma renin activity (PRA) will identify whether excess aldosterone or renin is contributing to hypertension and hypokalemia. Measuring urinary potassium excretion by

itself may suggest hyperaldosteronism; however, it is not specific and needs to be assessed in the context of PAC and PRA. When either computed tomographic (CT) scan or magnetic

resonance imaging (MRI) is negative in the setting of an elevated PAC:PRA ratio, microadenomas cannot be excluded and adrenal vein sampling would be necessary. Bilateral adrenal scintigraphy with NP-59 (131Iodine-labeled 6-beta-iodomethylnorcholesterol) would

be recommended if venous sampling could not be performed for localization of unilateral

qsid=148142&aid=3882&qid=61&checksum=d1bf63e489f7a1fce5418e5a13f15174&sesskey=9SD9XfnshK&new state=0

Page 96: Asap 2011

adenomas. Since the CT scan was negative, there is no benefit to obtaining an MRI of the adrenal glands because both tests are equally sensitive in detecting adenomas > 1 cm.

References

Avram AM, Fig LM, Gross MD. Adrenal gland scintigraphy. Seminars in Nuclear Medicine. 2006; 36:212-227

Doppman JL, Gill JR. Hyperaldosteronism: Sampling the adrenal veins. Radiology. 1996;198:309-312.

Ganguly, A. Current concepts: primary aldosteronism. N Engl J Med. 1998;339:1828-1834.

Incorrect

Marks for this submission: 0.00/1.00.

Question 20

Marks: 1.00

Primary hyperaldosteronism is suggested by an increased plasma aldosterone concentration with suppressed plasma renin activity with an increased PAC:PRA >30.

Which one of the following disorders is not associated with primary aldosteronism?

Choose one answer .

A. Glucocorticoid remediable aldosteronism

B. Aldosterone-producing adenoma

C. Adrenal carcinoma

D. Renin secreting tumor

E. Idiopathic hyperaldosteronism

qsid=148143&aid=3882&qid=62&checksum=07cf0dbde934ac9f82cb362745dbef6a&sesskey=9SD9XfnshK&new state=0

Page 97: Asap 2011

Educational Objective: To understand the use of the plasma aldosterone concentration to plasma renin activity ratio (PAC:PRA) to differentiate between primary and secondary

aldosteronism.

Rationale: Primary hyperaldosteronism is suggested by an increased plasma aldosterone concentration with a suppressed plasma renin activity level. Primary aldosteronism includes

aldosterone-producing adenoma, idiopathic hyperaldosteronism, adrenal hyperplasia, adrenal carcinoma, and glucocorticoid remediable aldosteronism, all of which are

characterized by elevated levels of aldosterone with suppressed renin levels. Elevated aldosterone levels with elevated renin levels are indicative of secondary

hyperaldosteronism. It is important to identify secondary causes of hyperaldosteronism, which are usually correctable. Although a renin-secreting tumor is a less frequent cause of

secondary aldosteronism, it is the most common cause of severe hypertension and hypokalemia in children.

References

McVicar M, Carman C, Chandra M, Abbi RJ, Teichberg S, Kahn E. Hypertension secondary to renin-secreting juxtaglomerular cell tumor: case report and review of 38 cases. Pediatr

Nephrol. 1993;7:404-412.

Weinberger M, Fineberg N. The diagnosis of primary aldosteronism and separation of two major subtypes. Arch Intern Med. 1993;153:2125-2129.

Wilcox CS. Use of angiotensin-converting enzyme inhibitors for diagnosing renovascular hypertension. Kidney Int. 1993;44:1379-1390.

Incorrect

Marks for this submission: 0.00/1.00.

Question 21

Marks: 1.00

Ingestion of licorice or other products that contain glycyrrhizinic acid produces hypertension and hypokalemia that appears to be due to primary aldosteronism.

This effect of glycyrrhizinic acid is due to which of the following factors?

Choose one answer .

qsid=148144&aid=3882&qid=63&checksum=7f7cc9386479484a9cfb2e3400850e83&sesskey=9SD9XfnshK&new state=0

Page 98: Asap 2011

A. Direct stimulation of cortisol production by the adrenal gland

B. Inhibition of 11-beta hydroxysteroid dehydrogenase in the kidney

C. Systemic increased conversion of cortisone to cortisol

D. Binding of glycyrrhizinic acid to adrenal mineralocorticoid receptors

E. Deceased clearance of cortisol by the kidney

Educational Objective: To understand the etiology of licorice induced hypertension

Rationale: Licorice purchased in the United States has usually been deglycyrrhizinized. However, some tobacco or herbal products contain licorice, and licorice imported from Europe or the Middle East may contain glycyrrhizinic acid. When ingested in significant

amounts, hypertension and hypokalemia can result, presenting as hyperaldosteronism that results from the glycyrrihizinic acid inhibiting 11-beta hydroxysteroid dehydrogenase-2 that

converts cortisol to cortisone in the kidney. The treatment is discontinuation of product containing licorice or glycyrrhizinic acid.

References

Edwards CR. Lessons from licorice. N Engl J Med. 1991;325:1242-1243.

Farese RV Jr, Biglieri EG, Shackleton CH, et al. Licorice-induced hypermineralocorticoidism. N Engl J Med. 1991;325:1223-1227.

Monder C. Corticosteroids, kidneys, sweet roots and dirty drugs. Mol Cell Endocrinol. 1991;78:C95-C98.

Incorrect

Marks for this submission: 0.00/1.00.

Question 22

Marks: 1.00

qsid=148145&aid=3882&qid=64&checksum=8b63f6e6db2f09f599cc66d54c409ca7&sesskey=9SD9XfnshK&new state=0

Page 99: Asap 2011

A 22-year old woman with hypertension and hypokalemia being evaluated for primary aldosteronism has a PAC:PRA >30 suggesting primary hyperaldosteronism. She has a family

history of early-onset hypertension. During evaluation of primary aldosteronism, it was found that in addition to elevated aldosterone and low renin levels, 18-hydroxycortisol and

18-oxocortisol levels were also elevated .

Which of the following is the most definitive test for the diagnosis of this disorder of mineralocorticoid hypertension?

Choose one answer .

A. Bilateral adrenal vein sampling

B. Measurement of morning supine 18-hydroxycorticosterone level

C. Performing genetic analysis

D. Captopril suppression during renal angiography

E. NP-59( 131Iodine-labeled 6-beta-iodomethylnorcholesterol) scan

Educational Objective: To understand the etiology diagnosis of glucococorticoid remediable aldosteronism (GRA).

Rationale: Screening results of a PAC:PRA >30 has a 90% specificity for primary aldosteronism. When associated with elevation in 18-hydroxycortisol and 18-oxocortisol, the diagnosis of GRA is considered. While 18-hydroxycorticosterone level may be elevated even

during morning supine measurement, a definitive diagnosis can only be made by genetic analysis. GRA is an autosomal dominant disorder resulting from a chimeric gene located in the zona fasciculata and in the zona glomerulosa that is responsive to adrenocorticotropic

hormone (ACTH), resulting in increased aldosterone production. Bilateral adrenal vein sampling will not be of any help in making the diagnosis because this disorder is not due to

an aldosterone-producing adenoma (APA) or idiopathic hyperaldosteronism (IHA) for which adrenal vein sampling is indicated. A captopril suppression test during renal angiography will

demonstrate renal artery stenosis, but is not helpful for the diagnosis of GRA. Similarly, an NP-59 (131Iodine-labeled 6-beta-iodomethylnorcholesterol) scan will not be helpful since

this disorder is not due to APA or IHA.

References

Doppman JL, Gill JR Jr. Hyperaldosteronism: sampling the adrenal veins. Radiology 1996; 198:309-312.

Page 100: Asap 2011

Doppman JL, Gill JR Jr, Miller DL, et al. Distinction between hyperaldosteronism due to bilateral hyperplasia and unilateral adosteronoma: reliability of CT. Radiology. 1992;

183:677-682 .

Ulick S, Chan CK, Gill JR Jr, et al. Defective fasciculate zone function as the mechanism of glucocorticoid-remediable aldosteronism. J Clin Endocrinol Metab 1990; 71:1151-1157.

Incorrect

Marks for this submission: 0.00/1.00.

Reminder:

You have three (3) attempts to successfully complete this assessment with a 75% score or higher.

You have 30 days to complete each attempt, once an attempt is started.

If you need to save your answers and come back to the assessment, you must click the 'Next' button at the bottom-left of the assessement prior to logging

out of ASAP.

Only click 'Submit all and finish' once you are satisfied with all your assessment answers.

Bottom of Form

Finish review

Skip Quiz navigation

Quiz navigation

i (Closed ) 1 (Incorrect ) 2 (Incorrect ) 3 (Incorrect ) 4 (Incorrect ) 5 (Incorrect ) 6 (Incorrect ) 7 (Incorrect ) 8 (Incorrect ) 9 (Incorrect ) 10 (Incorrect ) 11 (Incorrect ) 12 (Incorrect ) 13

(Incorrect ) 14 (Incorrect ) 15 (Incorrect ) 16 (Incorrect ) 17 (Incorrect ) 18 (Incorrect ) 19 (Incorrect ) 20 (Incorrect ) 21 (Incorrect ) 22 (Incorrect ) i (Closed )

Finish review

You are logged in as Mohammed Aldawish (Logout)

HYPER

 

Copyright: 2011, American Association of Clinical Endocrinologists (AACE).All material published on the Web site is the property of AACE and may not be reproduced in

any form or by any electronic meansincluding information, storage and retrieval systems without the written consent of AACE.

Page 101: Asap 2011

This activity is sponsored by the American Association of Clinical Endocrinologists.

Skip to main content

Hypoglycemic Disorders

You are logged in as Mohammed Aldawish (Logout)

Page path

Home

/ ► My courses

/ ► HYPO

/ ► Assessment

/ ► Hypoglycemic Disorders Assessment

/ ► Review of attempt 1

Review of attempt 1

Started onTuesday, 19 November 2013, 10:27 PM

Completed onTuesday, 19 November 2013, 10:27 PM

Time taken21 secs

Grade0.00 out of a maximum of 21.00 (0%)

FeedbackFailed

Top of Form

Directions: In this section, each item includes a lead question and a list of options labeled with letters. Select the ONE lettered option that is BEST in each case by selecting the radio

button corresponding to the correct answer .

9SD9XfnshK

Page 102: Asap 2011

Question 1

Marks: 1.00

Which of the following statements regarding the diagnostic evaluation of patients presenting with symptoms suggestive of hypoglycemia is CORRECT?

Choose one answer .

A. The 5-hour oral glucose tolerance test is of diagnostic use in a patient with postprandial symptoms compatible with hypoglycemia

B. Use of glucose/insulin ratios is a poor indication of hyperinsulinemia

C. Classification of hypoglycemic disorders based on the timing of symptoms (fasting versus postprandial) has significant diagnostic utility

D. In a patient with hyperinsulinemic hypoglycemia and negative computed tomography scan of the pancreas, measurement of insulin-like growth factor (IGF)-

1 should be carried out

E. Endoscopic ultrasonography is a useful localization procedure in patients with post Roux-en-Y gastric bypass hypoglycemia

Educational Objective: Understand the pitfalls in diagnostic testing for hyperinsulinemic hypoglycemia.

Rationale: Glucose/insulin ratios are a poor indicator of hyperinsulinemic hypoglycemia and should not be used in making the diagnosis of hyperinsulinemic hypoglycemia. A diagnostic

criterion based on absolute insulin values, such as a level of 3 µU/mL by ICMA (immunochemiluminometricassay) is far more accurate.

The timing of hypoglycemic symptoms is not helpful in the classification of hypoglycemia, as sometimes it may be difficult to distinguish postprandial from fasting symptoms. Moreover, timing does not necessarily give insights into the pathogenesis of disease. Similarly, fasting

hypoglycemia may be seen without associated hyperinsulinism in some inborn errors of metabolism.

A glucose tolerance test is rarely useful in the diagnosis of hypoglycemic disorders and should not be part of the diagnostic testing for hypoglycemic disorders. Ten percent of

qsid=148148&aid=3883&qid=66&checksum=dd7493b41fad7dd3d9bd0bf141a11ec7&sesskey=9SD9XfnshK&new state=0

Page 103: Asap 2011

healthy normal individuals will have glucose nadirs Insulin-like growth factor-1 (IGF-1) measurement would not be useful in the diagnosis of

hyperinsulinemic hypoglycemia as non-islet cell hypoglycemic syndrome (NICHS) is not associated with hyperinsulinemia.

Endoscopic ultrasonography is usually not feasible in patients who have undergone a Roux-en-Y gastric bypass.

Reference

Service GF, Thompson GB, Service FJ, et al. Hyperinsulinemic hypoglycemia with nesidioblastosis after gastric bypass surgery. N Engl J Med. 2005;353:249-254.

Incorrect

Marks for this submission: 0.00/1.00.

Question 2

Marks: 1.00

In assessing the patient with hypoglycemia, which ONE of the following observations/interpretations is most helpful?

Choose one answer .

A. The levels of β-cell polypeptides in sulfonylurea-induced hypoglycemia are clearly distinguishable from those in patients with insulinoma

B. Insulin concentrations of approximately 100 µU/mL during hypoglycemia are most likely related to a recent injection of insulin in the absence of insulin

antibodies

C. The best diagnostic criterion for hyperinsulinemic hypoglycemia in persons with insulinoma is the absolute insulin level, >3 µU/mL by ICMA

(immunochemiluminometric assay)

qsid=148149&aid=3883&qid=67&checksum=1d26893b27bb92b9776c85affe1f47ef&sesskey=9SD9XfnshK&new state=0

Page 104: Asap 2011

D. There is no need to proceed beyond 48 hours in any patient undergoing a prolonged fast

E. A negative 72-hour fast may be indicated by a consistent and progressive decrease of serum beta-hydroxybutyrate after 18 hours of fasting

Educational Objective: Understand the assessment of patients presenting with hypoglycemia.

Rationale: The levels of β-cell polypeptides may be similar in patients with sulfonylurea-induced hypoglycemia and patients with insulinoma and are therefore not useful in

differentiating these two potential sources of hypoglycemia.A serum insulin concentration of 100 µU/mL or less is usually seen with insulinomas. High

Insulin levels of 1000 µU/mL or more are usually seen after a recent insulin injection of insulin or in the presence of insulin antibodies.

The best diagnostic criterion for hyperinsulinemic hypoglycemia in patients with insulinoma is the absolute insulin concentration of ≥3 µU/mL at the time of hypoglycemia .

Approximately 8% of patients with hypoglycemia may a have a negative test at 48 hours and therefore need to be tested for the full 72 hours. There are also rare cases of patients with

hypoglycemia who are not identified by 72 hours and may require a repeat test.A progressive rise in serum β-hydroxybutyrate measured every 6 hours during the 72-hour

fast presages a negative fast and could be used to truncate the fast. Serum β-hydroxybutyrate is essentially an insulin surrogate. Insulin has an antiketogenic effect and

patients with insulin-mediated hypoglycemia have lower levels than in normal subjects or in patients with noninsulin-mediated hypoglycemia.

References

Service FJ, Natt N. Clinical perspective: the prolonged fast. J Clin Endocrinol Metab. 2000;85:3973-3974.

Service FJ, O'Brien PC. Increasing serum beta-hydoxybutyrate concentrations during the 72 hour fast: evidence against hyperinsulinemic hypoglycemia. J Clin Endocrinol Metab.

2005;90:4555-455

Incorrect

Marks for this submission: 0.00/1.00.

Question 3

Marks: 1.00

Page 105: Asap 2011

Which of the following patterns of laboratory tests indicate an insulinoma in a patient who has satisfied Whipple's triad (hypoglycemic symptoms, documented hypoglycemia, and

reversal of hypoglycemia associated with relief of symptoms)?

Choose one answer .

A. Elevated insulin (100 µU/mL), C-peptide, and proinsulin with a negative sulfonylurea screen

B. Elevated insulin (1000 µU/mL), undetectable C-peptide and proinsulin levels

C. Undetectable insulin, C-peptide, and proinsulin levels and positive insulin antibodies

D. Undetectable insulin, C-peptide, and proinsulin levels and low β-hydroxybutyrate concentrations

E. Elevated insulin (1000 µU/mL), normal C-peptide and proinsulin levels, and positive insulin antibodies

Educational Objective: Identify the β-cell polypeptide pattern for patients with insulinoma.Rationale: The classic β-cell polypeptide pattern for insulinoma is elevated serum insulin,

usually <100 µU/mL,and associated proportional elevation of C-peptide and proinsulin levels. Sulfonylurea-induced hypoglycemia may have similar measurements, but medical

history and further testing for sulfonylurea and other insulin secretogogues will differentiate hypoglycemia due to insulinoma versus sulfonylurea or other insulin secretogogues

(meglitinides).

Markedly elevated insulin levels of 1000 µU/mL are associated with recent insulin injection, usually surreptitiously. These levels may also be seen in patients with positive insulin

antibodies related to insulin therapy because of the interfering effect of the antibodies in the immunoassay for insulin.

The pattern of markedly elevated insulin with suppressed C-peptide and proinsulin is diagnostic of exogenous insulin administration.

The pattern of markedly elevated insulin levels with associated high C-peptide and proinsulin levels are consistent with insulin autoimmune syndrome with hypoglycemia.

Undetectable insulin, C-peptide, and proinsulin levels with positive insulin antibodies are seen in untreated type 1 diabetes mellitus. The same pattern without positive insulin

qsid=148150&aid=3883&qid=68&checksum=cec96b6ed362a3cd856603e641c42ecb&sesskey=9SD9XfnshK&new state=0

Page 106: Asap 2011

antibodies can be seen in some types of glycogen storage diseases, such as von Gierke's disease.

Low β-hydroxybutyrate is consistent with hyperinsulinemia and is not likely to be seen in conjunction with low β-polypeptide levels. Low β-polypeptide levels can be seen in alcohol

intoxication. Ketoacidosis can also be seen in alcohol intoxication.

References

1 .Service FJ. Hypoglycemic disorders. N Engl J Med. 1995;332:1144-1152 ..

2 .Service FJ, Natt N. Clinical perspective: the prolonged fast. J Clin Endocrinol Metab. 2000;85:3973-3974.

3.Service FJ, O’Brien PC. Increasing serum beta-hydroxybutyrate concentrations during the 72 hour fast: evidence against hyperinsulinemic hypoglycemia. J Clin Endocrinol

Metab. 2005;90:4555-4558.

Incorrect

Marks for this submission: 0.00/1.00.

Question 4

Marks: 1.00

Which of the following statements regarding non-insulinoma pancreatogenous hypoglycemia syndrome (NIPHS) is CORRECT?

Choose one answer .

A. It can occur in patients post Roux-en-Y gastric bypass surgery

B. It results in fasting hypoglycemia

C. It is due to insulin antibodies

D. It has characteristic findings on computed tomography

qsid=148151&aid=3883&qid=69&checksum=b370c4464b6569802ab3cf0579d5e955&sesskey=9SD9XfnshK&new state=0

Page 107: Asap 2011

E. It rarely requires surgical intervention

Educational Objective: Recognize characteristics of non-insulinoma pancreatogenous hypoglycemia syndrome (NIPHS)

Rationale: The hypoglycemia associated with this disorder is almost exclusively noted in the postprandial state. There are no characteristic findings on computed tomography. Insulin

antibodies are not associated with the condition. These patients require surgery to control symptoms. Usually the surgery entails gradient-guided partial pancreatectomy. No

insulinomas have been found in these patients. This disorder has recently been noted in patients who have had gastric bypass surgery for morbid obesity

Reference

Service FJ, Natt N, Thompson GB, et al. Noninsulinoma pancreatogenous hypoglycemia in adults independent of mutations in Kir6.2 and SUR 1 genes. J Clin Endocrinol Metab.

1999;84:1582-1589 .

Incorrect

Marks for this submission: 0.00/1.00.

Question 5

Marks: 1.00

Which of the following statements is most appropriate regarding the inter-relationship between the drugs listed below and hypoglycemia?

Choose one answer .

A. The effect of angiotensin converting enzyme inhibitors (ACEI) on post hypoglycemic effects of epinephrine release are independent of the type 1

angiotensin II receptor (AT1) action

B. Use of angiotensin-receptor blocking agents and ACEI is associated with a decreased risk of hypoglycemia in patients with type 2 diabetes mellitus on insulin

therapy

qsid=148152&aid=3883&qid=70&checksum=e48666b3c0bfb911b74518105daffb6f&sesskey=9SD9XfnshK&new state=0

Page 108: Asap 2011

C. Measurement of C-peptide is valuable in the differentiation between insulinoma and the surreptitious use of sulfonylurea

D. Diazoxide reverses hypoglycemia by potentiating glucagon release

E. Pramlintide is frequently associated with severe hypoglycemia (≤40 mg/dL) in type 1 diabetic patients

Educational Objective: To understand the complex interaction of pharmacologic agents and endogenous glycemic-related peptides in the causation of hypoglycemia.

Rationale: Both angiotensin-converting enzyme (ACE) inhibitors and angiotensin receptor blocking (ARB) agents may attenuate the post hypoglycemic epinephrine response. This

effect is independent of the type 1 angiotensin II receptor (AT1) action. Studies have shown an increased risk of hypoglycemia in insulin treated patients with type 2 diabetes mellitus on

ACE inhibitors or ARB agents. C-peptide may be elevated in both insulinoma and surreptitious use of sulfonylurea since sulfonylurea causes increased insulin release.

Diazoxide blocks insulin release rather than affecting glucagon release. Hypoglycemia may occur with pramlinitide therapy but severe hypoglycemia is rare in type 1 patients treated

with pramlintide.

References

Worck RH, Frandsen E, Ibsen H, Petersen JS. AT1 and AT2 receptor blockade and epinephrine release during insulin-induced hypoglycemia. Hypertension. 1998;31:384-390.

Palatnick W, Meatherall RC, Tenenbein M. Clinical spectrum of sulfonylurea overdose and experience with diazoxide therapy. Arch Int Med. 1991;151:1859-1862.

Deininger E, Oltmanns KM, Wellhoener P, Fruehwald-Schultes B, Kern W, Heuer B, Dominiak P, Born J, Fehm HL, Peters A. Losartan attenuates symptomatic and hormonal responses to

hypoglycemia in humans. Clin Pharmacol Ther. 2001;70:362-369.

Rubenstein AH, Kuzuya H, Horwitz DL. Clinical significance of C-peptide in diabetes and clinical hypoglycemia. Arch Intern Med. 1977;137:625-629.

Incorrect

Marks for this submission: 0.00/1.00.

Question 6

Marks: 1.00

Page 109: Asap 2011

A 24-year old woman is recovering after delivery of her first child. On awakening she is noted to have an episode of confusion and tachycardia which resolve with orange juice. A venous

blood glucose drawn at the time of symptoms is low at 39mg/dL. Insulin and C-peptide concentrations are undetectable.

Subsequent management should include which of the following?

Choose one answer .

A. Adrenal Ultrasound

B. Sulfonylurea screen

C. Adrenocorticotrophin hormone (ACTH) stimulation test

D. Searching the room for evidence of surreptitious insulin use

E. A 72-hour supervised fast

Educational Objective: Understand the differential diagnosis of hypoglycemia in the post-partum setting.

Rationale: Although extremely uncommon, non-insulin mediated hypoglycemia in the post-partum setting is usually caused by primary or secondary adrenal insufficiency. Given the

absence of β-cell secretion or hyperinsulinemia at the time of hypoglycemia, there will be little benefit in screening for sulfonylureas or insulin use. A 72-hour fast is not indicated as

Whipple's triad (hypoglycemia symptoms, documented hypoglycemia and relief of symptoms with reversal of hypoglycemia) has already been fulfilled and insights into the

mechanism of hypoglycemia clarified. Although adrenal hemorrhage may cause acute adrenal insufficiency, the first step is to document adrenal insufficiency and then determine

whether this is caused by adrenal or pituitary disease.

Reference :

Service FJ. Hypoglycemic disorders. N Engl J Med. 1995;332:1144-1152.

Incorrect

qsid=148153&aid=3883&qid=71&checksum=079c1724ea862d1e781831570869fc12&sesskey=9SD9XfnshK&new state=0

Page 110: Asap 2011

Marks for this submission: 0.00/1.00.

Question 7

Marks: 1.00

A 36-year old woman is referred for the management of insulinoma. She has had hyperinsulinemic hypoglycemia documented on several occasions and an endoscopic

ultrasound performed elsewhere suggests the presence of an islet tumor in the uncinate process. The relationship with food intake is variable. She has had nocturnal symptoms as

well as symptoms occurring during and within an hour of food ingestion. At the time of hypoglycemia, insulin and c-peptide concentrations were elevated. A screen for

tolbutamide, glyburide and glipizide was negative.

Subsequent management should include which ONE of the following?

Choose one answer .

A. Measurement of insulin antibodies

B. Request a mixed meal test

C. Refer for surgery

D. Schedule a selective arterial calcium stimulation test

E. Repeat sulfonylurea screen at the time of hypoglycemia

Educational Objective: Understand the limitations of testing for hypoglycemic agents

Rationale: Nateglinide, repaglinide and 4th generation sulfonylureas such as glimepiride are not routinely tested for by most laboratories performing sulfonylurea screening. Since the β-cell polypeptide pattern is indistinguishable from an insulinoma, the diagnosis depends on a reliable and comprehensive drug screen. In this case, the patient was taking glimepiride (her

father had type 2 diabetes mellitus treated with this medication). Repeat imaging did not show any evidence of a pancreatic tumor. A calcium stimulation test should only be requested if a firm diagnosis of hyperinsulinemic hypoglycemia has been made and

qsid=148154&aid=3883&qid=72&checksum=aad65bbb22f3b0814e701631e9eccfcd&sesskey=9SD9XfnshK&new state=0

Page 111: Asap 2011

localization is uncertain. A mixed meal test may be helpful in provoking postprandial symptoms if this is indeed the temporal pattern of a patient's symptoms.

Reference

1 .Service FJ. Classification of hypoglycemic disorders. Endocrinol Metab Clinic North AM. 1999:28: 501-507.

Incorrect

Marks for this submission: 0.00/1.00.

Question 8

Marks: 1.00

A 76-year old man has a 3 year history of a fibrous mesenchymal tumor arising in the left pleural cavity and spreading locally to involve the ribs and diaphragm. For the past year, he

has had occasional symptoms of hypoglycemia. The patient is admitted for a 72-hour fast which is terminated at 38 hours due to the development of symptomatic hypoglycemia.

Insulin and c-peptide concentrations are undetectable at the time that his glucose is 35mg/dL. Glucagon is administered at the end of the fast .

Which ONE of the following combinations of laboratory results would you expect?

Choose one answer .

A. No change in venous blood glucose and an elevated β-hydroxybutyrate

B. A blood glucose rise greater than 25mg/dL with a suppressed β-hydroxybutyrate

C. A blood glucose rise greater than 25mg/dL with an elevated β-hydroxybutyrate

D. A blood glucose rise less than 25mg/dL with a suppressed β-hydroxybutyrate

E. A blood glucose rise less than 25mg/dL with an elevated β-hydroxybutyrate

Educational Objective: Understand the use of biochemical tests in hypoglycemia

qsid=148155&aid=3883&qid=73&checksum=7d2c532919fa60fdb8567343f10518da&sesskey=9SD9XfnshK&new state=0

Page 112: Asap 2011

Rationale: Fibrous mesenchymal tumors as well as some more common ones such as lymphoma or renal cell carcinoma produce hypoglycemia by secreting an insulin-like factor (often insulin-like growth factor-2 [IGF-2]) which is undetected by insulin assays. However, other biochemical surrogates of insulin or insulin-like activity can help to make the correct

diagnosis. Hepatic production of ketones such as β-hydroxybutyrate are exquisitely sensitive to the presence of insulin and the lack of elevation after 38 hours of fasting would support

the presence of an insulin-like factor. Similarly, glycogen depletion that occurs during fasting is prevented by insulin or insulin-like activity and a rise in venous glucose > 25mg/dL in

response to glucagon is compatible with this process.

Reference:

1 .Service FJ, O'Brien PC. Increasing serum beta-hydroxybutyrate concentrations during the 72 hour fast: evidence against hyperinsulinemic hypoglycemia. J Clin Endocrinol Metab.

2005;90:4555-4558.

Incorrect

Marks for this submission: 0.00/1.00.

Question 9

Marks: 1.00

A 40-year old man is self-referred for the management of hypoglycemia which occurs infrequently and unpredictably. He has experienced a seizure-like episode and is currently

unable to drive. He is admitted for an inpatient fast. The fast was terminated with the development of neuroglycopenic symptoms (confusion, sweating and tachycardia). Venous

blood obtained at this point confirms a glucose value of 32mg/dL. Insulin was elevated at 990µU/ml but C-peptide was undetectable.

Which ONE of the following assessments would you recommend?

Choose one answer .

A. Measurement of insulin antibodies

B. Request a psychiatric consultation

C. Obtain a sulfonylurea screen

qsid=148156&aid=3883&qid=74&checksum=63ef46fc29afe534cb99889c48f26d2c&sesskey=9SD9XfnshK&new state=0

Page 113: Asap 2011

D. Schedule pancreatic imaging studies

E. Obtain a mixed meal study

Educational Objective: Understand the presentation of insulin-induced hypoglycemia

Rationale: Given the biochemistry reported with low blood glucose, elevated insulin levels and undetectable C-peptide, this patient has been taking insulin, most likely, surreptiously.

This is the only differential diagnosis to be entertained in this setting. In some circumstances insulin antibodies can produce similar biochemistry but in that circumstance C-peptide concentrations are usually elevated. Psychiatric consultation is indicated. Sulfonylurea

ingestion is associated with an increase in insulin concentration and proportionate increases in C-peptide and proinsulin. Mixed meal study or pancreatic imaging will not be helpful on

the basis of the data provided and will be negative.

Reference:

Service FJ. Classification of hypoglycemic disorders. Endocrinol Metab Clinic North AM. 1999:28: 501-507.

Incorrect

Marks for this submission: 0.00/1.00.

Question 10

Marks: 1.00

A 34 year-old Asian woman has persistent recurrent episodes of adrenergic symptoms with accompanying confusion. She has had documentation of hypoglycemia at the time of symptoms with markedly elevated C-peptide and insulin concentrations >1000µU/ml.

Thyroid function is unremarkable and the patient has no significant prior medical history. Insulin autoantibodies are present in high titer.

Which ONE of the following treatments is most feasible and reasonable to control symptoms?

Choose one answer .

qsid=148157&aid=3883&qid=75&checksum=63c7b8573790d144fe39902181ce8286&sesskey=9SD9XfnshK&new state=0

Page 114: Asap 2011

A. Methimazole

B. Partial pancreatectomy

C. Losartin

D. Propranolol

E. Clonidine

Educational Objective: Understand the management of hypoglycemia mediated by insulin auto-antibodies.

Rationale: Hypoglycemia caused by insulin-binding antibodies present in high titer is extremely uncommon. However, in Asians (the ethnic group in which it occurs most

frequently) it is associated with thyroid disease and exposure to anti-thyroid medication such as methimazole and carbimazole. Acarbose and partial pancreatectomy can lower

postprandial insulin secretion and in theory decrease severity of symptoms. Intermittent therapy with glucagon may be helpful in preventing hypoglycemic episodes as may enteral

feeding to prevent nocturnal hypoglycemia. Losartin, propanolol, and clonidine are not effective in controlling the symptoms and are not reasonable options in the management of

this hypoglycemic disorder.The occurrence of insulin autoimmunity and hypoglycemia has also been reported in Caucasian patients.

References:

Service FJ. Classification of hypoglycemic disorders. Endocrinol Metab Clinic North AM. 1999:28: 501-507.

Basu A, Service FJ, Yu L et al. Insulin autoimmunity and hypoglycemia in seven white patients. Endocr Pract. 2005:11: 97-103.

Incorrect

Marks for this submission: 0.00/1.00.

Question 11

Marks: 1.00

Page 115: Asap 2011

A 39 year-old man presents with a 5-year history of hypoglycemia. He sought medical attention after his wife noted that he was incoherent on awakening in the morning. On

questioning he reported the insidious onset of such symptoms which could be prevented by snacking, most especially at bedtime. Hyperinsulinemic hypoglycemia is documented after a

supervised fast. Transabdominal ultrasound and a triphasic computed tomography (CT) of the abdomen are negative.

A calcium stimulation test shows the following results:

 

Time MinutesGastroduodenal artery Insulin

Superior Mesenteric artery Concentrations

Splenic Artery µU/mL

0151817

20 4514812

40 8020018

60 12423425

 

At this point, appropriate management would include which ONE of the following?

Choose one answer .

A. Request a genetic test to exclude multiple endocrine neoplasia-1 (MEN 1)

B. Ask the surgeon to consider performing a Whipple procedure with total pancreatectomy

C. Obtain an endoscopic ultrasound to examine the uncinate process

D. Ask the radiologist to repeat the test as this is inconclusive

qsid=148158&aid=3883&qid=76&checksum=5feb1c7582b301d002aebaf9c2107fca&sesskey=9SD9XfnshK&new state=0

Page 116: Asap 2011

E. Consider medical treatment

Educational Objective: Appreciate the pitfalls in the interpretation of the calcium stimulation test.

Rationale: The calcium stimulation test has found favor in the regionalization of insulinoma especially when imaging identifies lesions which may or may not represent a functioning

insulinoma. Sometimes a positive response in more than one territory may suggest the presence of multiple functioning tumors as can occur in multiple endocrine neoplasia-1

(MEN 1). However, in this patient, the positive response in 2 adjacent territories likely occurred because of the tumor's location in a watershed area supplied by 2 adjacent arterial

territories (gastroduodenal and superior mesenteric arteries). Indeed an endoscopic ultrasound confirmed the presence of an islet tumor in the uncinate process. This was

subsequently enucleated successfully.

Reference:

Doppman JL, Chang R, Fraker DL et al. Localization of insulinomas to regions of the pancreas by intra-arterial stimulation with calcium. Ann Intern Med. 1995; 123: 269-273.

Service FJ. Classification of hypoglycemic disorders. Endocrinol Metab Clinic North AM. 1999:28: 501-507.

Incorrect

Marks for this submission: 0.00/1.00.

Question 12

Marks: 1.00

A 44 year-old woman is referred to you for recurrent adrenergic symptoms which occur after eating. She has taken to using a reflectance meter glucose at the time of her symptoms and

has frequently encountered readings in the 40's. The episodes seem to occur within hours of eating and are characterized by confusion, presyncope and tachycardia. Her history is

remarkable for a roux-en-y gastric bypass undertaken six years previously for medically-complicated obesity when she weighed ~ 400lbs. Since then she has lost about 190lbs. She

believes that her symptoms are easily reproduced by consuming ice-cream or pancakes with syrup.

At this time appropriate management would include which One of the following?

qsid=148159&aid=3883&qid=77&checksum=05e95452191ef01c3f5bde3be5b47de9&sesskey=9SD9XfnshK&new state=0

Page 117: Asap 2011

Choose one answer .

A. Surgical consultation

B. An oral glucose tolerance test

C. Acarbose

D. Somatostatin

E. Dietary consultation

Educational Objective: Understand the differential diagnosis of post-prandial symptoms after bariatric surgery.

Rationale: Hypoglycemia has been described after bariatric surgery and occurs in the postprandial setting. Therapeutic interventions that have been attempted include acarbose,

somatostatin or gradient-guided partial pancreatectomy. However, based on the provided information there is no objective documentation of hypoglycemia on which to base

recommendations. An oral glucose tolerance test has little place in the investigation of hypoglycemia especially in the setting of a roux-en-y-gastric bypass where caloric liquids and

foods such as ice-cream are likely to cause the symptoms described. Indeed the roux-en-y-gastric surgery is designed to produce such symptoms if these foods are ingested.

Appropriate dietary counseling is an important part of the care of such patients and is the most reasonable option in the present circumstances. Acarbose and somatostatin are not

effective and are not appropriate options.

Reference:

Service FJ. Classification of hypoglycemic disorders. Endocrinol Metab Clinic North AM. 1999:28: 501-507.

Incorrect

Marks for this submission: 0.00/1.00.

Question 13

Marks: 1.00

Page 118: Asap 2011

A 32 year-old man is referred for evaluation of postprandial symptoms that are characterized by hypotension and presyncope. For 4 months he has been treated with

midodrine for presumed autonomic dysfunction as a cause of his symptoms. However, this treatment has had little effect on his symptoms and during a recent emergency room visit

his venous glucose was 39mg/dL. This was within an hour of eating. He has not had prior abdominal surgery. He is unaware of symptoms that occur during fasting. On examination,

there is no evidence of orthostatism and he has appropriate slowing of his heart rate in response to Valsalva maneuver. A 72-hour fast fails to provoke symptoms. An

adrenocorticotrophin hormone (ACTH) stimulation test is normal .

At this point, which ONE of the following would you order?

Choose one answer .

A. Refer to a neurologist and attribute the abnormal glucose value to laboratory error

B. Obtain an oral glucose tolerance test

C. Obtain repeat glucose determination with insulin and C-peptide levels when symptoms recur

D. Request an endoscopic ultrasound of the pancreas

E. Request a triphasic computed tomography (CT) of the pancreas

Educational Objective: Understand the differential diagnosis of post-prandial hypoglycemia in otherwise healthy subjects.

Rationale: In this patient, there are 2 potential diagnoses that could explain postprandial hypoglycemia despite a negative 72-hour fast. Insulinomas occasionally produce exclusively post-prandial hypoglycemia and non-insulinoma pancreatogenous hypoglycemia syndrome (NIPHS) is also characterized by a negative 72-hour fast with postprandial symptoms. In such circumstances and prior to embarking on imaging studies, it is important to document that the hypoglycemia is insulin mediated and there is no evidence of sulfonylurea or insulin use at the time symptoms occur. Obtaining the appropriate tests at symptom recurrence and / or provoking an episode by means of a mixed meal test is a reasonable way forward. The oral glucose tolerance test produces a glucose nadir < 50mg/dL in about 10% of healthy

qsid=148160&aid=3883&qid=86&checksum=2e9112ee25b13c8c1282957e9989ea16&sesskey=9SD9XfnshK&new state=0

Page 119: Asap 2011

individuals. It has little role to play in the diagnostic evaluation of hypoglycemia. Referral to a neurologist is not indicated at this point without further evaluation of his hypoglycemia. Endoscopic ultrasound of the pancreas and computed tomography (CT) of the pancreas are not indicated or appropriate until the hypoglycemia has been demonstrated to be insulin mediated.

Reference:

1 .Service FJ. Classification of hypoglycemic disorders. Endocrinol Metab Clinic North AM. 1999:28: 501-507

Incorrect

Marks for this submission: 0.00/1.00.

Question 14

Marks: 1.00

The patient described above (item 13) undergoes a mixed meal test which reproduces his symptoms approximately 90 minutes after food ingestion. His glucose is 42mg/dL with

concomitant elevation of insulin and C-peptide concentrations. His ability to perform simple addition and subtraction is restored by intravenous dextrose. Sulfonylurea screen is

negative. He undergoes a transabdominal ultrasound which is negative .

At this point which ONE of the following would be the next best test to evaluate the etiology of this man's hypoglycemia?

Choose one answer .

A. Obtain a selective arterial calcium stimulation test

B. Request surgical consultation with a view to operative pancreatic exploration with or without intraoperative ultrasound

C. Obtain an octreotide scan

D. Request an endoscopic ultrasound of the pancreas

qsid=148161&aid=3883&qid=87&checksum=04ee74fb37961313720115f1c45a88ab&sesskey=9SD9XfnshK&new state=0

Page 120: Asap 2011

E. Request a triphasic computed tomography (CT) of the pancreas

Educational Objective: Understand the relative role of imaging studies in the evaluation of postprandial hypoglycemia.

Rationale: At this point, postprandial hypoglycemia due to hyperinsulinemia has been documented and sulfonylurea use has been excluded as a cause. The differential diagnosis

remains between an insulinoma and non-insulinoma pancreatogenous hypoglycemia syndrome (NIPHS). The latter can be a diagnosis of exclusion. In practice, insulinomas may be

difficult to localize by transabdominal ultrasound because of patient-related factors such as abdominal adiposity. Moreover if the insulinoma lies on the pancreatic surface, it can be

difficult to distinguish from the peripancreatic fat. A recent case series from the Mayo Clinic showed that the combination of computed tomography (CT) and transabdominal ultrasound

correctly localized an insulinoma in ~ 75% of cases. Scintigraphic imaging usually has poor resolution and is not helpful for localizing small insulinomas. Invasive imaging studies would be indicated after negative non-invasive imaging. Given that it is possible to use these tests

to achieve ~ 100% localization prior to operation, most surgeons have abandoned blind operation prior to localization.

Reference :

Placzkowski KA, Vella A, Thompson GB et al. Secular trends in the presentation and management of functioning insulinoma at the Mayo Clinic, 1987-2007. J Clin Endocrinol

Metab. 2009;94:1069-1073.

Incorrect

Marks for this submission: 0.00/1.00.

Question 15

Marks: 1.00

 

The patient (items 13 and 14) undergoes a computed tomography (CT) of the pancreas which is negative. Subsequently an endoscopic ultrasound is requested. This suggests the presence of a small 12mm hypoechoic lesion in the tail of the pancreas and another 5mm

lesion in the head of the pancreas. Since this is inconclusive and of little help to the surgeon, calcium stimulation is performed with the following results:

 

qsid=148162&aid=3883&qid=88&checksum=e8f6acf68ae0715ac7be60057db9c642&sesskey=9SD9XfnshK&new state=0

Page 121: Asap 2011

Time MinutesGastroduodenal Artery Insulin

Superior Mesenteric artery Concentrations

Splenic artery µU/mL

 

0151817

20201435

401922200

601820425

At this point, which ONE of the following conclusions is most appropriate on the basis of the findings?

Choose one answer .

A. The patient most likely has a functioning insulinoma in the tail of the pancreas

B. The findings to date are still inconclusive; an octeoscan should be performed

C. Non-insulinoma pancreatogenous hypoglycemia syndrome (NIPHS) has been excluded

D. The surgeon can limit the inspection of the pancreas to the tail of the pancreas.

E. A Whipple procedure with total pancreatectomy is likely to be necessary

Educational Objective: Interpret the calcium stimulation test correctly .

Rationale: The calcium stimulation test is positive in the splenic territory and suggests that the lesion in the tail is functioning. The response to arterial calcium in non-insulinoma

pancreatogenous hypoglycemia syndrome (NIPHS) usually occurs in multiple territories and is associated with a tripling or quadrupling (at most) of insulin concentrations over baseline. Nevertheless surgical excision and pathological confirmation of an insulinoma together with

the absence of the findings observed in NIPHS is required to exclude the latter diagnosis. The test also suggests that the lesion visualized in the head of the pancreas was an incidental

finding making the likelihood of the surgeon having to perform a Whipple procedure

Page 122: Asap 2011

extremely unlikely. The exploration of the pancreas can not be limited to the tail only as there is apparently a small non-functioning tumor in the head of the pancreas noted on

endoscopic ultrasound.

References:

Doppman JL, Chang R, Fraker DL et al. Localization of insulinomas to regions of the pancreas by intra-arterial stimulation with calcium. Ann Intern Med. 1995; 123: 269-273.

Service FJ. Classification of hypoglycemic disorders. Endocrinol Metab Clinic North AM. 1999:28: 501-507.

Incorrect

Marks for this submission: 0.00/1.00.

Question 16

Marks: 1.00

A 76-year old woman with chronic liver disease is being evaluated for liver transplant. At this time her MELD (model for end-stage liver disease) score is not high enough to require urgent

transplant. Her only complaint is abdominal distension and night cramps, which are treated by over the counter medications. She is now hospitalized with altered mental status. Her

plasma glucose concentration was found to be 32 mg/dL. Her mental status normalized completely after receiving intravenous glucose in the emergency room. Her serum insulin

level from the blood specimen drawn for glucose was later reported to be 20 µU/mL (normal, 2.6-24.9 in the fasting state).

What should be the next step in her management?

Choose one answer .

A. Measure beta-hydroxybutyrate

B. Perform computed tomography (CT) of the abdomen, and if needed selective arterial calcium stimulation test to determine region of pancreas causing

hyperinsulinemia

C. Perform a prolonged (up to 72 hours) fast to determine the cause of

qsid=148163&aid=3883&qid=90&checksum=eb1cb30ed916f3180ff256387eb09c58&sesskey=9SD9XfnshK&new state=0

Page 123: Asap 2011

D. Review the history, including the medication history

Educational Objective: Recognize the importance of history, particularly medication history, in assessing a patient with hypoglycemia.

Rationale: This patient clearly demonstrated Whipple's triad with hypoglycemia symptoms, documented hypoglycemia, and relief of symptoms with correction/treatment of the

hypoglycemia. After treating the patient for hypoglycemia with exogenous glucose, performing a complete medication history is the next most important step in the evaluation

of the patient presenting with hypoglycemia followed by history of medical problems and physical examination. Identification of the cause of the hypoglycemia is important in the evaluation and management of the patient. Only after a careful history concentrating on

medication history and medical problems can a plan of evaluation be devised. Any recent change in medication with a potential temporal relationship to the episode of hypoglycemia

will need to be assessed and the potential culprit medication evaluated for risk of hypoglycemia from previous reports .

The patient was on treatment with an over-the-counter compound containing quinine for night cramps. Quinine has been reported to be associated with increased risk of

hypoglycemia. The patient was advised to discontinue the medication for night cramps. Subsequently, there was no further recurrence of hypoglycemia. The patient's severe liver disease made her susceptible to risk of hypoglycemia from the drug because the diseased

liver is unable to effectively continue gluconeogenesis and clearance of insulin .

Serum insulin and C-peptide levels from the specimen showing hypoglycemia were at the upper limits of normal. If medication history is negative for an agent or medication known to

be associated with hypoglycemia, the medications that are temporally related to the episodes of hypoglycemia should be replaced with alternatives to see if such episodes stop .

If despite all attempts at modifications of medications fail to stop these episodes patient should be evaluated for insulinoma.

References

Service, FJ. Hypoglycemic Disorders. N Engl J Med. 1995;332:1144-1152.

Murad MH, Coto-Yglesias, Wang AT et al. Clinical Review: Drug-induced hypoglycemia: a systematic review. J Clin Endocrinol Metab. 2009;94:741-745.

Incorrect

Marks for this submission: 0.00/1.00.

Question 17

Marks: 1.00

Page 124: Asap 2011

An 87-year old man has been having recent episodes of confusion. During one of these episodes, his laboratory plasma glucose concentration was documented at 29 mg/dL. His

confusion cleared when he ingested orange juice. Insulin level was not obtained. He has an extensive history of cardiac and pulmonary disease and is taking several medications

including propoxyphene which he has recently been started for a sore back .

Which ONE of the following actions should be taken initially to assess whether this medication is causing these episodes of confusion?

Choose one answer .

A. Continue the to observe and hospitalize the patient if confusion recurs to document hypoglycemia

B. Document normalization of glycemia is associated with resolution of confusion

C. Discontinue the propoxyphene and see if episodes of confusion recur

D. Measure plasma insulin, C-peptide and proinsulin with next episode of confusion

Educational Objective: Recognize potential drug-induced hypoglycemia in a patient with multiple medical problems started on propoyphene for pain relief

Rationale: The medication history suggests that a newly added medication to this patient's multiple medications may be causing this hypoglycemia. The medication was propoxyphene,

which has been reported to be associated with hypoglycemia. Propoxyphene was discontinued and the patient had no further episodes of hypoglycemia. Repeating the

glucose level when symptoms recur without stopping the suspected medication of inducing hypoglycemia places this elderly man in jeopardy of significant cerebral, neurological and

cardiovascular complications from a recurrent episode of hypoglycemia. Whipple's triad has already been demonstrated in this patient as described in the vignette. If the hypoglycemia

recurs after stopping propoxyphene, further studies for adrenal and/or pituitary insufficiency and for insulin-mediated hypoglycemia are indicated..

References:

Service, FJ. Hypoglycemic Disorders. N Engl J Med. 1995;332:1144-1152

qsid=148164&aid=3883&qid=92&checksum=626b9f1c6f2720ec3f1b69501d654272&sesskey=9SD9XfnshK&new state=0

Page 125: Asap 2011

Muard MH, Coto-Yglesias, Wang AT et al. Clinical Review: Drug-induced hypoglycemia: a systematic review. J Clin Endocrinol Metab. 2009;94:741-745

Incorrect

Marks for this submission: 0.00/1.00.

Question 18

Marks: 1.00

A 56-year old woman with type 2 diabetes mellitus is being treated with metformin (1000 mg twice a day) and glimepiride (2 mg once a day). She continues to have capillary glucose

measurements above her target of 100-150 mg/dL; her HbA1c was found to be 8.6 %. Based on these measurements, acarbose was added to her therapy and the dose was increased

over several weeks to the present dose of 100 mg three times a day. Her husband calls you and tells you that she is having hypoglycemia, as suggested by the results of fingerstick

capillary blood glucose of 52 mg/dL. He was advised to follow the "Rule of 15." He gave her 4 oz of orange juice and waited for 15 minutes, but the blood glucose did not improve. He is

calling you for further advice .

The best option for management of her hypoglycemia at this point is which one of the following?

Choose one answer .

A. Continue to observe; her glucose rise in response to the orange juice is delayed

B. It does not matter what she takes, as long as she takes 15 g of carbohydrate and waits for 15 more minutes

C. Take 15 grams of glucose and wait for 15 more minutes

D. Take 15 grams of honey and wait for 15 more minutes

E. Take one slice of bread and wait for 15 more minutes

qsid=148165&aid=3883&qid=96&checksum=45166dae7dfae92bb56657899f6577bd&sesskey=9SD9XfnshK&new state=0

Page 126: Asap 2011

Educational Objective: Recognize the mechanism of action of acarbose and other α-glucosidase inhibitors when treating diabetic patients with hypoglycemia .

Rationale: α-glucosidase inhibition therapy blocks the effectiveness of complex carbohydrates to reverse hypoglycemia and its symptoms. Table sugar (sucrose) is a dimmer,

which needs to be acted upon by sucrase (an α-glucosidase) to release glucose for relief of hypoglycemia. The initial orange juice did not provide enough glucose for reversal of the

hypoglycemia in the patient described probably due to the α-glucosidase inhibition. Table-sweetened orange juice will not be effective because the sucrase for the breakdown of

sucrose to glucose is blocked by the α-glucosidase inhibitor, acarbose. Therefore, the best recommendation for treatment of the persistent hypoglycemia in this patient is 15 grams of

glucose, which bypasses theα-glucosidase inhibition. Honey, bread, other complex carbohydrate will not reverse her hypoglycemia for the reasons stated above. Continued

observation without further treatment could result in significant persistent hypoglycemia with significant risk of mental impairment, cerebral dysfunction and deficit, neurological

deficit, and cardiovascular complications.

References:

1 .Service, FJ. Hypoglycemic Disorders. N Engl J Med. 1995;332:1144-1152.

2 .Muard MH, Coto-Yglesias, Wang AT et al. Clinical Review: Drug-induced hypoglycemia: a systematic review. J Clin Endocrinol Metab. 2009;94:741-745.

Incorrect

Marks for this submission: 0.00/1.00.

Question 19

Marks: 1.00

A baby presents with hypoglycemia and acidosis. A metabolic disorder is suspected and urine amino acids are measured. The baby is found to have elevated urinary concentrations

of leucine, valine, and isoleucine.

What is the most likely diagnosis?

Choose one answer .

A. Hereditary tyrosinemia

qsid=148166&aid=3883&qid=108&checksum=94cf198d4d0f752c9658bf93c7d95a95&sesskey=9SD9XfnshK&new state=0

Page 127: Asap 2011

B. Methylmalonic acidemia

C. 3-hydroxy-3-Methyl glutaric acidemia

D. Familial leucine-sensitive hypoglycemia

E. Maple syrup urine disease

Educational Objective: Recognize the aminoaciduria characteristics of maple syrup urine disease.

Rationale: Hereditary tyrosinemia, methylmalonic acidemia, 3-hydroxy-3-methyl glutaric acidemia, familial leucine-sensitive hypoglycemia and maple syrup urine disease are

metabolic disorders associated with hypoglycemia, but only maple syrup urine disease leads to elevated urinary and plasma concentrations of leucine, valine, and isoleucine.

References

Chuang DT. Maple syrup urine disease: it has come a long way. J Pediatr. 1998;132:S17-23

Rezvani I, Rosenblatt DS. Valine, leucine, isoleucine, and related organic acidemias. In: Textbook of Pediatrics, 16th edition, Behrman RE, Kliegman RM, Jenson HB (Eds), W.B.

Saunders Company, Philadelphia, 2000. p 354-362.

Shih VE. Detection of hereditary metabolic disorders involving amino acids and organic acids. Clin Biochem. 1991;24:301-309.

Incorrect

Marks for this submission: 0.00/1.00.

Question 20

Marks: 1.00

An infant with a family history of maturity-onset diabetes of the young (MODY) presents with macrosomia and transient hyperinsulinemic hypoglycemia.

A mutation in which of the following genes is most likely responsible for this infant's condition?

qsid=148167&aid=3883&qid=104&checksum=4a11e7d31690f7a88b8f6339c71dc382&sesskey=9SD9XfnshK&new state=0

Page 128: Asap 2011

Choose one answer .

A. ABCC8

B. KCJN11

C. HNF4A

D. GLUD1

E. Glucokinase

Educational Objective: Recognize gene mutations associated with hypoglycemia and their clinical characteristics.

Rationale: Mutations in the genes of ABCC8, KCJN11, HNF4A, GLUD1, and glucokinase are associated with hypoglycemia. However, only mutations in HNF4A gene are associated with

a positive family history of MODY.

References:

James C, Kapoor RR, Ismail D, et al. The genetic basis of congenital hyperinsulinism. J Med Genet. 2009;46:289-299.

Kapoor R, Locke J, Colclough K, et al. Persistent hyperinsulinaemic hypoglycemia and maturity onset diabetes of the young (MODY) due to heterozygous HNF4A. Diabetes

2008;57:1659-1663.

Pearson ER, Boj SF, Steele AM, et al. Macrosomia and hyperinsulinaemic hypoglycemia in patients with heterozygote mutations in the HNF4A gene. PLoS Med. 2007; 4:e 118.

Incorrect

Marks for this submission: 0.00/1.00.

Question 21

Marks: 1.00

qsid=148168&aid=3883&qid=107&checksum=f562dc5764bee63c2fbe10076c8060ac&sesskey=9SD9XfnshK&new state=0

Page 129: Asap 2011

A 17-year old female with a history of depression presents with headache, sweating, and tachycardia. A central blood glucose is measured and is found to be 48 mg/dL. The patient's grandmother has type 2 diabetes and you suspect that the patient has taken an overdose of

her grandmother's glyburide.

What pattern of laboratory results would you expect in this patient?

Choose one answer .

A. A low concentration of insulin and a high concentration of C-peptide

B. A high concentrations of insulin with a low concentration of C-peptide

C. A high concentration of insulin and a high concentration of C-peptide

D. A low concentration of insulin and a low concentration of C-peptide

E. Normal concentrations of both insulin and C-peptide

Educational Objective: Recognize the laboratory characteristics of sulfonylurea-induced hypoglycemia.

Rationale: Glyburide is a sulfonylurea that causes insulin release from the pancreas. Because this insulin is endogenously produced, elevated insulin and C-peptide levels would be

expected in this patient with suspected sulfonylurea ingestion. In the patient taking exogenous insulin leading to hypoglycemia, elevated insulin levels but low C-peptide levels

would be noted. Low concentration of insulin with high C-peptide levels is not likely to occur since insulin and C-peptide are secreted simultaneously in equimolar fashion. The patient

could be taking C-peptide injections, however this is a highly unlikely clinical scenario. Low concentrations of insulin and C-peptide are seen in the fasting state.

References

Marks V. Hypoglycaemia-Real and unreal, lawful and unlawful: the 1994 Banting lecture. Diabetic Med. 1995;12:850-864.

Horwitz DL. Factitious and artifactual hypoglycemia. Endocrinol Clin North Am. 1989:18:203-241.

Seltzer HS. Drug-induced hypoglycemia. A review of 1418 cases. Endocrinol Clin North Am. 1989;18:163-183.

Incorrect

Page 130: Asap 2011

Marks for this submission: 0.00/1.00.

Reminder:

You have three (3) attempts to successfully complete this assessment with a 75% score or higher.

You have 30 days to complete each attempt, once an attempt is started.

If you need to save your answers and come back to the assessment, you must click the 'Next' button at the bottom-left of the assessement prior to logging

out of ASAP.

Only click 'Submit all and finish' once you are satisfied with all your assessment answers.

Bottom of Form

Finish review

Skip Quiz navigation

Quiz navigation

i (Closed ) 1 (Incorrect ) 2 (Incorrect ) 3 (Incorrect ) 4 (Incorrect ) 5 (Incorrect ) 6 (Incorrect ) 7 (Incorrect ) 8 (Incorrect ) 9 (Incorrect ) 10 (Incorrect ) 11 (Incorrect ) 12 (Incorrect ) 13

(Incorrect ) 14 (Incorrect ) 15 (Incorrect ) 16 (Incorrect ) 17 (Incorrect ) 18 (Incorrect ) 19 (Incorrect ) 20 (Incorrect ) 21 (Incorrect ) i (Closed )

Finish review

You are logged in as Mohammed Aldawish (Logout)

HYPO

 

Copyright: 2011, American Association of Clinical Endocrinologists (AACE).All material published on the Web site is the property of AACE and may not be reproduced in

any form or by any electronic meansincluding information, storage and retrieval systems without the written consent of AACE.

This activity is sponsored by the American Association of Clinical Endocrinologists.

Skip to main content

Page 131: Asap 2011

Reproductive Endocrinology Disorders

You are logged in as Mohammed Aldawish (Logout)

Page path

Home

/ ► My courses

/ ► REPRO

/ ► Assessment

/ ► Reproductive Endocrinology Disorders Assessment

/ ► Review of attempt 1

Review of attempt 1

Started onTuesday, 19 November 2013, 10:31 PM

Completed onTuesday, 19 November 2013, 10:32 PM

Time taken21 secs

Grade0.00 out of a maximum of 31.00 (0%)

FeedbackFailed

Top of Form

Directions: In this section, each item includes a lead question and a list of options labeled with letters. Select the ONE lettered option that is BEST in each case by selecting the radio

button corresponding to the correct answer .

Question 1

Marks: 1.00

9SD9XfnshK

qsid=148171&aid=3884&qid=286&checksum=290bc132de03221161acd205727efd4a&sesskey=9SD9XfnshK&new state=0

Page 132: Asap 2011

A 32-year-old man and his wife (aged 31 years) present with a history of 24 months of secondary infertility. Both had been fertile in their 20s. On initial evaluation, semen analysis

showed 2.0 mL with 12 X 106 sperm/mL (normal, >20 million/mL), 50% motility, and 40% normal morphology. Due to the lower sperm count, a random endocrine profile was

ordered, which showed modest reduction in gonadotropins and low total testosterone. The husband smokes 20 cigarettes daily and is employed as a long-distance truck driver. His

maternal grandmother and a paternal uncle have type 2 diabetes. His father had a myocardial infarction at age 59 years. During a review of symptoms, he complained of

marked reduction of libido .

Physical examination showed a body mass index of 33 kg/m2 and blood pressure of 146/90 mm Hg; the testes were normal in size, and no scrotal masses were palpable .

Which is the most appropriate next step in evaluating this couple's infertility?

Choose one answer .

A. Supplemental androgen replacement therapy with testosterone

B. Repeat free testosterone and sex hormone—binding globulin (SHBG) in the morning

C. Perform trans-scrotal ultrasound for better evaluation of possible subclinical varicocele

D. Clomiphene plus letrozole therapy to augment gonadotropin stimulation of spermatogenesis

E. Perform a hysterosalpingogram (HSG) on the wife to screen for tubal obstruction

Educational Objective: To recognize the proper clinical way to screen for hypogonadism

Rationale: Screening for male hypogonadism should occur with levels drawn in the morning due to the diurnal pattern of testosterone secretion. Exogenous androgens depress the

hypothalamic-pituitary-gonadal axis and can worsen spermatogenesis. Clomiphene citrate therapy has been in and out of favor, but clear evidence of its effectiveness is lacking. Early

results are encouraging on the use of aromatase inhibitors, which may improve estradiol/testosterone ratio and spermatogenesis, but it is too early to make

recommendations for its use. Doppler studies can improve the diagnostic accuracy of a physical examination, but correction of the subclinical varicocele — some reports state any

varicocele — is of limited value in the treatment of infertility. The decision for varicocele repair should take into consideration its size, symptoms, age of female partner, and other fertility etiologies. Alterations in semen analysis can be a harbinger of underlying medical

Page 133: Asap 2011

problems that need evaluation and therapy above and beyond fertility issues. Although this man's general health may or may not be the cause of the couple's infertility, evaluation is

indicated. Other findings may be potentially modifiable risk factors, including smoking and the occupational risk of increased stress or scrotal temperature. Changing the type of male

underwear from briefs to boxers has not been shown to improve sperm counts. Although ovulation testing is in order, unless there is a strong history of pelvic disease, anatomic

testing other than screening ultrasonography can be postponed .

References

Vandekerckhove P, Lilford R, Vail A, et al. Androgens versus placebo or no treatment for idiopathic oligo/asthenospermia. Cochrane Database Syst Rev. 2000; (2):CD000150.

Raman JD, Schlegel PN. Aromatase inhibitors for male infertility. J Urol. 2002; 167:624-629.

Evers JL, Collins JA. Assessment of efficacy of varicocele repair for male subfertility: a systematic review. Lancet. 2003; 361:1849-1852.

Kapoor D, Malkin CJ, Channer KS, et al. Androgens, insulin resistance, and vascular disease in men. Clin Endocrinol. 2005; 63:239-250.

Incorrect

Marks for this submission: 0.00/1.00.

Question 2

Marks: 1.00

Which method of tracking ovulation is a better retrospective indicator, rather than prospective indicator?

Choose one answer .

A. Basal body temperature (BBT) tracking

B. Estradiol level

C. Ovulation predictor kits that monitor luteinizing hormone

qsid=148172&aid=3884&qid=290&checksum=e4b30d7976d9f6b9b63098b4e5ff7d5b&sesskey=9SD9XfnshK&new state=0

Page 134: Asap 2011

D. Cervical mucus

Educational Objective: To evaluate methods of ovulation detection for conception and prospective pregnancy

Rationale: The 0.5°F to 1.0°F increase in body temperature that occurs from the thermogenic effect of progesterone may indicate that ovulation has occurred in the previous 24 to 30

hours. Usually a pattern is established within 3 cycles, and basal body temperature can be used as a retrospective indicator of ovulation. Cervical mucus changes and ovulation kits precede ovulation. Serum estradiol is not used to predict ovulation; serum progesterone

approximately 7 days postovulation can be used to detect egg release.

Reference

Stanford JB, White GL, Hatasaka H. Timing intercourse to achieve pregnancy: current evidence. Obstet Gynecol. 2002; 100:1333-1341.

Incorrect

Marks for this submission: 0.00/1.00.

Question 3

Marks: 1.00

A 37-year-old woman, with a recent early first-trimester spontaneous pregnancy loss, presents for fertility counseling. It had taken about 6 months to establish this first pregnancy

after stopping oral contraceptives, which she had used since her marriage 4 years earlier. Her periods, which had been 28 to 29 days apart for most of her reproductive life, are now

26 to 27 days apart. She thinks she is ovulating, but no specific testing has been performed. She and her husband are busy professionals who have intercourse once or twice weekly, mostly on weekends. Both are in excellent health and have negative medical and family histories. It was suggested by her primary care physician (PCP) that there was "no rush".

Which statement describes the best course of action for this woman at the current stage of counseling?

Choose one answer .

A. Agree with her PCP and ask her to return in 6 months if a pregnancy has not been achieved

qsid=148173&aid=3884&qid=293&checksum=ad07c651a8c410ef9c443c9116f2ac6d&sesskey=9SD9XfnshK&new state=0

Page 135: Asap 2011

B. Order a karyotype, at least on the female partner, to ensure lack of a transmissible chromosomal disorder

C. Order tests for lupus-like anticoagulant and antiphospholipid antibodies to exclude immunologic causes of pregnancy loss

D. Conduct blood testing on cycle day 2 or 3 for follicle-stimulating hormone and estradiol

Educational Objective: To emphasize the importance of reproductive aging in the context of infertility

Rationale: Although the embryo lost in the initial pregnancy had a greater than 50% chance of a genetic abnormality, this is more a statistical occurrence than a rationale for maternal or

paternal karyotyping in the absence of a specific family history. Even though phospholipid antibody syndrome can result in severe fetal growth restriction, its relationship to first-

trimester losses is tenuous. Proactive testing to identify major barriers to fertility should be performed due to the relatively limited time remaining to affect therapy. These can include

semen analysis and baseline ultrasound scan to identify pelvic pathology such as ovarian cysts, adnexal changes, and fibroids, and, given her age, should include baseline follicle-stimulating hormone (FSH) and estradiol determinations. The most common reason for

pregnancy loss is egg quality. With aging, not only does the number of oocytes decline but quality also decreases. A marked and accelerated decline in fertility occurs at age 37 to 38.

Age may represent the greatest barrier to successful fertility therapy for this couple. A delay of 6 months or more could have profound consequences for their possibility of conception. Although a more conservative approach may be taken in terms of therapy, a determination

of ovarian reserve (egg stores) is important. As follicle/egg stores decline, inhibin levels increase, resulting in increased FSH levels. Higher FSH levels accelerate follicle growth, as

evidenced by increased basal estradiol levels with resultant shortening of menstrual cycle length. While pregnancies are possible even with high basal FSH levels, even modest

elevations are associated with significant decline in fertility and, specifically, response to controlled ovarian stimulation. In addition to determining FSH on cycle day 2 or 3, an

estradiol level is necessary to validate basal conditions, and it is an independent marker of eggs stores.

References:

2003 National Summary and Fertility Clinic Report. Available at: http://www.cdc.gov.ART/index.htm.

Macklon NS, Fauser BC. Ovarian reserve. Semin Reprod Med. 2005; 23:248-256.

Incorrect

Marks for this submission: 0.00/1.00.

Page 136: Asap 2011

Question 4

Marks: 1.00

Which clinical feature is most likely to be associated with testosterone deficiency?

Choose one answer .

A. Cold intolerance

B. Reduced muscle bulk

C. Decreased body fat

D. Erythrocytosis

E. Decreased incidence of bone fractures

Educational Objective: To recognize the signs and symptoms of testosterone deficiency

Rationale: Signs and symptoms of testosterone deficiency include hot flashes and episodes of sweating (seen in patients with primary hypogonadism), reduced muscle bulk, and

increased body fat. In addition, low-trauma fracture and osteoporosis in a man should alert one to the possibility of low testosterone. The incidence of fractures, including compression

fractures of the vertebrae and long bones, is increased. Cold intolerance is not usually associated with testosterone deficiency unless hypothyroidism is present. Erythrocytosis, on

the other hand, may be seen as an adverse effect of testosterone treatment and can potentially cause a stroke or heart attack in men who have underlying vascular disease.

Reference:

Petak SM, Nankin HR, Spark RF, et al. American Association of Clinical Endocrinologists Medical Guidelines for clinical practice for the evaluation and treatment of hypogonadism in

adult male patients — 2002 update. Endocr Pract. 2002; 8:440-456.

Incorrect

Marks for this submission: 0.00/1.00.

qsid=148174&aid=3884&qid=295&checksum=58a835b9147e8c454ce474cd25f5a811&sesskey=9SD9XfnshK&new state=0

Page 137: Asap 2011

Question 5

Marks: 1.00

Which condition or medication is known to increase the level of sex hormone—binding globulin (SHBG)?

Choose one answer .

A. Obesity

B. Hypothyroidism

C. Glucocorticoids

D. Anticonvulsant drugs

E. Nephrotic syndrome

Educational Objective: To recognize which conditions and medications may be associated with elevated levels of SHBG

Rationale: Obesity, hypothyroidism, and glucocorticoids decrease SHBG synthesis by the liver. Nephrotic syndrome allows increased SHBG excretion by the kidney. Several

anticonvulsant drugs have been reported to increase SHBG synthesis by the liver.

Reference:

Petak SM, Nankin HR, Spark RF, et al. American Association of Clinical Endocrinologists Medical Guidelines for clinical practice for the evaluation and treatment of hypogonadism in

adult male patients — 2002 update. Endocr Pract. 2002; 8:440-456.

Incorrect

Marks for this submission: 0.00/1.00.

Question 6

Marks: 1.00

qsid=148175&aid=3884&qid=297&checksum=71c59b4f3addbfda7f022b740fcb3840&sesskey=9SD9XfnshK&new state=0

Page 138: Asap 2011

A 60-year-old male teacher and coach presented to his primary care provider complaining of erectile dysfunction and some reduction in libido, energy, and stamina. He was found to

have an early morning total testosterone level of 230 ng/dL (normal, 241-827 ng/dL), a luteinizing hormone (LH) level of 4 mIU/mL (normal, 1-9 mIU/mL), and a prolactin level of 6 ng/mL (normal, 3.0-30.0 ng/mL). Two years earlier, his prostate-specific antigen (PSA) level

was 6 ng/mL (normal, <4 ng/mL), and prostate biopsies were positive for prostate cancer. He had a nerve-sparing radical prostatectomy.

His body mass index is 26 kg/m2 and his physical examination findings are normal, but his testes had some reduction in firmness. A repeat early-morning testosterone level was 215

ng/dL (normal, 241-827 ng/dL), sex hormone—binding globulin (SHBG) level was 22 nmol/L (normal, 13-71 nmol/L), calculated free testosterone was 4.0 ng/dL (normal, 6.0-27.0 ng/dL),

and the highly sensitive PSA level was undetectable. His complete metabolic profile and complete blood count were normal.

Which action is most appropriate for managing this man's presenting symptoms?

Choose one answer .

A. Provide him with a prescription for sildenafil (Viagra)

B. Obtain magnetic resonance imaging (MRI) of the pituitary and hypothalamus

C. Advise him that his symptoms are not due to his low testosterone level

D. Discuss testosterone treatment with the patient and his urologist

E. Review the pathology report to learn if surgical margins were clear of cancer and how much of the resected prostate contained malignant cells

Educational Objective: Management of the patient with prostate cancer and erectile dysfunction

Rationale: In older men, low testosterone levels generally are not due to anatomical disease involving the hypothalamus or pituitary; therefore, recognition, evaluation, and treatment

considerations are different for older men. This patient has a history of prostate cancer, which probably has been cured by radical prostatectomy. Although erectile dysfunction is common in men who have had a radical prostatectomy, they usually do not have reduced

qsid=148176&aid=3884&qid=299&checksum=9e43369a23f285b09460e53f21ac169e&sesskey=9SD9XfnshK&new state=0

Page 139: Asap 2011

libido, decreased energy, and reduced total and calculated free testosterone levels. An assessment of depression is appropriate because of his low testosterone. The patient's

urologist should be consulted to determine the patient's prognosis and the potential benefits and risks of testosterone treatment. The surgical margins, extent of disease, time

from surgery, and PSA level should be considered. If all agree that the benefit:risk ratio appears favorable, then testosterone treatment can be initiated, but PSA levels must be

monitored more frequently, in addition to the usual measures of monitoring testosterone treatment in men of this age. The patient may still need a phosphodiesterase type 5

inhibitor for erectile dysfunction, but testosterone therapy would alleviate the symptoms of reduced libido, decreased energy, and associated depression. Therefore, the most

appropriate action for managing this man's presenting symptoms is consideration of testosterone therapy.

References:

Petak SM, Nankin HR, Spark RF, et al. American Association of Clinical Endocrinologists Medical Guidelines for clinical practice for the evaluation and treatment of hypogonadism in

adult male patients — 2002 update. Endocr Pract. 2002; 8:440-456.

Calof OM, Singh AB, Lee ML, et al. Adverse events associated with testosterone replacement in middle-aged and older men: a meta-analysis of randomized, placebo-controlled trials. J

Gerontol A Biol Sci Med Sci. 2005; 60:1451-1457.

Incorrect

Marks for this submission: 0.00/1.00.

Question 7

Marks: 1.00

Which finding is common in polycystic ovary syndrome (PCOS)?

Choose one answer .

A. Acanthosis nigricans

B. Vitiligo

qsid=148177&aid=3884&qid=301&checksum=7ee13f208415738ce734d18b4c7eca3e&sesskey=9SD9XfnshK&new state=0

Page 140: Asap 2011

C. Muscle weakness

D. Purple striae

E. Hypoinsulinemia

Educational Objective: To review the clinical features of PCOS, insulin resistance, and other conditions association with anovulation and hyperandrogenism

Rationale: The clinical features of insulin resistance include skin tags, acanthosis nigricans, and abdominal obesity. Acne is related to androgen-mediated increased sebum production,

and the sporadic finding of estrogenic cervical mucus is consistent with the patient's anovulatory state, a situation associated with increased risk of endometrial hyperplasia and cancer. The finding of muscle weakness and purple striae in such a patient should alert one

to the possibility of Cushing's syndrome and a corticotroph adenoma. Vitiligo and hypoinsulinemia are not common in PCOS.

References :

Ehrmann DA. Polycystic ovary syndrome. N Engl J Med. 2005; 352:1223-1236.

Azziz R, Woods KS, Reyna R, et al. The prevalence and features of the polycystic ovary syndrome in an unselected population. J Clin Endocrinol Metab. 2004; 89:2745-2749.

Incorrect

Marks for this submission: 0.00/1.00.

Question 8

Marks: 1.00

Which statement is correct about the medical treatment of polycystic ovary disease (PCOS)?

Choose one answer .

A. Metformin is the treatment of choice for infertility in PCOS

qsid=148178&aid=3884&qid=304&checksum=4ea3cf9289e98a6c85c675e916c74706&sesskey=9SD9XfnshK&new state=0

Page 141: Asap 2011

B. Antiandrogens (e.g., spironolactone, flutamide) are useful for treatment of infertility in PCOS

C. Oral contraceptive treatment increases hepatic sex hormone—binding globulin (SHBG) production, reducing free androgen levels

D. Thiazolidinediones (TZDs) are the most commonly prescribed insulin-sensitizing drugs in PCOS

E. Glucocorticoids are frequently used to treat insulin resistance in PCOS

Educational Objective: To review the medical treatment of polycystic ovary syndrome (PCOS)

Rationale: Oral contraceptive treatment increases hepatic sex hormone—binding globulin (SHBG) production and reduces free androgen levels in PCOS

Metformin is commonly used to treat infertility in PCOS, because it frequently results in ovulation but current clinical trials have indicated that metformin is not considered to be an

effective primary therapy for infertility in PCOS. Metformin is the most commonly prescribed insulin-sensitizing drug in PCOS.

Antiandrogens are potentially teratogenic and are not used to treat infertility. Glucocorticoid treatment does not improve insulin resistance in PCOS.

References

Palomba S, Falbo A, Zulio F, et al. Evidence-based and potential benefits of metformin in PCOS: a comprehensive review. Endocr. Rev. 2009; 30:1-50.

Korytkowski MT, Mokan M, Horwitz MJ, et al. Metabolic effects of oral contraceptives in women with PCOS. J Clin Endocrinol Metab. 1995; 80:3327-3334.

Incorrect

Marks for this submission: 0.00/1.00.

Question 9

Marks: 1.00

qsid=148179&aid=3884&qid=306&checksum=524b9728cbac51eb2cfebe521c5d44a5&sesskey=9SD9XfnshK&new state=0

Page 142: Asap 2011

The metabolic syndrome secondary to insulin resistance in polycystic ovary syndrome (PCOS) is frequently associated with which finding?

Choose one answer .

A. Fatty liver

B. Increased high-density lipoprotein (HDL) cholesterol

C. Hypokalemia

D. Reduced triglycerides

E. Decreased intra-abdominal visceral fat

Educational Objective: To review the metabolic syndrome secondary to insulin resistance in PCOS

Rationale: The metabolic syndrome includes glucose intolerance, hyperinsulinemia, elevated blood pressure, increased waist circumference, reduced HDL cholesterol, elevated

triglyceride levels, fatty liver, and increased intra-abdominal visceral fat. Hypokalemia is not associated with the metabolic syndrome.

Reference

Grundy SM, Cleeman JI, Daniels SR, et al; American Heart Association; National Heart, Lung, and Blood Institute. Diagnosis and management of the metabolic syndrome: an American

Heart Association / National Heart, Lung and Blood Institute Scientific Statement. Circulation. 2005; 112:2735-2752.

Incorrect

Marks for this submission: 0.00/1.00.

Question 10

Marks: 1.00

qsid=148180&aid=3884&qid=309&checksum=eed1e573688a7a6aa23ba8e6824fcbb5&sesskey=9SD9XfnshK&new state=0

Page 143: Asap 2011

Which diagnosis is associated with elevated concentrations of serum follicle-stimulating hormone (FSH)?

Choose one answer .

A. Craniopharyngioma

B. Müllerian agenesis

C. Lymphocytic hypophysitis

D. Autoimmune oophoritis

E. Hypogonadotropic hypogonadism

Educational Objective: To review the causes of gonadal failure

Rationale: Both Turner's syndrome and XY gonadal agenesis (Swyer syndrome) are associated with accelerated germ cell loss, germ cell deficiency, and gonadal failure, and

hence high serum FSH (lack of inhibin). In autoimmune oophoritis, the inflammatory process may destroy germ cells and the ovarian follicular apparatus, and thus inhibin, leading to high

FSH concentrations. Women harboring the fragile X premutations (associated with mental retardation in male offspring) may develop premature ovarian failure and high FSH levels.

Women with Müllerian agenesis almost always have normally functioning ovaries; their FSH level is thus normal. Craniopharyngioma and lymphocytic hypophysitis are associated with

hypopituitarism and low FSH levels. Hypogonadotropic hypogonadism is, by definition, associated with low FSH levels.

Reference

Rebar RW, Connolly HV. Clinical features of young women with hypergonadotropic amenorrhea. Fertil Steril. 1990; 53:804-810.

Incorrect

Marks for this submission: 0.00/1.00.

Question 11

Marks: 1.00

Page 144: Asap 2011

Which disorder is most commonly associated with amenorrhea?

Choose one answer .

A. Kallmann's syndrome

B. Hypokalemia

C. Hypercalcemia

D. Empty sella

Educational Objective: To review the hypothalamic-pituitary causes of amenorrhea

Rationale: Kallmann's syndrome is hypothalamic hypogonadism in association with anosmia. Although more common in men, who can have X-linked disease as well as autosomal

dominant and recessive forms, it also occurs in women. Hyperprolactinemia is often related to medications, in particular neuroleptics. Craniopharyngioma is a developmental lesion, a

tumor of the remnants of Rathke's pouch. Both prior pituitary radiation and lymphocytic hypophysitis (an autoimmune disease that often occurs postpartum) may be associated with

hypopituitarism. Empty sella represents an anatomic variation and is rarely associated with endocrine dysfunction or amenorrhea. Hypokalemia and hypercalcemia, per se, are not

associated with amenorrhea.

Reference:

Yazigi RA, Quintero CH, Salameh WA. Prolactin disorders. Fertil Steril. 1997; 67:215-225.

Incorrect

Marks for this submission: 0.00/1.00.

Question 12

Marks: 1.00

qsid=148181&aid=3884&qid=312&checksum=9c9103575b18cb643bce781b47d6bb5d&sesskey=9SD9XfnshK&new state=0

Page 145: Asap 2011

Which statement concerning the menstrual cycle and amenorrhea is correct?

Choose one answer .

A. Luteinizing hormone (LH) induces the aromatization of androgens to estrogen

B. Follicle-stimulating hormone (FSH) surge occurs through the positive feedback effect of estradiol on the anterior pituitary

C. A karyotype study is required in an 18 year old who has amenorrhea and high FSH concentrations

D. Primary hypothyroidism paradoxically causes chronic ovulation and amenorrhea

E. Anorexia nervosa is associated with low endorphins

Educational Objective: To identify the mechanisms for amenorrhea associated with abnormalities of the menstrual cycle

Rationale: In women, it is FSH that induces aromatization of androgens to estrogen — not LH. In men, LH stimulates aromatization of androgens .

The surge in LH does occur through estrogen positive feedback. Gonadal failure (high FSH) in an 18 year old necessitates karyotyping, because the possibility of finding a Y-line exists, and this carries an increased risk of gonadal cancer; the streak gonads should be removed if a Y-

line is found. Primary hypothyroidism may lead to chronic anovulation, possibly through altering the metabolism of sex steroids. One of the mechanisms for amenorrhea in anorexia

nervosa relates to stress and high endorphins.

Reference

Berga SL. Behaviorally induced reproductive compromise in women and men. Semin Reprod Endocrinol. 1997; 15:47-53.

Incorrect

Marks for this submission: 0.00/1.00.

qsid=148182&aid=3884&qid=313&checksum=fd5f6b6c18f84e978dcc31ab9236720e&sesskey=9SD9XfnshK&new state=0

Page 146: Asap 2011

Question 13

Marks: 1.00

A 25-year-old female marathon runner has suffered a repeat fracture of her right foot. The last fracture occurred less than 6 months ago. She has been amenorrheic for the past 2 years

while training. She is 5 feet 3 inches tall and weighs 110 pounds. A dual energy x-ray absorptiometry (DEXA) scan shows a T-score of -2.7 of the lumbar spine and T-score of -1.8

at the femoral neck.

Which is the most appropriate next step in the management of this patient?

Choose one answer .

A. Human parathyroid hormone

B. Bisphosphonate therapy

C. Combination oral contraceptives with oral calcium and vitamin D

D. Cyclic progesterone therapy

E. Restricted activity plus weight gain

Educational Objective: Recognize low bone mass and athletic amenorrhea

Rationale: Intensive athletic training can lead to disruption of gonadotropin-releasing hormone (GnRH) secretion, with resultant amenorrhea and low estrogen levels. The latter

may predispose to bone loss of 1% to 2% per year, with increased risk of fracture even in reproductive-age women. The DEXA findings are consistent with osteoporosis, and the best

management would be an antiresorptive agent such as oral contraceptives with the addition of calcium and vitamin D. Bisphosphonates are contraindicated in women of childbearing

age due to theoretical risk of harm to the fetus. Recombinant parathyroid hormone, cyclic progesterone, or weight gain would not be the best choice based on the clinical

presentation.

Reference:

qsid=148183&aid=3884&qid=314&checksum=e2ae4fbde31bfdeeecbaaae5e82f7265&sesskey=9SD9XfnshK&new state=0

Page 147: Asap 2011

Laughlin GA, Yen SS. Nutritional and endocrine-metabolic aberrations in amenorrheic athletes. J Clin Endocrinol Metab. 1996; 81:4301-4309.

Incorrect

Marks for this submission: 0.00/1.00.

Question 14

Marks: 1.00

A 19-year-old woman comes to the clinic for evaluation of secondary amenorrhea. Her medical history is negative. She reports 2 previous cycles at age 14. She is 5 feet 3 inches tall and weighs 155 pounds. Physical examination shows Tanner stage 3 for breast development

and Tanner stage 4 for pubic hair. Ultrasonography demonstrates a normal uterus with cervix, but no ovaries can be visualized. Her follicle-stimulating hormone (FSH) level is 55

mIU/mL (normal, 0.7-12.8 mIU/mL) and estradiol level is 15 pg/mL (normal, 15-320 pg/mL).

The next step to determine the diagnosis is to order which of the following tests?

Choose one answer .

A. Fasting glucose

B. Fragile X screening

C. Karyotype

D. Serum testosterone level

E. Thyroid-stimulating hormone (TSH)

Educational Objective: Diagnosis of turner's syndrome

Rationale: Turner's syndrome occurs in 1 of 2000 liveborn female infants. Puberty may occur depending on the degree of ovarian development, with primary or secondary amenorrhea.

Therefore, patients may present with normal stature and may not have classic physical findings. A karyotype is recommended in young patients who have amenorrhea and

qsid=148184&aid=3884&qid=315&checksum=be9556e8be4ab174b7defe62956c0f47&sesskey=9SD9XfnshK&new state=0

Page 148: Asap 2011

elevated FSH levels. Other testing such as TSH, fasting glucose, or genetic screening for fragile X may be done later, after the initial diagnosis is made. Serum testosterone level

would provide little if any useful information in this patient with Turner's syndrome. Patients with Turner's syndrome require thorough evaluation with autoimmune and endocrine

testing, and may conceive with donor egg in vitro fertilization, but they should be counseled about the increased risk of mortality related to cardiac problems (e.g., aortic dissection) that

may develop in pregnancy despite a normal echocardiogram.

References:

Achrmann JC, Ozicick G, Meeks JJ, et al. Genetic causes of human reproductive disease. J Clin Endocrinol Metab. 2002; 87:2447-2454.

Sybert VP. Cardiovascular malformations and complications in Turner's syndrome. Pediatrics. 1998; 101:E11.

Incorrect

Marks for this submission: 0.00/1.00.

Question 15

Marks: 1.00

A 24-year-old woman presents to the office for secondary amenorrhea; she stopped using birth control pills 6 months earlier. Her medical history is negative. Growth and development are normal. Her father has a neurologic disorder with ataxia, and a nephew has mild autism.

Her follicle-stimulating hormone (FSH) level is markedly elevated, at 75 mIU/mL (normal, 0.7-12.8 mIU/mL). Her endocrine evaluation shows Hashimoto's thyroiditis with a normal

karyotype .

Screening for which condition should be performed next ?

Choose one answer .

A. Cystic fibrosis

B. Fragile X syndrome

C. Gaucher disease

qsid=148185&aid=3884&qid=317&checksum=479a1e2f9dc51aab99210425c1b707e5&sesskey=9SD9XfnshK&new state=0

Page 149: Asap 2011

D. Maple syrup urine disease

E. Canavan's disease

Educational Objective: The need for patient counseling in premature ovarian failure

Rationale: Besides autoimmune and endocrine testing, the patient diagnosed with premature ovarian failure (POF) should have appropriate genetic screening, with karyotype and fragile X carrier screening, because this may affect future offspring or be present in the

family and is associated with other neurologic abnormalities such as learning disabilities, attention hyperactivity disorder, autism, mental retardation, and ataxia tremor syndrome in

the older male (over age 50). Approximately 15% to 20% of patients that are fragile X premutation carriers of the fragile X mental retardation 1 (FMR1) gene will develop POF.

Gaucher disease, maple syrup urine disease, and Canavan's disease are autosomal recessive conditions that are not closely associated with POF.

References:

Devi AS, Metzger DA, Luciano AA, et al. 45X/46XX mosaicism in patients with idiopathic premature ovarian failure. Fertil Steril. 1998; 70:89-93.

Davison RM, Fox M, Conway GS. Mapping of the POF 1 locus and identification of putative genes for POF. Mol Hum Reprod. 2000; 6:314-318.

Doherty E, Pakarenin P, Tiitinen A, et al. A novel mutation in the FSH receptor inhibiting signal transduction and causing primary ovarian failure. J Clin Endocrinol Metab. 2002;

87:1151-1155.

Incorrect

Marks for this submission: 0.00/1.00.

Question 16

Marks: 1.00

A 25-year-old nulligravid woman presents with a 5-year history of secondary amenorrhea. Her body mass index (BMI) is 15 kg/m2. Follicle-stimulating hormone (FSH) level is 1.0 mIU/mL (normal, 0.7-12.8 mIU/mL) and luteinizing hormone (LH) level is 1.0 mIU/mL

(normal, 1.2-12.9 mIU/L). Prolactin is 10 ng/mL (normal, 3-27 ng/mL) and thyroid-stimulating hormone (TSH) is 1.5 mIU/mL (normal, 0.3-4.5 mIU/mL). Estradiol level is 15 pg/mL (normal,

qsid=148186&aid=3884&qid=321&checksum=dbcc1117754b91a9b9cc09181400381a&sesskey=9SD9XfnshK&new state=0

Page 150: Asap 2011

15-320 pg/mL). The patient denies any medical history, but reports always being on a diet because she is "too fat" She and her spouse would like to proceed with having a child .

Which would be the best recommendation for this patient?

Choose one answer .

A. Clomiphene citrate therapy

B. Hormone replacement therapy

C. Gonadotropin therapy

D. Dietary counseling

E. Check the level of beta human chorionic gonadotropin (hCG) hormone

Educational Objective: Understand the management of functional hypothalamic amenorrhea

Rationale: The patient with a BMI below 18.5 kg/m2 is considered to have low body weight, and this may cause a physiologic disturbance of gonadotropin-releasing hormone (GnRH) and gonadotropin secretion, resulting in hypothalamic hypogonadism with amenorrhea.

Gonadotropin levels and estrogen levels are low. This is often associated with severe psychological stress, intense exercise, and eating disorders. With functional hypothalamic

amenorrhea, medical therapy for ovulation induction is not effective. The best recommendation is dietary counseling to restore a normal body weight, which will re-

establish normal menstrual function and ovulation. Anovulation with infertility is common, and obtaining the beta hCG hormone level would not be beneficial. Initiating hormone

therapy would not establish ovulatory cycles.

Reference:

Falsetti L, Gambera A, Barletti L, et al. Long term follow up of functional hypothalamic amenorrhea and prognostic factors. J Clin Endocrinol Metab. 2002; 87:500-505.

Incorrect

Marks for this submission: 0.00/1.00.

Question 17

Marks: 1.00

Page 151: Asap 2011

A 30-year-old nulligravid single woman desires contraception. She is in a monogamous relationship. Her menstrual cycles vary from 50 to 85 days. She reports bilateral breast

discharge that appears milky in appearance. She is otherwise asymptomatic, with no headaches or visual changes. She is 5 feet 8 inches tall and weighs 140 pounds. Her thyroid-

stimulating hormone (TSH) level is 2.0 mIU/mL (normal, 0.3-4.5 mIU/mL) with a fasting prolactin level of 60 ng/mL (normal, 3-27 ng/mL). Pituitary MRI demonstrates an 8-mm

microadenoma. The patient reports that she had side effects from bromocriptine therapy in the past .

Which would be the best treatment option for this patient?

Choose one answer .

A. Oral cabergoline (Dostinex)

B. Levonorgestrel intrauterine device (Mirena)

C. Combination hormonal contraceptives

D. Norethindrone acetate (Micronor)

E. Vaginal bromocriptine mesylate (Parlodel)

Educational Objective: Management of amenorrhea and galactorrhea in a patient with a prolactin-secreting microadenoma of the pituitary

Rationale: Amenorrhea and galactorrhea often develop in patients with prolactin-secreting microadenoma due to the elevated prolactin levels, which have a direct effect on the

gonadotropin-releasing hormone (GnRH) receptors, causing hypogonadotropic hypogonadism. Elevated prolactin levels may occur with breastfeeding, medications,

pregnancy, or medical disease. Women who do not desire fertility may be treated with combination hormonal contraceptives to provide exogenous estrogen that will protect

bones and establish regular cyclic menses. The dopamine agonists (e.g., bromocriptine and cabergoline) are not without side effects and are not always the preferred treatment for patients with stable tumors and lower prolactin levels. Progesterone-only contraceptives

qsid=148187&aid=3884&qid=326&checksum=14d4ccf38fd740da8b144cbb19155fc2&sesskey=9SD9XfnshK&new state=0

Page 152: Asap 2011

(intrauterine device and oral contraceptive) do not provide the estrogen that would maintain normal bone mass and regulate cycles.

Reference:

Reindollar RH, Novak M, Tho SPT, et al. Adult onset amenorrhea: a study of 262 patients. Am J Obstet Gynecol. 1986; 155:531-543.

Incorrect

Marks for this submission: 0.00/1.00.

Question 18

Marks: 1.00

Which factor is associated with male sexual dysfunction (erectile dysfunction)?

Choose one answer .

A. Metformin therapy

B. Thiazide diuretic therapy

C. High-impact contact sports

D. Repaglinide therapy

E. Thiazolidinedione therapy

Educational Objective: To review risk factors for sexual dysfunction

Rationale: Numerous conditions and situations may lead to male sexual dysfunction. These include diabetes mellitus (through neural and vascular lesions as well as testosterone

deficiency), hypertension (often related to medications used for its treatment, including alpha- and beta- blockers and diuretics), cigarette smoking (probably vascular effect), and depression (related to psychologic factors and medications used for treatment, including

those inducing hyperprolactinemia). Erectile dysfunction has not been reported to be a

qsid=148188&aid=3884&qid=329&checksum=ea9cc104d6f6aa283a808d779e42f3dc&sesskey=9SD9XfnshK&new state=0

Page 153: Asap 2011

significant adverse side effect of metformin, repaglinide, or thiazolidinedione. Contact sports is not associated with erectile dysfunction.

Reference:

Dhindsa S, Prabhakar S, Sethi M, et al. Frequent occurrence of hypogonadotropic hypogonadism in type 2 diabetes. J Clin Endocrinol Metab. 2004; 89:5462-5468.

Incorrect

Marks for this submission: 0.00/1.00.

Question 19

Marks: 1.00

Which of the following hormone panels would be most common in an obese man with erectile dysfunction?

Choose one answer .

A. Low serum testosterone level and elevated luteinizing hormone (LH) level

B. Low serum testosterone level and undetectable LH level

C. Low serum testosterone level and low sex hormone—binding globulin (SHBG) level

D. Normal serum testosterone level and elevated SHBG level

Educational Objective: To review the interpretation of common laboratory tests in evaluating male hypogonadism and sexual dysfunction

Rationale: Testosterone deficiency in the presence of high LH delineates primary (testicular) hypogonadism, whereas low-normal LH is found in men with hypogonadism and pituitary

disease. An obese man will have not only low serum testosterone but also low SHBG (related to insulin resistance), but an elderly man with erectile dysfunction may have a relatively

normal testosterone level. Because SHBG increases with age, his free testosterone level may actually be low. An athlete using an androgenic steroid other than testosterone will have a

low testosterone level with a markedly suppressed (undetectable) LH level.

qsid=148189&aid=3884&qid=332&checksum=d6c159baa597dfc93d5735fcb6fdaf91&sesskey=9SD9XfnshK&new state=0

Page 154: Asap 2011

Reference

Buvat J, Lemaire A. Endocrine screening in 1,022 men with erectile dysfunction: clinical significance and cost-effective strategy. J Urol. 1997; 158:1764-1767.

Incorrect

Marks for this submission: 0.00/1.00.

Question 20

Marks: 1.00

An athlete using androgenic steroids (not testosterone) will present with which of these hormonal panels?

Choose one answer .

A. Low serum testosterone level and elevated luteinizing hormone (LH) level

B. Low serum testosterone level and undetectable LH level

C. Low serum testosterone level and low sex hormone—binding globulin (SHBG) level

D. Normal serum testosterone level and elevated SHBG level

Rationale: An athlete using androgenic steroids (not testosterone) will present with low serum testosterone level and undetectable LH level. Testosterone deficiency in the presence of high LH delineates primary (testicular) hypogonadism, whereas low-normal LH is found in men with hypogonadism and pituitary disease. An obese man will have not only low serum

testosterone but also low SHBG (related to insulin resistance), but an elderly man with erectile dysfunction may have a relatively normal testosterone level. Because SHBG

increases with age, his free testosterone level may actually be low. An athlete using an androgenic steroid other than testosterone will have a low testosterone level with a

markedly suppressed (undetectable) LH level.

Reference:

qsid=148190&aid=3884&qid=334&checksum=1f9a69615c7b5752868c8512e936e0de&sesskey=9SD9XfnshK&new state=0

Page 155: Asap 2011

Buvat J, Lemaire A. Endocrine screening in 1,022 men with erectile dysfunction: clinical significance and cost-effective strategy. J Urol. 1997; 158:1764-1767.

Incorrect

Marks for this submission: 0.00/1.00.

Question 21

Marks: 1.00

A 33-year-old woman with polycystic ovary syndrome, associated with hirsutism and a family history of diabetes mellitus, presents for evaluation of infertility. Her body mass index (BMI)

is 29 kg/m2 and her blood pressure is 120/80 mm Hg. Investigations confirm that she is anovulatory, and ovulation induction is recommended .

Which test should be performed first, prior to induction of ovulation?

Choose one answer .

A. Pelvic ultrasound scan

B. Testosterone level

C. Fasting serum lipids

D. Fasting plasma insulin

E. Oral glucose tolerance test

Educational Objective: To emphasize the need for glucose testing prior to ovulation induction and conception

Rationale: An oral glucose tolerance test should be carried out prior to ovulation induction. It is very important to detect diabetes mellitus and treat it before pregnancy .

Although pelvic ultrasonography may show polycystic ovaries, it is not needed to diagnose polycystic ovary syndrome because the patient already has anovulation and hirsutism. Her

testosterone is likely to be elevated, given her diagnosis. And her family history and BMI are

qsid=148191&aid=3884&qid=337&checksum=0f7459f29adf080ad832e50742b5a2c7&sesskey=9SD9XfnshK&new state=0

Page 156: Asap 2011

consistent with insulin resistance. A fasting insulin level is not helpful or indicated. If she has a lipid disorder, its treatment would have to wait until after her pregnancy .

Reference

American Association of Clinical Endocrinologists position statement on metabolic and cardiovascular consequences of polycystic ovary syndrome. Endocr Pract. 2005; 11:126-134.

Incorrect

Marks for this submission: 0.00/1.00.

Question 22

Marks: 1.00

 

A 42-year-old woman with long-standing irregular menses, a history of infertility, and hirsutism presents for a routine physical examination, complaining of headaches. She has 2

children from prior ovulation induction. Due to pre-eclampsia, she had bilateral tubal ligation. Her body mass index is 32 kg/m2 and her blood pressure is 145/90 mm Hg.

 

Laboratory tests show the following:

Fasting plasma glucose115 mg/dL (normal, 80-100 mg/dL)

2-hour post 75-g glucose load149 mg/dL (normal, 80-140 mg/dL)

Total cholesterol204 mg/dL (normal, 100-200 mg/dL)

Total triglycerides320 mg/dL (normal, 50-150 mg/dL)

High-density lipoprotein cholesterol40 mg/dL (normal, 40-50 mg/dL)

 

In addition to optimizing blood pressure, which approach would be the next step in the management of this patient?

qsid=148192&aid=3884&qid=344&checksum=3d2173017fdd0df2e38343458ac3adac&sesskey=9SD9XfnshK&new state=0

Page 157: Asap 2011

Choose one answer .

A. Behavior modification with reduction of calories and increased exercise

B. Glimepiride 4 mg daily

C. Exenatide 1 mg subcutaneously twice daily

D. Atorvastatin 20 mg daily

E. Fenofibrate 140 mg daily

Educational Objective: To review the management of the metabolic syndrome in a patient with polycystic ovary syndrome

Rationale: The patient presents with features of the metabolic syndrome. These are hypertension, dyslipidemia (high triglycerides and low high-density lipoprotein [HDL]

cholesterol), obesity, and glucose intolerance. It is always appropriate to start with behavior modification. If behavior modification does not help, fibrates are more appropriate as lipid-

altering agents than atorvastatin. However, either could be used in the treatment of dyslipidemia, after behavior modification, exercise, and weight reduction have been tried for

3 to 6 months. Her blood pressure should certainly be treated, and angiotensin-converting enzyme inhibitors are suitable agents. Metformin therapy can help reduce hepatic glucose output and lower her blood sugar and serum insulin, and it may restore regular menses by

lowering androgens and inducing ovulation. Sulfonylureas (e.g., glimepiride) have no indication for the treatment of metabolic syndrome. Thiazolidinediones (TZDs) can be used for insulin sensitization, and newer studies with GLP-1 analogs show promising results, but

these agents would not be the next step in this patient's management.

Reference

Talbott E, Guzick D, Clerici A, et al. Coronary heart disease risk factors in women with polycystic ovary syndrome. Arterioscler Thromb Vasc Biol. 1995; 15:821-826.

Incorrect

Marks for this submission: 0.00/1.00.

Question 23

Marks: 1.00

Page 158: Asap 2011

Which statement regarding the menstrual cycle is correct?

Choose one answer .

A. Basal temperature is elevated in the follicular phase of the menstrual cycle

B. The presence of cervical mucus is an indirect indication that the patient is in the follicular phase of the menstrual cycle

C. Progesterone production is associated with the follicular phase of the menstrual cycle .

D. Blood hormone levels change least in the periovulatory phase of the menstrual cycle

Educational Objective: Evaluation of the menstrual cycle

Rationale: Hormonal evaluation of the menstrual cycle requires precise documentation of ovulation to establish the length of the follicular phase of the cycle. In the follicular phase of

the cycle, estrogen is the predominant hormone, whereas progesterone is associated with the luteal phase. Estrogen stimulates the production of cervical mucus and is associated with

a lower basal temperature. Ovulation is followed by progesterone production, disappearance of cervical mucus, and a rise in basal temperature that persists as long as progesterone is produced. At midcycle, large hormonal changes occur, namely surges of luteinizing hormone (LH) and, to a lesser degree, follicle-stimulating hormone, a drop in

estradiol following the LH surge, and the production of progesterone after exposure to high LH.

Reference:

Davis ME, Fugo NW. The cause of physiologic basal temperature changes in women. J Clin Endocrinol. 1948; 8:550-563.

Incorrect

Marks for this submission: 0.00/1.00.

Question 24

Marks: 1.00

qsid=148193&aid=3884&qid=345&checksum=294df9aee53aeca342fea98569c68368&sesskey=9SD9XfnshK&new state=0

Page 159: Asap 2011

Which of the following is related to menopausal changes seen in the menstrual cycle?

Choose one answer .

A. Increase in inhibin

B. Increasing prolactin

C. A lower basal temperature

D. Decreasing anti-Müllerian hormone

Educational Objective: To review hormonal changes in the menstrual cycle

Rationale: Menopausal changes include decreasing inhibin from decreasing follicles, which leads to increased levels of follicle-stimulating hormone (FSH) (loss of negative feedback).

Prolactin and basal temperature vary intracycle (monthly) but are not associated with menopausal changes. Pituitary temperature regulation is dysregulated by the lack of

estrogen, causing vasomotor symptoms. Anti-Müllerian hormone is an indirect measurement of follicular reserve, and low levels signify loss of follicles without intracycle

variability in FSH.

Reference:

Cobin RH, Futterweit W, Ginzburg SB, et al; AACE Menopause Guidelines Revision Task Force. American Association of Clinical Endocrinologists medical guidelines for clinical

practice for the diagnosis and treatment of menopause. Endocr Pract. 2006; 12:315-337.

Incorrect

Marks for this submission: 0.00/1.00.

Question 25

Marks: 1.00

qsid=148194&aid=3884&qid=347&checksum=cb9fd59ce9ed0be1cbd2384108c15fd6&sesskey=9SD9XfnshK&new state=0

Page 160: Asap 2011

Which statement about the menstrual cycle is correct?

Choose one answer .

A. Hyperandrogenism lengthens the luteal phase of the cycle

B. Any menstruation in a woman not receiving some form of hormonal therapy is a direct indication that ovulation has occurred

C. Progesterone production is associated with the presence of cervical mucus

D. Mastodynia, a cyclic edema, and mood swings are more common in the follicular phase of the menstrual cycle

E. Oral contraceptives that contain a mixture of estrogen and progesterone compounds are expected to reduce the amount of cervical mucus

Educational Objective: Evaluation of the menstrual cycle

Rationale: Hyperandrogenism is often initially associated with lengthening of the follicular phase, followed by complete anovulation. Hormonal evaluation of the menstrual cycle

requires precise documentation of ovulation to establish the length of the follicular phase of the cycle. Menstruation can occur in the absence of ovulation, so-called estrogen

breakthrough bleeding. In the follicular phase of the cycle, estrogen is the predominant hormone, whereas progesterone is associated with the luteal phase. Estrogen stimulates the

production of cervical mucus and is associated with a lower basal temperature. Ovulation is followed by progesterone production, disappearance of cervical mucus, and a rise in the

basal temperature that persists as long as progesterone is produced. Combination oral contraceptives would be expected to reduce the production of cervical mucus. Premenstrual

molimina are associated with progesterone production (luteal phase) .

Reference

Moghissi KS, Syner FN, Borin B. Cyclic changes of cervical mucus enzymes related to the time of ovulation. I Alkaline phosphatase. Am J Obstet Gynecol. 1976; 125:1044-1048.

Incorrect

Marks for this submission: 0.00/1.00.

qsid=148195&aid=3884&qid=350&checksum=5086feaded9981248faa1dfb658d633a&sesskey=9SD9XfnshK&new state=0

Page 161: Asap 2011

Question 26

Marks: 1.00

A 27-year-old woman complains of premenstrual symptoms, acne, and hirsutism, associated with regular menstrual cycles occurring every 28 to 32 days. On physical examination, her

thyroid gland is diffusely enlarged, but she is clinically euthyroid. On pelvic examination, her cervix is extruding copious amounts of clear mucus.

Which laboratory test could and should be performed at the time you see this patient to assess her presenting symptoms?

Choose one answer .

A. Serum testosterone, dehydroepiandrosterone sulfate, and cortisol

B. Serum prolactin

C. Serum estradiol

D. Serum progesterone

E. Thyroid-stimulating hormone (TSH)

Educational Objective: Determining the most appropriate test to obtain on the day a patient presents with premenstrual symptoms, an enlarged thyroid, and findings suggestive of

polycystic ovary syndrome

Rationale: The patient is likely in the periovulatory phase of her cycle and can be expected to have considerable variation in levels of androgens, prolactin, and estradiol. If progesterone

were present, she would not have cervical mucus. TSH is not affected by the menstrual cycle.

References:

Moghissi KS, Syner FN, Borin B. Cyclic changes of cervical mucus enzymes related to the time of ovulation. I. Alkaline phosphatase. Am J Obstet Gynecol. 1976; 125:1044-1048

qsid=148196&aid=3884&qid=353&checksum=9d91c5d4d6fe3157c63c64233836c2bd&sesskey=9SD9XfnshK&new state=0

Page 162: Asap 2011

Lundy LE, Lee SG, Levy W, et al. The ovulatory cycle: a histologic, thermal, steroid, and gonadotropin correlation. Obstet Gynecol. 1974; 44:14-25.

Incorrect

Marks for this submission: 0.00/1.00.

Question 27

Marks: 1.00

A 33-year-old woman stopped her birth control pills 6 months ago and has not menstruated since then. She does not have hot flashes, night sweats, galactorrhea, oily

complexion, acne, or hirsutism. Her body mass index is 23 kg/m2. Her physical examination findings are normal except for slightly atrophic vaginal mucosa and a small uterus. Prior to

starting oral contraceptives, she had irregular menstrual cycles.

Which is the most important laboratory test to assess this woman's secondary amenorrhea?

Choose one answer .

A. Serum follicle-stimulating hormone (FSH)

B. Serum prolactin

C. Serum estradiol

D. Serum progesterone

E. Serum human chorionic gonadotropin (hCG)

Educational Objective: Understand the appropriate evaluation of patients who have a low estradiol level

Rationale: The physical examination findings suggest low estradiol. The serum FSH level should help establish the cause of what appears to be low estradiol. The absence of hot

flashes and night sweats suggest that FSH will be low. Levels of prolactin, estradiol,

qsid=148197&aid=3884&qid=364&checksum=150ed411c4c8027535794fc10568fe05&sesskey=9SD9XfnshK&new state=0

Page 163: Asap 2011

progesterone, and hCG would not establish the cause of low estradiol in this particular patient.

Reference:

Lundy LE, Lee SG, Levy W, et al. The ovulatory cycle: a histologic, thermal, steroid, and gonadotropin correlation. Obstet Gynecol. 1974; 44:14-25.

Incorrect

Marks for this submission: 0.00/1.00.

Question 28

Marks: 1.00

A 30-year-old woman stopped her birth control pills 6 months ago and presents with heat intolerance, fatigue, mastodynia, and amenorrhea. Her body mass index is 23 kg/m2.

Physical examination shows a borderline enlarged thyroid gland, warm skin, and a pulse of 72 beats per minute. Pelvic examination demonstrates vaginal mucosa that is well supported

hormonally, with no cervical mucus and a uterus that is slightly enlarged.

Which test should be ordered first?

Choose one answer .

A. Thyroid-stimulating hormone (TSH) and free thyroxine

B. Serum follicle-stimulating hormone (FSH)

C. Serum estradiol

D. Serum progesterone

E. Serum human chorionic gonadotropin (hCG)

Educational Objective: Recognize possible pregnancy in a woman with amenorrhea who has discontinued oral contraceptives

qsid=148198&aid=3884&qid=367&checksum=45c7c0ae249af3954e5324a47fcc71a0&sesskey=9SD9XfnshK&new state=0

Page 164: Asap 2011

Rationale: Even though this woman has had no menstrual periods since discontinuing her oral contraceptives, she is likely to be pregnant. Determination of her hCG level is the most

important initial test. All physical findings are consistent with early pregnancy. A normal pulse makes hyperthyroidism unlikely. Levels of FSH, estradiol, or progesterone will not

establish the cause of amenorrhea in this clinical setting. TSH and free thyroxine should be measured as well, but the most important initial step is obtaining the hCG level.

Reference:

Lundy LE, Lee SG, Levy W, et al. The ovulatory cycle: a histologic, thermal, steroid, and gonadotropin correlation. Obstet Gynecol. 1974; 44:14-25.

Incorrect

Marks for this submission: 0.00/1.00.

Question 29

Marks: 1.00

Which clinical problem is encountered in patients with Turner's syndrome (TS)?

Choose one answer .

A. Hypercalcemia

B. Paradoxical precocious puberty

C. Aortic dissection

D. Growth hormone deficiency

E. Hyperacusis

Educational Objective: To review the clinical picture and pathophysiology of TS

Rationale: Although patients with TS are short, they do not actually have growth hormone deficiency. The majority have pubertal arrest due to depletion of the ovarian follicular

apparatus. Conductive hearing loss, often secondary to otitis media, is very common, as is

qsid=148199&aid=3884&qid=373&checksum=a762d6de045cab1ade9dcd939dc23b6e&sesskey=9SD9XfnshK&new state=0

Page 165: Asap 2011

autoimmune thyroid disease. Disturbingly, aortic dissection has been reported in a number of TS patients in the setting of pregnancy, and close monitoring is recommended.

Hypercalcemia is not part of the comorbidity of TS.

Reference:

Karnis MF, Zimon AE, Lalwani SI, et al. Risk of death in pregnancy achieved through oocyte donation in patients with Turner syndrome: a national survey. Fertil Steril. 2003; 80:498-501.

Incorrect

Marks for this submission: 0.00/1.00.

Question 30

Marks: 1.00

Which clinical finding is associated with Turner's syndrome (TS)?

Choose one answer .

A. Sclerodactyly

B. Hypertension

C. Down's syndrome

D. Megacolon

E. Macular degeneration

Educational Objective: Recognize comorbidities of TS

Rationale: Coarctation of the aorta and hypertension are common comorbidities and clinical characteristics of TS. Sclerodactyly, Down's syndrome, megacolon, and macular

degeneration are not clinical characteristics or common comorbidities of TS. The amenorrhea of TS relates to accelerated germ cell loss, and webbing of the neck is

associated with lymphedema. Short stature relates to haploinsufficiency of the short-stature

qsid=148200&aid=3884&qid=376&checksum=7ce4e43537b4589bb316702e61adbde0&sesskey=9SD9XfnshK&new state=0

Page 166: Asap 2011

homeobox-containing gene (SHOX). The presence of Y-mosaicism necessitates gonadectomy because of the increased risk of gonadoblastoma and other germ cell tumors.

Reference:

Elsheikh M, Dunger DB, Conway GS, et al. Turner's syndrome in adulthood. Endocr Rev. 2002; 23:120-140.

Incorrect

Marks for this submission: 0.00/1.00.

Question 31

Marks: 1.00

A 23-year-old woman with a history of Turner's syndrome (TS) and primary amenorrhea comes for evaluation. She was last seen by her pediatrician just before her 19th birthday,

and this is her first visit to an endocrinologist.

Which statements is correct regarding the management of TS?

Choose one answer .

A. Inform her that hormonal therapy with 17-β-estradiol alone is appropriate for patients with TS

B. Arrange for echocardiography, and repeat it in 3 to 5 years if findings are normal

C. Inform her that adoption is the only option available because of her infertility

D. Since her gonads were removed as a child, she does not need a Papanicolaou smear

Educational Objective: To review treatment and follow-up guidelines for Turner's syndrome (TS)

Rationale: Since TS patients have uteri, estrogen alone is not appropriate therapy and progestins are required to protect the endometrium. Echocardiography is necessary to

qsid=148201&aid=3884&qid=377&checksum=78009c2bfcee9bcd07d48d58375ffb2c&sesskey=9SD9XfnshK&new state=0

Page 167: Asap 2011

delineate cardiac lesions. Patients with TS have the option of borrowing oocytes to achieve pregnancy, as well as adoption. It is appropriate to check the patient's thyroid-stimulating

hormone level and thyroid antibodies, because autoimmune thyroid disease is very common in TS patients. Gonads are removed only because of the presence of Y-mosaicism. Under

these circumstances, the uterus is not removed, so childbearing can be made possible using donated oocytes. Thus, patients with TS will still need annual gynecologic examinations and

Papanicolaou smears.

Reference:

Gravholt CH, Fedder J, Naerraa RW, et al. Occurrence of gonadoblastoma in patients with Turner syndrome and Y-chromosome material: a population study. J Clin Endocrinol Metab.

2000; 85:3199-3202.

Incorrect

Marks for this submission: 0.00/1.00.

Reminder:

You have three (3) attempts to successfully complete this assessment with a 75% score or higher.

You have 30 days to complete each attempt, once an attempt is started.

If you need to save your answers and come back to the assessment, you must click the 'Next' button at the bottom-left of the assessement prior to logging

out of ASAP.

Only click 'Submit all and finish' once you are satisfied with all your assessment answers.

Bottom of Form

Finish review

Skip Quiz navigation

Quiz navigation

i (Closed ) 1 (Incorrect ) 2 (Incorrect ) 3 (Incorrect ) 4 (Incorrect ) 5 (Incorrect ) 6 (Incorrect ) 7 (Incorrect ) 8 (Incorrect ) 9 (Incorrect ) 10 (Incorrect ) 11 (Incorrect ) 12 (Incorrect ) 13

(Incorrect ) 14 (Incorrect ) 15 (Incorrect ) 16 (Incorrect ) 17 (Incorrect ) 18 (Incorrect ) 19 (Incorrect ) 20 (Incorrect ) 21 (Incorrect ) 22 (Incorrect ) 23 (Incorrect ) 24 (Incorrect ) 25 (Incorrect ) 26 (Incorrect ) 27 (Incorrect ) 28 (Incorrect ) 29 (Incorrect ) 30 (Incorrect ) 31

(Incorrect ) i (Closed )

Finish review

You are logged in as Mohammed Aldawish (Logout)

REPRO

Page 168: Asap 2011

 

Copyright: 2011, American Association of Clinical Endocrinologists (AACE).All material published on the Web site is the property of AACE and may not be reproduced in

any form or by any electronic meansincluding information, storage and retrieval systems without the written consent of AACE.

This activity is sponsored by the American Association of Clinical Endocrinologists.

Skip to main content

Serum Lipid Disorders

You are logged in as Mohammed Aldawish (Logout)

Page path

Home

/ ► My courses

/ ► LIPIDS

/ ► Assessment

/ ► Serum Lipids Disorders Assessment

/ ► Review of attempt 1

Review of attempt 1

Started onTuesday, 19 November 2013, 10:36 PM

Completed onTuesday, 19 November 2013, 10:36 PM

Time taken32 secs

Grade0.00 out of a maximum of 37.00 (0%)

FeedbackFailed

Page 169: Asap 2011

Top of Form

Directions: In this section, each item includes a lead question and a list of options labeled with letters. Select the ONE lettered option that is BEST in each case by selecting the radio

button corresponding to the correct answer .

Question 1

Marks: 1.00

Which goal level for the lipid measurements listed below is most appropriate for persons with two or more cardiovascular risk factors?

Choose one answer .

A. Low-density lipoprotein (LDL) cholesterol 100 mg/dL

B. Apolipoprotein B (apoB) 65 mg/dL

C. LDL particle number 1300 nmol/L

D. Non-high-density lipoprotein (non-HDL) cholesterol 100 mg/dL

Educational Objective: Recognize the goal level for various lipid measurements at different cardiovascular risk levels .

Rationale: The American College of Cardiology and American Diabetes Association consensus goal for low density lipoprotein (LDL) cholesterol for persons with two or more

cardiovascular (CV) risk factors is 100 mg/dL . For those patients with diabetes mellitus without other risk factors, the goal for apoB is 85 mg/dL, for LDL particle number, 1000

nmol/L and for non-HDL cholesterol, 130 mg/dL .

References

1 .Contois JH, McNamara JR, Lammi-Keefe CJ, et al. Reference intervals for plasma apolipoprotein B determined with a standardized commercial immunoturbidimetric assay:

results from the Framingham Offspring Study. Clin Chem. 1996; 42:515-523.

9SD9XfnshK

qsid=148204&aid=3885&qid=146&checksum=fc1e2a831d62241d2fb3fc3fc933b6ea&sesskey=9SD9XfnshK&new state=0

Page 170: Asap 2011

2 .Bachorik PS, Lovejoy KL, Carroll MD, et al. Apolipoprotein B and AI distributions in the United States, 1988-1991: results of the National Health and Nutrition Examination Survey III

(NHANES III). Clin Chem. 1997; 43:2364-2378.

3 .Mora S, Szklo M, Otvos JD, et al. LDL particle subclasses, LDL particle size, and carotid atherosclerosis in the Multi-Ethnic Study of Atherosclerosis (MESA). Atherosclerosis. 2007;

192:211-217.

Incorrect

Marks for this submission: 0.00/1.00.

Question 2

Marks: 1.00

Which goal level for the lipid measurements listed below is most appropriate for persons with known ischemic cardiovascular disease or diabetes and ≥1 other risk factor?

Choose one answer .

A. Non-high-density lipoprotein (HDL) cholesterol <130 mg/dL

B. Low-density lipoprotein (LDL) cholesterol <100 mg/dL

C. Nuclear magnetic resonance (NMR) LDL particle number <700 nmol/L

D. Apolipoprotein B (apoB) <80 mg/dL

Educational Objective: To understand the target lipid levels from current medical guidelines

Rationale: The goal for apoB according to the consensus statement of the American College of Cardiology (ACC) and American Diabetes Association (ADA) is <80 mg/dL, with

recommended LDL cholesterol goal being <70 mg/dL, and non-HDL cholesterol goal being <100 mg/dL .

The consensus statement recognized the concept of global cardiometabolic risk, which included lipid parameters associated with insulin resistance (ie, high triglycerides, low HDL

cholesterol, and increased small LDL measured by non-HDL cholesterol, NMR number of LDL

qsid=148205&aid=3885&qid=162&checksum=1e5ff1d059b9591151c8893aaa6d17d8&sesskey=9SD9XfnshK&new state=0

Page 171: Asap 2011

particles, or apoB). Trials of treatment with statins in patients with diabetes decreases cardiovascular (CV) risk, but only to the level of nondiabetic patients treated with placebo.

Part of the reason may be underestimated risk using standard LDL cholesterol measurements and goals .

The statement examines all four methods of measuring this lipid risk, with only the standard LDL cholesterol measure requiring a fasting state (based on a calculated value using the

Friedewald equation). The table below gives consensus goals for LDL cholesterol, non-HDL cholesterol, and apoB. No goal is given for NMR LDL particle number because the method in

2007 was not widely available, was relatively expensive, and there were concerns about accuracy as published in a study the same year as the consensus statement.

Goals suggested were as follows:

LDL cholesterol

mg/dL

Non-HDL cholesterol

mg/dL

apoB

mg/dL

Highest risk1 (known CVD*2 (DM*+ 1 addtl

RF*

>70>100>80

High risk1 (2 major RF only2 (DM + no other

RF

>100>130>90

 

*CVD, cardiovascular disease; DM, diabetes mellitus; addtl, additional; RF, risk factor(s).

Matching the <70 and <100 mg/dL for LDL and non-HDL cholesterol as a 5th percentile population value would be apoB <65 mg/dL and LDL particle number <700 nmol/L. These

lower goals for apoB and LDL particle number are untested for better CV prevention results.

Incorrect

Marks for this submission: 0.00/1.00.

Question 3

Marks: 1.00

Page 172: Asap 2011

A 42-year-old normotensive, nonsmoking male with type 2 diabetes mellitus, body mass index (BMI)25 kg/m2, high-density lipoprotein (HDL) cholesterol 43 mg/dL, and no personal

or family history of macrovascular disease comes to you for management of his diabetes mellitus.

In which ONE of the following categories of cardiovascular risk and low-density lipoprotein cholesterol (LDL-C) goal would he be classified ?

Choose one answer .

A. Lower risk with LDL-C goal <160 mg/dL

B. Moderate risk with LDL-C goal <130 mg/dL

C. Moderately high risk with LDL-C goal <130 mg/dL

D. High risk with LDL-C goal <100 mg/dL

E. Very high risk with LDL-C goal <70 mg/dL

Educational Objective: To understand nuances of recent medical practice guidelines and consensus statements

Rationale: National Cholesterol Education Program Adult Treatment Panel III 2006 (NCEP ATPIII 2006) classifies persons with diabetes mellitus as "high risk" with a LDL-Cgoal of <100

mg/dL. Diabetic patients with a history of cardiovascular disease (CVD) or one major additional risk factor are "very high risk" with a nLDL-C goal of <70 mg/dL. The patient in the

clinical vignette of the question is not overweight or obese, is a nonsmoker, is not hypertensive and has no history of macrovascular disease. Therefore, his LDL-C goal is <100

mg/dL since CVD or an additional risk factor is not identified.

References

Smith SC Jr., Allen J, Blair SN, et al. AHA/ACC guidelines for secondary prevention for patients with coronary and other atherosclerotic vascular disease: 2006 update: endorsed by the

National Heart,Lung, and Blood Institute. Circulation. 2006; 113:2363-2372 .

qsid=148206&aid=3885&qid=165&checksum=54946b66641019a5f4b50ee1c959e1be&sesskey=9SD9XfnshK&new state=0

Page 173: Asap 2011

Brunzell JD, Davidson M, Furberg CD, et al.Lipoprotein management in patients with cardiometabolic risk: consensus statement from the American Diabetes Association and the

American College of Cardiology Foundation. Diabetes Care. 2008; 31:811-222 .

Incorrect

Marks for this submission: 0.00/1.00.

Question 4

Marks: 1.00

The Fenofibrate Intervention and Event Lowering in Diabetes (FIELD) and the Action to Control Cardiovascular Risk in Diabetes (ACCORD) have assessed the lowering of triglycerides

with fibrates in patients with diabetes mellitus whether treated or not treated with 3-hydroxy-3-methyl-glutaryl coenzyme A reductase inhibitors (statins).

Which of the following statements is most consistent with the findings of these studies in patients with diabetes mellitus?

Choose one answer .

A. Fibrates consistently reduce cardiovascular (CVD) death and CVD events regardless of statin treatment .

B. Fibrates have little benefit in persons with diabetes mellitus .

C. Fibrates benefit patients with diabetes mellitus with elevated serum triglycerides and low high-density lipoprotein cholesterol (HDL-C) by reducing CVD

events but not CVD mortality .

D. Fibrates benefit patients with diabetes mellitus with elevated serum triglycerides and low HDL-C by reducing both CVD events and CVD mortality .

E. Fibrates have their maximum beneficial effect on CVD events and all-cause mortality in patients with diabetes mellitus whose serum triglyceride levels are

<160 mg/dL .

qsid=148207&aid=3885&qid=168&checksum=68152df8fb5b714543f46246996c7992&sesskey=9SD9XfnshK&new state=0

Page 174: Asap 2011

Educational Objective: To understand the results of recent large clinical trials in patients with diabetes mellitus treated with fibric acid derivatives (fibrates) .

Rationale: Clinical trials have consistently demonstrated a beneficial effect of fibric acid derivatives on CVD events but not on CVD mortality in patients with elevated serum

triglycerides and low HDL-C levels. However, no apparent added fibrate benefit has been noted beyond that attributed to statins in patients with normal triglyceride and HDL-C levels.

The FIELD trial of fenofibrate versus placebo without statins showed a reduction of CVD events only in diabetic patients with serum triglycerides >200 mg/dL, or HDL-C <40 mg/dL

(men) or <50 mg/dL (women), or both. The (ACCORD)-Lipid trial of fenofibrate added to statins showed that clinical events are reduced only in diabetic patients with serum

triglycerides >200 mg/dL and HDL-C <35 mg/dL. These findings are consistent with the current medical practice recommendations of using fibrates only in patients with serum triglyceride levels >200 mg/dL, expecting that the best preventive benefit will be seen in

those with both high serum triglycerides and low HDL-C .

References

The FIELD study investigators. Effects of long-term fenofibrate therapy on cardiovascular events in 9795 people with type 2 diabetes mellitus (the FIELD study): randomized

controlled trial. Lancet. 2005; 366:1849-1861.

Scott R, O'Brien R, Fulcher G, et al, for the FIELD study investigators. Effects of fenofibrate treatment on cardiovascular disease risk in 9,795 individuals with type 2 diabetes and

various components of the metabolic syndrome. Diabetes Care. 2009; 32:493-498.

The ACCORD Study Group. Effects of combination lipid therapy in type 2 diabetes mellitus. N Engl J Med. 2010: 0.1056/NEJMoa1001282.

Incorrect

Marks for this submission: 0.00/1.00.

Question 5

Marks: 1.00

Recent studies have assessed the effect of treatment with 3-hydoxy-3-methyl-glutaryl(HMG-CoA) reductase inhibitors (statins) on cardiovascular disease (CVD) events and CVD mortality

in patients with low-density lipoprotein cholesterol (LDL-C)<130 mg/dL and with varying levels of the inflammatory marker, high-sensitivity C-reactive protein (hsCRP) .

qsid=148208&aid=3885&qid=172&checksum=fa9087ff6e5e046b8127a6c7fa177168&sesskey=9SD9XfnshK&new state=0

Page 175: Asap 2011

Which of the following statements is correct regarding the effect of statin therapy on CVD events and CVD death on patients with LDL-C levels<130 mg/dL ?

Choose one answer .

A. Lowering LDL-C below 130 mg/dL with statin therapy has no further beneficial effect on CVD events or CVD death .

B. Lowering LDL-C in patients with LDL-C <130 mg/dL and hsCRP levels >2.0 mg/L results in decreased CVD death and CVD events .

C. hsCRP is not useful to stratify ischemic CVD risk for primary prevention of CVD events and mortality

D. Specific therapy for an elevated hsCRP level is clearly indicated in the prevention and management of CVD

E. No benefit in stroke reduction occurs with lowering LDL-C below 100 mg/dL with a concomitant reduction in hsCRP to <1.0 mg/L

Educational Objective: To understand the nuances of therapy with HMG-CoA reductase inhibitors (statins) on CVD events and CVD mortality in patients with LDL-C <130 mg/dL and

with varying levels of the inflammatory marker hsCRP .

Rationale: The Justification for the Use of statins in Primary prevention: an Intervention Trial Evaluating Rosuvastatin (JUPITER) trial is considered a landmark study.It demonstrates that

apparently healthy persons with LDL-C levels <130 mg/dLand not requiring further LDL-C lowering according to medical practice guidelines but whose baseline hsCRP was >2.0 mg/L

had a significant benefit in reducing ischemic CVD events and CVD-related death. Prior studies had demonstrated a clear statin benefit in patients with higher LDL-C levels not at

goal, many of whom had prior CVD events. The exact role of hsCRP as an atherogenic factor, a treatment target, or a CVD marker remains unclear, but the JUPITER study has increased interest in hsCRP as a possible target for therapy. Patients on treatment who achieved an LDL-C <70 mg/dL and hsCRP< 1 mg/L had a striking and significant (79%) reduction in CVD

events .

References

Ridker PM, Danielson E, Fonseca FA, et al, for the JUPITER Study Group. Rosuvastatin to prevent vascular events in men and women with elevated C-reactive protein. N Engl J Med.

2008; 359: 2195-2207 .

Page 176: Asap 2011

RidkerPM, Danielson E, Fonseca FA, et al, for the JUPITER Trial Study Group.Reduction in C-reactive protein and LDL cholesterol and cardiovascular event rates after initiation of

rosuvastatin: a prospective study of the JUPITER trial. Lancet. 2009; 373:1175-1182 .

Incorrect

Marks for this submission: 0.00/1.00.

Question 6

Marks: 1.00

Which of the following statements is most consistent with current medical practice guidelines regarding lipoprotein measurements other than low-density lipoprotein (LDL)

cholesterol in patients with type 2 diabetes mellitus or with cardiometabolic risk ?

Choose one answer .

A. Apolipoprotein B (apoB) and nuclear magnetic resonance (NMR) LDL particle number are no more informative than LDL cholesterol or non-high-density

lipoprotein (non-HDL) cholesterol in assessing the residual ischemic cardiovascular risk in patients with type 2 diabetes mellitus

B. The upper limit of the target level for apoB is <80 mg/dL in patients with uncomplicated type 2 diabetes mellitus and no other cardiovascular risk factors.

C. The optimal LDL particle number in patients with type 2 diabetes mellitus has been set at <1000 nmol/L

D. apoB and NMR particle number provide equally informative further information about lipoprotein risk beyond LDL cholesterol and non-HDL cholesterol in persons

with cardiometabolic risk l .

E. apoB is a far superior measure of lipoprotein risk beyond LDL cholesterol and non-HDL cholesterol compared with NMR LDL particle number .

qsid=148209&aid=3885&qid=175&checksum=e158e0ccc7975e0c820fa83d2545b0f3&sesskey=9SD9XfnshK&new state=0

Page 177: Asap 2011

Educational Objective: To understand the other measures of cardiometabolic risk beyond LDL cholesterol and non-HDL cholesterol in patients with diabetes mellitus or with

cardiometabolic risk .

Rationale: LDL cholesterol lowering has been shown to reduce ischemic cardiovascular disease in patients with diabetes mellitus.Patients with type 2 diabetes mellitus and insulin

resistance have many cardiometabolic risk factors beyond increased LDL cholesterol, as one might seein persons with familial heterozygous hypercholesterolemia. Such cardiometabolic

risk factors include a more complicated dyslipidemia consisting of increased very-low-density lipoprotein (VLDL) cholesterol and VLDL remnants manifesting as

hypertriglyceridemia, reduced HDL cholesterol, and increased small dense LDL particles with standard LDL cholesterol measurements underestimating LDL risk.Both apoB and NMR LDL

particle number have been shown to be equally informative in better predicting this increased risk than LDL cholesterol and non-HDL cholesterol in most studies. The 2008

consensus statement of the American College of Cardiology and the American Diabetes Association explored lipoprotein measures other than LDL cholesterol in persons with type 2 diabetes mellitus and/or with cardiometabolic risk. At the time of the consensus statement,

there were concerns about the cost, availability, and value of NMRLDL particle number among diverse ethnic groups that a specific LDL particle number goal was not given. Two

goal levels of apoB, however, were suggested: <80 mg/dL in those at very high risk; and <90 mg/dL in those at high risk, which includes diabetes mellitus without other cardiovascular

risk factors. The optimal level of NMRLDL lipid particle number based on the respective laboratory standardization of the test is reported to be<1000 nmol/L but no treatment

target goal has been set by any consensus statement to date. An NMR LDL lipid particle level of <1000 nmol/L is equivalent to an LDL cholesterol <100 mg/dL in the Multi-Ethnic Study of

Atherosclerosis (MESA) population .

References

Brunzell JD, Davidson M, Furberg CD, et al. Lipoprotein management in patients with cardiometabolic risk: consensus statement from the American Diabetes Association and the

American College of Cardiology Foundation. Diabetes Care. 2008; 31:811-822 .

Mora S,Szklo M, Otvos JD, et al. LDL particle subclasses, LDL particle size, and carotid atherosclerosis in the Multi-Ethnic Study of Atherosclerosis (MESA). Atherosclerosis. 2007;

192:211-217

Incorrect

Marks for this submission: 0.00/1.00.

Question 7

Marks: 1.00

Page 178: Asap 2011

Nicotinic acid has been used in the treatment of dyslipidemia for many years .

Which of the following disorders is most likely to be adversely affected by its use ?

Choose one answer .

A. Glycogen storage disease type 1

B. Islet cell tumors of the pancreas

C. Obesity

D. Type 2 diabetes mellitus

E. Cholesterol ester transfer protein deficiency

Educational Objective: To understand the sideeffects of nicotinic acid therapy

Rationale: Nicotinic acid adversely affects glucose tolerance andis associated with hyperglycemia, which occurs with increasing dosages ofnicotinic acid. In patients with diabetes mellitus, administration of nicotinicacid for treatment of hyperlipidemia will

generally aggravate hyperglycemia, particularly with increasingdosages of the drug.

Nicotinic acid is not indicated inthe treatment of glycogen storage disease associated with hypoglycemia becauseof its potential hepatotoxic effect. Nicotinic acid has not been used in theusual treatment of functioning islet cell tumors with hypoglycemia. Such tumorsrequire

surgery. Nicotinic acid has no apparent adverse effect on obesity perse unless impaired glucose tolerance or diabetes mellitus is present.

Cholesterol ester transfer proteindeficiency is associated with elevated high-density lipoprotein cholesterol.Nicotinic acid is not a treatment for this disorder.

References

Whayne TF. High density lipoproteincholesterol: current perspectives for clinicians. Angiology. 2009: 60:644-649 .

qsid=148210&aid=3885&qid=177&checksum=fd08baa74a237168d5457327c4d238c2&sesskey=9SD9XfnshK&new state=0

Page 179: Asap 2011

Hirano K, Yamashita S, Kuga Y,et al. Atheroscleroticdisease in marked hyperalphalipoproteinemia: combined reduction of cholesterylester transfer protein and

hepatic triglyceride lipase. Arterioscler ThrombVasc Biol. 1995; 15:1849-1856 .

Canner PL, Berge KG, WengerNK, et al. Fifteen year mortality in coronary drug projectpatients: long-term benefits with niacin. J Am Coll Cardiol. 1986; 8:1245-1255 .

Incorrect

Marks for this submission: 0.00/1.00.

Question 8

Marks: 1.00

Which of the following statements regarding Tangier disease is correct?

Choose one answer .

A. Tangier disease is associated with an increased efflux of cholesterol from macrophages to high-density lipoprotein (HDL) .

B. Patients with Tangier disease have a decreased risk of cardiovascular disease (CVD) .

C. Tangier disease is associated with mutations in the ABC1 transporter gene with decreased efflux of cholesterol to nascent and mature HDL particles .

D. Tangier disease is associated with decreased chylomicron formation and low triglyceride levels

E. Tangier disease is associated with increased levels of HDL cholesterol and increased levels of low-density lipoprotein (LDL) cholesterol .

Educational Objective: To recognize the clinical features and lipid abnormalities of Tangier disease

Rationale: Tangier disease is caused by mutations in the ABC1 gene,which leads to decreased efflux of cholesterol from macrophages to nascent and mature HDL particles.

These patients have low LDL cholesterol, and though initially it was thought they had a

qsid=148211&aid=3885&qid=179&checksum=f033cb4be1035ab1298e4d3bccdd655e&sesskey=9SD9XfnshK&new state=0

Page 180: Asap 2011

reduced risk of CVD, more recent evidence suggests that they infact have an increased risk. Clinically, affected patients have extremely low HDL cholesterol levels, large orange tonsils,

splenomegaly, and peripheral neuropathy. Moderate hypertriglyceridemia is also found. These patients were initially believed not to be at increased risk for atherosclerosis because

of concomitant reduction in LDL cholesterol levels, but subsequent reports have documented a four- to sixfold increase in CVD. The diagnosis is made by finding low HDL

cholesterol levels, normal or moderately increased triglyceride levels,and hyper plastic orange-yellow tonsils. There is no specific treatment for Tangier disease.

Reference

Serfaty-Lacrosniere C, CiveiraF, Lanzberg A, et al. Homozygous Tangier disease and cardiovascular disease. Atherosclerosis. 1994; 107:85-98

Incorrect

Marks for this submission: 0.00/1.00.

Question 9

Marks: 1.00

A 48-year-old man with establishedcoronary artery disease has a high-density lipoprotein (HDL) cholesterol levelof 30 mg/dL after lifestyle modifications.

Which of the following medications should he begin taking?

Choose one answer .

A. Omega-3 fatty acids

B. Ezetimibe

C. Atorvastatin

D. Fenofibrate

qsid=148212&aid=3885&qid=181&checksum=87c5d91f5d5e2da6b8dc62fc8a013523&sesskey=9SD9XfnshK&new state=0

Page 181: Asap 2011

E. Gemfibrozil

Educational Objective: To understand the pharmacologic approach to low HDLcholesterol levels

Rationale: Drug treatmentfor low HDL cholesterol is required when lifestyle changes such as exercise,weight loss, and smoking are inadequate. The drug of choice in this setting isa 3-

hydroxy-3-methyl-glutaryl-coenzyme A ( HMG-CoA) reductase inhibitor such asatorvastatin. This drug will decrease low-density lipoprotein (LDL) cholesterolbut has modest effects on

HDL cholesterol. Nicotinic acid could also beconsidered. Ezetimibe will have little, if any, significant effect on HDLcholesterol levels.

Omega-3 fatty acids, fenofibrate,and gemfibrozil increase HDL cholesterol by lowering triglyceride levels. Thereis no apparent history of elevated triglyceride levels in the

patientdescribed, but HMG-CoA reductase inhibitors also reduce mild to moderatelyelevated triglyceride levels.

Reference

Executive Summary of the Thirdreport of the National Cholesterol Education Program (NCEP) Expert Panel on Detection, Evaluationand Treatment of High Blood Cholesterol in Adults.

JAMA. 2001; 285:2486-2497 .

Incorrect

Marks for this submission: 0.00/1.00.

Question 10

Marks: 1.00

A 48-year-old man with established coronary artery disease and low high-density lipoprotein (HDL) cholesterol is on a statin (3-hydroxy-3-methyl-glutaryl coenzyme A [HMG-CoA])

reductase inhibitor, but levels are still low at 25 mg/dL.

Which of the following medications should be added?

Choose one answer .

A. Nicotinic acid

qsid=148213&aid=3885&qid=182&checksum=4f1b51d63f73c3d31a6454f481bb1042&sesskey=9SD9XfnshK&new state=0

Page 182: Asap 2011

B. Ezetimibe

C. Pioglitazone

D. Omega-3 fatty acids

E. Gemfibrozil

Educational Objective: To understand the pharmacologic approach to low HDL cholesterol levels

Rationale: Patients on a statin (HMG-CoA) reductase inhibitor with HDL cholesterol levels below target should have nicotinic acid added to their drug program. Nicotinic acid is the

most potent of the currently available agents to increase HDL cholesterol levels.

Ezetimibe will not significantly affect HDL cholesterol levels. Omega-3 fatty acids and gemfibrozil can increase HDL cholesterol by reducing triglyceride levels if

hypertriglyceridemia is present. Pioglitazone is used in the treatment of type 2 diabetes mellitus and has a beneficial effect on the lipid profile, including an increase in HDL

cholesterol. However, nicotinic acid remains the drug of choice for increasing persistently low HDL cholesterol levels.

References

Whayne TF. High density lipoprotein cholesterol: current perspectives for clinicians. Angiology 2009; 60:644-649 .

Canner PL, Berge KG, Wenger NK, et al. Fifteen year mortality in coronary drug project patients: long-term benefits with niacin. J Am Coll Cardiol. 1986; 8:1245-1255.

Incorrect

Marks for this submission: 0.00/1.00.

Question 11

Marks: 1.00

qsid=148214&aid=3885&qid=185&checksum=fe371aba9bb263828f6ea9cc3583a5fb&sesskey=9SD9XfnshK&new state=0

Page 183: Asap 2011

Which of the following statements is true in relation to patients with elevated high-density lipoprotein (HDL) cholesterol?

Choose one answer .

A. An elevated HDL cholesterol level is always cardioprotective

B. The target for low-density lipoprotein (LDL) cholesterol in a patient with a high HDL cholesterol level (60 mg/dL) and premature coronary artery disease should be

between 100 and 130 mg/dL

C. An elevated HDL cholesterol level may be due to a mutation in cholesterol ester transfer protein (CETP) .

D. An elevated HDL cholesterol level is seen in Tangier disease .

Educational Objective: Torecognize causes and management of patients with elevated HDL cholesterol levels

Rationale: Elevated HDLcholesterol levels are associated with reduced cardiovascular risk inepidemiology studies, but individuals with this lipid finding have beenreported with

coronary artery disease. Patients with an elevated HDLcholesterol level with coronary artery disease should have aggressive attemptsto lower LDL cholesterol. CETP mutations may cause

elevated HDL cholesterollevels. Tangier disease is associated with low HDL cholesterol. Patients withtype 1 diabetes mellitus may have elevated HDL cholesterol levels but

shouldstill have LDL targets of less than 100 mg/dL; some would suggest less than 70mg/dL .

Reference

Hirano K, Yamashita S, Kuga Y, et al. Atheroscleroticdisease in marked hyperalphalipoproteinemia: combined reduction of cholesterylester transfer protein and

hepatic triglyceride lipase. Arterioscler Thromb Vasc Biol. 1995; 15:1849-1856 .

Incorrect

Marks for this submission: 0.00/1.00.

Question 12

Marks: 1.00

qsid=148215&aid=3885&qid=188&checksum=1ac5f3d618b0b78cc84953c17fb43065&sesskey=9SD9XfnshK&new state=0

Page 184: Asap 2011

A girl age 3 years and 10 months is referred for increased weight gain and elevated cholesterol. Her diet consists of foods with high saturated fat. She has six healthy siblings.

Both parents are from Bangladesh. Her mother is 40 years old with a body mass index (BMI) of33 kg/m2; she is otherwise healthy. Her father is 43 years old with a BMI of 30 kg/m2 and

hypercholesterolemia.

On examination, the girl's height is 102.2 cm (75th percentile for age), weight 23.7 kg (>97thpercentile for age), BMI 22.7 kg/m2 (>97th percentile for age), and blood pressure

100/60 mm Hg. She has minimal acanthosis nigricans around the neck. The rest of the examination is unremarkable.

Laboratory results are as follows:total cholesterol 288 mg/dL (normal, <170), triglycerides 152 mg/dL (normal,<150), high-density lipoprotein cholesterol (HDL-C) 38 mg/dL

(normal,>35), low-density lipoprotein cholesterol (LDL-C) 220 mg/dL (normal,<110), thyroid-stimulating hormone (TSH) 1.94 µIU/mL (normal, 0.6-5.5). A complete metabolic profile is

normal.

Which of the following is the most appropriate intervention for this child?

Choose one answer .

A. Nutrition therapy

B. 3-hydroxy-3-methylglutaryl coenzyme A (HMG-CoA) reductase inhibitor

C. Cholestyramine

D. Niacin

E. Gemfibrozil

Educational Objective: To learn the initial treatment of hypercholesterolemia in children

Rationale: Nutrition therapy is the cornerstone in children with elevated cholesterol, LDL-C, or triglyceride levels. Drug therapy is considered in children over the age of 8 years after an

adequate trial of dietary modification for 6 to 12 months if they still have LDL-C ≥ 190mg/dL, or LDL-C ≥ 160mg/dL with other cardiovascular risk factors. Therefore, cholestyramine

orHMG-CoA reductase inhibitors are not the first choice in the child. Niacin is not recommended in the pediatric population. There are not enough studies regarding

gemfibrozil use in children.

References

Page 185: Asap 2011

American Academy of Pediatrics. National Cholesterol Education Program: Report of the expert panel on blood cholesterol levels in children and adolescents. Pediatrics .1992;

89:525-584 .

Daniels SR, Greer FR, Committee on Nutrition.Lipid screening and cardiovascular health in childhood. Pediatrics. 2008; 122:198-208.

Incorrect

Marks for this submission: 0.00/1.00.

Question 13

Marks: 1.00

A boy age 13 years and 2 months is referred for elevated cholesterol. His 43-year-old father has a history of elevated cholesterol and coronary artery stenting within the last 6 months.

Hypercholesterolemia and early coronary heart disease are present in other paternal family members. His mother is being treated for hypothyroidism. He eats a balanced diet.

On examination, the boy's height is 140.8 cm (<5th percentile for age), weight 49.1 kg (50th-75th percentile for age), body mass index 24.7 kg/ m2 (90th-95th percentile for age), and

blood pressure 98/72 mm Hg. The rest of the examination is unremarkable.

Laboratory results are as follows: total cholesterol 451 mg/dL (normal, <170), triglyceride 101 mg/dL (normal, <150), high-density lipoprotein cholesterol (HDL-C) 58 mg/dL (normal,

>35), low-density lipoprotein cholesterol (LDL-C) 373 mg/dL (normal, <110), and thyroid-stimulating hormone (TSH) 2.2 µIU/mL (normal, 0.6-5.5).

Which of the following is the most appropriate intervention for this adolescent boy?

Choose one answer .

A. Niacin

B. Colestipol

C. 3-hydroxy-3-methylglutaryl coenzyme A (HMG-CoA) reductase inhibitor

qsid=148216&aid=3885&qid=198&checksum=eb451ac91509a7e2ab54adf9bfc9e746&sesskey=9SD9XfnshK&new state=0

Page 186: Asap 2011

D. Cholestyramine

E. Gemfibrozil

Educational Objective: To learn the treatment of hypercholesterolemia in adolescents

Rationale: After nutrition therapy, there is a growing consensus that HMG-CoA reductase inhibitors are now the first choice for treating many high-risk children and adolescents with

familial hypercholesterolemia,especially in a family in which the parent has such early expression of coronary heart disease. The US Food and Drug Administration (FDA)-approved

statins for this purpose showing 2-year efficacy and safety include lovastatin,simvastatin, pravastatin, atorvastatin, and rosuvastatin. Niacin is not recommended in the pediatric

population. Colestipol and cholestyramine have been used in children, but compliance is poor due to gastrointestinal side effects. Cholestyramine has generally demonstrated LDL-C

reductions of 15% to20%. This is not adequate for this patient. There are not enough studies regarding gemfibrozil use in children.

References

McCrindle BW, Urbina EM, et al. Drug therapy of high-risk lipid abnormalities in children and adolescents: a scientific statement from the American Heart Association

Atherosclerosis,Hypertension, and Obesity in Youth Committee, Council of Cardiovascular Disease in the Young, with the Council on Cardiovascular Nursing. Circulation. 2007;

115:1948-1967 .

Arambepola C, Farmer AJ, Perera R, et al. Statin treatment for children and adolescents with heterozygous familial hypercholesterolaemia: a systematic review and meta-analysis.

Atherosclerosis. 2007; 195:339-347 .

Colletti RB, Neufeld EJ, RoffNK, et al. Niacin treatment of hypercholesterolemia in children. Pediatrics . 1993; 92:78-82 .

Tonstad S,Knudtzon J, Sivertsen M, et al. Efficacy and safety of cholestyramine therapy in peripubertal and prepubertal children with familial hypercholesterolemia.J Pediatr. 1996;

129:42-49 .

Incorrect

Marks for this submission: 0.00/1.00.

Question 14

Marks: 1.00

Page 187: Asap 2011

A 17-year-old male with a history of asthma presents to the emergency department with dizziness, polydipsia,polyuria, and polyphagia over 1 month. He lost 9 kg over the past few

months. On examination, his blood pressure is 154/71 mm Hg (>95th percentile for age), heart rate131 beats/minute, respiratory rate 19 breaths/minute, and temperature36°C. His height is 166 cm (10th percentile for age), weight 93 kg (97th percentile for age), and body

mass index (BMI) 34 kg/m2 (>97th percentile for age). Hehas moderate acanthosis nigricans. The rest of the examination is unremarkable.

Finger stick glucose is 500 mg/dL(normal fasting glucose, 70-100).

The patient is started on intravenous fluids and insulin. Therapeutic lifestyle changes are started.Fasting laboratory evaluation 2 months later shows the following: total cholesterol

214 mg/dL (normal, < 170), triglycerides 370 mg/dL (normal, <150), high-density lipoprotein (HDL) cholesterol 40 mg/dL (normal, >35), and low-density lipoprotein (LDL) cholesterol 100

mg/dL (normal, <110). Hishemoglobin A1C level has decreased from 13% to 10% (normal, <6.5).

Which of the following would be the next step in the management of this young man's dyslipidemia?

Choose one answer .

A. Decrease simple sugars and calories in the diet, increase physical activity, and repeat lipid profile in 3 months

B. Initiate gemfibrozil along with therapeutic lifestyle changes

C. Initiate 3-hydroxy-3-methyl-glutaryl coenzyme A reductase inhibitor along with therapeutic lifestyle changes

D. Initiate niacin along with therapeutic lifestyle changes

Educational Objective: To understand the management of dyslipidemiain children and adolescents with diabetes mellitus

Rationale: This young man most likely has type 2 diabetes mellitussupported by the presence of acanthosis nigricans and obesity. In such patients with type 2 diabetes mellitus, triglyceride levels usually decline with better glycemic control. Therefore, initial therapy for

hypertriglyceridemia is behavioral modification with weight loss, reduction in

qsid=148217&aid=3885&qid=204&checksum=cd6591e5adaf5ea3bc8372905605fb44&sesskey=9SD9XfnshK&new state=0

Page 188: Asap 2011

simplecarbohydrates, and increased physical activity. Drug therapy is considered if triglyceride levels are greater than 1000 mg/dL because a significant risk of pancreatitis is

present at this level.

References

American Diabetes Association.Management of dyslipidemia in children and adolescents with diabetes. Diabetes Care. 2003; 26:2194-2197.

Starc TJ,Shea S, Cohn LC, et al. Greater dietary intake of simple carbohydrate is associated with lower concentrations of high-density-lipoprotein cholesterol in hypercholesterolemic

children. Am JClin Nutr . 1998; 67:1147-1154 .

Incorrect

Marks for this submission: 0.00/1.00.

Question 15

Marks: 1.00

Which ONE of the following statements regarding serum lipid levels in the pediatric population is correct ?

Choose one answer .

A. High-density lipoprotein cholesterol (HDL-C) is similar in boys and girls in the first decade; thereafter, HDL-C increases in girls but remains relatively constant in

boys

B. Total cholesterol (TC) levels are steady during the prepubertal years, rise slightly during puberty, and decrease thereafter

C. For children, the acceptable upper limits of the levels of total cholesterol and low-density lipoprotein cholesterol (LDL-C) are 180 mg/dL and 100 mg/dL,

respectively

D. Adolescents who are at risk for obesity (body mass index [BMI] ≥85th and <95th percentile) tend to have serum lipid levels (TC, HDL-C, LDL-C, triglycerides) within

the 25th to 75th percentile

qsid=148218&aid=3885&qid=206&checksum=4c5710964f1cad1bc8b7a9ebd71c30fc&sesskey=9SD9XfnshK&new state=0

Page 189: Asap 2011

E. Non-Hispanic black children have higher levels of HDL-C and of LDL-C than Hispanic or non-Hispanic whites

Educational Objective: To understand how the lipid levels vary in children

Rationale: The acceptable upper limit for LDL-C level in children is 110 mg/dL (not 100 mg/dL). The upper limit for total cholesterol in children is 170 mg/dL (not 180 mg/dL). The Third National Health and Nutrition Examination Survey and the Lipid Research Clinic data

showed that after the first decade boys experience a decrease in HDL-C levels, and triglycerides rise in both boys and girls during childhood. Non-Hispanic blacks have higher

HDL-C but also higher LDL-C levels than Hispanic or non-Hispanic white children.

Adolescents who are at risk for obesity (BMI ≥85thand <95th percentile) have a higher prevalence of high LDL-C(>95th percentile), low HDL-C (<5th percentile),high triglycerides

(>150), and hypertension (blood pressure ≥95thpercentile) compared with adolescents with normal BMI (<85thpercentile). This suggests that even children who are at risk of being

overweight should potentially be screened for dyslipidemia with a fasting lipid profile.

References

American Academy of Pediatrics.National Cholesterol Education Program: Report of the expert panel on blood cholesterol levels in children and adolescents. Pediatrics . 1992;

89:525-584 .

Hickman TB, Briefel RR, CarrollMD, et al. Distributions and trends of serum lipid levels among United States children and adolescents ages 4-19 years: data from the third national

health and nutrition examination survey. PrevMed . 1998; 27:879-890 .

Jago R, Harrell JS, McMurrayRG, et al. Prevalence of abnormal lipid and blood pressure values among anethnically diverse population of eighth-grade adolescents and screening

implications. Pediatrics . 2006;117:2065-2073 .

Daniels SR,Greer FR, Committee on Nutrition. Lipid screening and cardiovascular health in childhood. Pediatrics. 2008; 122:198-208 .

Incorrect

Marks for this submission: 0.00/1.00.

Question 16

Marks: 1.00

qsid=148219&aid=3885&qid=209&checksum=11195d812ca3d949ed10ccfd68e1170b&sesskey=9SD9XfnshK&new state=0

Page 190: Asap 2011

Which of the following statements regarding nutrition therapy in children is correct ?

Choose one answer .

A. In children over the age of 8 years, drug therapy is recommended when low-density lipoprotein cholesterol (LDL-C) is greater than 100 mg/dL with a family

history of cardiovascular risk factors

B. Children with hypercholesterolemia should receive no more than 20% of calories from fat

C. Plant sterols paradoxically elevate total cholesterol and LDL-C in children

D. Children with dyslipidemia are at increased risk for becoming adults with serum lipid disorders

E. Daily use of oat bran cereals significantly decreases LDL-C in children

Educational Objective: To review nutrition therapy for dyslipidemiain children

Rationale: Children with dyslipidemia are at increased risk for becoming adults with serum lipid disorders. Drug therapy is recommended in children who are older than 8 years of age

with elevated LDL-C >190 mg/dL without other cardiovascular risk factors or >160 mg/dL with one or more additional cardiovascular risk factors. The National Cholesterol Education

Program (NCEP) and the American Academy of Pediatrics (AAP) recommend a low-fat(an average of no more than 30% of total calories and no less than 20% of total calories), low-

saturated-fat (<10% of total calories), and low-dietary-cholesterol (<300 mg/day) diet for all healthy children older than 2 years. For children with dyslipidemia, the NCEP generally

recommends a diet similar to that suggested for all healthy children. If following this diet for 3 months does not result in an acceptable LDL-C level, further reduction of saturated fat

intake to <7% of total calories and cholesterol intake to<200 mg/day is recommended. Short-term studies have demonstrated plant sterols to be useful adjuncts to lower total

cholesterol and LDL-C levels in children.Daily use of oat bran cereals did not reduce LDL-C (–6 mg/dL, NS) but did significantly reduce apolipoprotein B (a better long-term event

predictor in adults) by 11 mg/dL. Thus, their use may be recommended.

References

American Academy of Pediatrics. National Cholesterol Education Program: Report of the expert panel on blood cholesterol levels in children and adolescents. Pediatrics .

1992;89:525-584 .

Daniels SR, Greer FR, Committeeon Nutrition. Lipid screening and cardiovascular health in childhood. Pediatrics.2008; 122:198-208.

Page 191: Asap 2011

Moruisi KG, Oosthuizen W,Opperman AM. Phytosterols/stanols lower cholesterol concentrations in familial hypercholesterolemic subjects: A systematic review with meta-

analysis. J Am Coll Nutr . 2006; 25:41-48 .

GoldK, Wong N, Tong A, et al. Serum apolipoprotein and lipid profile effects of anoat-bran-supplemented, low-fat diet in children with elevated serumcholesterol. Ann N Y Acad

Sci .1991; 623:429-431 .

Incorrect

Marks for this submission: 0.00/1.00.

You are involved in the management of a 58-year-old East Indian woman, status post myocardial infarction (MI) andstents in the right coronary and left anterior descending

coronary arteries 2days previously. She was on simvastatin 40 mg daily prior to her MI and is nowon clopidrogil 75 mg daily and aspirin 81 mg daily .

Physical examination: height, 62in, weight, 139 lb, waist circumference 34 in, blood pressure, 110/70, pulse 76beats/minute. The remainder of her examination is

unremarkable.

Laboratory studies on admission: total cholesterol 189 mg/dL (normal,<180), triglyceride 285 mg/dL (normal, <150), high-density lipoprotein(HDL) cholesterol, 33 mg/dL (>50), low-

density lipoprotein (LDL) cholesterol, 99 mg/dL (normal<100), high-sensitivity C-reactive protein (hsCRP), 8.8 mg/dL (low risk level <1), A1C, 6.3% (normal, <6.5%) .

Question 17

Marks: 1.00

Which of the following statements is correct regarding this patient's risks?

Choose one answer .

A. This patient is at high risk for a recurrent cardiovascular event

B. This patient is at high risk of developing diabetes mellitus while on statin therapy

C. This woman with coronary artery disease is more likely to have a recurrent

qsid=148221&aid=3885&qid=212&checksum=de8de2f43b362bc31edc37f754f8a44f&sesskey=9SD9XfnshK&new state=0

Page 192: Asap 2011

D. The use of clopidrogel is contraindicated in this patient

Educational Objective: To understand goals and risks following acute coronary syndrome

Rationale: In patients with acute coronary syndrome, the greatest risk for a recurrent cardiovascular event was seen in patients with a low density lipoprotein (LDL) cholesterol

>70 mg/dL and a high sensitivity C-reactive protein (HS-CRP) >2. The lowest risk was seen in patients with LDL-cholesterol <70 mg/dL and Hs-CRP >2.

The patient has four criteria for the "metabolic syndrome": dyslipidemia; impaired fasting glucose/impaired glucose tolerance by the HbA1c criteria of the American Diabetes

Association;an elevated waist circumference for women of East Indian (South Asian) origin by the International Diabetes Federation. She has an increased risk of progression to

diabetes mellitus. There is no contraindication to clopidrogel therapy in this patient. It is standard therapy to treat patients with acute coronary syndrome and post coronary artery

stenting with clopidrogel and aspirin therapy.

References

RidkerPM, Cannon CP, Morrow D, et al. C-reactive protein levels and outcomes after statin therapy. N Engl J Med. 2005; 352:20-28.

Ridker PM, Danielson E, Fonseca FA, et al. Reductionin C-reactive protein and LDL cholesterol and cardiovascular event rates after initiation of rosuvastatin: a prospective

study of the JUPITER trial. Lancet.2009; 373:1175-1182.

Ridker PM, Danielson E, Fonseca FA, et al.Rosuvastatin to prevent vascular events in men and women with elevated C-reactive protein. N Engl J Med. 2008; 359:2195-2207 .

Incorrect

Marks for this submission: 0.00/1.00.

Question 18

Marks: 1.00

Which ONE of the following treatment options is best to optimize cardiovascular risk reduction for this woman?

Choose one answer .

qsid=148222&aid=3885&qid=213&checksum=50e7f7f540e96df54585711ef732d415&sesskey=9SD9XfnshK&new state=0

Page 193: Asap 2011

A. Change simvastatin 40 mg to atorvastatin 80 mg daily .

B. Start a fibrate to optimize control of triglyceride levels and lower the risk of a recurrent event .

C. Start pharmacotherapy to prevent progression to diabetes

D. Add omega-3 fatty acids .

Educational Objective: To understand the therapeutic options following acute coronary syndrome (ACS)

Rationale: In patients with ACS, atorvastatin 80 mg daily is superior to the other 3-hydroxy-3-methyl-glutaryl coenzyme A reductaseinhibitors in the reducing the risk of a recurrent

cardiovascular event. To date, there are no studies showing any benefit to post-ACS cardiovascular risk from lowering triglyceride levels with fibrates or even for primary or

secondary prevention once low densitylipoprotein (LDL) cholesterol is at goal level.

Pharmacotherapy (metformin) has been shown to be effective in preventing the development of diabetes mellitus(DM) in persons with impaired fasting glucose (IFG) or

impaired glucosetolerance (IGT). However, metformin has not been approved by the FDA for the prevention of DM in patients at risk for the disorder. Therapeutic lifestyle changes have

been shown to be effective in preventing the development of DM inpatients with IFG or IGT. At this point, therapeutic lifestyle change is the treatment of choice for this patient's pre-

diabetes state.

Omega-3 fatty acids have minimal effects on LDL-cholesterol. Atorvastatin at 80 mg daily should effectively control the mildly elevated triglyceride levels in this patient.

References

de Lemos JA, Blazing MA,Wiviott SD, et al. Early intensive versus delayed conservative simvastatin strategy in patients with acute coronary syndromes: phase Z of the A to Ztrial.

JAMA. 2004; 292: 1307-1316.

Knowler WC, Barrett-Conner E,Fowler SE, et al: Reduction in the incidence of type 2 diabetes with life style intervention or metformin N Engl J Med2002; 346:393-403.

DeFronzo RA, Banerji M, Bray GA, et al. Actos Now for the prevention of diabetes (ACT NOW) study. BMC Endocr. Disord. 2009: 9; 17-24.

Hoy SM, Keating GM. Omega-3ethylester concentrate: a review of its use in secondary prevention post myocardial infarction and the treatment of hypertriglyceridaemia. Drugs.

2009; 69:1077-1105.

Page 194: Asap 2011

Incorrect

Marks for this submission: 0.00/1.00.

Question 19

Marks: 1.00

A 67-year-old woman with type 2diabetes mellitus and hyperlipidemia has been intolerant of every available statin,developing profound muscle weakness and some myalgias, though she has never had any documented evidence of elevations of creatinine kinase. She is on a

combination of a moderate dose of thiazolidinedione (TZD) and metformin 1500mg/day.

Biochemical profile: HbA1C, 6.7% (normal, <6.5%), total cholesterol256 mg/dL (normal, <180), triglycerides, 190 mg/dL (normal, <150), high-density lipoprotein (HDL) cholesterol,

38 mg/dL (normal, >50), low-density lipoprotein (LDL) cholesterol,180 mg/dL (normal, <100), alanine transferase (ALT), 55 U/L (normal,<50), thyroid-stimulating hormone (TSH), 7.5

µIU/mL (normal, 0.3-5).

Which ONE of the following factors may account for the patient's intolerance to 3-hydroxy-3-methyl-glutaryl coenzyme A (HMG-CoA) reductase inhibitors(statins) ?

Choose one answer .

A. Elevated ALT

B. TZD therapy

C. Hypothyroidism

D. Type 2 diabetes mellitus

E. Female gender

Educational Objective: To recognize possible sources of intolerance to HMG-CoA reductase inhibitors (statins)

qsid=148223&aid=3885&qid=216&checksum=6a16da155e43e1f365df710d4956ac35&sesskey=9SD9XfnshK&new state=0

Page 195: Asap 2011

Rationale: Hypothyroidism has been associated with statin intolerance. Individuals older than 65 years of age have been reported to have an increased frequency of statin

intolerance. Statin intolerance mayrun in families, and two recent studies have verified a significantly high incidence of statin intolerance in patients with a gene variation,

termedSLCO1B1. Liver dysfunction especially as mild as this and likely due to fatty infiltration of the liver is not associated with statin intolerance. Type 2diabetes mellitus, TZD therapy,

and female gender have not been reported to be associated with an increased frequency of statin intolerance.

References

Ahmed W, Khan N, Glueck CJ, etal. Low serum 25(OH) vitamin D levels (<32 ng/mL) are associated with reversible myositis-myalgia in statin treated patients. Transl Res. 2009;

153:11-16.

Voora D, ShahSH, Spasojevic I, et al. The SLCO1B1*5 genetic variant is associated with statin induced side effects. J Am Coll Cardiol. 2009; 54:1609-1616

Incorrect

Marks for this submission: 0.00/1.00.

Question 20

Marks: 1.00

A 56-year-old man with hyperlipidemia and type 2 diabetes mellitus on a thiazolidinedione (TZD) andmetformin is intolerant of therapy with 3-hydroxy-3-methyl-glutaryl coenzyme A

(HMG-CoA)reductase inhibitors (statins).

Which ONE of the following therapeuticoptions would be best for this patient?

Choose one answer .

A. Fenofibrate

B. Ezetimibe

C. Niacin

qsid=148224&aid=3885&qid=222&checksum=ea7e7e1b17a4392abe8f22a006fb29e0&sesskey=9SD9XfnshK&new state=0

Page 196: Asap 2011

D. Stop the TZD

E. Colesevelam

Educational Objective: To understand alternative therapies inpatients with intolerance to HMG-CoA reductase inhibitors (statins)

Rationale: Colesevelam is the next best therapy for effective control of this patient's hyperlipidemia. Colesevelam may be associated with amild elevation in serum triglycerides,

which is not considered to be a significant cardiovascular risk. Niacin in therapeutic doses will aggravate hyperglycemia in patients with diabetes mellitus and can be considered in

addition to colesevelam if colesevelam alone is not able to achieve target lipid levels. Niacin usually increases high-density lipoprotein cholesterol (HDL-C) levels by 13% to 16% and

causes a decrease in triglyceride levels of about 16% to 24% and a decrease in low-density lipoprotein cholesterol (LDL-C) levels of 8% to 11%. If niacin is necessary for therapy in this

patient,an adjustment in her antidiabetes therapy may be necessary. Hopefully, a lower dose of niacin will be needed in conjunction with coleselam to achieve target lipid levels.

The fibrates will decrease triglyceride levels by 26% to 35% and increase HDL-C levels around 10%, with a minimal 2% to 6% decrease in LDL-C levels. They would therefore not help this

patient to achieve target lipid levels.

Omega-3 fatty acids in therapeutic doses would decrease triglycerides and total cholesterol levels between 6% and10%, with minimal changes in HDL-C and LDL-C levels. Thus, this

patient would not achieve goal LDL-C levels.

TZDs increase HDL-C by 10% to 13%and have variable effects on triglycerides. Pioglitazone lowers serum triglycerides by 16% to 18%, whereas rosiglitazone may increase serum

triglycerides by 8% to12% and both are usually associated with an increase in LDL-C level.Importantly, glycemic control is positively affected. Discontinuation would not

appreciably lower the LDL-C and would impact glycemic control negatively.

Ezetimibe lowers LDL-C level by 15%to 20% but used alone has insignificant effects on HDL-C and triglycerides and no effect on glucose control. Recent studies have failed to show any

significant benefit on cardiovascular events or carotid intimal-media thickness in spite of the LDL lowering. Thus, at present, primary therapy with ezetimibe would not be a first choice

for the treatment of hyperlipidemia in this patient intolerant to statins.

References

Ahmed W, Khan N, Glueck CJ, etal. Low serum 25(OH) vitamin D levels (<32 ng/mL) are associated with reversible myositis-myalgia in statin treated patients. Transl Res. 2009;

153:11-16.

Voora D, Shah SH, SpasojevicI, et al. The SLCO1B1*5 genetic variant is associated with statin induced side effects. J Am Coll Cardiol. 2009; 54:1609-1616.

Page 197: Asap 2011

Incorrect

Marks for this submission: 0.00/1.00.

Question 21

Marks: 1.00

Which one of the following biochemical features is MOST LIKELY to be seen in the patient with insulin resistance ?

Choose one answer .

A. Increased levels of large, fluffy low-density lipoprotein (LDL) cholesterol particles

B. Low apolipoprotein B (<60 mg/dL)

C. Lower levels of free fatty acids (FFA) throughout the day

D. Increased concentration of sex hormone binding globulin

E. Increased levels of postprandial triglyceride-rich lipoproteins

Educational Objective: To review the pathophysiology of diabetic dyslipidemia

Rationale: Insulin resistance is manifest in adipose tissue by the inability to block the release of FFA from adipose tissue, so that higher, not lower, levels of FFA remain in the circulation.

At the same time, these fatty acids are combined with glycerol in the liver to form excess triglycerides, resulting in increased hepatic secretion of very-low-density lipoprotein (VLDL).

Triglycerides are not adequately broken down by lipoprotein lipase, whose activity is diminished by insulin resistance, and the lipoprotein lipase is saturated by the increased

triglyceride levels, giving rise to increased triglyceride levels in the fasting and postprandial states. Increased VLDL in the circulation, through interactions with LDL and high-density

lipoprotein (HDL) via cholesterol ester transfer protein, enrich these lipoproteins with triglycerides, which after reacting with hepatic lipase result in the classic increased small,

dense LDL and apolipoprotein B and decreased HDL.

qsid=148225&aid=3885&qid=225&checksum=c110b394a55d29a5cfce017f3b5b22eb&sesskey=9SD9XfnshK&new state=0

Page 198: Asap 2011

Impaired glucose tolerance, or the insulin resistant syndrome, in women in the child bearing years results in hyperinsulinemia which with a susceptible ovary results in

hyperandrogenism and a lowering of sex hormone binding globulin.

Reference

Ginsberg HN, Zhang YL, Hernandez-Ono A. Regulation of plasma triglycerides in insulin resistance and diabetes. Arch Med Res. 2005; 36:232-240 .

Incorrect

Marks for this submission: 0.00/1.00.

Question 22

Marks: 1.00

A 30-year-old man receives a hearttransplant due to idiopathic dilated cardiomyopathy. He has no family history of coronary artery disease or hyperlipidemia. He has no history of

diabetes mellitus or hypertension and is a nonsmoker.

His fasting lipid profile prior to his transplant was: total cholesterol 167 mg/dL (normal, <180) low-density lipoprotein (LDL) cholesterol 98 mg/dL (normal, <100), high-density

lipoprotein (HDL) cholesterol 40 mg/dL (normal, >45), and serum triglycerides148 mg/dL (normal, <150).

Which of the following statements best describes this patient's risk of hyperlipidemia following transplantation?

Choose one answer .

A. He has a lower risk of post-transplant hyperlipidemia due to the effect of his immunosuppressive medications

B. There will be no change in his risk of hyperlipidemia post transplantation

C. He may be at increased for hyperlipidemia due to the effect of his immunosuppressive medications

D. All patients, posttransplantation, develop hyperlipidemia and require

qsid=148226&aid=3885&qid=227&checksum=a041aaad362e9ef81356c3afd6d2d867&sesskey=9SD9XfnshK&new state=0

Page 199: Asap 2011

medications to lower cholesterol levels

E. He is at decreased risk because transplant medications increase HDL levels

Educational Objective: To review prevalence rates of hyperlipidemiapost transplantation

Rationale: Hyperlipidemia is very common following solid-organ transplantation. Prevalence rates are highest in cardiac transplant recipients, reportedly as high as 60% to 80%. The

etiology of post-transplant hyperlipidemia is multifactorial and likely reflects the combination of traditional risk factors and the impact of factors unique to transplant

recipients, such as the use of immunosuppressive medications.

References

Miller LW, Schlant RC, Kobashigawa J, et al. 24th Bethesda conference:Cardiac transplantation. Task Force 5: Complications. J Am Coll Cardiol. 1993;22:41-54 .

Bilchick KC, Henrikson CA, Skojec D, et al. Treatment of hyperlipidemia in cardiac transplant recipients. Am Heart J. 2004; 148:200-210 .

Ballantyne CM, Radovancevic B, Farmer JA, et al. Hyperlipidemia after heart transplantation: report of a 6-year experience, with treatment recommendations. J Am Coll Cardiol. 1992;

19:1315-1321

Incorrect

Marks for this submission: 0.00/1.00.

Question 23

Marks: 1.00

A 40-year-old Caucasian male with history of end-stage renal disease (ESRD) due to diabetes mellitus received a kidney transplant 1 year ago. He has had no episodes of rejection and his

kidney function has been stable, with a creatinine of 1.2 mg/dL (normal,0.8-1.3) and an estimated glomerular filtration rate of 75 ml/min (normal,>60). His immunosuppressive

regimen consists of mycophenolate mofetil,prednisone, and cyclosporine. He has no family history of diabetes mellitus or cardiac disease.

A routine fasting lipid panel is as follows: total cholesterol, 228 mg/dL (normal, <180) ;

qsid=148227&aid=3885&qid=230&checksum=a3196ecd97bd699379817c7746fe49e5&sesskey=9SD9XfnshK&new state=0

Page 200: Asap 2011

low-density lipoprotein (LDL) cholesterol, 147mg/dL (normal, <100); high-density lipoprotein (HDL) cholesterol, 32 mg/dL(normal, >45); serum triglycerides, 248 mg/dL (normal, <150) .

Which of the following options is an appropriate intervention at this point ?

Choose one answer .

A. Continue to observe with repeat fasting lipid profile in 6 months

B. Refer patient for dietary counseling and therapeutic lifestyle changes (TLC), with repeat of fasting lipid profile in 3 months

C. Change immunosuppressive regimen

D. Refer patient for dietary counseling and TLC and start patient on gemfibrozil

E. Initiate dietary counseling and TLC and start therapy with atorvastatin

Educational Objective: To discuss target LDL cholesterol goals in the renal transplant recipient

Rationale: Management of hyperlipidemia in kidney transplant patients should follow the National Kidney Foundation Kidney/Disease Outcome Quality Improvement (NKF/KDOQI)

guidelines, which place patients with chronic kidney disease (including renal transplant) in the highest risk category. This was based on evidence that the 10-year cumulative risk for

coronary heart disease in this population is at least 20%, equivalent to that seen in patients with previous cardiovascular disease. As a result, target goals for LDL cholesterol are <100

mg/dL. For an LDL cholesterol range between 100 and 129mg/dL, NKF guidelines recommend initiation of TLC followed by 3-hydroxy-3methyl-glutaryl coenzyme A reductase

inhibitor (a statin) therapy in 3 months if the LDL cholesterol level fails to fall below 100 mg/dL. For LDL cholesterolvalues >130 mg/dL, TLC should be initiated concurrent with

pharmacologic therapy, with initial choice being a statin. National Cholesterol Education Program guidelines have no specific recommendations for transplant patients,and the risk

stratification is the same as for the general population. Changing immunosuppressive regimen is not a recommendation of the NKF; however, when target lipid levels are not

achievable by medical management, thought should be given to changing the immunosuppressive regimen. Adjustments are best made by the nephrologist.

References

Kasiske B, Cosio FG, Beto J, et al; National Kidney Foundation. Clinical practice guidelines for managing dyslipidemias in kidney transplant patients: a report from the Managing

Page 201: Asap 2011

Dyslipidemias in Chronic Kidney Disease Work Group of the National Kidney Foundation Kidney Disease Outcomes Quality Initiative. Am J Transplant. 2004; 4(suppl 7):13-53 .

Executive Summary of The Third Report of The National Cholesterol Education Program (NCEP) Expert Panel on Detection, Evaluation, And Treatment of High Blood Cholesterol

In Adults (Adult Treatment Panel III). JAMA. 2001; 285:2486-2497 .

Artz MA, Boots JM, Ligtenberg G,etal. Improved cardiovascular risk profile and renal function in renal transplant patients after randomized conversion from cyclosporine to tacrolimus. J

Am Soc Nephrol. 2003; 14:1880-1888

Incorrect

Marks for this submission: 0.00/1.00.

Question 24

Marks: 1.00

A 55-year-old African-American male with past medical history significant for myocardial infarction 10 years previously, hyperlipidemia, and diabetes mellitus received a renal

transplant 3 years ago. His immunosuppressive regimen consists of cyclosporine, prednisone,and mycophenolate mofetil. Immediately post transplantation he was continued

on his pre-transplant dose of atorvastatin 20 mg daily but this was increased at 3months post transplantation to 40 mg daily, with resulting improvement in his lipid parameters to

target levels.

Three years later his fasting lipid panel is as follows: total cholesterol, 258 mg /dL (normal, <180); low-density lipoprotein (LDL) cholesterol, 155 mg/dL (normal, <100); high-density

lipoprotein (HDL) cholesterol, 33 mg/dL (normal, >45); serum triglycerides,350 mg/dL. His baseline creatinine has worsened in the intervening years to 2.9 mg/dL(normal, 0.8-1.3) with

estimated glomerular filtration rate of 29 mL/min (normal, >60).

Which of the following is a potential strategy for managing his hyperlipidemia at this juncture ?

Choose one answer .

A. Increase atorvastatin to 80 mg daily

B. Leave atorvastatin dose unchanged but add gemfibrozil and counsel on diet and

qsid=148228&aid=3885&qid=232&checksum=7e72b698d29b73248a9cd25cd0d08f90&sesskey=9SD9XfnshK&new state=0

Page 202: Asap 2011

exercise

C. Reevaluate immunosuppressive regimen

D. There are no other options for managing his hyperlipidemia given his renal insufficiency

Educational Objective: To review side effects of lipid-lowering therapy, especially in high doses and in combination with immunosuppressive therapy

Rationale: There have been reports of rhabdomyolysis with high doses of statins (3-hydroxy-3-methyl-glutaryl coenzyme A inhibitors) as well as with statins in combination with fibrates,

such as gemfibrozil. Currentrecommendations are for lower doses of statins in transplant patients,especially with the concomitant use of cyclosporine. In addition, some statins need

dose adjustment in renal failure. For patients with creatinine clearances <30mL/min, atorvastatin and fluvastatin may be beneficial, as there was little observed modification of

pharmacokinetic parameters of these drugs in patients with renal failure.

The patient is already at 40 mg of atorvastatin. There is concern that increasing this further may lead to adverse side effects. As a result, considering changing his immunosuppressive regimen may have a role. Some data have indicated that conversion from cyclosporine to

tacrolimus may lead to some improvement in lipid profile parameters.

References

East C, Alivizatos PA, Grundy SM, et al. Rhabdomyolysisin patients receiving lovastatin after cardiac transplantation. N Engl J Med.1988; 318:47-48 .

Artz MA, Boots JM, Ligtenberg G,et al. Improvedcardiovascular risk profile and renal function in renal transplant patients after randomized conversion from cyclosporine to tacrolimus. J

Am Soc Nephrol.2003; 14:1880-1888 .

Launay-Vacher V, Izzedine H, Deray G.Statins' dosage in patients with renal failure and cyclosporine drug-drug interactions in transplant recipient patients. Int J Cardiol. 2005;

101:9-17 .

Incorrect

Marks for this submission: 0.00/1.00.

Question 25

Marks: 1.00

Page 203: Asap 2011

A 30-year-old Caucasian female with end-stage renal disease due to polycystic kidney disease received a kidney transplant 9 months ago. At 6 months following transplantation,

she complained of hair loss and was thus switched from a tacrolimus-based immunosuppressiveregimen to cyclosporine, prednisone, and mycophenolate mofetil. Her

baselinecreatinine remained stable at 0.9 mg/dL (normal, 0.8-1.3), with an estimated glomerular filtration rate of

86 mL/min (normal, >60).

At 3 months post transplantation,she had had a fasting lipid profile that showed: total cholesterol, 182 mg/dL(normal, <180); low-density lipoprotein (LDL) cholesterol, 98 mg/dL

(normal,<100); high-density lipoprotein (HDL) cholesterol, 55 mg/dL (normal,>50); serum triglycerides, 162 mg/dL (normal, <150). She has no family history of cardiac disease,

hyperlipidemia, or diabetes mellitus.

When should she have a repeat fasting lipid profile ?

Choose one answer .

A. Her fasting lipid profile should be repeated in 5 years

B. Her fasting lipid profile should be repeated annually

C. Lipid profiles of renal transplant recipients should be repeated at 6-month intervals

D. Her fasting lipid profile should be repeated now

E. Lipid profiles of renal transplant recipients should be repeated at 3-month intervals

Educational Objective: To review timing of hyperlipidemia monitoring in renal transplant recipients

Rationale: Management of hyperlipidemia in kidney transplant patients should follow the National Kidney Foundation/ Kidney Disease Outcome Quality Improvement (NKF/KDOQI) guidelines, which place transplant recipients in the highest cardiovascular risk. As a result, recommendations for monitoring are that lipid profiles be checked at presentation, after a

change in status,and annually.

qsid=148229&aid=3885&qid=235&checksum=02f469337266e4edf2f40e3e6e956796&sesskey=9SD9XfnshK&new state=0

Page 204: Asap 2011

This patient has had a change in her immunosuppressive regimen, which warrants repeating her lipid profile earlier than her annual screen. She has also been switched to a cyclosporine-

based regimen, which is considered to cause more hyperlipidemia than a tacrolimus-based regimen. This is in contrast to the National Cholesterol Education Program(NCEP) guidelines,

which do not have specific renal transplant guidelines.Transplant patients are risk stratified in the same manner as the generalpopulation, and given this patient's lack of additional risk

factors, NCEP guidelines would recommend rechecking lipid profile after 5 years.

References

Kasiske B, Cosio FG, Beto J, et al; National Kidney Foundation. Clinical practice guidelines for managing dyslipidemias in kidney transplant patients: a report from the Managing

Dyslipidemias in Chronic Kidney Disease Work Group of the National Kidney Foundation Kidney Disease Outcomes Quality Initiative. Am J Transplant. 2004; 4(suppl 7):13-53 .

Executive Summary of The Third Report of The National Cholesterol Education Program (NCEP) Expert Panel on Detection, Evaluation, And Treatment of High Blood Cholesterol

In Adults (Adult Treatment Panel III). JAMA. 2001; 285:2486-2497 .

Artz MA, Boots JM, Ligtenberg G,etal. Improved cardiovascular risk profile and renal function in renal transplant patients after randomized conversion from cyclosporine to tacrolimus. J

Am Soc Nephrol. 2003; 14:1880-1888

Incorrect

Marks for this submission: 0.00/1.00.

Question 26

Marks: 1.00

Following a routine clinic visit, acardiac transplant recipient's laboratory work reveals that the low-density lipoprotein (LDL) cholesterol and total cholesterol (TC) levels are elevated.

Which of the following immunosuppressive medication, if a part of the regimen, would most likely contribute to hyperlipidemia ?

Choose one answer .

A. Cyclosporine

qsid=148230&aid=3885&qid=238&checksum=7c2027663a68554508bd0b60223f0233&sesskey=9SD9XfnshK&new state=0

Page 205: Asap 2011

B. Mycophenolate mofetil

C. Tacrolimus

D. All immunosuppressive medications contribute to hyperlipidemia

Educational Objective: To discuss role of immunosuppressive medications in the etiology of post-transplantation hyperlipidemia

Rationale: Etiology of post-transplant hyperlipidemia is multifactorial and is likely due to combination of traditional risk factors and factors uniqueto transplant recipients, such as

immunosuppressive medications. Cyclosporineis associated with a dose-dependent impairment in lipid metabolism characterized by an increase in TC, LDL cholesterol, serum

triglycerides (TG),and non-high-density lipoprotein (non-HDL) cholesterol. Tacrolimus, on the other hand, has been associated with a more favorable lipid profile than cyclosporine in

numerous studies involving transplant recipients.Antiproliferative medications such as mycophenolate and azathioprine have not been shown to have an effect on lipid profiles.

Sirolimus also has a significant dose-dependent effect on TC, LDL cholesterol, and TG.

References

Deleuze S, Garrigue V, Delmas S, et al. New onsetdyslipidemia after renal transplantation: is there a difference between tacrolimus and cyclosporine? Transplant Proc. 2006 ;

38:2311-2313 .

Johnson C, Ahsan N, Gonwa T, et al. Randomized trialof tacrolimus (Prograf) in combination with azathioprine or mycophenolate mofetil versus cyclosporine (Neoral)

with mycophenolate mofetil after cadaveric kidney transplantation. Transplantation. 2000; 69:834-841 .

Wlodarczyk Z, Vitko S, Salmela K, et al; TERRA Study Group. Lipid metabolism in renal transplant patients receiving tacrolimus/sirolimus combination therapy. Transplant Proc.

2005; 37:1871-1873 .

Incorrect

Marks for this submission: 0.00/1.00.

Question 27

Marks: 1.00

Page 206: Asap 2011

Which ONE of the following serum lipid changes is characteristically associated with human immunodeficiency virus (HIV) infection ?

Choose one answer .

A. Increased low-density lipoprotein (LDL) cholesterol levels

B. Increased serum triglyceride levels

C. Increased high-density lipoprotein (HDL) cholesterol levels

D. Increased total serum cholesterol levels

Educational Objective: To recognize the characteristic lipid profile of patients with untreated HIV infection

Rationale: Hypertriglyceridemia is characteristically seen inpatients with untreated HIV infection. The more advanced the disease, the higher the serum triglyceride levels tend to

be. LDL cholesterol, HDL cholesterol, and total serum cholesterol levels tend to be lower in patients with untreated HIV infection than in the general population.

References

Riddler SA, Li X, Chu H, et al. Longitudinal changes in serum lipids among HIV-infected men on highly active antiretroviral therapy. HIV Med. 2007; 8:280-287.

GrunfeldC, Kotler DP, Shigenaga JK, et al. Circulating interferon-alpha levels and hypertriglyceridemia in the acquired immunodeficiency syndrome. Am J Med. 1991 ;

90:154-162.

Grunfeld C, Pang M, Doerrler W, et al.Lipids, lipoproteins, triglyceride clearance, and cytokines in human immunodeficiency virus infection and the acquired immunodeficiency

syndrome. JClin Endocrinol Metab. 1992; 74:1045-1052 .

Incorrect

Marks for this submission: 0.00/1.00.

Question 28

qsid=148231&aid=3885&qid=240&checksum=ca6f2298eead982a882150d8f10421bc&sesskey=9SD9XfnshK&new state=0

Page 207: Asap 2011

Marks: 1.00

Which ONE of the following drugs or combination of drugs for the treatment of human immunodeficiency (HIV) infection is most likely to be associated with atherogenic serum lipid

changes ?

Choose one answer .

A. Atazanavir-ritonavir

B. Lopinavir-ritonavir

C. Nelfinavir

D. Nevirapine

Educational Objective: To recognize which treatments for HIV infection have the greatest and lowest risk of inducing atherogenic dyslipidemia

Rationale: The protease inhibitors lopinavir-ritonavir,fosamprenavir-ritonavir and tipranavir-ritonavir have the greatest tendency to induce adverse serum lipid changes whereas

nelfinavir has a moderate capacity to do so. The protease inhibitors atazanavir-ritonavir and darunavir-ritonavirhave the least adverse effects on serum lipid levels. They are often used

in place of other protease inhibitors, when virologically appropriate, in order to have the least adverse effects on serum lipid levels and atherogenesis. Nevirapine is a non-nucleoside reverse transcriptase inhibitor with essentially no adverse atherogenic effect on serum lipid

levels.

References

Dubé MP, Stein JH, Aberg JA, et al. Guidelines for the evaluation and management of dyslipidemia in HIV-infected adults receiving antiretroviral therapy.Recommendations of the

HIV Medical Association of the Infectious Disease Society of America and the Adult AIDS Clinical Trials Group. Clin Infect Dis. 2003; 37:613-627.

Calza L, Manfredi R, Colangeli V, et al. Efficacy and safety of atazanavir-ritonavirplus abacavir-lamivudine or tenofovir-emtricitabine in patients with hyperlipidaemia switched

qsid=148232&aid=3885&qid=244&checksum=74c9308dbc094ab64d9c06dd2f29b0a5&sesskey=9SD9XfnshK&new state=0

Page 208: Asap 2011

from a stable protease inhibitor-based regimenincluding one thymidine analogue. AIDS Patient Care STDs. 2009; 23:2043-2050

Soriano V, Garcia-Gasco P, Vispo E, et al. Efficacy and safety of replacing lopinavirwith atazanavir in HIV-infected patients with undetectable plasma viraemia:final results of the

SLOAT trial. J Antimicrob Chemother. 2008; 61:200-205.

Incorrect

Marks for this submission: 0.00/1.00.

Question 29

Marks: 1.00

Which of the following serum lipid changes is commonly associated with protease inhibitor use?

Choose one answer .

A. Reduced high-density lipoprotein (HDL) cholesterol levels

B. Isolated elevation of low-density lipoprotein (LDL) cholesterol levels

C. Increased very-low-density lipoprotein (VLDL) cholesterol levels

D. Increased large, buoyant LDL particle composition

Educational Objective: To recognize the serum lipid changes associated with protease inhibitor therapy for human immunodeficiency virus(HIV) infection

Rationale: HDL cholesterol levels tend to increase with protease inhibitor use, although generally to a lesser extent than with the non-nucleoside reverse transcriptase inhibitors

such as efavirenz and nevirapine. Increased serum triglyceride levels and VLDL cholesterol levels are characteristic of many of the protease inhibitors. With protease inhibitors,there is a tendency for a greater proportion of more atherogenic small, dense LDL particles, but LDL

cholesterol is not significantly elevated.

References

qsid=148233&aid=3885&qid=247&checksum=8ca870f1a4ee3ee1b7a56c8065ebf31e&sesskey=9SD9XfnshK&new state=0

Page 209: Asap 2011

Dubé MP, Stein JH, Aberg JA, et al. Guidelines for the evaluation and management of dyslipidemia in HIV-infected adults receiving antiretroviral therapy.Recommendations of the

HIV Medical Association of the Infectious Disease Society of America and the Adult AIDS Clinical Trials Group. Clin Infect Dis. 2003; 37:613-627

Negredo E, Ribalta J, Paredes R, et al. Reversal of atherogenic lipoprotein profile in HIV-1 infected patients with lipodystrophy after replacing protease in hibitorsby nevirapine. AIDS.

2002; 16:1383-1389.

Soriano V, Garcia-Gasco P, Vispo E, etal. Efficacy and safety of replacing lopinavir with atazanavir in HIV-infected patients with undetectable plasma viraemia: final results of the

SLOAT trial. J Antimicrob Chemother. 2008;61:200-205 .

Incorrect

Marks for this submission: 0.00/1.00.

Question 30

Marks: 1.00

Which of the following agents is the optimal first-line agent or agents for elevated triglycerides levels (>500 mg/dL) in human immunodeficiency virus (HIV)-infected patients?

Choose one answer .

A. 3-hydroxy-3-methyl-glutaryl coenzyme A (HMG-CoA) reductase inhibitors (statins)

B. Extended-release niacin

C. Bile acid binding resins

D. Fish oils or fibrates

E. Insulin-sensitizers such as metformin

Educational Objective: To identify the optimum medication for hypertriglyceridemia <500 mg/dL

qsid=148234&aid=3885&qid=249&checksum=ed946f65a5717d607b713a36adc20353&sesskey=9SD9XfnshK&new state=0

Page 210: Asap 2011

Rationale: Fish oil (6 g/day) and either gemfibrozil or fenofibrateare effective treatments for hypertriglyceridemia. Fibrates plus fish oil appear to be additive. HMG-CoA reductase

inhibitors (statins) and niacin have modest triglyceride-lowering effects but are not considered first-line agents.Insulin sensitizers should not be used solely for their lipid-

lowering effects.Bile acid binding resins may increase serum triglyceride levels.

References

Dubé MP, Stein JH, Aberg JA, et al. Guidelines for the evaluation and management of dyslipidemia in HIV-infected adults receiving antiretroviral therapy.Recommendations of the

HIV Medical Association of the Infectious Disease Society of America and the Adult AIDS Clinical Trials Group. Clin Infect Dis. 2003; 37:613-627.

GerberJG, Kitch DW, Fichtenbaum CJ, et al. Fish oil and fenofibrate for the treatment of hypertriglyceridemia in HIV-infected subjects on antiretroviral therapy:results of ACTG

A5186. J Acquir Immune Defic Syndr. 2008; 47:459-466.

Dubé MP, Wu JW, Aberg JA, et al. Safety and efficacy of extended-release niacin for the treatment of dyslipidaemia inpatients with HIV infection: AIDS Clinical Trials Group Study

A5148. Antiviral Ther. 2006; 11:1081-1089 .

Incorrect

Marks for this submission: 0.00/1.00.

Question 31

Marks: 1.00

Which of the following 3-hydroxy-3methyl-glutaryl coenzyme A (HMG-CoA)reductase inhibitors (statins) carries a low risk of serious drug interactions with drugs such as human immunodeficiency virus (HIV) protease inhibitors (PIs)(suppressors of hepatic enzyme CYP

3A4) and non-nucleoside reverse transcriptase inhibitors (inducers of CYP 3A4)?

Choose one answer .

A. Pravastatin

B. Simvastatin

qsid=148235&aid=3885&qid=252&checksum=c028ba87d88dace671ee88cda5d7086c&sesskey=9SD9XfnshK&new state=0

Page 211: Asap 2011

C. Atorvastatin

D. Rosuvastatin

Educational Objective: To understand the mechanism underlying the potential adverse interaction between antiviral therapy and antilipid treatment

Rationale: Simvastatin levels increase markedly with PI therapy forHIV infection and levels of atorvastatin and rosuvastatin may increase bythree- to six-fold. Pravastatin levels increase

modestly with darunavir but tend to be lower with other PIs and may be the safest with antiretroviral drugs.

References

Dubé MP, Stein JH, Aberg JA, et al. Guidelines for the evaluation and management of dyslipidemia in HIV-infected adults receiving antiretroviral therapy.Recommendations of the

HIV Medical Association of the Infectious Disease Society of America and the Adult AIDS Clinical Trials Group. Clin Infect Dis. 2003; 37:613-627.

Fichtenbaum CJ, Gerber JG. Interactions between antiretroviral drugs and drugs used for the therapy of the metabolic complications encountered during HIV infection. Clin

Pharmacokinet. 2002; 41:1195-1211.

Fichtenbaum CJ, Gerber JG, RosenkranzSL, et al. Pharmacokinetic interactions between protease inhibitors and statins in HIV seronegative volunteers: ACTG Study A5047. AIDS.

2002; 16:569-577 .

Incorrect

Marks for this submission: 0.00/1.00.

Question 32

Marks: 1.00

Your patient has failed to respond to fenofibrate or gemfibrozil therapy for hypertriglyceridemia while onantiviral therapy for human immunodeficiency virus (HIV)

infection. He has persistently elevated serum triglyceride levels (>500 mg/dL).

Which of the following therapies is the most appropriate next agent ?

qsid=148236&aid=3885&qid=254&checksum=7b3503b241ff9539b4e4ffe0aad08b9c&sesskey=9SD9XfnshK&new state=0

Page 212: Asap 2011

Choose one answer .

A. 3-hydroxy-3-methyl-glutaryl coenzyme A (HMG-CoA) reductase inhibitor (statin)

B. Ezetimibe

C. Bile acid binding resin

D. Fish oils

Educational Objective: To identify second-line agent for treatment of hypertriglyceridemia when fibrates are not effective or are not tolerated

Rationale: Fish oils are potent triglyceride-lowering agents and are additive to fibrates in patients with HIV infection. HMG-CoA reductase inhibitor therapy is not usually effective in

decreasing serum triglyceride levels >500 mg/dL. Ezetimibe is not effective in significantly reducing triglyceride levels. Bile acid binding resins tends to raise serum triglyceride levels.

Reference

GerberJG, Kitch DW, Fichtenbaum CJ, et al. Fish oil and fenofibrate for the treatment of hypertriglyceridemia in HIV-infected subjects on antiretroviral therapy:results of ACTG

A5186. J Acquir Immune Defic Syndr.. 2008; 47:459-466 .

Incorrect

Marks for this submission: 0.00/1.00.

Question 33

Marks: 1.00

A 54-year-old woman presents to your office with the chief complaint of intermittent chest pain, not associated with exercise, lasting for a few minutes. She denies fatigue or lethargy.Family history: Patient's mother died of a myocardial infarction at age 56.

Physical examination: height5'2", weight 175 lb, calculated body mass index (BMI) 32 kg/m2, waist circumference 37". Blood pressure (BP) 142/92mm Hg, pulse 66 beats/minute and

regular. The remainder of the physical examination is unremarkable.

qsid=148237&aid=3885&qid=258&checksum=36d036d168dc85a58801f0a75fcd73c0&sesskey=9SD9XfnshK&new state=0

Page 213: Asap 2011

Laboratory values:

Total cholesterol228 mg/dL (normal,<180)

Low-density lipoprotein cholesterol (LDL-C)125mg/dL (normal, <100)

High-density lipoprotein cholesterol (HDL-C)

45 mg/dL(normal, >50)

Serum triglycerides175 mg/dL (normal,<150)

Fasting blood sugar98 mg/dL (normal, 70-100)

HbA1C5.7%) normal, <6.5%(

Thyroid-stimulating hormone (TSH)3.2 µIU/ml (normal, 0.3-4.5)

Hematocrit36)% normal, 35-45(

Hemoglobin12.5 g/dL (normal, 12-15)

What is her 10-year cardiovascular risk using the Framingham scoring system

Choose one answer .

A. <5%

B. 5% to 9.9 %

C. 10% to 14.9%

D. 15% to 20%

E. >20%

Educational Objective: To understand the use of the Framingham Cardiovascular Risk Score in the evaluation of cardiovascular risk in patients in clinical practice

Page 214: Asap 2011

Rationale: Using the Framingham Cardiovascular Risk Score system, the patient in the clinical vignette presented has 6 points for age, 4 points for total cholesterol, 1 point for HDL-C, and

3 points for systolic BP, giving a total of 14 points, which represents a 10-year risk of 2%.

Reference

Executive Summary of the Third Report of the National Cholesterol Education Program (NCEP) Expert Panel on Detection, Evaluation, and Treatment of High Blood Cholesterol in

Adults (Adult Treatment Panel III). JAMA 2001; 285:2486-2497 .

Incorrect

Marks for this submission: 0.00/1.00.

Question 34

Marks: 1.00

To minimize cardiovascular risk,guidelines for target serum lipid levels have been developed for use in clinical practice.

Which ONE of the following targets for serum lipid levels is most consistent with current guidelines for serum lipid levels in women ?

Choose one answer .

A. Low-density lipoprotein cholesterol (LDL-C) <160 mg/dL with two cardiovascular risk factors

B. High-density lipoprotein cholesterol (HDL-C) >100 mg/dL

C. Serum triglycerides <300 mg/dL

D. Non-HDL-C <130 mg/dL

Educational Objective: To identify target serum lipid levels defined by current medical practice guidelines

Rationale: The target level for LDL-C with two cardiovascular risk factors is <100 mg/dL. In women, the desirable HDL-C level is >50mg/dL. The target serum triglyceride level is <150

mg/dL. The target non-HDL-C level is <130 mg/dL. The National Cholesterol Education

qsid=148238&aid=3885&qid=259&checksum=6c9c4578856909229cb293e511a619f2&sesskey=9SD9XfnshK&new state=0

Page 215: Asap 2011

Program Adult Treatment Panel emphasizes the importance of treating elevated LDL-C to a level of ≤100 mg/dL with associated cardiovascular risk factors. Decreasing serum

triglycerides, increasing HDL-C, and decreasing non-HDL-C are hopefully achieved with treatment of elevated LDL-C levels. There are suggestive data that lowering elevated serum

triglyceride levels and non-HDL-C and increasing HDL-C may reduce cardiovascular risk/events independent of the effect of the treatment on LDL-C levels.

Reference

Mosca L, Appel LJ, Benjamin EJ, et al. American Heart Association. Evidence-based guidelines for cardiovascular disease prevention in women. Circulation. 2004; 109:672-693.

Incorrect

Marks for this submission: 0.00/1.00.

Question 35

Marks: 1.00

Which of the following statements regarding coronary artery disease in women is correct?

Choose one answer .

A. The clinical presentation of cardiac ischemia in women is the same as in men

B. In women, cardiac ischemia may be evident in the absence of occlusive coronary artery disease (CAD)

C. In women with suspected myocardial ischemia, the exercise treadmill test is an important diagnostic modality

D. Sestamibi scan is the gold standard for the diagnosis of CAD in women .

Educational Objective: To recognize the gender differences in clinical presentation and in the diagnosis of CAD in women versus men

Rationale: In women, myocardial ischemia may often be evident in the absence of occlusive CAD. The clinical presentation of myocardial ischemiain women often differs from that in

men, and it is important to recognize this factor in assessing chest pain in women. The

qsid=148239&aid=3885&qid=262&checksum=3d6deaf33414f0b2078271fe3a9bcbfc&sesskey=9SD9XfnshK&new state=0

Page 216: Asap 2011

standard treadmill exercise testis not sensitive enough for determining the presence of CAD in women. Sestamibi also has limited value in identifying CAD in women. The gold standard

for the diagnosis of CAD remains coronary angiography .

Reference

Bairey Merz CN, Shaw LJ, Reis SE, et al; WISE Investigators. Insights from the NHLBI-Sponsored Women's Ischemia Syndrome Evaluation (WISE) Study: part II: gender differences

in presentation, diagnosis,and outcome with regard to gender-based pathophysiology of atherosclerosis and macrovascular and microvascular coronary disease. J Am Coll

Cardiol.2006; 47(3 suppl):S21-S29

Incorrect

Marks for this submission: 0.00/1.00.

Question 36

Marks: 1.00

The Justification for the Use of statins in Primary prevention: an Intervention Trial Evaluating Rosuvastatin (JUPITER) trial assessed the effect of the3-hydroxy-3-methyl-glutaryl coenzyme

A (HMG-CoA) reductase inhibitor (statin),rosuvastatin, in a large number of women in the primary prevention of coronaryartery disease.

Which of the following statements is correct regarding this trial ?

Choose one answer .

A. The use of the statin was associated with a decreased incidence of diabetes mellitus in women

B. The statin was beneficial in women whose only risk factors were age and elevated high-sensitivity C-reactive protein (hsCRP) .

C. There was a <50% reduction in the need for arterial revascularization in the women treated with the statin

D. There was a paradoxical increase in all-cause mortality in the women on the statin .

qsid=148240&aid=3885&qid=264&checksum=ee94c0b9fc0a856e08796107817628fd&sesskey=9SD9XfnshK&new state=0

Page 217: Asap 2011

Educational Objective: To understand the potential impact of statin therapy in the primary prevention of coronary artery disease in women with minimal cardiovascular risk factors

Rationale: The JUPITER trial was carried out in a large cohort of women with minimal cardiovascular risk, and the statin (HMG-CoA reductase inhibitor) used in the trial was

beneficial in women whose only risk factors were age and an elevated hsCRP level. This trial has pertinence in women ≥60years of age with low-density lipoprotein cholesterol <130 mg/dL and hsCRP ≥2 mg/L. The absolute increase in nonadjudicated, physician-reported

diabetes mellitus in women receiving the statin was 0.5 per 100 person-years. This increase in the incidence of diabetes mellitus was not seen in the men in the JUPITER trial. There was

a >76% reduction in the need for arterial revascularization in the women treated with the statin. There was also a significant decrease in all-cause mortality in the women treated with

the statin.

Reference

Mora S, Glynn RJ, Hsia J, et al. Statins for the primary prevention of cardiovascular events in women with elevated high-sensitivity C-reactive protein or dyslipidemia. Circulation.

2010;121:1069-1077

Incorrect

Marks for this submission: 0.00/1.00.

Question 37

Marks: 1.00

Which of the following statements regarding risk factors for cardiovascular disease in women is correct ?

Choose one answer .

A. Women with low-density lipoprotein cholesterol (LDL-C) <100 mg/dL have not been shown to benefit from the use of a statin (3-hydoxy-3-methyl-glutaryl

coenzyme A [HMG-CoA] reductase inhibitor)

B. Hormone replacement therapy has an important role in the treatment of coronary artery disease (CAD) in women .

qsid=148241&aid=3885&qid=267&checksum=fcf7001f6940f9bba3e87644529a20e4&sesskey=9SD9XfnshK&new state=0

Page 218: Asap 2011

C. High-density lipoprotein cholesterol (HDL-C) <50 mg/dL is an independent risk factor for coronary artery disease in women, irrespective of the LDL-C level .

D. Women have a decreased response to extended release niacin compared with men .

E. Hormonal changes occurring after menopause are associated with a less atherogenic lipid profile .

Educational Objective: To recognize the potential beneficial effect of HMG-CoA reductase inhibitor(statin) therapy even in women with (LDL-C<100 mg/dL

Rationale: Women with an LDL-C level <100 mg/dL accrued substantial benefits with the use of a statin in the Heart Protection Study.Hormone replacement therapy with estrogen has

been shown to increase cardiovascular events in women. The hormonal changes of menopause are associated with a more atherogenic lipid profile. HDL-C is an independent risk factor for CAD, with low HDL-C levels being associated with an increased risk of CAD.

Women have been shown to respond to niacin therapy with a lipid-lowering effect comparable to that for men.

Reference

Heart Protection Study Collaborative Group. MRC/BHF Heart Protection Study of cholesterol lowering with simvastatin in 20,536 high-risk individuals:a randomized placebo-controlled

trial. Lancet. 2002; 360:7-22 .

Incorrect

Marks for this submission: 0.00/1.00.

Reminder:

You have three (3) attempts to successfully complete this assessment with a 75% score or higher.

You have 30 days to complete each attempt, once an attempt is started.

If you need to save your answers and come back to the assessment, you must click the 'Next' button at the bottom-left of the assessement prior to logging

out of ASAP.

Only click 'Submit all and finish' once you are satisfied with all your assessment answers.

Bottom of Form

Finish review

Skip Quiz navigation

Page 219: Asap 2011

Quiz navigation

i (Closed ) 1 (Incorrect ) 2 (Incorrect ) 3 (Incorrect ) 4 (Incorrect ) 5 (Incorrect ) 6 (Incorrect ) 7 (Incorrect ) 8 (Incorrect ) 9 (Incorrect ) 10 (Incorrect ) 11 (Incorrect ) 12 (Incorrect ) 13

(Incorrect ) 14 (Incorrect ) 15 (Incorrect ) 16 (Incorrect ) i (Closed ) 17 (Incorrect ) 18 (Incorrect ) 19 (Incorrect ) 20 (Incorrect ) 21 (Incorrect ) 22 (Incorrect ) 23 (Incorrect ) 24 (Incorrect ) 25 (Incorrect ) 26 (Incorrect ) 27 (Incorrect ) 28 (Incorrect ) 29 (Incorrect ) 30 (Incorrect ) 31 (Incorrect ) 32 (Incorrect ) 33 (Incorrect ) 34 (Incorrect ) 35 (Incorrect ) 36

(Incorrect ) 37 (Incorrect ) i (Closed )

Finish review

You are logged in as Mohammed Aldawish (Logout)

LIPIDS

 

Copyright: 2011, American Association of Clinical Endocrinologists (AACE).All material published on the Web site is the property of AACE and may not be reproduced in

any form or by any electronic meansincluding information, storage and retrieval systems without the written consent of AACE.

This activity is sponsored by the American Association of Clinical Endocrinologists.

Skip to main content

Nutrition Disorders and Management

You are logged in as Mohammed Aldawish (Logout)

Page path

Home

/ ► My courses

Page 220: Asap 2011

/ ► NUTRITION

/ ► Assessment

/ ► Nutrition Disorders and Management Assessment

/ ► Review of attempt 1

Review of attempt 1

Started onTuesday, 19 November 2013, 10:40 PM

Completed onTuesday, 19 November 2013, 10:40 PM

Time taken23 secs

Grade0.00 out of a maximum of 30.00 (0%)

FeedbackFailed

Top of Form

Directions: In this section, each item includes a lead question and a list of options labeled with letters. Select the ONE lettered option that is BEST in each case by selection the

appropriate radio button corresponding to the correct answer .

Question 1

Marks: 1.00

For which of the following is the use of the dietary supplement supported by at least one placebo controlled randomized controlled trial ?

Choose one answer .

A. Fish oil for primary prevention of myocardial infarction

9SD9XfnshK

qsid=148244&aid=3886&qid=91&checksum=de8a1b42bc29276c6a3f448ffd44237e&sesskey=9SD9XfnshK&new state=0

Page 221: Asap 2011

B. Folic acid during pregnancy for prevention of neural tube defects

C. Selenium for prevention of prostate cancer

D. Saw palmetto for lower urinary tract symptoms in men with prostatic hyperplasia

E. Gingko biloba for slowing cognitive decline in older adults

Educational objective: To be familiar with latest data supporting use of certain dietary supplements

Rationale: Many supplements are used with little data to support them. Randomized Control trials (RCTs) provide the most rigorous evidence in support of dietary supplements. Fish oil

has been shown to be of benefit for secondary prevention of cardiovascular events primarily in open label trials. Selenium, saw palmetto, and gingkobiloba demonstrated no benefit in

adequately powered RCTs; these results stand in contrast to data with folic acid .

Reference

WolffT, Witkop CT, Miller T and Syed SB, Folic acid supplementation for theprevention of neural tube defects: an update of the evidence for the U.S.Preventive Services Task Force.

Ann Intern Med, 2009. 150: p. 632-9 .

Incorrect

Marks for this submission: 0.00/1.00.

Question 2

Marks: 1.00

Supplements containing which of the following appear to have reversible effects on renal function ?

Choose one answer .

A. Chromium

qsid=148245&aid=3886&qid=94&checksum=be69985d8cf02d54c4d659d3fa38cf85&sesskey=9SD9XfnshK&new state=0

Page 222: Asap 2011

B. Creatine

C. Aristolochic acid

D. Daidzein

E. Coenzyme Q10

Educational Objective: To be aware of the reversible effect on serum creatinine with the commonly used supplement, creatine .

Rationale: Aristolochic acid can cause irreversible renal damage. Chromium, daidzein (a soyisoflavone), and coenzyme Q 10 have no significant renal effects

References

1 .Willis J, Jones R, Nwokolo N and Levy J. Protein and creatine supplements and misdiagnosis of kidney disease. BMJ. 2010; 340:5027 .

2 .Groeneveld GJ, Beijer C, Veldink JH, Kalmijn S, et al. Few adverse effects of long-term creatine supplementation in a placebo-controlled trial. Int J Sports Med. 2005; 26: 307-13 .

3 .Nortier JL and Vanherweghem JL. For patients taking herbaltherapy--lessons from aristolochic acid nephropathy. Nephrol Dial Transplant.2007; 22:1512-7 .

Incorrect

Marks for this submission: 0.00/1.00.

Question 3

Marks: 1.00

Which of the following statements is true about dietary supplements and nutraceuticals ?

Choose one answer .

A. The Food and Drug Administration (FDA) ensures that the dietary supplement is

qsid=148246&aid=3886&qid=111&checksum=7693c3b57342de93e2aba0ac0fdbe0ad&sesskey=9SD9XfnshK&new state=0

Page 223: Asap 2011

B. The manufacturer must obtain FDA approval before marketing any dietary supplement .

C. All nutraceuticals are available without prescription .

D. Disease claims can be made on labels of nutraceuticals without FDA approval

E. The FDA is expected to take action against any dietary supplement if it is judged to be unsafe after marketing .

Educational objective: To be familiar with the limited role that the Food and Drug Administration (FDA) has with regard to dietary supplements and nutraceuticals arising from

the 1994 Dietary Supplement Health and Education Act ( DSHEA)

Rationale : The FDA is expected to take action againstany dietary supplement if it is judged to be unsafe after marketing.The FDA does not ensure that a dietary supplement is safe

before marketing.The manufacturer of dietary supplements does not need to obtain FDA approval before marketing any dietary supplement. Some nutraceuticals, such as high doseextended-release nicotinic acid and aspecific formulation of a fish oil supplement

require a prescription. Disease claims can not be made on labels of nutraceuticals without FDA approval .

Reference

Administration USFaD. Dietary Supplements. 2010;Available from: http://www.fda.gov/food/DietarySupplements/default.htm .

Incorrect

Marks for this submission: 0.00/1.00.

Question 4

Marks: 1.00

Which of the following statements is correct regarding chronically critically ill patients ?

Choose one answer .

qsid=148247&aid=3886&qid=112&checksum=4c103ddfddf8a94490eaee6970b40dd5&sesskey=9SD9XfnshK&new state=0

Page 224: Asap 2011

A. The majority have marasmus-type malnutrition .

B. Most do not tolerate enteral feeding owing to delayed gastric emptying .

C. Vertebral compression fractures are common during hospital admission .

D. Less than one third of patients require insulin treatment .

E. Respiratory failure and tracheotomy are defining components of the syndrome.

Educational Objective: To understand the characteristics of thechronically critically ill patient and the appropriate management of nutrition therapy and life-support strategies

Rationale: Chronic critical illness begins when the acute illness has resolved and a tracheotomy is placed. All patients have adult kwashiorkor-like malnutrition. The majority of patients are successfully fed long-term through gastrostomy tubes. Bone hyperresorption is

common, but fractures have not been reported during hospitalization. The vast majority(>60%) have hyperglycemia requiring insulin treatment .

References

1 .Mechanick JI, Brett EM. Endocrine and metabolic issues in the management of the chronically critically ill patient. Crit Care Clin. 2002; 18:619-641 .

2 .Mechanick JI, Brett EM. Nutrition support of the chronically critically ill patient. Crit Care Clin. 2002; 18:597-618 .

3 .Mechanick JI, Brett EM. Nutrition and the chronically critically ill patient. Curr Opin Clin Nutr Metab Care. 2005; 8:33-39 .

Incorrect

Marks for this submission: 0.00/1.00.

Question 5

Marks: 1.00

qsid=148248&aid=3886&qid=113&checksum=836594047d0f60e11f22c06b2db97e2b&sesskey=9SD9XfnshK&new state=0

Page 225: Asap 2011

Typical protein requirements in patients with acute renal failure receiving continuous venovenous hemofiltration (CVVH) are within which range listed below ?

Choose one answer .

A. 0.3-0.6 g/kg/day

B. 0.7-1.0 g/kg/day

C. 1.1-1.4 g/kg/day

D. 1.5-1.8 g/kg/day

Educational Objective: To understand the appropriate protein needs for catabolic patients receiving CVVH

Rationale: Patients receiving CVVH or other forms or renal replacement therapy are more catabolic, and high amounts of protein are required to maintain positive nitrogen balance.

Protein losses due to the CVVH itself range from 1.2 to 7.5 g/day, which is less than was previously thought .

References

1 .Macias WL, Alaka KJ, Murphy MH, et al. Impact of the nutritional regimen on protein catabolism and nitrogen balance in patients with acute renal failure. JPEN J Parenter Enteral

Nutr. 1996; 20:56-62 .

2 .Mokrzycki MH, Kaplan AA. Protein losses in continuous renal replacement therapies. J Am Soc Nephrol. 1996; 7:2259-2263 .

Incorrect

Marks for this submission: 0.00/1.00.

Question 6

Marks: 1.00

qsid=148249&aid=3886&qid=114&checksum=6ca1ed0c9dc6d8b8b22759ef15ba48fc&sesskey=9SD9XfnshK&new state=0

Page 226: Asap 2011

Overfeeding carbohydrates in critically ill patients is associated with which one of the following ?

Choose one answer .

A. Respiratory difficulty

B. Dehydration

C. Hypertension

D. Hypoglycemia

E. Acute renal failure

Educational Objective: To recognize the risks of overfeeding carbohydrates

Rationale: Overfeeding carbohydrates in critically ill patients can cause several problems. Liver dysfunction (including cholestasis and steatosis) is a significant risk, especially in patients receiving parenteral nutrition. Overproduction of carbon dioxide (CO2) from

carbohydrate oxidation can impair respiratory drive. Resultant hyperinsulinemia causes fluid retention. Excess carbohydrates can worsen glycemic control in diabetic patients and

exacerbate stress hyperglycemia in non diabetic patients. Hypoglycemia may occur with insulin therapy or with the abrupt withdrawal of high carbohydrate solutions in patients not on insulin therapy. Acute renal failure and hypertension are not known risks of overfeeding

of carbohydrates .

Reference

Mechanick JI, Brett EM. Nutrition support of the chronically critically ill patient. Crit Care Clin. 2002; 18:597-618 .

Incorrect

Marks for this submission: 0.00/1.00.

Question 7

Marks: 1.00

Page 227: Asap 2011

Which of the following patients meets Medicare criteria for home parenteral nutrition ?

Choose one answer .

A. A 66-year-old woman with intestinal obstruction due to ovarian cancer that cannot be relieved by surgery

B. A 72-year-old man with severe diarrhea and abdominal pain due to Crohn's disease awaiting intestinal resection next month

C. An 88-year-old woman with dysphagia due to a recent stroke who is not expected to recover normal swallowing function

D. A 45-year-old woman with type 1 diabetes for 38 years who was recently diagnosed with gastroparesis; she has lost 10% of her usual body weight and her

serum albumin is 3.6 mg/dL

E. A 66-year-old man with severe cachexia due to colon cancer who has lost 18% of his usual body weight in the past 3 months; serum albumin is 1.7 mg/dL

Educational Objective: To recognize the conditions approved for Medicare coverage for home parenteral nutrition

Rationale: Medicare covers home parenteral nutrition "for conditions involving the small intestine and/or its exocrine glands which significantly impairs the absorption of nutrients or disease of the stomach and/or intestine which is a motility disorder and impairs the ability of

nutrients to be transported through the gastrointestinal system." The attending physician must document that the condition is of long and indefinite duration(ordinarily at least 3

months). Patients with motility disorders must be severely malnourished (albumin <3.4 and <10% usual body weight) and must have tried maximal doses of a prokinetic medication.

Reference

Centers for Medicare &Medicaid Services.Available at: https://coverage.cms.fu.com/lcd/view_article_popup_front.asp?

article_number=37215&article_version=14&contractor_id=137. Accessed October 11, 2010.

Incorrect

qsid=148250&aid=3886&qid=115&checksum=3ddc63d02523116ccfbd383be5f3fe2b&sesskey=9SD9XfnshK&new state=0

Page 228: Asap 2011

Marks for this submission: 0.00/1.00.

Question 8

Marks: 1.00

Which ONE of the following statements is correct regarding gastric residual volumes (GRV)?

Choose one answer .

A. Signs and symptoms of feeding intolerance are a better predictor of aspiration risk than GRV .

B. Persistently elevated GRV should be treated by changing to a lower-osmolality formula before starting a prokinetic agent .

C. Enteral feeds should routinely be held for GRV >150 mL .

D. GRV directly correlates with aspiration risk .

E. Critically ill, intubated patients require less frequent monitoring of GRV than more stable patients (eg, every 8 hours) .

Educational Objective: To understand the role of management of GRV in reducing aspiration risk

Rationale: All tube-fed patients should be monitored for aspiration risk. Signs and symptoms of feeding intolerance are a better predictor of aspiration risk than GRV, which does not

directly correlate with aspiration risk. GRV should initially be checked every 4 hours; persistently elevated GRV >250 mL is an indication for a prokinetic agent .

Critically ill patients have a higher incidence of delayed gastric emptying. Recommendations for monitoring GRV and holding feeds vary. Routinely withholding feeds without signs or

symptoms of feeding intolerance can result in inadequate calories delivered .

References

qsid=148251&aid=3886&qid=122&checksum=e628383ab89d869aefe3619b27dd7a48&sesskey=9SD9XfnshK&new state=0

Page 229: Asap 2011

1 .Bankhead R, Boullata J, Brantley S, et al.; A.S.P.E.N. Board of Directors. Enteral nutrition practice recommendations. JPEN J Parenter Enteral Nutr. 2009; 33:122-167.

2 .Lin HC, Citters GWV. Stopping enteral feeding for arbitrary gastric residual volume may not be physiologically sound: results of a computer simulation model. JPEN J Parenter

Enteral Nutr. 1997; 21:286-289.

Incorrect

Marks for this submission: 0.00/1.00.

Question 9

Marks: 1.00

In order to minimize both diabetes mellitus and cardiovascular risk in a healthy overweight individual, which of the following weight loss targets is considered effective, acceptable, and

safe?

Choose one answer .

A. 15% in 6 months or a weight loss of >2 lbs/week

B. 5% in 1 year or a weight loss of <0.5 lb/week

C. 7% in 6 months irrespective of the amount of weight loss/week

D. 7% or a weight loss of 0.5 to 2 lbs/week

E. >2 lbs/week

Educational Objective: To know the effects of weight loss on diabetes and cardiovascular risk reduction

Rationale: In order to minimize both diabetes and cardiovascular risk in a healthy overweight person, he or she should follow a caloric restriction diet that leads to a weight reduction of

about 7% or a weight loss of 0.5 to 2 lbs/week. This can be achieved by reducing daily caloric intake by almost 500 calories/day. The Diabetes Prevention Program found that 7% weight

qsid=148252&aid=3886&qid=127&checksum=3a28eb0c5c7f9d6ae276de79ce765081&sesskey=9SD9XfnshK&new state=0

Page 230: Asap 2011

reduction through caloric restriction and increased physical activity reduces diabetes risk by 58% in a high-risk population .

Reference

1 .Knowler WC, Barrett-Connor E, Fowler SE, et al.; Diabetes Prevention Program Research Group. Reduction in the incidence of type 2 diabetes with lifestyle intervention or

metformin. N Engl J Med. 2002;346:393-403 .

Incorrect

Marks for this submission: 0.00/1.00.

Question 10

Marks: 1.00

The recommended dietary fat intake for a normal healthy individual is which of the following?

Choose one answer .

A. 15% to 20% of total daily calories. Saturated fat < 5%

B. 20% to 35% of total daily calories. Saturated fat < 10%

C. 30% to 40% of total daily calories. Saturated fat < 15%

D. 20% to 35% of total daily calories. Saturated fat < 5%

E. 20% to 35% of total daily calories. Saturated fat < 15%

Educational Objective: To know the recommended fat intakes for a healthy diet

Rationale: Fat consumption should be between 20% and 35% of total daily calories. Saturated fat should be limited to < 10% of total caloric intake or < 7% in individuals with

low-density lipoprotein (LDL)-cholesterol > 100 mg/dL. Polyunsaturated fat should comprise up to 10% of total calories, and monounsaturated fat up to 15% to 20% of total calories. It is

qsid=148253&aid=3886&qid=131&checksum=f41b3a2bbe57e408253f0ae29afbb9ac&sesskey=9SD9XfnshK&new state=0

Page 231: Asap 2011

recommended that cholesterol consumption should be <300 mg/day or <200 mg/day in individuals with LDL-cholesterol >100 mg/dL. Replacement of saturated fats by

monounsaturated and polyunsaturated fats is encouraged however intake of polyunsaturated fats should be restricted to < 10% of total calories. Higher intakes are

associated lowering cardioprotective effects of high-density lipoprotein (HDL) cholesterol.

 

References

1 .Mata P, Garrido JA, Ordovas JM, et al . Effects of long-term monounsaturated vs. polyunsaturated- enriched diets on lipoproteins in healthy men and women. Am J Clin Nutr

1992; 55:846.

2 .Department of Health and Human Services (HHS) and the Department of Agriculture (USDA). Dietary Guidelines for Americans 2005. Available at

http://www.health.gov/dietaryguidelines/dga2005/recommendations.htm. Accessed September 23, 2006.

3 .American Diabetes Association. Nutrition recommendations and interventions for diabetes — 2006. Diabetes Care. 2006;29:2140-2157.

4 .National Cholesterol Education Program (NCEP) Expert Panel on Detection, Evaluation, and Treatment of High Blood Cholesterol in Adults (Adult Treatment Panel III). Third Report

of the National Cholesterol Education Program (NCEP) Expert Panel on Detection, Evaluation, and Treatment of High Blood Cholesterol in Adults (Adult Treatment Panel III)

final report. Circulation. 2002;106:3143—3421.

Incorrect

Marks for this submission: 0.00/1.00.

Question 11

Marks: 1.00

Which of the following statements regarding dietary and supplemental calcium intake is correct?

Choose one answer .

A. The average nondairy diet contains only 100 to 200 mg of calcium per day

qsid=148254&aid=3886&qid=134&checksum=d15bab9a7c2d0431043fbc2718b8a10c&sesskey=9SD9XfnshK&new state=0

Page 232: Asap 2011

B. Clinical trials have failed to demonstrate clear-cut superiority of one calcium supplement preparation over another in the prevention of osteoporotic fractures

C. Calcium citrate delivers more calcium per pill

D. Calcium carbonate is better absorbed in a low acidity environment

E. Calcium supplements should be administered once daily in the morning to maintain compliance and avoid missing doses

Educational Objective: To know the principles of calcium intake for a healthy diet

Rationale: Clinical trials have failed to demonstrate clear-cut superiority of one calcium supplement preparation over another in the prevention of osteoporotic fractures. Calcium

carbonate delivers more calcium per pill, but calcium citrate is better absorbed in a low acidity environment that frequently exists in the elderly. As the average nondairy diet

contains only 300 to 500 mg of calcium, it is recommended that people consume diary products with reduced fat content, such as skim milk or 1% or 2% milk products, as sources

of calcium or take calcium supplements. Calcium supplements should be administered in divided dosages of 500 to 600 mg or less per dose and taken with meals to optimize

absorption .

References

1 .Heller HJ, Stewart A, Haynes S, et al. Pharmakinetics of calcium absorption from two commercial calcium supplements. J Clin Pharmacol. 1999;39:1151-1154.

2 .Heaney RP. There should be a dietary guideline for calcium. Am J Clin Nutr. 2000;71:658-661.

Incorrect

Marks for this submission: 0.00/1.00.

Question 12

Marks: 1.00

qsid=148255&aid=3886&qid=136&checksum=4a95552396bb1a4c6c8bc734ffe2716f&sesskey=9SD9XfnshK&new state=0

Page 233: Asap 2011

Your patient is a 42 year old woman who seeks your advice for dietary change for health promotion as she seeks to better manage her weight. You calculate her body mass index

(BMI) and it is 34 kg/m2. Her waist circumference is 38 inches. She is otherwise healthy.

Which of the following recommendations would you provide her?

Choose one answer .

A. Initiate a regular exercise program with aerobic exercises and weight training for 40 minutes each day four times a week before coping with dietary

B. Change your diet to a low carbohydrate diet (<130g/day), it is the best way to lose and maintain weight .

C. Recommend gastric banding for effective and sustainable weight loss and maintenance .

D. Introduce a fatty fish containing meal twice weekly to reduce your risk for cardiovascular disease .

E. Pursue dietary change to consistently achieve a modest calorie restriction of 250 to 500 kcal/day .(

Educational objective: Emphasize the concept that calorie intake and not dietary macronutrient composition is the best determinant of effective weight management.

Rationale: Calorie intake and not dietary macronutrient (carbohydrate or fat) composition is the best determinant of effective weight management. Dietary changes to consistently

achieve a modest calorie restriction of 250-500 kcal/day should be the basic recommendation as weight reduction and maintenance can not be achieved without calorie

restriction. Exercise alone will not do it. Diet is first with the addition of regular exercise to assist with calorie expendure for weight reduction. Fatty fish meals may have a beneficial

effect on cardiovascular risk but weight reduction is the primary goal for the patient described in the clinical vignette of the question to reduce the development of co-

morbidities and cardiovascular risk. It is not appropriate to recommend gastric banding for weight reduction before an adequate trial of diet and exercise and assessment of her

cardiovascular risk factors.

References:

Sacks FM, Bray GA, Carey VJ et al. Comparison of weight loss diets with different compositions of fat, protein and carbohydrates. N Engl J Med 2009; 360(9): 859-73.

Page 234: Asap 2011

Jacobson TA. Beyond Lipids: The Role of Omega-3 Fatty Acids from Fish Oil in the Prevention of Coronary Heart Disease. Current Atherosclerosis Reports. 2007; 9:145-153.

Incorrect

Marks for this submission: 0.00/1.00.

Question 13

Marks: 1.00

An obese 50-year-old man seeks advice about weight loss after reading about low-carbohydrate diets in the newspapers .

Which of the following statements about low-carbohydrate diets is correct?

Choose one answer .

A. Low-carbohydrate diets produce superior weight loss over 6 months compared with low-fat diets .

B. Over 12 months, low-fat diets generally produce superior weight loss versus low-carbohydrate diets .

C. Low-carbohydrate diets are often associated with significant increases in low-density lipoprotein (LDL) cholesterol levels 6 months after initiation compared

with baseline .

D. Low-carbohydrate diets are generally associated with higher systolic blood pressure after 12 months versus low-fat diets .

E. Low-carbohydrate diets have less favorable effects on abdominal obesity than do low-fat diets .

Educational Objective: To know the basic information contained in the published literature about popular weight-loss diets

Rationale: There are few differences between different popular diets when they are followed for 12 months or more. However, there are few published deleterious effects of

low-carbohydrate diets from randomized clinical trials. Low-carbohydrate diets often

qsid=148256&aid=3886&qid=138&checksum=ec9b3535ca83f652e389ca5b3aed5809&sesskey=9SD9XfnshK&new state=0

Page 235: Asap 2011

produce superior weight loss over 6 months compared with low-fat diets, although by 12 months, the differences between diets usually disappear.

Low-carbohydrate diets are not associated with an increase in LDL cholesterol 6 months after initiation of the diet compared with baseline LDL cholesterol levels, and neither low-

carbohydrate diets nor low-fat diets have any significant effect on blood pressure or abdominal obesity other than through weight reduction.

References

Samaha FF, Iqbal N, Seshadri P, et al. A low-carbohydrate as compared with a low-fat diet in severe obesity. N Engl J Med. 2003; 348:2074-2081.

Brehm BJ, Seeley RJ, Daniels SR, et al. A randomized trial comparing a very low carbohydrate diet and a calorie-restricted low fat diet on body weight and cardiovascular risk factors in

healthy women. J Clin Endocrinol Metab. 2003; 88:1617-1623.

Incorrect

Marks for this submission: 0.00/1.00.

Question 14

Marks: 1.00

A 60-year-old woman with a strong family history of type 2 diabetes and a prior personal history of impaired glucose tolerance started low-dose (600 mg of eicosapentenoic acid [EPA] and docosahexeneoic acid [DHA]) n-3 fatty acid supplements 6 months ago on the

advice of her prior physician. She would like to stop taking them, as she finds the odor unpleasant .

Which of the following statement about n-3 fatty acid supplementation is correct ?

Choose one answer .

A. Clinical trials show that n-3 fatty acid supplementation can lead to a small but significant weight loss .

B. Clinical bleeding events are statistically increased with n-3 fatty acid supplementation .

qsid=148257&aid=3886&qid=142&checksum=0dee6a3dac3e679b5218a31feb4014d3&sesskey=9SD9XfnshK&new state=0

Page 236: Asap 2011

C. The risk of developing diabetes mellitus is not reduced with n-3 fatty acid supplementation .

D. The principal effect on the lipoprotein profile with n-3 fatty acid supplementation is a decrease in low-density lipoprotein (LDL) cholesterol .

E. n-3 fatty acid supplementation improves insulin sensitivity in patients with polycystic ovary syndrome .

Educational Objective: To understand the potential benefits of n-3 fatty acid supplements

Rationale: Although n-3 fatty acids can reduce serum triglycerides, they have no effect on the incidence of diabetes mellitus or on weight loss. Clinical bleeding events are not

statistically significantly increased with n-3 fatty acid supplements. LDL cholesterol usually increases with n-3 fatty acid supplementation. Insulin sensitivity is not increased with n-3

fatty acid supplements in patients with polycystic ovary syndrome.

Reference

Eslick GD, Howe PR, Smith C, et al. Benefits of fish oil supplementation in hyperlipidemia: a systematic review and meta-analysis. Int J Cardiol. 2009; 136:4-16.

Incorrect

Marks for this submission: 0.00/1.00.

Question 15

Marks: 1.00

Which of the following statements about low-glycemic-index (GI) diets is correct?

Choose one answer .

A. Low-GI diets have proven benefits in obese adults with type 2 diabetes .

B. Low-GI diets have been shown to be beneficial for women with gestational diabetes .

qsid=148258&aid=3886&qid=143&checksum=4567238e14c9bf51c38011749505a8d8&sesskey=9SD9XfnshK&new state=0

Page 237: Asap 2011

C. The GI of a food is fixed and is not affected by food preparation .

D. Rigid adherence to a low-GI diet has been associated with an increased risk of gallstones .

E. Low-GI diets in patients with impaired glucose tolerance reduce the risk of diabetes mellitus more effectively than does metformin .

Educational Objective: To be familiar with the literature concerning benefits and limitations of the low-GI diet

Rationale: The proven benefits of a low-GI diet are limited, mainly because the GI is difficult to assess in the real world. Food is usually not eaten as a single component, but is prepared

and mixed with other foods during the meal. The benefit of a low-GI diet in diabetes mellitus and/or obesity has not been established, though some benefit has been demonstrated in

the very special situation of gestational diabetes. There is no known risk of gallstones in patients on a rigid low-GI diet. Rapid weight loss may be associated with risk of gallstones.

Low-GI diets have not been clearly shown to reduce the risk of diabetes mellitus.

References

Thomas DE, Elliott EJ, Baur L. Low glycaemic index or low glycaemic load diets for overweight and obesity. Cochrane Database Syst Rev. 2007; CD005105.

Moses RG, Barker M, Winter M, et al. Can a low-glycemic index diet reduce the need for insulin in gestational diabetes mellitus? A randomized trial. Diabetes Care. 2009; 32:996-

1000.

Incorrect

Marks for this submission: 0.00/1.00.

Question 16

Marks: 1.00

A 60-year-old man comes to your office with newly diagnosed type 2 diabetes. He is started on metformin, aspirin, an angiotensin-converting enzyme (ACE) inhibitor and a 3-hydroxy-3-

methyl-glutaryl coenzyme A inhibitor. He is very motivated about improving his health and inquires which dietary supplement may be of help to him .

qsid=148259&aid=3886&qid=144&checksum=dbb77b7f7da7971ff348bf7766a7e33d&sesskey=9SD9XfnshK&new state=0

Page 238: Asap 2011

Which of the following statements regarding dietary nutrients and cardiovascular disease is correct?

Choose one answer .

A. Cinnamon has been shown to decrease glycosylated hemoglobin (A1C) levels by 0.8% .

B. Folate, vitamin B6, and vitamin B12, when taken as supplements, have been shown to reduce the risk of cardiovascular events .

C. Chromium repletion has been shown to statistically improve control of diabetes mellitus .

D. Fish oils, particularly from cold-water fish rich in n-3 polyunsaturated fats, can be useful to decrease plasma triglycerides .

E. Vitamin E as a lipid-soluble antioxidant at 400 IU/day has been demonstrated to have an effect on cardiovascular events .

Educational Objective: To be familiar with the clinical evidence pertaining to dietary supplements and cardiovascular disease

Rationale: Fish oils can be useful to decrease plasma triglycerides. Although folate, vitamin B6, and vitamin B12 can reduce homocysteine, there is no evidence that reducing their

levels actually has a causal relationship with reductions in cardiovascular disease. The effect of cinnamon on glycemia and glycemic control and the role of chromium repletion in the

treatment of diabetes mellitus have not been established. Vitamin E has not been demonstrated to have a beneficial effect on cardiovascular events.

ReferenceAmerican Diabetes Association. Evidence-based nutrition principles and recommendation for

the treatment and prevention of diabetes and related complications. Diabetes Care. 2002; 25:S50-S60 .

Incorrect

Marks for this submission: 0.00/1.00.

Question 17

Marks: 1.00

Page 239: Asap 2011

A 25-year-old woman with a history of gestational diabetes comes to your office for a 1-year follow-up after her last pregnancy. She has been well and has had a normal glucose

tolerance test since her delivery. She is very concerned about her increased lifetime risk for diabetes mellitus and wants to know what she can do to prevent the onset of diabetes

mellitus.

Which of the following statements regarding risk or progression of diabetes mellitus in women with a history of gestational diabetes is correct?

Choose one answer .

A. Green tea has been shown to decrease the progression to diabetes mellitus by 12% in patients at risk of diabetes mellitus .

B. Exercise and maintaining a healthy weight have been shown to be the most effective tools to decrease the 5-year risk of developing diabetes mellitus .

C. Thiazolidinediones are superior to diet, exercise, and healthy weight maintenance with regard to risk reduction for progression of diabetes mellitus .

D. Metformin is superior to diet, exercise, and healthy weight maintenance with regard to risk reduction for progression of diabetes mellitus .

E. Diet and exercise have limited roles in high-risk patients with a history of gestational diabetes .

Educational Objective: To be familiar with the evidence pertaining to modifiers of progression of gestational diabetes to diabetes mellitus

Rationale: Exercise and maintaining a healthy weight have been shown to be the most effective measures to decrease the 5-year risk of developing diabetes mellitus. The benefits of green tea in reducing diabetes risk or progression have not been demonstrated in sound scientific studies. Thiazolidinediones and metformin are not superior to diet, exercise, and

healthy weight maintenance with regard to risk reduction for progression of diabetes mellitus.

Reference

qsid=148260&aid=3886&qid=145&checksum=dd7a02b47dee836afcd2de4cde579d2a&sesskey=9SD9XfnshK&new state=0

Page 240: Asap 2011

Knowler WC, Barrett-Connor E, Fowler SE, et al. Reduction in the incidence of type 2 diabetes with lifestyle intervention or metformin. N Engl J Med. 2002; 346:393-403.

Incorrect

Marks for this submission: 0.00/1.00.

Question 18

Marks: 1.00

A 23-year-old woman with a body mass index (BMI) of 38 kg/m2 and a history of menstrual irregularity, facial hirsutism, and acne is referred to you for evaluation of infertility. Your

history documents that attempts at achieving a pregnancy have been unsuccessful for the past 2 years, since she was married. Her last menstrual period was 6 weeks ago. She is taking

a prenatal multivitamin pill every day but no medications otherwise.

Physical examination confirms her BMI and a waist circumference of 43 in. Her acne is mild in nature, and her facial hirsutism is adequately treated with electrolysis. She has normal

secondary sex characteristics, including normal speculum and bimanual pelvic examinations, with no adnexal masses. The uterine fundus is not palpable given her body habitus.

Abdominal exam is unremarkable, without any overt hepatosplenomegaly, and without tenderness.

Accompanying laboratory studies include a normal hemoglobin level, elevated aspartate aminotransferase (AST) and alanine aminotransferase (ALT), normal fasting glucose, normal

thyroid stimulating hormone (TSH), normal 17-hydroxyprogesterone, mildly elevated total testosterone, and a normal 24-hour urine collection for free cortisol.

The most appropriate next step in her evaluation is which of the following?

Choose one answer .

A. Qualitative urinary beta-human chorionic gonadotropin

B. Computed tomography (CT) scan of the abdomen and pelvis

C. Ultrasound of the pelvis

qsid=148261&aid=3886&qid=149&checksum=3f348df468f382b419b4d1ae1006bc59&sesskey=9SD9XfnshK&new state=0

Page 241: Asap 2011

D. Plasma prolactin

E. Fasting lipid panel

Educational Objective: To understand the appropriate assessment of a woman with infertility

Rationale: Any woman who is sexually active could be pregnant. Before exposure to x-rays is done, a pregnancy must be excluded. A CT scan of the abdomen to evaluate elevated liver

enzymes also should not be considered unless there is documentation that a pregnancy is not established. Likewise, an ultrasound of the pelvis is unwarranted. The patient does not

have any pelvic symptoms. Polycystic ovarian syndrome (PCOS), the patient's diagnosis, does not

include ovarian cysts as a diagnostic criterion.

A plasma prolactin level should be considered in any woman with menstrual irregularity, but this is not an immediate priority in this case.

PCOS is a risk factor for premature CVD, and lipid management in PCOS is important. However, medications to treat the cholesterol are contraindicated in patients who are

attempting to become pregnant.

Reference

1 .American Association of Clinical Endocrinologists Position Statement on Metabolic and Cardiovascular Consequences of Polycystic Ovary Syndrome. Endocr Pract. 2005; 11:126-

134.

Incorrect

Marks for this submission: 0.00/1.00.

Question 19

Marks: 1.00

A 43-year-old man comes to see you with a chief complaint of fatigue. His past medical history is significant for hypertriglyceridemia, low high-density lipoprotein (HDL) cholesterol

levels, hypertension controlled with an angiotensin-converting enzyme (ACE) inhibitor, and a laparoscopic cholecystectomy for symptomatic cholelithiasis. His review of symptoms is

qsid=148262&aid=3886&qid=151&checksum=a62ae1b8a80bd787f52fce8ceff84313&sesskey=9SD9XfnshK&new state=0

Page 242: Asap 2011

negative for snoring, but he does complain of erectile dysfunction. He reports that he does have occasional spontaneous morning erections.

On physical examination, he has a neck circumference of 18 in (normal <17 in), normal examination of the heart and lungs, normal peripheral pulses, and normal external genitalia

with testicles of normal size.

The next best step in evaluating this patient is which of the following?

Choose one answer .

A. Total plasma testosterone

B. Overnight oximetry

C. Plasma prolactin

D. Nocturnal penile tumescence

E. A trial of sildenafil

Educational Objective: To recognize possible sleep apnea in a man with fatigue and increased neck circumference in the absence of a history of snoring

Rationale: Despite the absence of a history of snoring, an increased neck circumference makes obstructive sleep apnea very likely, and overnight oximetry should be ordered to test

for obstructive sleep apnea. Neck circumference >17 in for men and >16 in for women is associated with an increased risk of obstructive sleep apnea.

Total plasma testosterone may be low due to sleep apnea. Plasma prolactin is not appropriate unless there is documentation of hypogonadotropic hypogonadism.

Nocturnal penile tumescence is unnecessary because the patient does have spontaneous morning erections. A trial of sildenafil may be appropriate after an evaluation to exclude

other causes of hypogonadism, including sleep apnea.

Reference

Davies RJ, Stradling JR. The relationship between neck circumference, radiographic pharyngeal anatomy, and the obstructive sleep apnoea syndrome. Eur Respir J. 1990; 3:509-

514.

Incorrect

Page 243: Asap 2011

Marks for this submission: 0.00/1.00.

Question 20

Marks: 1.00

A 34-year-old woman comes to see you with a chief complaint of unwanted weight gain from 200 to 245 lb in the past 24 months. The weight gain happened despite repeated

efforts to improve nutrition and increase physical activity. Her past medical history is remarkable for a total abdominal hysterectomy and bilateral salpingo-oophorectomy at age

30 years for cervical cancer, depression treated with paroxetine, polycystic ovarian syndrome, and impaired fasting glucose, which was first documented at age 32 years.

Her physical examination reveals acanthosis nigricans in the axillae and the neck but is otherwise unremarkable. Laboratory studies reveal an elevated follicle-stimulating hormone

(FSH) level and low estradiol and testosterone levels, a fasting glucose of 116 mg/dL (normal, 65-99 mg/dL), low HDL cholesterol, and elevated serum triglycerides.

Which of the following statements is correct regarding this patient?

Choose one answer .

A. Phentermine is contraindicated because the patient is taking paroxetine .

B. Because she has had a bilateral salpingo-oophorectomy, metformin would be of little benefit to treat her polycystic ovarian syndrome .

C. Testosterone deficiency causes a loss of lean muscle mass and a redistribution of fat to the abdomen .

D. A sulfonylurea is the best treatment for her impaired fasting glucose .

E. She should give up physical activity because it has not helped her so far .

Educational Objective: To recognize testosterone deficiency in women after bilateral oophorectomy

Rationale: Female hypogonadism from her previous bilateral salpingo-oophorectomy results in signs and symptoms of testosterone deficiency in women, including the accumulation of

qsid=148263&aid=3886&qid=155&checksum=c185b121241ce0d08f7204644f9d3521&sesskey=9SD9XfnshK&new state=0

Page 244: Asap 2011

visceral fat, fatigue, loss of lean muscle mass, loss of libido, and female sexual dysfunction. Testosterone replacement, independent of estrogen replacement, may have metabolic

benefits for these women.

Phentermine is not contraindicated with paroxetine because phentermine is not a serotonin reuptake inhibitor, and there is no risk of the serotonin syndrome. Metformin is indicated

because the patient has acanthosis nigricans (a sign of insulin resistance) and hyperglycemia. Metformin decreases the risk of progression to diabetes mellitus. Sulfonylureas would be

expected to cause a significant risk of hypoglycemia given the patient's mild fasting glucose elevation. She is a candidate for an insulin sensitizer as first-line therapy, given her

acanthosis nigricans. Finally, daily physical activity is integral to weight management for every patient.

References

Davis S. Androgen replacement in women: a commentary. J Clin Endocrinol Metab. 1999; 84:1886-1891.

Knowler WC, Barrett-Connor E, Fowler SE, et al. Reduction in the incidence of type 2 diabetes with lifestyle intervention or metformin. N Engl J Med. 2002; 346:393-403.

Incorrect

Marks for this submission: 0.00/1.00.

Question 21

Marks: 1.00

A 45-year-old woman comes to see you 2 years after undergoing a Roux-en-Y procedure for weight loss. Her weight decreased from 320 lb to 210 lb following the procedure, but she

stopped losing weight, and at her height of 5 ft 4 in, her current body mass index (BMI) classifies her as obese. Her presurgical diabetes mellitus, hypertension, and dyslipidemia are

corrected and she is no longer using medications for them. She has been very consistent taking 2 multivitamin tablets every day, and 500 mg of calcium carbonate 2x/day. She feels

fatigued and complains of mental fog and memory impairment.

On her physical examination, the patient has redundant skin folds due to the loss of subcutaneous fat but is otherwise unremarkable. Laboratory studies reveal a normal

albumin with a low total serum calcium level of 8.4 mg/dL (normal, 8.6-10.2 mg/dL). Her total 25-hydroxy-vitamin D level is 45 ng/mL (normal, 40-100 ng/mL).

Which of the following is the next best step in this patient's care?

qsid=148264&aid=3886&qid=156&checksum=1b662c147d2067b6c2a9175c88a4b324&sesskey=9SD9XfnshK&new state=0

Page 245: Asap 2011

Choose one answer .

A. Treatment with 1 capsule of vitamin D, 50,000 IU every week, to increase her 5-hydroxy-vitamin D plasma level

B. Treatment with calcitriol, 0.25-µg capsule every day

C. Discontinuation of calcium carbonate and starting treatment with calcium citrate, 950 mg 5x/day

D. Ordering a parathyroid sestamibi scan

E. Reversal of her Roux-en-Y procedure

Educational Objective: To recognize the increased risk of hypocalcemia and its appropriate management in patients post Roux-N-Y procedure

Rationale: Calcium carbonate is poorly absorbed in patients with a Roux-en-Y procedure, despite normal vitamin D levels in the blood. Calcium citrate is better absorbed in many such

individuals, and switching this patient from calcium carbonate to calcium citrate is appropriate.

Vitamin D is not indicated because the patient has normal plasma vitamin D levels. Calcitriol, the active form of vitamin D, also is not necessary when the patient has normal renal

function.

The patient is expected to have secondary hyperparathyroidism due to the hypocalcemia from poor gastrointestinal absorption of calcium. A sestamibi parathyroid scan is indicated in

the evaluation of hypercalcemia due to primary hyperparathyroidism, and not in this case. The patient has derived significant benefit from the Roux-en-Y procedure. Reversal of the

procedure is not appropriate for her at this time.

References

Mechanick JI, Kushner RF, Sugerman HJ, et al. Executive summary of the recommendations of the American Association of Clinical Endocrinologists, the Obesity Society, and American

Society for Metabolic & Bariatric Surgery medical guidelines for clinical practice for the perioperative nutritional, metabolic, and nonsurgical support of the bariatric surgery patient.

Endocr Pract. 2008; 14:318-336.

Goode LR, Brolin RE, Chowdhury HA, Shapses SA. Bone and gastric bypass surgery: effects of dietary calcium and vitamin D. Obes Res. 2004; 12:40-47.

Incorrect

Page 246: Asap 2011

Marks for this submission: 0.00/1.00.

Question 22

Marks: 1.00

A 68-year-old man comes to see you for help with weight loss. He has degenerative osteoarthritis of the knees and the pain has become severe. An orthopedic surgeon wants to perform a total knee arthroplasty on the left side, which has the most advanced disease, but

will not consider the surgery until the patient loses at least 20 lb. His past medical history is remarkable for coronary artery disease treated with endoluminal stents to the right

coronary artery and the left anterior descending coronary artery at age 61. He has been asymptomatic since that time. He has hypertension, well controlled with metoprolol, erectile

dysfunction, dyslipidemia treated with simvastatin, and depression, treated with fluoxetine. Review of systems is significant for fatigue, loud snoring, absent morning erections and

claudication with a block of walking. He denies any further chest pain since his coronary artery stenting .

On physical examination, the patient has advanced osteoarthritis of the knees with genu varum ,

a waist circumference of 54 in, and a body mass index (BMI) of 41 kg/m2. The pulses at the femoral, popliteal, dorsalis pedis, and posterior tibia sites are decreased bilaterally, and

there is acrocyanoisis at the feet .

Which of the following should be the first step in the management of this patient?

Choose one answer .

A. Start orlistat, 120 mg with meals .

B. Start phentermine, 15 mg every morning .

C. Refer for diagnostic studies for sleep apnea, with the intent to treat.

D. Refer for angiography of the lower extremities .

qsid=148265&aid=3886&qid=158&checksum=d8d15455c4705cc5b581f673a93a7be5&sesskey=9SD9XfnshK&new state=0

Page 247: Asap 2011

E. Refer to physical therapy for knee pain .

Educational Objective: To recognize the possibility of sleep apnea in the assessment of the obese patient as an obstacle to weight loss

Rationale: Sleep apnea is both a complication of obesity, and an obstacle to weight loss. Overnight hypoxia from sleep apnea places patients at risk for sudden death and must be

addressed first.

Phentermine is contraindicated in patients with established arteriosclerotic disease because of their adrenergic and pressor properties. Orlistat could be considered but may not be very

effective in weight reduction without treatment of obstructive sleep apnea. It also is not well-tolerated due to side effects.

Angiography of the lower extremities with the aim to revascularize the limbs is appropriate, but the fact that femoral pulses are diminished suggests aortic disease, and this must be

addressed first. Physical therapy for a patient with severe degenerative osteoarthritis is also appropriate, but hypoxia from sleep apnea and ischemia from arteriosclerosis must be

addressed before addressing rehabilitation problems.

Reference

1 .Wolk R, Shamsuzzaman AS, Somers VK. Obesity, sleep apnea, and hypertension. Hypertension. 2003; 42:1067-1074.

Incorrect

Marks for this submission: 0.00/1.00.

Question 23

Marks: 1.00

A 32-year-old man sees you for type 2 diabetes mellitus. He is referred by his primary care physician because glipizide and metformin have not been able to bring his glycosylated

hemoglobin (A1C) level below 8%.

On physical examination, the patient has a blood pressure of 150/94 mm Hg, a waist circumference of 52 in, a rounded face, hyperpigmented striae on the abdomen, proximal

muscle weakness (he is unable to rise from the seated position without the use of his arms), a scar on

qsid=148266&aid=3886&qid=161&checksum=68461030a326ce6003b7e878852aa5cd&sesskey=9SD9XfnshK&new state=0

Page 248: Asap 2011

the left forearm from a radial shaft fracture that required open reduction and internal fixation, and ecchymoses on the arms.

Which of the following diagnostic studies is the most appropriate for this patient?

Choose one answer .

A. Cortrosyn (adrenocorticotropic hormone [ACTH]) stimulation test

B. 24-hour urinary free cortisol

C. Plasma creatine kinase (total)

D. CT scan of the adrenal glands

E. Pituitary magnetic resonance imaging (MRI)

Educational Objective: To recognize Cushing's syndrome in a patient with obesity

Rationale: The patient has Cushing's syndrome by clinical description. An assessment of excess cortisol production is most appropriate. Cortrosyn (ACTH) stimulation test is used to

stimulate the adrenal cortex to evaluate patients for a lack of cortisol.

Although the patient has muscle weakness, creatine kinase is not the best diagnostic test, as excess cortisol production is not usually associated with elevate creatine kinase levels in the

blood.

Pituitary or adrenal imaging is not appropriate as the first step in the evaluation of this patient.

Reference

Schteingart DE. Cushing's syndrome. Endocrinol Metab Clin North Am. 1989; 18:311-338.

Incorrect

Marks for this submission: 0.00/1.00.

Question 24

Marks: 1.00

Page 249: Asap 2011

A 20-year-old college student comes to see you for secondary amenorrhea. She has not menstruated since age 16. She has had multiple negative pregnancy tests because she has

been sexually active. These tests have been done at home and at her physician's office. The last one was done 2 weeks prior as part of her yearly examination with her physician. Her past medical history is relevant for acne, treated with topical metronidazole, dyslipidemia

with elevated triglycerides and low high-density lipoprotein (HDL) cholesterol, and hirsutism, for which her doctor has started treatment with spironolactone. After her physical

examination and laboratory testing, you diagnose her with polycystic ovarian syndrome (PCOS), and prescribe metformin.

Which of the following statements regarding this patient is correct?

Choose one answer .

A. Metformin treatment may raise her serum triglycerides .

B. Metformin treatment increases the likelihood of pregnancy, and the patient must be counseled on contraception use .

C. Spironolactone treatment is safe should a pregnancy occur .

D. Metformin will worsen her acne .

E. Metformin will increase her circulating insulin levels .

Educational Objective: To understand the role of metformin in the management of PCOS to improve fertility

Rationale: Metformin decreases hepatic glucose production, increases insulin sensitivity and lowers circulating insulin, which allows the ovaries to resume normal function, and increases

the likelihood of a pregnancy. Metformin is used as an adjunct to improve fertility in PCOS.

Metformin treatment usually leads to a decrease in serum triglycerides and the total circulating testosterone in women with PCOS. Metformin does not aggravate acne.

Spironolactone is teratogenic and must be avoided prior to conception and after a pregnancy is established. Continued spironolactone use in women with PCOS who are

qsid=148267&aid=3886&qid=164&checksum=1c40ab85a69a134f185ea1dc867fc41a&sesskey=9SD9XfnshK&new state=0

Page 250: Asap 2011

sexually active should prompt a discussion about contraceptive use, and this discussion should be documented in the medical record.

Reference

American Association of Clinical Endocrinologists Position Statement on Metabolic and Cardiovascular Consequences of Polycystic Ovary Syndrome. Endocr Pract. 2005; 11:126-

134.

Incorrect

Marks for this submission: 0.00/1.00.

Question 25

Marks: 1.00

A 53-year-old man comes to the emergency room for epigastric abdominal pain. The onset of the pain was acute, 30 minutes earlier. He describes the pain as stabbing, radiating

through to the back. The pain is constant, associated with strong nausea and one emesis, "10/10" at worst, "“7/10" now, but not associated with dyspnea, palpitations, irregular

heartbeats, presyncope, syncope, or lower-extremity edema. The patient's past medical history is remarkable for known sleep apnea, cholelithiasis, hypertriglyceridemia with levels

>800 mg/dL (normal, <150 mg/dL), obesity with a body mass index (BMI) of 44 kg/m2, hypertension, tobacco use (1 pack per day), and type 2 diabetes mellitus. Medications

include aspirin 81 mg every day, metformin, exenatide, lisinopril, and fenofibrate. He denies alcohol use. His father died at age 35 years in a motor vehicle accident; his mother is alive

and well at age 77 years.

On physical examination he is in visible distress, with a cool sweat. He has a respiratory rate of 24 breaths/minute, and his oxygen saturation is 98% on pulse oximetry. His heart rate is

120 beats/minute and he is in sinus tachycardia by telemetry. Lung sounds are normal in all lung fields. Other than tachycardia his cardiovascular exam is normal. The abdomen is

tender in the epigastrium, with no organomegaly or masses, and with active bowel sounds in all 4 quadrants. Peripheral vascular examination shows full pulses throughout, and there is

no peripheral edema.

Which of the following is the immediate next step in the management of this patient ?

Choose one answer .

qsid=148268&aid=3886&qid=167&checksum=4df10e82cd516ebd1c92fc5b89b57749&sesskey=9SD9XfnshK&new state=0

Page 251: Asap 2011

A. Computed tomography (CT) scan of the chest and abdomen

B. Plasma amylase and lipase

C. 12-lead electrocardiogram

D. Abdominal ultrasound

E. Emergent consultation by a gastroenterologist for consideration of endoscopic retrograde cholangiopancreatography (ERCP)

Educational Objective: To actively exclude myocardial ischemia in a patient with abdominal pain and multiple cardiovascular risk factors

Rationale: An acute myocardial infarction must be considered given this patient's risk factors, and a 12-lead electrocardiogram will provide the most immediate assessment of his

cardiac function.

The most likely diagnosis for this patient is acute pancreatitis. Plasma amylase and lipase are appropriate, but a cardiac evaluation takes precedence.

CT scanning of the chest and abdomen may be appropriate to exclude a pulmonary embolus or acute biliary disease, and to evaluate the pancreas, given the likely diagnosis of acute

pancreatitis, but a cardiac evaluation takes precedence.

An abdominal ultrasound may document biliary disease but does not visualize the pancreas well.

ERCP should not be considered until cardiac clearance is achieved.

Reference

Kushner RF, Weinsier RL. Evaluation of the obese patient. Practical considerations. Med Clin North Am. 2000; 84:387-399.

Incorrect

Marks for this submission: 0.00/1.00.

Question 26

Marks: 1.00

Page 252: Asap 2011

Which of the following is the most direct measure of body fat?

Choose one answer .

A. Body mass index

B. Waist circumference

C. Stunkard Figure Rating Scale

D. Dual-energy X-ray absorptiometry body composition analysis

E. Bioimpedance

Educational Objective: To be aware of the most direct measure of body composition available in clinical practice.

Rationale: Dual-energy X-ray absorptiometry body composition analysis is a direct measure of body composition. It provides a table with grams of fat and lean tissue by body part.

Body mass index is a measure of kilograms of total body weight divided by height in meters squared. Waist circumference reflects the space occupied by visceral and subcutaneous

abdominal fat, as well as other visceral organs.

The Stunkard Figure Rating Scale utilizes gender-specific body figures for which a subjective choice is made.

Bioimpedance estimates body fat through an assessment of electrical signals passing through various body tissues, including fat, lean body mass, and water.

References

Bays H, Dujovne CA. Adiposopathy is a more rational treatment target for metabolic disease than obesity alone. Curr Atheroscler Rep. 2006; 8(2):144-156.

Bays HE, Bazata DD, Fox KM, et al. Perceived body image in men and women with type 2 diabetes mellitus: correlation of body mass index with the figure rating scale. Nutr J. 2009;

57:1475-2891.

qsid=148269&aid=3886&qid=171&checksum=1ddf8f25985e4f8415386422f92f982b&sesskey=9SD9XfnshK&new state=0

Page 253: Asap 2011

Incorrect

Marks for this submission: 0.00/1.00.

Question 27

Marks: 1.00

During positive caloric balance, which of the following statements regarding the pathogenic potential of adipose tissue is correct?

Choose one answer .

A. The pathogenic endocrine and immune responses of adipose tissue during positive calorie balance contribute to metabolic disease depending on the

capabilities and limitations of other body organs to handle these responses .

B. The unencumbered recruitment, differentiation, and proliferation of adipocytes leads to obesity and thus increases the risk of metabolic disease .

C. An increase in hip circumference is associated with a decreased risk of metabolic disease .

D. An increase in body fat is associated with an increase in testosterone levels in men .

E. Adipose tissue is mostly an inert storage organ of energy .

Educational Objective: To understand that the pathogenic and immune potential of adipose tissue during positive caloric balance is largely dependent upon the capabilities and

limitations of other body organs

Rationale: The pathogenic potential of adipose tissue during positive caloric balance is largely dependent on the capabilities and limitations of other body organs in adjusting to

adipose tissue endocrine and immune responses.

An increase in functional adipocytes may increase fat mass, and contribute to fat-mass pathology. However, if positive caloric balance results in the unencumbered addition of

qsid=148270&aid=3886&qid=174&checksum=b5272cb5e1efb8ad39f2d77ab2833fc6&sesskey=9SD9XfnshK&new state=0

Page 254: Asap 2011

functional adipocytes with absence or limited pathogenic endocrine and/or immune responses, then metabolic abnormalities may be mitigated.

Although an increase in hip circumference (reflecting peripheral subcutaneous adiposity) may be associated with reduced risk of developing metabolic disease compared to an

increase in waist circumference (reflecting visceral adiposity), even subcutaneous adipose tissue has pathogenic potential. Furthermore, patients with an increase in hip circumference

have an increased risk (not decreased risk) of metabolic disease.

Adiposity is mainly associated with a decrease in testosterone levels in men. Adipose tissue is an active endocrine and immune organ.

References

Bays H, Dujovne CA. Adiposopathy is a more rational treatment target for metabolic disease than obesity alone. Curr Atheroscler Rep. 2006; 8:144-156.

Mason C, Katzmarzyk PT. Effect of the site of measurement of waist circumference on the prevalence of the metabolic syndrome. Am J Cardiol. 2009, 103:1716-1720.

Mason C, Katzmarzyk PT. Variability in waist circumference measurements according to anatomic measurement sites. Obesity. 2009; 17:1789-1785.

Incorrect

Marks for this submission: 0.00/1.00.

Question 28

Marks: 1.00

Which of the following statements regarding weight reduction in overweight patients with metabolic disease is correct?

Choose one answer .

A. The first goal of a weight management program is to recommend a structured diet and physical activity program that is a total departure from the patient's prior

lifestyle and meets with patient expectations. A realistic objective for most patients is at least a 10% weight loss within the first 2 months .

B. Behavior modification is essential because weight regain after weight loss is common and almost solely due to the underlying psychological profile of the

qsid=148271&aid=3886&qid=176&checksum=b87bd6863d129fe34091fdfcfe7fce07&sesskey=9SD9XfnshK&new state=0

Page 255: Asap 2011

patient, which originally caused the overweight status .

C. Improvement in the pathogenic responses from adipose tissue, which translates into improvements in metabolic diseases, can occur with as little as 5% to 10%

weight loss .

D. Nutritional counseling, increased physical activity, and behavior modification are not necessary and would unduly delay therapy in patients for whom the

decision is made to proceed with drug therapy or bariatric surgery .

E. Patients should be warned that even with an increase in physical activity, gradual weight loss through appropriate nutrition often results in a substantial

increase in bone turnover, and thus a substantial increased risk of bone loss .

Educational Objective: To understand the importance of setting realistic goals and expectations for weight reduction

Rationale: Setting an initial goal that is a "total departure" from the patient's prior lifestyle, and a loss of 10% body weight in the first 2 months may be desirable, but not necessarily a

realistic objective for most patients. Although behavior modification is important after weight loss, physiologic barriers also exist that often make persistence of weight loss

challenging.

As little as 5% to 10% of weight loss can improve adipose tissue endocrine and immune responses.

Many, if not most, anti-obesity drug trials (and bariatric surgery trials) included patients who received nondrug and nonsurgical weight-loss interventions.

An increase in physical activity can mitigate bone loss, which is another reason to incorporate structured physical activity in a weight management program.

References

Minnesota Medical Association Obesity Task Force. How to evaluate and treat obesity. Minn Med. 2005; 88:40-46.

Varady KA, Tussing L, Bhutani S, et al. Degree of weight loss required to improve adipokine concentrations and decrease fat cell size in severely obese women. Metabolism. 2009;

58:1096-1101.

Villareal DT, Fontana L, Weiss EP, et al. Bone mineral density response to caloric restriction-induced weight loss or exercise-induced weight loss: a randomized controlled trial. Arch

Intern Med. 2006; 166:2502-2510.

Incorrect

Page 256: Asap 2011

Marks for this submission: 0.00/1.00.

Question 29

Marks: 1.00

Which of the following statements regarding drug therapies for patients who are overweight or obese is correct?

Choose one answer .

A. The lack of development or withdrawal of most anti-obesity agents was not because of safety or tolerability .

B. Pharmacotherapy for obesity does not work because obesity simply reflects poor judgment on the part of the patient .

C. Phentermine, approved for long-term clinical use in the 1970s, is the most widely prescribed anti-obesity agent .

D. The main cardiovascular concern with sibutramine is the increased risk of left-sided cardiac valve pathology

E. Reducing adiposity with anti-obesity drugs improves the pathogenic responses of adipose tissue .

Educational Objective: To recognize the benefits and limitations of currently available anti-obesity agents

Rationale: Anti-obesity drug development has mainly been impaired by safety concerns that either negated approval of anti-obesity drugs or resulted in withdrawal from the market of

previously approved anti-obesity drugs.

Fat mass accumulation leading to physical complications (adiposity) is a disease. Pathogenic adipose tissue leading to multiple metabolic abnormalities that result in ill health

(adiposopathy) is also a disease. The degree by which adiposity and adiposopathy result in ill health is substantially determined by genetic and environmental factors, and not simply

"poor judgment" on the part of patients.

qsid=148272&aid=3886&qid=184&checksum=68235343f045af95851169c9b4f76ea7&sesskey=9SD9XfnshK&new state=0

Page 257: Asap 2011

Phentermine was approved in the 1950s, but only for short-term (12-week) use because of concerns at the time of its approval that any centrally acting agent might be habit forming or

cause addiction. Fenfluramine was commonly used with phentermine for weight loss ("fen-phen"). However, the occurrence of left-sided cardiac valve pathology with fenfluramine and

dexfenfluramine (not sibutramine) resulted in their withdrawal from the market.

References

Bays H, Rodbard HW, Schorr AB, et al. Adiposopathy: treating pathogenic adipose tissue to reduce cardiovascular disease risk. Curr Treat Options Cardiovasc Med. 2007 ;

9:259-271.

Centers for Disease Control and Prevention (CDC).Cardiac valvulopathy associated with exposure to fenfluramine or dexfenfluramine: U.S. Department of Health and Human

Services interim public health recommendations, November 1997. MMWR Morbid Mortal Wkly Rep. 1997; 46:1061-1066.

Incorrect

Marks for this submission: 0.00/1.00.

Question 30

Marks: 1.00

Which of the following statements regarding bariatric surgery is correct?

Choose one answer .

A. Bariatric surgery substantially improves metabolic diseases (such as type 2 diabetes mellitus) and is associated with a reduction in total mortality .

B. Gastric banding is solely a restrictive procedure; gastric bypass is solely a macronutrient malabsorptive procedure .

C. After gastric bypass, micronutrient abnormalities leading to metabolic diseases are usually apparent within the first 6 weeks postoperatively .

D. As opposed to nutritional and drug interventions, the advantage of bariatric surgery is that weight regain is not common, unless complications or flaws

qsid=148273&aid=3886&qid=187&checksum=5a72b3671cb5e4a72bd59e2cf4c0a0b4&sesskey=9SD9XfnshK&new state=0

Page 258: Asap 2011

E. Bariatric surgeries should be considered only for refractory patients with body mass index ≥40 kg/m2 and comorbid conditions .

Educational Objective: To recognize the benefits and limitations of bariatric surgery

Rationale: Gastric banding may alter satiety; gastric bypass may have a restrictive component to its efficacy. Gastric bypass may result in more distal digestion and

micronutrient malabsorption but does not necessarily result in substantial macronutrient malabsorption. The micronutrient abnormalities may take many months to years to be

clinically or biochemically detected and may require life-long therapy.

Weight regain after bariatric surgery does not necessarily mean that the surgery was flawed. Maintenance of weight loss after bariatric surgery requires continued diligence on the part

of clinician and patient.

Consideration of bariatric surgery is appropriate for patients with comorbidities and a body mass index ≥35kg/m2 who have not responded to other forms of treatment. There is

evidence that the weight loss that follows bariatric surgery effectively treats adiposopathy and that mortality is decreased

References

Sjostrom L, Narbro K, Sjostrom CD, et al. Effects of bariatric surgery on mortality in Swedish obese subjects. N Engl J Med. 2007; 357:741-752.

Mechanick JI, Kushner RF, Sugerman HJ, et al. Executive summary of the recommendations of the American Association of Clinical Endocrinologists, the Obesity Society, and American

Society for Metabolic & Bariatric Surgery medical guidelines for clinical practice for the perioperative nutritional, metabolic, and nonsurgical support of the bariatric surgery patient.

Endocr Pract. 2008; 14:318-336.

Dixon AF, Dixon JB, O'Brien PE. Laparoscopic adjustable gastric banding induces prolonged satiety: a randomized blind crossover study. J Clin Endocrinol Metab. 2005; 90:813-819.

Incorrect

Marks for this submission: 0.00/1.00.

Reminder:

You have three (3) attempts to successfully complete this assessment with a 75% score or higher.

You have 30 days to complete each attempt, once an attempt is started.

If you need to save your answers and come back to the assessment, you must click the 'Next' button at the bottom-left of the assessement prior to logging

out of ASAP.

Page 259: Asap 2011

Only click 'Submit all and finish' once you are satisfied with all your assessment answers.

Bottom of Form

Finish review

Skip Quiz navigation

Quiz navigation

i (Closed ) 1 (Incorrect ) 2 (Incorrect ) 3 (Incorrect ) 4 (Incorrect ) 5 (Incorrect ) 6 (Incorrect ) 7 (Incorrect ) 8 (Incorrect ) 9 (Incorrect ) 10 (Incorrect ) 11 (Incorrect ) 12 (Incorrect ) 13

(Incorrect ) 14 (Incorrect ) 15 (Incorrect ) 16 (Incorrect ) 17 (Incorrect ) 18 (Incorrect ) 19 (Incorrect ) 20 (Incorrect ) 21 (Incorrect ) 22 (Incorrect ) 23 (Incorrect ) 24 (Incorrect ) 25

(Incorrect ) 26 (Incorrect ) 27 (Incorrect ) 28 (Incorrect ) 29 (Incorrect ) 30 (Incorrect ) i (Closed )

Finish review

You are logged in as Mohammed Aldawish (Logout)

NUTRITION

 

Copyright: 2011, American Association of Clinical Endocrinologists (AACE).All material published on the Web site is the property of AACE and may not be reproduced in

any form or by any electronic meansincluding information, storage and retrieval systems without the written consent of AACE.

This activity is sponsored by the American Association of Clinical Endocrinologists.

Skip to main content

Parathyroid and Bone Mineral Disorders

You are logged in as Mohammed Aldawish (Logout)

Page path

Home

/ ► My courses

Page 260: Asap 2011

/ ► PBMD

/ ► Assessment

/ ► Parathyroid and Bone Mineral Disorders Assessment

/ ► Review of attempt 1

Review of attempt 1

Started onTuesday, 19 November 2013, 10:43 PM

Completed onTuesday, 19 November 2013, 10:44 PM

Time taken1 min 14 secs

Grade0.00 out of a maximum of 34.00 (0%)

FeedbackFailed

Top of Form

Directions: In this section, each item includes a lead question and a list of options labeled with letters. Select the ONE lettered option that is BEST in each case by selecting the radio

button corresponding to the correct answer .

Question 1

Marks: 1.00

Which ONE of the following biochemical markers best reflects bone formation?

Choose one answer .

A. Serum bone-specific alkaline phosphatase

B. Serum carboxy-terminal propeptide of type I collagen

9SD9XfnshK

qsid=148276&aid=3887&qid=117&checksum=14fc974311e4aa97b79ffcf7ffd4ca32&sesskey=9SD9XfnshK&new state=0

Page 261: Asap 2011

C. Serum osteocalcin

D. Serum amino-terminal propeptide of type I collagen

Educational Objective: To describe the current clinical markers of osteoblast activity

Rationale: The best markers of bone formation are those that reflect osteoblast function. These include serum bone-specific alkaline phosphatase and serum osteocalcin. Between

these two markers, bone-specific alkaline phosphatase more directly reflects osteoblast function. Serum osteocalcin more accurately reflects total bone turnover, rather than just

bone formation. This is due to the fact that serum osteocalcin is derived from both new synthesis by osteoblasts and release of osteocalcin previously incorporated in bone matrix

by osteoclast resorption. The serum carboxy- and amino-terminal propeptides of type I collagen have not been found to be as helpful in assessment of bone formation as serum

bone-specific alkaline phosphatase or serum osteocalcin.

References:

Hodgson SF, Watts NB, Bilezikian JP, et al. American Association of Clinical Endocrinologists Medical Guidelines for Clinical Practice for the Prevention and Management of

Postmenopausal Osteoporosis: 2001 edition, with selected updates for 2003. Endocr Pract. 2003;9:544-564.

Miller PD, Baran DT, Bilezikian JP, et al. Practical clinical application of biochemical markers of bone turnover: consensus of an expert panel. J Clin Densitom. 1999;2:323-342.

Woitge HW, Seibel MJ. Biochemical markers to survey bone turnover. Rheum Dis Clin North Am. 2001;27:49-80.

Delmas PD, Eastell R, Garnero P, et al. Committee of Scientific Advisors of the International Osteoporosis Foundation. The use of biochemical markers of bone turnover in osteoporosis.

Osteoporosis Int. 2000;11(Suppl 6):S2-S17.

Incorrect

Marks for this submission: 0.00/1.00.

Question 2

Marks: 1.00

Page 262: Asap 2011

Which of the following biochemical markers best reflects bone resorption?

Choose one answer .

A. Serum tartrate-resistant acid phosphatase

B. Serum carboxy-terminal cross-linked (CTx)-telopeptide of type-I collagen

C. Urine hydroxyproline

D. Urine deoxypyridinoline

Educational Objective: To describe the current clinical markers of osteoclast activity

Rationale: The best markers of bone resorption are those that reflect osteoclast function. These include serum carboxy-terminal cross-linked (CTx)-telopeptide of type I collagen and

urine deoxypyridinoline. Serum CTx-telopeptide is currently considered to be the best marker of bone resorption. Urine deoxypyridinoline is more reflective of bone resorption,

whereas urine pyridinoline reflects total body connective tissue turnover. Tartrate-resistant acid phosphatase may be useful for identifying osteoclasts in bone histomorphometric

sections, but it is not sufficiently reliable to serve as a serum marker of osteoclast activity. Urine hydroxyproline is influenced by dietary intake of a number of foods, and therefore

does not typically reflect bone resorption accurately.

References

Hodgson SF, Watts NB, Bilezikian JP, et al. American Association of Clinical Endocrinologists Medical Guidelines for Clinical Practice for the Prevention and Management of

Postmenopausal Osteoporosis: 2001 edition, with selected updates for 2003. Endocr Pract. 2003;9:544-564.

Miller PD, Baran DT, Bilezikian JP, et al. Practical clinical application of biochemical markers of bone turnover: consensus of an expert panel. J Clin Densitom. 1999;2:323-342.

Woitge HW, Seibel MJ. Biochemical markers to survey bone turnover. Rheum Dis Clin North Am. 2001;27:49-80.

Incorrect

qsid=148277&aid=3887&qid=119&checksum=6c10c36e72561adce52ab7b28f291f46&sesskey=9SD9XfnshK&new state=0

Page 263: Asap 2011

Marks for this submission: 0.00/1.00.

Question 3

Marks: 1.00

Which ONE of the following statements regarding markers of bone turnover best describes their usefulness in clinical practice?

Choose one answer .

A. Markers of bone turnover correlate well with bone strength

B. Markers of bone turnover are suitable surrogates for bone mineral density

C. Markers of bone turnover are most helpful in following response to antiresorptive therapy

D. Markers of bone turnover remain constant in a given patient with low intra-individual variability from day-to-day

Educational Objective: To describe the potential clinical utility of measurement of markers of bone turnover

Rationale: In clinical practice, the most helpful application of bone turnover markers is in following response to antiresorptive therapy. Markers of bone turnover have been

demonstrated to predict response to antiresorptive therapy, risk of future fractures, and rate of future bone loss in postmenopausal women with osteoporosis. Markers of bone turnover correlate relatively poorly with bone strength, and therefore are unsuitable as

surrogates for bone mineral density. The available serum and urine markers of bone turnover are difficult to apply in clinical practice to a given patient due to their day-to-day

intraindividual variability

References

Melton LJ III, Khosla S, Atkinson EJ, et al. Relationship of bone turnover to bone density and fractures. J Bone Miner Res. 1997; 12:1083-1091.

qsid=148278&aid=3887&qid=120&checksum=8777b2546bdbe9ee79490296f6e26d93&sesskey=9SD9XfnshK&new state=0

Page 264: Asap 2011

Garnero P, Hausherr E, Chapuy MC, et al. Markers of bone resorption predict hip fracture in elderly women: the EPIDOS Prospective Study. J Bone Miner Res. 1996;11:1531-1538.

Ravn P, Clemmesen B, Christiansen C, et al. for the Alendronate Osteoporosis Prevention Study Group. Biochemical markers can predict the response in bone mass during alendronate treatment in early postmenopausal women. Bone. 1999;24:237-244.

Incorrect

Marks for this submission: 0.00/1.00.

Question 4

Marks: 1.00

Which ONE of the following statements regarding markers of bone turnover is correct?

Choose one answer .

A. There is minimal circadian variation in the levels of markers of bone turnover

B. Markers of bone turnover have a small coefficient of variation which enhances their clinical usefulness in patients with decreased muscle mass

C. The reference population for bone turnover and markers of bone turnover has been standardized to optimize their usefulness in clinical practice

D. Markers of bone turnover have been demonstrated to predict risk of bone fractures

Educational Objective: To recognize the limitations of theuse of clinical markers of bone turnover

Rationale: The use of markers of bone turnover in clinical practice is limited by a number of factors related to the biology of the available markers and their interpretation. Most of the markers have some degree of circadian variation. Even if this is controlled for by measuring

the markers at the same time each day, there is still a fair amount of day-to-day variability in the marker level. Because of this, interpretation of a single marker value in a patient is

associated with some uncertainty. In addition, there is still debate as to what reference population a marker value should be compared to, although most experts feel that using

qsid=148279&aid=3887&qid=121&checksum=2d72fcabf0ec640e65d1597a3161a261&sesskey=9SD9XfnshK&new state=0

Page 265: Asap 2011

premenopausal women or younger men as the reference standard for comparison makes the most sense. Markers of bone turnover cannot be used to predict bone density or

strength, as they correlate relatively poorly with these variables. With the current coefficient of variation of 5% for the measurement of markers of bone turnover, any other factor such as decreased muscle mass, the circadian variation in bone markers, and the intra-individual

day-to-day variability in bone markers, would add further variability in the test result. This variability increases the percentage change needed to assess the benefit of any therapy.

References

1 .Hodgson SF, Watts NB, Bilezikian JP, et al. American Association of Clinical Endocrinologists Medical Guidelines for Clinical Practice for the Prevention and

Management of Postmenopausal Osteoporosis: 2001 edition, with selected updates for 2003. Endocr Pract.

2003;9:544-564.

2.Miller PD, Baran DT, Bilezikian JP, et al. Practical clinical application ofbiochemical markers of bone turnover: consensus of an expert panel. J Clin

Densitom. 1999;2:323-342.

3 .Woitge HW, Seibel MJ. Biochemical markers to survey bone turnover. Rheum Dis Clin North Am. 2001;27:49-80.

4 .Delmas PD, Eastell R, Garnero P, et al. Committee of Scientific Advisors of the International Osteoporosis Foundation. The use of biochemical markers of bone turnover in

osteoporosis. Osteoporosis Int. 2000;11(Suppl 6):S2-S17.

Incorrect

Marks for this submission: 0.00/1.00.

Question 5

Marks: 1.00

A 58-year old postmenopausal female has been treated for osteoporosis with generic alendronate and with calcium and vitamin D supplementation for the last three years.

Despite taking her alendronate faithfully each week for three years, her follow-up bone density test shows a decrease of 10% at her lumbar spine, 6.8% at her left femoral neck, and

7.2% at her left total hip, compared to her bone density on the same densitometer three years earlier. Her serum bone-specific alkaline phosphatase is 65 mcg/L (normal

postmenopausal female, <22), and serum beta-C-terminal cross-linked (CTx)-telopeptide 1,500 pg/mL (normal postmenopausal female, 104 to 1,008) .

qsid=148280&aid=3887&qid=124&checksum=aa188c71e35a0d830e18445116227851&sesskey=9SD9XfnshK&new state=0

Page 266: Asap 2011

Which of the following statements provides the most likely explanation for the patient's continued bone loss?

Choose one answer .

A. She is taking her alendronate less than 50% of the time

B. She has an unrecognized or untreated secondary causes of osteoporosis

C. She is malabsorbing her alendronate due to unrecognized celiac disease

D. She is taking less than 1600 mg of oral calcium daily and less than 800 IU of vitamin D daily

Educational Objective: To describe potential explanations for bone loss and increased clinical markers of bone turnover while on treatment with antiresorptive therapy

Rationale: The use of serum markers of bone turnover in clinical practice may offer insight into the unexpected finding of bone loss at multiple skeletal sites in patients being treated

with antiresorptive therapy. There are multiple potential explanations for bone loss on antiresorptive therapy. The chief consideration should be noncompliance with therapy,

especially after several years of therapy, because many women stop taking therapy within a year of starting treatment. In those who are apparently compliant with therapy,

unrecognized or untreated secondary causes of bone loss may limit effectiveness of antiresorptive therapy and lead to bone loss but unlikely in this patient. Malabsorption of oral bisphosphonate medications is always a possibility but rare, even in patients without

diarrhea, steatorrhea, rapid intestinal transit, or unexpected weight loss. Some patients may persist in taking their medication, but fail to take adequate calcium or vitamin D

supplementation. However, patients are more likely to take their calcium and vitamin D supplementation than their alendronate. Any of these conditions might explain unexpected bone loss, particularly when markers of bone turnover remain increased. While markers of bone turnover cannot be used to predict bone density or strength, they may give insight as

to the mechanism behind bone loss while on therapy.

References

Hodgson SF, Watts NB, Bilezikian JP, et al. American Association of Clinical Endocrinologists Medical Guidelines for Clinical Practice for the Prevention and Management of

Postmenopausal Osteoporosis: 2001 edition, with selected updates for 2003. Endocr Pract. 2003;9:544-564.

Miller PD, Baran DT, Bilezikian JP, et al. Practical clinical application of biochemical markers of bone turnover: consensus of an expert panel. J Clin Densitom. 1999;2:323-342.

Page 267: Asap 2011

Szulc P, Delmas PD. Biochemical markers of bone turnover in men. Calcif Tiss Int. 2001;69:229-234.

Woitge HW, Seibel MJ. Biochemical markers to survey bone turnover. Rheum Dis Clin North Am. 2001;27:49-80.

Incorrect

Marks for this submission: 0.00/1.00.

Question 6

Marks: 1.00

A 42-year old man is evaluated for mild persistent hypercalcemia. His serum calcium has ranged from 9.9-10.8 mg/dL (normal, 8.9-10.1) over the last two years, with serum

phosphate levels 2.5-3.0 mg/dL (normal, 2.5-4.5) when checked. His serum creatinine is normal. He has no documentation of hypercalcemia prior to five years ago, and no family

history of hypercalcemia. His serum parathyroid hormone level is mildly increased at 74 pg/mL (normal, 10-55).

The most likely diagnosis for this patient's hypercalcemia is which of the following disorders?

Choose one answer .

A. Sporadic primary hyperparathyroidism

B. Multiple Endocrine Neoplasia-1 (MEN 1)

C. MEN IIA

D. Familial benign (hypocalciuric) hypercalcemia

Educational Objective: Appreciate the differential diagnosis of mild hypercalcemia associated with increased serum PTH

Rationale: The most likely diagnosis in a patient of this age is primary hyperparathyroidism. Patients with biochemical evidence of primary hyperparathyroidism may have sporadic

qsid=148281&aid=3887&qid=128&checksum=1d01099caad4a58a93e7fb4e9662dcdb&sesskey=9SD9XfnshK&new state=0

Page 268: Asap 2011

primary hyperparathyroidism or a familial disorder causing primary hyperparathyroidism, or familial benign (hypocalciuric) hypercalcemia. Younger patients with biochemical evidence

of primary hyperparathyroidism should be assumed to have a familial disorder causing their primary hyperparathyroidism until proven otherwise, and should generally be considered for

screening for Multiple Endocrine Neoplasia, type 1 (MEN1) especially, and MEN IIA. However, routine analysis of the MEN1 gene is not recommended. These patients should be

questioned carefully about their family history for hypercalcemia or endocrine tumors. There was no family history of hypercalcemia reported in this patient. It is important to note that no serum biochemical test reliably distinguishes between primary hyperparathyroidism

and familial benign hypercalcemia, and that lack of family history does not rule out familial benign hypercalcemia. .

References:

Bilezikian JP, Khan AA, Potts JT, Jr ; Third International Workshop on the Management of Asymptomatic Primary Hyperparathyroidism. Guidelines for the management of asymptomatic primary hyperparathyroidism: summary statement from the Third

International Workshop. J Clin Endocrinol Metab. 2009;94:335-339.

Nesbit MA, Hannan FM, Graham U, et al. Identification of a second kindred with Familial Hypocalciuric Hypercalcemia Type 3 (FHH3) narrows localization to a <3.5 megabase pair

region on chromosome 19q13.3. J Clin Endocrinol Metab. 2010 Feb 4. [Epub ahead of print]

Powell AC, Libutti SK. Multiple endocrine neoplasia type 1: clinical manifestations and management. Cancer Treat Res. 2010;153:287-302 .

Traugott AL, Moley JF. Multiple Endocrine Neoplasia Type 2: clinical manifestations and management. Cancer Treat Res. 2010;153:321-337.

Incorrect

Marks for this submission: 0.00/1.00.

Question 7

Marks: 1.00

Which of the following statements about hypercalcemia is correct?

Choose one answer .

qsid=148282&aid=3887&qid=132&checksum=a25ce8eee184b770f8783fc0db9a9550&sesskey=9SD9XfnshK&new state=0

Page 269: Asap 2011

A. Intravenous bisphosphonate may be given before hydration or diuresis to treat hypercalcemia

B. Most granulomatous diseases cause hypercalcemia by increasing the production of 25-dihydroxyvitamin D

C. Humoral hypercalcemia of malignancy is due to underlying primary hyperparathyroidism

D. Most cases of familial benign (hypocalciuric) hypercalcemia are due to inactivating mutations of the calcium-sensing receptor

E. Patients with both primary hyperparathyroidism and postmenopausal osteoporosis respond poorly to long-term cinacalcet

Educational Objective: Recognize common clinical features of hypercalcemia

Rationale: Intravenous bisphosphonate may be the most effective therapy for hypercalcemia in many situations, but in general this should not be given before patients have been adequately hydrated and diuresed. Most granulomatous diseases occasionally cause

hypercalcemia, and when they do, it is usually by increasing extra-renal production of 1,25-dihydroxyvitamin D via 1α-hydroxlyase expression. Granulomatous tissue has the ability to

produce 1α-hydroxylase activity in some disease states, but the regulation of 1α-hydroxylase activity is not well understood. Malignant solid tumors are capable of producing many

cytokines, some of which may cause hypercalcemia. PTHrp is the most commonly recognized cytokine in these tumors, but other cytokines may be responsible for causing hypercalcemia

of malignancy when PTHrp levels are low or suppressed. Familial benign (hypocalciuric) hypercalcemia appears to be due most commonly to an inactivating mutation in one of the

calcium-sensing receptor alleles. Inactivating mutations in both alleles of the calcium-sensing receptor typically causes neonatal severe hyperparathyroidism. Activating mutations of the

calcium sensing receptor cause hypocalcemia and hypercalciuria. Patients with primary hyperparathyroidism and postmenopausal osteoporosis treated with cinacalcet for up to 5

years maintain stable bone density, and suppress their hyperparathyroidism and hypercalcemia.

References:

Kantham L, Quinn SJ, Egbuna OI, et al. The calcium-sensing receptor (CaSR) defends against hypercalcemia independently of its regulation of parathyroid hormone secretion. Am J

Physiol Endocrinol Metab. 2009;297:E915-923 .

Mundy GR, Edwards JR. PTH-related peptide (PTHrp) in hypercalcemia. J Am Soc Nephrol. 2008;19:672-675.

Page 270: Asap 2011

Peacock M, Bolognese MA, Borofsky M, et al. Cinacalcet treatment of primary hyperparathyroidism: biochemical and bone densitometric outcomes in a five-year study. J

Clin Endocrinol Metab. 2009;94:4860-4867.

Stewart AF. Clinical practice. Hypercalcemia associated with cancer. N Engl J Med. 2005;352:373-379.

Incorrect

Marks for this submission: 0.00/1.00.

Question 8

Marks: 1.00

An 82-year old man is referred for evaluation of increased bone alkaline phosphatase. He had mild benign prostate hypertrophy diagnosed three years ago, and was previously given

α-adrenergic blocking agent therapy for two years with improvement in his urinary symptoms. He has a history of hypertension, currently well controlled on lisinopril, and

passed a single kidney stone three years ago. His only medication now is lisinopril 10 mg a day.

Physical exam shows BMI, 26.7 kg/m2, blood pressure 126/82 mmHg, right arm sitting, and heart rate 70 beats per minute. He appears well. Examination of his lungs, heart, abdomen,

and lymph nodes is within normal limits.

His laboratory studies are as follows:

Serum calcium9.6 mg/dL)normal, 8.9-10.1 mg/dL(

Serum phosphate3.6 mg/dL)normal, 2.5-4.5 mg/dL(

Total alkaline phosphatase320 U/L)normal, 45-115 U/L (

Bone alkaline phosphatase220 U/L)normal, 11-67 U/L(

Serum creatinine1.2 mg/dL)normal, 0.8-1.3 mg/dL (

Thyroid-stimulating 1.3 mIU/L)normal, 0.3-5 mIU/L(

qsid=148283&aid=3887&qid=135&checksum=4471710f9451e545a203671969deefd8&sesskey=9SD9XfnshK&new state=0

Page 271: Asap 2011

hormone

Serum 25-hydroxyvitamin D30 ng/mL)normal,>30 ng/mL (

Serum parathyroid hormone

64 pg/mL)normal, 10-55 pg/mL(

Serum protein electrophoresis

Normal

 

His bone scan is negative except for osteoarthritis.

What is the next best step in evaluation of his increased bone alkaline phosphatase?

Choose one answer .

A. 24-hour urinary calcium and creatinine

B. Serum 1,25-dihydroxyvitamin D

C. Metastatic bone survey

D. Serum angiotensin-converting enzyme level

E. Fungal serologies

Educational Objective: Recognize the differential diagnosis of physiological hyperparathyroidism

Rationale: Subsequent evaluation showed that his 24-hour urinary calcium was increased at 450 mg, with an appropriate 24-hour urinary creatinine, indicating complete 24-hour urine

collection. His increased parathyroid hormone (PTH) level was due to physiological hyperparathyroidism due to his hypercalciuria. His KUB with tomograms showed small

bilateral calcium-containing kidney stones. He was started on hydrochlorthiazide 25 mg a day, with repeat 24-hour urinary calcium mid-normal at 175 mg, serum calcium normal at 9.7 mg/dL, and repeat PTH mid-normal at 32 pg/mL. Unexplained increased total or bone

alkaline phosphatase is not uncommonly due to physiological hyperparathyroidism, which is frequently due to decreased calcium intake or absorption, or increased urinary calcium

excretion. Idiopathic hypercalciuria may cause increased bone alkaline phosphatase. Vitamin

Page 272: Asap 2011

D level for sarcoidosis or vitamin D deficiency, metastatic bone survey for cancer metastases, serum angiotensin-converting enzyme for sarcoidosis, and fungal serologies for infections

associated with ectopic vitamin D production, would provide little if any useful information regarding the patient's elevated alkaline phosphatase in the presence of a normal serum

calcium and normal bone scan.

References

Brown EM. The calcium-sensing receptor: physiology, pathophysiology and CaR-based therapeutics. Subcell Biochem. 2007:45:139-167.

Fraser WD. Hyperparathyroidism. Lancet 2009;374:145-158.

Joshi D Center JR, Eisman JA. Investigation of incidental hypercalcemia. BMJ 2009;339:b4613.

Moe OW, Bonny O.Genetic hypercalciuria. J Am Soc Nephrol. 2005;16:729-745.

Incorrect

Marks for this submission: 0.00/1.00.

Question 9

Marks: 1.00

A 75-year old woman is referred for evaluation and management of newly discovered hypercalcemia. Her serum calcium was found to be 14.0 mg/dL during evaluation in her local

emergency department for progressive fatigue, sweats, polyuria, and recent 25-pound weight loss. She currently takes no medications. She has never smoked cigarettes.

Her physical examination shows BMI, 26.0 kg/m2, blood pressure 122/68 mmHg, left arm, sitting, and heart rate 96 beats per minute. She appears chronically ill. Her lungs, heart,

abdomen, spine, and lymph node examination are within normal limits .

Laboratory studies show repeat serum calcium of 13.6 mg/dL (normal,8.9-10.1), serum phosphate, 4.3 mg/dL (normal, 2.5-4.5), parathyroid hormone, 10 pg/mL (normal, 15-65),

and normal alkaline phosphatase and thyroid-stimulating hormone. Her serum creatinine is mildly increased at 1.6 mg/dL (normal, 0.8-1.3), and serum 25-hydroxyvitamin D optimal at

35 ng/mL (normal,>30).

What is the next best step in evaluation of her hypercalcemia?

qsid=148284&aid=3887&qid=137&checksum=a4ad094013bbb4d7303f0980a9dd5beb&sesskey=9SD9XfnshK&new state=0

Page 273: Asap 2011

Choose one answer .

A. Serum 1,25-dihydroxyvitamin D

B. Angiotensin-converting enzyme level

C. 24-hour urinary calcium

D. Fungal serologies

E. Tuberculin skin test

Educational Objective: Appreciate the differential diagnosis of hypercalcemia

Rationale: The patient's serum 1,25-dihydroxyvitamin D level was increased at 88 pg/mL (normal, 22-67), and angiotensin-converting enzyme (ACE) was upper-normal at 44.0 U/L

(normal, 7-46). Her chest x-ray was negative, and her high-normal ACE level interpreted as due to her mild renal insufficiency. Fungal serologies were negative, and her PPD skin test for tuberculosis was negative, with negative controls. Further work-up for causes of extra-renal 1α-hydroxylase activity included a computed tomography (CT) scan of her abdomen,

which revealed lower pelvic lymphadenopathy. CT-guided needle biopsy of an enlarged pelvic lymph node revealed non-Hodgkin's lymphoma. Simultaneous hypercalcemia and

hyperphosphatemia may be due to hypervitaminosis D due to exogenous or endogenous sources. If serum 25-hydroxyvitamin D is normal, 1,25-dihydroxyvitamin D should always be checked. Numerous causes of endogenous increased serum 1,25-dihydroxyvitamin D exist, including sarcoidosis, other non-infectious granulomatous diseases, Crohn's disease, fungal

infections, tuberculosis, cat-scratch disease, and malignant lymphoproliferative disorders. A 24 hour urinary calcium would most likely show hypercalciuria but that would be consistent

with her hypercalcemia and provide no new information as to the patient's underlying diagnosis.

References

Horwitz MJ, Hodak SP, Stewart AF. Non-parathyroid hypercalcemia. In: Rosen CJ, ed. Primer on the Metabolic Bone Diseases and Disorders of Mineral Metabolism. 7th ed. Washington,

DC: American Society for Bone and Mineral Research, 2008: 307-312.

Joshi D Center JR, Eisman JA. Investigation of incidental hypercalcemia. BMJ. 2009;339:b4613.

Seymour JF and Gagel RF. Calcitriol: the major humoral mediator of hypercalcemia in Hodgkin's disease and non-Hodgkin's lymphomas. Blood. 1993;82:1383-1394.

Page 274: Asap 2011

Incorrect

Marks for this submission: 0.00/1.00.

Question 10

Marks: 1.00

A 33-year old woman is referred for further evaluation and management of hyperparathyroidism of about three years duration. She complains of chronic back, hip and thigh pain, and generally feels unwell. She takes only acetominophen for pain control. She

has never smoked cigarettes.

On physical examination her BMI is 24.2 kg/m2, blood pressure, 96/64, and heart rate, 82 beats per minute. Her lungs are clear, and heart, abdomen, lymph node, and extremity

examinations are normal.

Laboratory studies are as follows:

Serum calcium9.2 mg/dL)normal, 8.9-10.1 mg/dL(

Serum phosphate4.7 mg/dL)normal, 2.5-4.5 mg/dL (

Total alkaline phosphatase120 IU/L)normal, 81-213 IU/L(

Serum creatinine0.9 mg/dL)normal, 0.6-0.9 mg/dL(

Parathyroid hormone level325 pg/mL)normal, 15-65 pg/ML(

Serum 25-hydroxyvitamin D40 ng/mL)normal, 25-80 ng/mL(

Serum 1,25-dihydroxyvitamin D

34 pg/mL)normal, 22-67 pg/mL(

What is the next best step in evaluation and management?

Choose one answer .

qsid=148285&aid=3887&qid=139&checksum=525d60f08a4fe8bb4aeda46144225e86&sesskey=9SD9XfnshK&new state=0

Page 275: Asap 2011

A. X-rays of the hands

B. Computed tomography of the head

C. 24-hour urinary calcium and creatinine

D. Urinary cyclic adenosine monophosphate (AMP)

E. Ultrasound of the Neck

Educational Objective: Recognize that not all hyperparathyroidism is associated with hypercalcemia

Rationale: The patient's low-normal serum calcium, mild hyperphosphatemia, and significantly increased parathyroid hormone level suggest parathyroid hormone resistance

due to pseudohypoparathyroidism. The lack of a classical skeletal phenotype for psuedohypoparathyroidism (Albright's hereditary osteodystrophy), including short stature

and shortened fourth and fifth metacarpals and metatarsals, suggests a diagnosis of pseudohypoparathyroidism Ib. Patients with pseudohypoparathyroidism type Ib typically

have normal Gsα activity and hormone resistance limited to PTH target tissues such as bone and kidney, but bone lesions associated with hyperparathyroidism may be seen. Patients

with pseudohypoparathyroidism types Ia and Ib have lower baseline urinary cyclic AMP levels than normal subjects, and fail to stimulate appropriately with intravenous infusion of PTH. Hypocalcemia or low-normal serum calcium, mild hyperphosphatemia, and increased

parathyroid hormone with normal skeletal phenotype suggest a diagnosis of pseudohypoparathyroidism type Ib. X-rays of the hands, computed tomography of the head,

24 hour urinary calcium and creatinine and ultrasound of the neck will provide little if any useful information of the patient's underlying diagnosis.

References

Mantovani G, de Sanctis L, Barbieri AM, et al. Pseudohypoparathyroidism and GNAS epigenetic defects: clinical evaluation of Albright Hereditary Osteodystrophy and molecular

analysis in 40 patients. J Clin Endocrinol Metab. 2010;95:651-658.

Lecumberri B, Fernandez-Rebollo E, Sentchordi L, et al. Coexistence of two different pseudohypoparathyroidism subtypes (Ia and Ib) in the same kindred with independent

Gs{alpha} coding mutations and GNAS imprinting defects. J Med Genet. 2009 Nov 4. [Epub ahead of print]

Shalitin S, Davidovits M, Lazar L, Weintrob N. Clinical heterogeneity of pseudohypoparathyroidism: from hyper- to hypocalcemia. Horm Res. 2008;70:137-144.

Page 276: Asap 2011

Incorrect

Marks for this submission: 0.00/1.00.

Question 11

Marks: 1.00

Hypervitaminosis A may be associated with hypercalcemia.

Which of the following mechanisms is most likely to account for the hypercalcemia of hypervitaminosis A?

Choose one answer .

A. Increased catecholamine production

B. Increased osteoclast activity

C. Increased serum 1,25-dihydroxyvitamin D level

D. Increased production of parathyroid hormone related protein (PTHrp)

E. Increased ectopic production of parathyroid hormone

Educational Objective: Briefly review the mechanisms of hypercalcemia typically seen in various disorders

Rationale: Hypervitaminosis A is believed to cause hypercalcemia by stimulating osteoclast bone resorption Crohn's disease most commonly causes hypercalcemia by causing extra-

renal production of 1,25-dihydroxyvitamin D, due to increased macrophage 1α-hydroxylase activity in granulomas. Multiple myeloma may cause hypercalcemia by a variety of cytokine-

mediated mechanisms, and increased parathyroid hormone related protein (PTHrp) production by myeloma cells is fairly common.. Pheochromocytoma may cause

hypercalcemia via the effects of increased catecholamines on the bones. Small cell lung cancer may cause hypercalcemia due to ectopic over-secretion of parathyroid hormone.

References:

qsid=148286&aid=3887&qid=140&checksum=5cd19e0c68709093ef0dc997c702e4d2&sesskey=9SD9XfnshK&new state=0

Page 277: Asap 2011

Horwitz MJ, Hodak SP, Stewart AF. Non-parathyroid hypercalcemia. In: Rosen CJ, ed. Primer on the Metabolic Bone Diseases and Disorders of Mineral Metabolism. 7th ed. Washington,

DC: American Society for Bone and Mineral Research, 2008: 307-312.

Joshi D Center JR, Eisman JA. Investigation of incidental hypercalcemia. BMJ. 2009;339:b4613.

Mundy GR, Edwards JR. PTH-related peptide (PTHrp) in hypercalcemia. J Am Soc Nephrol. 2008;19:672-675.

Incorrect

Marks for this submission: 0.00/1.00.

Question 12

Marks: 1.00

A 16-year old girl had the following head computed tomography scan during evaluation for a seizure.

qsid=148287&aid=3887&qid=141&checksum=5798bdd12bd6eb955f1c845df028c26c&sesskey=9SD9XfnshK&new state=0

Page 278: Asap 2011
Page 279: Asap 2011

Which one of the following should you measure?

Choose one answer .

A. Thyroid-stimulating hormone (TSH)

B. Calcium

C. Potassium

D. Sodium

E. Prolactin

Educational Objective: Recognize sequelae of long-standing hypocalcemia

Rationale: The patient's head computed tomography (CT) scan shows basal ganglia calcifications that are seen in chronic hypocalcemia. While hyponatremia can lead to

seizures, it would not explain the findings on the CT scan. Measurement of thyroid-stimulating hormone for thyroid disease, or serum potassium for hyper- or hypo-kalemia or

prolactin for a pituitary tumor would not be helpful nor indicated in the evaluation of this patient's seizures in the presence of the findings on CT of the brain.

Reference

Thakker RV. Hypocalcemia: pathogenesis, differential diagnosis, and management. In: Primer on the Metabolic Bone Diseases and Disorders of Mineral Metabolism. 6th ed. Washington,

DC: Cadmus Professional Communications, 2006: 213-216.

Incorrect

Marks for this submission: 0.00/1.00.

Question 13

Marks: 1.00

A 30-year old woman presents with hypocalcemia and this hand x-ray.

qsid=148288&aid=3887&qid=147&checksum=79718768bf7b604cd617bc96414bc9fc&sesskey=9SD9XfnshK&new state=0

Page 280: Asap 2011

Which one of the following is the cause of her hypocalcemia?

Choose one answer .

A. Activating mutation of the calcium-sensing receptor

B. Inactivating mutation of the calcium-sensing receptor

C. Postsurgical hypoparathyroidism

Page 281: Asap 2011

D. Deficiency of Gsα activity

E. Mutation of parathyroid hormone gene

Educational Objective: Identify Albright's hereditary osteodystrophy

Rationale: The hand films show the shortening of the third and fourth metacarpals, which is seen in Albright's hereditary osteodystrophy in a patient with pseudohypoparathyroidism

type 1a. The molecular defect in this disorder is a decrease in Gsα activity.

Reference:

Levine MA. Pseudohypoparathyroidism: from bedside to bench and back. J Bone Min Res. 1999;14:1255-1260.

Incorrect

Marks for this submission: 0.00/1.00.

Question 14

Marks: 1.00

A 68-year old man is hospitalized for newly diagnosed multiple myeloma with lytic bone lesions. Admission laboratory results show normal serum calcium. He receives an infusion of

zoledronic acid .

Laboratory tests the next day show calcium 7.2 mg/dL (normal, 8.5-10.1) and phosphorus 2.4 mg/dL (normal, 2.5-4.5) .

What is the most likely underlying problem?

Choose one answer .

A. Hypoparathyroidism

B. Vitamin D deficiency

C. Magnesium deficiency

qsid=148289&aid=3887&qid=148&checksum=09ba8ddee7e549628e5a3788c2d561be&sesskey=9SD9XfnshK&new state=0

Page 282: Asap 2011

D. Abnormal monoclonal protein with abnormal calcium binding affinity

E. Pseudohypoparathyroidism

Educational Objective: Identify the underlying cause of bisphosphonate-induced hypocalcemia

Rationale: Potent intravenous bisphosphonates can precipitate hypocalcemia in individuals with unrecognized vitamin D deficiency or mild hypoparathyroidism. The low serum

phosphorus and the prevalence of vitamin D deficiency make this the most likely etiology. Magnesium deficiency, pseudohypoparathyroidism, and hypoparathyroidism are usually

associated with higher serum phosphorus. Abnormal monoclonal proteins with high affinity binding of calcium have been reported to cause increases in total calcium, but not

decreases.

Reference

Maalouf NM, Heller HJ, Odvina CV, et al. Bisphosphonate-induced hypocalcemia: report of 3 cases and review of literature. Endocr Pract. 2006;12:48-53.

Incorrect

Marks for this submission: 0.00/1.00.

Question 15

Marks: 1.00

A 54-year old man from a family known to have multiple endocrine neoplasia (MEN) 1 is seen for routine endocrine follow-up. His only known manifestation is a history of

hyperparathyroidism treated with 3.5-gland parathyroid resection 5 years ago. He feels well. He had recently had a spine (MRI) scan for evaluation of disc disease and blood tests

performed later the same day. The blood results are as follows: calcium 7.6 mg/dL (normal, 8.5-10.1), phosphorus 3.2 mg/dL (normal, 2.5-4.5) .

What should you do next?

Choose one answer .

qsid=148290&aid=3887&qid=150&checksum=ff2202d18c2a3f118fc57f95d6296f7c&sesskey=9SD9XfnshK&new state=0

Page 283: Asap 2011

A. Initiate 1,25-dihydroxyvitamin D and calcium therapy

B. Measure parathyroid hormone-related peptide

C. Administer oral calcium

D. Measure prolactin

E. Repeat calcium measurement

Educational Objective: Recognize an error in blood calcium measurement

Rationale: The patient has spurious hypocalcemia related to the gadolinium contrast given for the magnetic resonance imaging (MRI) scan interfering with the standard colorimetric

calcium assay. He had his blood work drawn right after the MRI scan. Parathyroid hormone-related peptide is not associated with multiple endocrine neoplasia (MEN) 1. Measuring prolactin is probably not necessary given the normal MRI and will not help to explain his

hypocalcemia.

Reference

1 .Moore CD, Newman RC, Caridi JG. Spurious hypocalcemia after gadodiamide-enhanced magnetic resonance imaging: a case report and review of the literature. Rev Urol.

2006;8:165-168.

Incorrect

Marks for this submission: 0.00/1.00.

Question 16

Marks: 1.00

A 36-year old woman is referred for evaluation of hypocalcemia. The hypocalcemia was discovered on routine laboratory tests performed as part of a pre-employment physical

examination. She feels well.

qsid=148291&aid=3887&qid=152&checksum=0e2aee1bfe91e4217b7c318a8ed16d27&sesskey=9SD9XfnshK&new state=0

Page 284: Asap 2011

Past medical history reveals Graves' disease treated with radioactive iodine resulting in permanent hypothyroidism.

Medications: Levothyroxine 125 mcg daily.

Physical examination reveals vitiligo on both hands. No Chvostek's sign is demonstrable. The examination is otherwise unremarkable.

Laboratory studies are as follows:

Serum calcium8.0 mg/dL)8.9-10.2 mg/dL(

Serum phosphorus4.7 mg/dL)2.5-4.5 nmg/dL(

Parathyroid hormone20 pg/mL)15-65 pg/mL(

Serum creatinine0.8 mg/dL)0.6-1.1 mg/dL(

Her other laboratory tests were normal and included: complete blood count, serum electrolytes, aspartate aminotransferase,, alkaline phosphatase, and thyroid-stimulating

hormone.

Which of the following conditions is the cause of her hypocalcemia?

Choose one answer .

A. Low serum magnesium

B. Calcium sensing receptor antibodies

C. Hypoalbuminemia

D. Impaired Gs-alpha function

E. Vitamin D deficiency

Educational Objective: Recognize autoimmune cause of hypoparathroidism related to calcium-sensing receptor antibodies

Rationale: The low total serum calcium, elevated phosphorus, and inappropriately normal parathyroid hormone level (PTH) are due to hypoparathyroidism. The patient has evidence

of autoimmunity. Destruction of the parathyroid glands due to antibodies against the

Page 285: Asap 2011

calcium sensing receptor resulting in hypoparathyroidism is the most likely cause of her hypocalcemia. Low magnesium can result in altered PTH secretion/action but in the absence

of diarrhea, kidney disease, or medications causing magnesium wasting, this is not likely. Low albumin can cause the total calcium to be low but would not explain the elevated

phosphorus. Impaired Gs-alpha function, and vitamin D deficiency would cause the PTH level to be high.

Reference:

Brown EM. Anti-parathyroid and anti-calcium sensing receptor antibodies in autoimmune hypoparathyroidism. Endocrinol Metab Clin North Am. 2009;38:437-45.

Incorrect

Marks for this submission: 0.00/1.00.

Question 17

Marks: 1.00

A 42-year old man is referred for evaluation of a mild persistently increased serum alkaline phosphatase level. His serum calcium has been normal, but his serum phosphate has been

mildly decreased at 1.7 to 2.1 mg/dL, when checked on several occasions over the last 2 years. His serum creatinine is normal. He has no documentation of hypophosphatemia prior

to 2 years ago and no family history of hypophosphatemia, rickets, or osteomalacia. His lower extremities are not bowed. His serum 1,25-dihydroxyvitamin D level is mildly

decreased at 20 pg/mL (normal, 22-67).

This patient's hypercalcemia is most likely due to which one of the following disorders?

Choose one answer .

A. Tumor-induced osteomalacia

B. Chronic use of aluminum-containing antacids

C. X-linked hypophosphatemic rickets

D. Autosomal dominant hypophosphatemic rickets

qsid=148292&aid=3887&qid=153&checksum=06cda33affe5a2b749f486e03684bb08&sesskey=9SD9XfnshK&new state=0

Page 286: Asap 2011

Educational Objective: To appreciate the differential diagnosis of hypophosphatemic osteomalacia

Rationale: Adult patients with newly discovered hypophosphatemic osteomalacia may well have tumor-induced osteomalacia. Most patients with genetically inherited familial rickets,

such as X-linked hypophosphatemic rickets or autosomal dominant hypophosphatemic rickets, develop bowed legs as they begin to bear weight when starting to walk. Lack of

bowed legs suggests that this patient's osteomalacia began after skeletal maturity. Chronic use of aluminum-containing antacids may cause hypophosphatemia and osteomalacia, but

typically is associated with hyperparathyroidism and an increased serum 1,25-dihydroxyvitamin D level. Tumor-induced osteomalacia is classically associated with low-

normal or decreased serum 1,25-dihydroxyvitamin D levels. Tumor-induced osteomalacia may be difficult to diagnose because it is rare and tumors are often difficult to localize.

References:

Ruppe MD, Jan de Beur S. Disorders of phosphate metabolism. In: Rosen CJ, ed. Primer on the Metabolic Bone Diseases and Disorders of Mineral Metabolism. 7th ed. Washington, DC:

American Society for Bone and Mineral Research, 2008:317-325 .

Habra MA, Jimenez C, Huang SC, et al. Expression analysis of fibroblast growth factor-23, matrix extracellular phosphoglycoprotein, secreted frizzled-related protein-4, and fibroblast

growth factor-7: identification of fibroblast growth factor-23 and matrix extracellular phosphoglycoprotein as major factors involved in tumor-induced osteomalacia. Endocr

Pract. 2008;14:1108-1114.

3 .Maricic M. Osteomalacia. Curr Osteoporos Rep. 2008;6:130-133.

Incorrect

Marks for this submission: 0.00/1.00.

Question 18

Marks: 1.00

A 22-year old man is referred for evaluation of long-standing increased bone alkaline phosphatase and rickets. He has had bowed legs since childhood and was diagnosed with

hypocalcemic seizures and tetany in infancy. He has mild diffuse skeletal aching pain, but has not noticed body hair loss to date.

His laboratory studies are as follows:

qsid=148293&aid=3887&qid=154&checksum=ae6c84f37e313b5bbd59e68a543bff01&sesskey=9SD9XfnshK&new state=0

Page 287: Asap 2011

Serum calcium8.0 mg/dL)normal, 8.9-10.1 mg/dL(

Serum phosphate1.5 mg/dL)normal, 2.5-4.5 mg/dL(

Total alkaline phosphatase520 U/L)normal, 45-115 U/L(

Bone alkaline phosphatase420 U/L)normal, 11-67 U/L(

Serum creatinine1.2 mg/dL)normal, 0.8-1.3 mg/dL(

Serum 25-hydroxyvitamin D30 ng/mL)normal, >30 ng/mL(

Serum 1,25-dihydroxyvitmamin D

225 pg/mL)normal, 22 to 67 pg/mL(

Parathyroid hormone level94 pg/mL)normal, 10 to 65 pg/mL(

His bone scan shows diffusely increased uptake without evidence of stress fractures.

What is the next best step in the evaluation of his increased bone alkaline phosphatase?

Choose one answer .

A. 24-hour urinary phosphate and creatinine

B. Serum vitamin A

C. Metastatic bone survey

D. Iliac crest bone biopsy

E. Vitamin D receptor mutation analysis

Educational Objective: Recognize the clinical features of hereditary vitamin D-resistant rickets

Rationale: Hereditary vitamin D-resistant rickets classically causes rickets in infancy and childhood. A young man with long-standing increased bone alkaline phosphatase and bowed

Page 288: Asap 2011

legs might potentially have any of several genetically inherited disorders causing rickets, including X-linked hypophosphatemic rickets, autosomal dominant hypophosphatemic

rickets, pseudovitamin D deficiency rickets, or hereditary vitamin D-resistant rickets. His history of hypocalcemic seizures and tetany in infancy suggests a moderately severe form of

rickets. Lack of progressive alopecia universalis beginning in the first year of life is unusual for hereditary vitamin D-resistant rickets, but not all kindreds reported to date have

alopecia. His laboratory studies show mineral abnormalities likely to cause rickets, and his increased serum 1,25-dihydroxyvitmamin D level suggests 1,25-dihydroxyvitamin D

resistance. Vitamin D receptor mutation analysis will confirm the presence of a mutation associated with this disorder.

References:

Zhou Y, Wang J, Malloy PJ, et al. Compound heterozygous mutations in the vitamin D receptor in a patient with hereditary 1,25-dihydroxyvitamin D-resistant rickets with alopecia.

J Bone Miner Res. 2009;24:643-651.

Malloy PJ, Wang J, Srivastava T, et al. Hereditary 1,25-dihydroxyvitamin D-resistant rickets with alopecia resulting from a novel missense mutation in the DNA-binding domain of the

vitamin D receptor. Mol Genet Metab. 2010;99:72-79.

Malloy PJ, Pike JW, Feldman D. The vitamin D receptor and the syndrome of hereditary 1,25-dihydroxyvitamin D-resistant rickets. Endocrine Rev. 1999;20:156-188.

Incorrect

Marks for this submission: 0.00/1.00.

Question 19

Marks: 1.00

A 75-year old woman is referred for evaluation and management of newly discovered hypocalcemia. Her serum calcium was found to be 6.5 mg/dL (normal, 8.9-10.1) during

evaluation in her local emergency department for progressive tingling paresthesias, cramps, and tetany. She currently takes no vitamin or mineral supplements. She appears pale and

chronically ill, and notes that she does not leave her city apartment very often due to difficulty walking because of her weak leg muscles .

The remainder of her laboratory studies show serum phosphate 2.1 mg/dL (normal, 2.5-4.5), total alkaline phosphatase 681 U/L (normal, 45-115) and parathyroid hormone 104 pg/mL

(normal, 10 to 65). Her serum creatinine is normal at 1.2 mg/dL, and serum 25-hydroxyvitamin D is deficient at 8 ng/mL (optimal, 30 to 80).

qsid=148294&aid=3887&qid=157&checksum=612c2a818d96b03001d47e2760934fe4&sesskey=9SD9XfnshK&new state=0

Page 289: Asap 2011

Which of the following treatments will best treat her mineral and vitamin D deficiencies?

Choose one answer .

A. Vitamin D3 (cholecalciferol) 50,000 IU three times a week for three months, before resuming vitamin D3 maintenance intake of 800 IU daily

B. Calcium intake of 1,500 mg elemental calcium each day

C. Adequate dietary caloric intake to achieve daily phosphate requirement

D. Direct sunlight exposure for several minutes several times each week as allowed by the weather

E. All of the above

Educational Objective: To appreciate that osteomalacia should be treated with a multi-pronged approach to replace mineral and vitamin D deficiencies

Rationale: The elderly patient described has hypocalcemia, hypophosphatemia, increased alkaline phosphatase, and lower extremity weakness due to osteomalacia resulting from

nutritional vitamin D deficiency. Repletion of her vitamin D deficiency with vitamin D3 50,000 IU three times a week will require several months, before she resumes her vitamin

D3 recommended daily allowance of 800 IU daily. Daily calcium intake of 1,500 mg elemental calcium is appropriate for her age and postmenopausal status. Elderly patients

who are house- or apartment-bound may not always maintain adequate nutrition, so adequate dietary caloric intake should be assured to achieve her daily phosphate

requirement. Direct sunlight exposure to her face and forearms for several minutes several times each week after vitamin D repletion, as allowed by the weather, should prevent

recurrence of vitamin D deficiency.

References

Pearce SH and Cheetham TD. Diagnosis and management of vitamin D deficiency. BMJ. 2010;340:b5664.

Bikle DD. Vitamin D: newly discovered actions require reconsideration of physiologic requirements. Trends Endocrinol Metab. 2010 Feb 8. [Epub ahead of print].

Holick MF. High prevalence of vitamin D inadequacy and implications for health. Mayo Clin Proc. 2006;81:353-373.

Incorrect

Marks for this submission: 0.00/1.00.

Page 290: Asap 2011

Question 20

Marks: 1.00

X-linked hypophosphatemic rickets is caused by which of the following mechanisms?

Choose one answer .

A. Genetic mutations in vitamin D receptor gene

B. Genetic renal 1-α hydroxylase deficiency

C. Overproduction of FGF-23 by mesenchymal tumor

D. Genetic mutations in FGF-23 gene

E. Genetic mutations in PHEX gene

Educational Objective: Briefly review the mechanisms of osteomalacia caused by various disorders

Rationale: X-linked hypophosphatemic rickets is caused by genetic mutations in the PHEX gene, which result in the production of defective PHEX protein, an endopeptidase that

normally cleaves serum FGF-23. The resultant increased serum FGF-23 levels cause renal tubular phosphate wasting. Autosomal dominant hypophosphatemic rickets is caused by

genetic mutations in the FGF-23 gene, which result in production of FGF-23 protein lacking normal cleavage sites. Similar to the situation with X-linked hypophosphatemic rickets,

resultant increased serum FGF-23 levels cause renal tubular phosphate wasting. Tumor-induced osteomalacia is caused by overproduction of FGF-23 by mesenchymal tumors.

Pseudovitamin D deficiency rickets is caused by genetic renal 1-α hydroxylase deficiency, resulting in inability to produce 1,25-dihydroxyvitamin D in the kidney. Hereditary vitamin D-

resistant rickets is due to genetic mutations in the vitamin D receptor gene, which causes tissue resistance to 1,25-dihydroxyvitamin D and resultant high levels of 1,25-

dihydroxyvitamin D.

References:

qsid=148295&aid=3887&qid=159&checksum=0dde1c7a9f14f904141749faaae6c202&sesskey=9SD9XfnshK&new state=0

Page 291: Asap 2011

Lips P, van Schoor NJ, Bravenboer N. Vitamin D-related disorders. In: Rosen CJ, ed. Primer on the Metabolic Bone Diseases and Disorders of Mineral Metabolism. 7th ed. Washington, DC:

American Society for Bone and Mineral Research, 2008:329-335.

Gaucher C, Walrant-Debray O, Nguyen TM, et al. PHEX analysis in 118 pedigrees reveals new genetic clues in hypophosphatemic rickets. Hum Genet. 2009;125:401-411 .

Levy-Litan V, Hershkovitz E, Avizov L, et al. Autosomal-recessive hypophosphatemic rickets is associated with an inactivation mutation in the ENPP1 gene. Am J Hum Genet. 2010 Feb 3.

[Epub ahead of print]

Incorrect

Marks for this submission: 0.00/1.00.

Question 21

Marks: 1.00

Which of the following statements about osteomalacia or rickets is correct?

Choose one answer .

A. Osteomalacia is due to defective mineralization at skeletal sites of bone turnover, including sites of periosteal or endosteal apposition, in the mature

skeleton of adults

B. Rickets is distinguished from osteomalacia in that it does not cause growth plate abnormalities in addition to defective mineralization in the growing skeleton of

children

C. Nutritional rickets is currently uncommon in areas of the world with high sunlight exposure

D. An iliac crest bone biopsy is the least reliable laboratory method to prove a diagnosis of osteomalacia

E. Subclinical vitamin D deficiency never causes osteomalacia

Educational Objective: Recognize common clinical features of osteomalacia and rickets

qsid=148296&aid=3887&qid=160&checksum=d5614445ce7c316176a7a5fdb204ac90&sesskey=9SD9XfnshK&new state=0

Page 292: Asap 2011

Rationale: Defective mineralization at skeletal sites of bone turnover, including sites of periosteal or endosteal apposition, causes osteomalacia to develop in the mature skeleton of adults. Rickets causes growth plate abnormalities in addition to defective mineralization at skeletal sites of bone turnover in the growing skeleton of children. Nutritional rickets is

unfortunately still prevalent in areas of the world with increased sunlight exposure for a variety of reasons, including cultural practices that limit sunlight exposure and low-calcium

diets in some countries. An iliac crest bone biopsy is the only available laboratory method to prove a diagnosis of osteomalacia. Bone histomorphometry criteria for diagnosis of osteomalacia include increased osteoid surface, width, and volume, and prolonged

mineralization lag time. Subclinical vitamin D deficiency may potentially cause osteomalacia in some settings, particularly when associated with low-calcium or low-phosphate diets,

renal insufficiency, or use of aluminum-containing antacids.

References:

Lips P, van Schoor NJ, Bravenboer N. Vitamin D-related disorders. In: Rosen CJ, ed. Primer on the Metabolic Bone Diseases and Disorders of Mineral Metabolism. 7th ed. Washington, DC:

American Society for Bone and Mineral Research, 2008:329-335.

Priemel M, von Domarus C, Klatte TO, et al. Bone mineralization defects and vitamin D deficiency: histomorphometric analysis of iliac crest bone biopsies and circulating 25-

hydroxyvitamin D in 675 patients. J Bone Miner Res. 2010;25:305-312.

Addison W, Masica D, Gray J, McKee MD. Phosphorylation-dependent inhibition of mineralization by osteopontin ASARM peptides is regulated by PHEX cleavage. J Bone Miner

Res. 2009 Sep 23. [Epub ahead of print]

Incorrect

Marks for this submission: 0.00/1.00.

Question 22

Marks: 1.00

A 58-year old man with newly diagnosed Paget's disease is referred for evaluation and management by his primary care physician. He was incidentally found to have mixed lytic

and sclerotic lesions involving L4 that were diagnostic for Paget's disease. He has had pain referable to this region for two years. His total serum alkaline phosphate is 4 times normal.

Zoledronate 5 mg intravenously was prescribed .

Which one of the following side effects is most likely to occur in this patient after the infusion of zoledronate?

qsid=148297&aid=3887&qid=163&checksum=eea21e0ed45ced043014233680496961&sesskey=9SD9XfnshK&new state=0

Page 293: Asap 2011

Choose one answer .

A. Osteonecrosis of the jaw

B. Symptomatic hypocalcemia

C. Fever and myalgias

D. Acute renal insufficiency

E. Eye inflammation

Educational Objective: To understand the complications of intravenous (IV) bisphosphonate infusions and frequency of occurrence in patients with Paget's disease treated with

bisphosphonates

Rationale: Complications reported with IV bisphosphonates include osteonecrosis of the jaw, renal dysfunction, acute phase reaction (fever and myalgias), hypocalcemia, and eye

inflammation. Jaw osteonecrosis (ONJ) has been reported with both pamidronate and zoledronate, but is limited to a few case reports in patients with Paget's disease. The

incidence of ONJ is considerably higher in patients with malignancy on high dose IV bisphosphonates. Dental extraction and oral bone surgery increase the risk of developing ONJ in patients receiving bisphosphonates. Patients receiving IV bisphosphonates should

consider dental surveillance prior to and after the IV therapy and avoid elective oral surgeries after treatment that may increase the risk of ONJ. Hypocalcemia has also been

recognized in 20% of treated patients 9-11 days after infusion, but is rarely symptomatic. It is specifically recommended that Paget's patients receiving zoledronate take 1200 mg of

elemental calcium and 800-1000 international units of vitamin D daily to reduce the risk of hypocalcemia. A cytokine-mediated febrile reaction with IV

bisphosphonates is reported in 18% of patients. This typically occurs within the first 3 days of treatment and resolves within 3 days. It typically does not recur with subsequent infusions.

Acetaminophen at the time of the IV infusion and as needed 72 hours after the infusion reduces the febrile reaction frequency to 7%. Although acute tubular necrosis has been

reported with IV bisphosphonates, especially in patients with predisposing factors to kidney disease it is limited to case reports at this time. Conjunctivitis, iritis, uveitis, and episcleritis

are rare complications of nitrogen-containing bisphosphonates, such as zoledronate, and typically respond to corticosteroid treatment.

 

References:

Page 294: Asap 2011

Khosla S, Burr D, Cauley J, et al. Bisphosphonate-associated osteonecrosis of the jaw: Report of a task force of the American Society for Bone and Mineral Research. J Bone Miner Res.

2007;22:1479-1491.

Reclast [package insert]. East Hanover, NJ: Novartis Pharmaceuticals Corporation;2009.

Siris ES, Roodman GD. Paget's disease of bone. In: Favus MJH, ed. Primer on the Metabolic Bone Diseases and Disorders of Mineral Metabolism. 7th ed. Washington, DC: American

Society of Bone and Mineral Research, 2008: 335-343.

Incorrect

Marks for this submission: 0.00/1.00.

Question 23

Marks: 1.00

Which one of the following benefits has been demonstrated after intensive treatment with bisphosphonates in patients with Paget's disease who are asymptomatic?

Choose one answer .

A. Pain reduction

B. Fracture prevention

C. Reduction in hearing loss

D. Improved quality of life

E. Reduction in use of nonsteroidal anti-inflammatory drugs (NSAID)

Educational Objective: To understand the benefits associated with treatment of asymptomatic Paget's disease

Rationale: The newer, more potent bisphosphonates can restore the indices of bone turnover to normal in most patients and restore a normal, lamellar architecture to pagetic bone. These findings and the observation that osteolytic lesions can heal with antipagetic

qsid=148298&aid=3887&qid=166&checksum=490c9985898251794c8a06dbc9e185ba&sesskey=9SD9XfnshK&new state=0

Page 295: Asap 2011

therapy have led to the concept that maximal suppression of disease activity for prolonged periods may prevent complications. In light of the progressive nature of the disease, the

potential for severe and disabling complications, and the ability to safely achieve biochemical remission in most patients, treatment of patients to prevent complications as

well as to relieve symptoms seems plausible. The Paget's Disease: Randomized Trial of Intensive versus Symptomatic Management (PRISM) was designed to assess if intensive

bisphosphonate therapy provided clinical benefit over symptom driven treatment in fracture prevention, quality of life, pain reduction, and hearing loss. The study included 1,324

patients with Paget's disease followed for a median of 3 years (range 2-5 years). Despite significantly lower serum alkaline phosphatase levels in the intensive treatment group, there were no differences in any of the outcome measures. However, intensively treated patients

did have lower use of nonsteroidal anti-inflammatory drugs (NSAID) than those in the symptomatic treatment patients. Longer term follow-up of these patients is ongoing to see if

beneficial effects may emerge with time .

References:

Langston AL, Campbell MK, Fraser WD, et al. for the PRISM Trial Group. Randomized trial of intensive bisphosphonate treatment versus symptomatic management in Paget's disease of

Bone. J Bone Miner Res. 2010;25:20-31.

Ralston SH, Langston AL, Reid IR. Pathogenesis and management of Paget's disease of bone. Lancet. 2008;372:155-163.

Siris ES, Roodman GD. Paget's disease of bone. In: Favus MJH, ed. Primer on the Metabolic Bone Diseases and Disorders of Mineral Metabolism. 7th ed. Washington, DC: American

Society of Bone and Mineral Research, 2008: 335-343.

Incorrect

Marks for this submission: 0.00/1.00.

Question 24

Marks: 1.00

Which of the following statements best characterizes bone turnover in Paget's disease?

Choose one answer .

qsid=148299&aid=3887&qid=170&checksum=48fd1acc027876a4ad3863ff66afcf0e&sesskey=9SD9XfnshK&new state=0

Page 296: Asap 2011

A. Increased bone resorption and normal bone formation

B. Increased bone resorption and bone formation

C. Lower bone resorption compared to bone formation

D. Reduced bone resorption and formation

E. Normal bone resorption and formation

Educational Objective: To understand the pathophysiology of Paget's disease

Rationale: Paget's disease of bone is a progressive, focal disorder of bone remodeling. The underlying defect is an increase in osteoclast-mediated bone resorption. Osteoclasts in

pagetic bone are substantially increased in size and number and contain up to 100 nuclei, in comparison with 5 to 10 nuclei in a normal osteoclast. Because bone resorption and

formation remain coupled in Paget's disease, the increase in bone resorption results in a secondary increase in bone formation and bone turnover (up to 10-fold). Bone that is

remodeled by this pathologic process becomes enlarged and mechanically weakened. Pain, skeletal deformity, and fracture may develop as a consequence of these changes.

Postmenopausal osteoporosis is the classical bone disease characterized by excessive bone resorption compared to bone formation. Osteopetrosis (marble bone disease) would be the

prototypical bone disease characterized by excessive bone formation compared to decreased bone resorption leading to structural weakened bone .

References:

Siris ES, Roodman GD. Paget's disease of bone. In: Favus MJH, ed. Primer on the Metabolic Bone Diseases and Disorders of Mineral Metabolism. 7th ed. Washington, DC: American

Society of Bone and Mineral Research, 2008: 335-343.

Ralston SH, Langston AL, Reid IR. Pathogenesis and management of Paget's disease of bone. Lancet. 2008;372:155-163.

Kanis JA. Pathophysiology and histopathology. In: Pathophysiology and Treatment of Paget's Disease of Bone. 2nd ed. London: Martin Dunitz Ltd, 1998:1-310.

Incorrect

Marks for this submission: 0.00/1.00.

Question 25

Marks: 1.00

Page 297: Asap 2011

A 66-year old woman with longstanding Paget's disease involving the pelvis and right femur presents due to the recent development of severe right hip pain. She had previous

treatment with risedronate for her Paget's disease 5 years prior. Her current alkaline phosphatase level is in the high normal range. The radiograph of her right hip is consistent

with Paget's disease, but is otherwise unrevealing.

What would you recommend next for this patient?

Choose one answer .

A. Magnetic resonance imaging (MRI) of the right hip

B. Bone scan

C. Bone alkaline phosphatase measurement

D. Treatment with zoledronate

E. Biopsy of right femur

Educational Objective: To understand evaluation of Paget's disease when sarcomatous transformation is suspected

Rationale: Sarcomatous transformation is a rare but serious complication occurring in less than 1% of patients with Paget's disease. Most of the tumors are osteosarcomas, but

fibrosarcomas, chondrosarcomas, malignant fibrous histiocytomas, and giant cell tumors have also been reported. The prognosis of patients with sarcoma is poor. The 6-month

survival is approximately 50%; less than 10% of patients survive more than 5 years. The development of new pain, an exacerbation of existing pain, pain that is unresponsive to

treatment, and a pathologic fracture suggest the possibility of sarcomatous transformation. Alkaline phosphatase activity is not a reliable indicator of the presence of sarcoma. In

addition, sarcomas may not be evident on bone radiographs. If bone radiography does not show a lesion and sarcoma is a diagnostic possibility, magnetic resonance imaging or

computed tomography should be considered.

References:

qsid=148300&aid=3887&qid=173&checksum=c8d827c486f716959999629e6c3a79e0&sesskey=9SD9XfnshK&new state=0

Page 298: Asap 2011

Ralston SH, Langston AL, Reid IR. Pathogenesis and management of Paget's disease of bone. Lancet. 2008;372:155-163.

Siris ES, Roodman GD. Paget's disease of bone. In: Favus MJH, ed. Primer on the Metabolic Bone Diseases and Disorders of Mineral Metabolism. 7th ed. Washington, DC: American

Society of Bone and Mineral Research, 2008: 335-343.

Kaplan FS. Paget's disease of bone: orthopedic complications. Semin Arthritis Rheum. 1994;23:250-252.

Incorrect

Marks for this submission: 0.00/1.00.

Question 26

Marks: 1.00

An 80-year old man is referred for evaluation of primary hyperparathyroidism. His current serum calcium is 10.2 mg/dL (normal, 8.9-10.1), phosphorus, 2.9 mg/dL (normal, 2.5-4.5),

and parathyroid hormone level, 77 pg/mL (normal, 15-65). He does not have complications related to primary hyperparathyroidism such as kidney stones, fractures, or osteoporosis. As part of his evaluation, you request a bone density study, and note that the density of his hips are discordant (right femur neck T-score of -0.9 and left femur neck T-score of +5.0). His total

alkaline phosphatase has also been significantly elevated for several years and is currently twice the upper limits of normal. To further evaluate the bone density finding, you order an

x-ray of his left hip (see image). The image indicates a small fissure fracture in the lateral cortex of the subtrochanteric area of the left femur (arrow). In retrospect, he acknowledges

mild left leg and hip pain for at least the last year. The pain is present at rest and with weight bearing .

 

qsid=148301&aid=3887&qid=178&checksum=a7fa0cd59511c6fdebc1035de167ce0a&sesskey=9SD9XfnshK&new state=0

Page 299: Asap 2011

 

 

Which of the following treatments would you initially recommend for this patient?

Choose one answer .

A. Referral to an orthopedic surgeon

B. Surgery for primary hyperparathyroidism

C. Treatment with cinacalcet

D. Treatment with zoledronate

E. Observation with follow-up in 1 year

Educational Objective: To understand the treatment of Paget's disease and relationship of Paget's disease to primary hyperparathyroidism

Page 300: Asap 2011

Rationale: Although this patient has a fissure fracture in the lateral cortex of the left femur, these lesions are usually asymptomatic. Fissure fractures may remain unchanged for many

years and generally would not require orthopedic intervention. Hypercalcemia has been reported in patients with Paget's disease. It is often in the setting of concomitant primary

hyperparathyroidism, the most common cause of hypercalcemia in elderly patients seen in the outpatient setting. Rarely, in the setting of immobilization in patients with extensive Paget's disease, severe hypercalcemia has been reported. Community-based estimates

suggest that hypercalcemia occurs in 5% of Paget's disease subjects, with the majority of these cases (83%) attributable to underlying primary hyperparathyroidism. The majority

(80%) of patients with primary hyperparathyroidism and Paget's disease are women. In this particular patient, the alkaline phosphatase is elevated and he has symptoms referable to

Paget's disease. The significant elevation in the alkaline phosphatase level is unlikely due to primary hyperparathyroidism since the patient's hypercalcemia is mild and uncomplicated.

Given the mild and uncomplicated nature of his primary hyperparathyroidism, neither surgery nor cinacalcet would be the initial treatment of choice. Rather, zoledronate would

be the treatment of choice for Paget's disease .

References:

Wermers RA, Tiegs RD, Atkinson EJ, et al. Morbidity and mortality associated with Paget's Disease of Bone: A population-based study. J Bone Miner Res. 2008;23:819-825.

Bilezikian JP, Potts JT Jr, Fuleihan Gel-H, et al. Summary statement from a workshop on asymptomatic primary hyperparathyroidism: a perspective for the 21st century. J Bone

Miner Res. 2002;17 (Suppl 2): N2-11.

Brandi ML, Falchetti A. What is the relationship between Paget's disease of bone and hyperparathyroidism? J Bone Miner Res. 2006;21;S2;69-74.

Incorrect

Marks for this submission: 0.00/1.00.

Question 27

Marks: 1.00

A 48-year old female nurse from the Philippines is referred to you for consideration of therapy with teriparatide for her osteoporosis because she failed to respond to 8 years of

bisphosphonate therapy. At age 44, 2 years after her last spontaneous menstrual period she had a bone mineral density (BMD) study; the lumbar spine T-score was -1.6 and femoral

neck T-score was -2.5. She had no relevant personal or family history of increased risk for fracture. Her height at presentation was 61 in and her weight 124 lb. She claimed to take her

qsid=148302&aid=3887&qid=180&checksum=e731872489ab456fcd2bef2757f5e1f1&sesskey=9SD9XfnshK&new state=0

Page 301: Asap 2011

medication regularly and every 2 years she had a recheck of her BMD. At each study there had been no change in BMD at either site. Repeat history and physical examination added

no new pertinent information .

Based on these data, what should you do next?

Choose one answer .

A. Agree that she has failed bisphosphonate therapy and change her medication to teriparatide

B. Agree that she has failed bisphosphonate therapy and add teriparatide to her treatment

C. Measure a biochemical marker of bone resorption to determine whether or not she has indeed failed bisphosphonate therapy

D. Order an extensive series of studies to explain why she might not have responded to bisphosphonate therapy

E. Measure the blood level of bisphosphonate to confirm that she had been compliant with therapy

Educational Objective: Understand that bisphosphonates inhibit bone resorption and prevent further bone loss

Rationale: There are several issues to address in this patient history. Based on the data presented, there was no real indication to measure bone mineral density (BMD) when this

patient first presented. The reference data for women from the Philippines living in the United States is very limited and it is difficult to estimate her fracture risk. There is no a

priori evidence to suggest that her initial BMD was not simply her peak bone mass. If there was a concern about her not responding to the bisphosphonate, the referral should have

been made after the first 2 years of therapy when BMD had not changed.

There is very little evidence from controlled clinical trials that there is a group of patients who have a delayed response to a bisphosphonate. There are no methods for measuring

serum bisphosphonate in clinical practice. The best early indication of a response to therapy is to determine that the biochemical markers of bone resorption are within the

premenopausal reference interval. A more likely BMD response in a patient who does indeed fail bisphosphonate therapy would be a progressive reduction in BMD. In such

patients a search for secondary causes of bone loss is indicated.

There is no contraindication for starting teriparatide therapy in this patient, but it is more expensive than bisphosphonate therapy, and the patient does not really meet any of the

Page 302: Asap 2011

usual indications for teriparatide (markedly low BMD, personal history of fracture, strong family history of fracture). The data on combination bisphosphonate/teriparatide therapy

are quite limited, but there seems to be no benefit to using both drugs at the same time.

References:

Miller PD, Leonard MB. Clinical use of bone mass measurements in adults for the assessment and management of osteoporosis. In: Favus MJ, ed. Primer on the Metabolic Bone Diseases

and Disorders of Mineral Metabolism. 6th ed. Washington, DC: American Society for Bone and Mineral Research, 2006: 150-161 .

Black D, Rosen CJ. Bisphosphonates for the prevention and treatment of osteoporosis. In: Favus MJ, ed. Primer on the Metabolic Bone Diseases and Disorders of Mineral Metabolism.

6th ed. Washington, DC: American Society for Bone and Mineral Research, 2006: 277-282 .

Cosman F. PTH treatment for osteoporosis. In: Favus MJ, ed. Primer on the Metabolic Bone Diseases and Disorders of Mineral Metabolism. 6th ed. Washington, DC: American Society

for Bone and Mineral Research, 2006: 285-290.

Incorrect

Marks for this submission: 0.00/1.00.

Question 28

Marks: 1.00

Menopause is associated with changes in the receptor activator of nuclear factor kappa β ligand (RANKL)

Which of the following statements is correct regarding the activity of RANKL in menopause?

Choose one answer .

A. With the hormonal and cytokine changes that occur in menopause, RANKL is decreased to prevent osteoblast formation

B. RANKL is an activator of osteoclast recruitment and activity which is enhanced during menopause

C. The osteoclast is the source of both RANKL and osteoprotegerin

qsid=148303&aid=3887&qid=183&checksum=1363dc38400083a904ea08cba42308ed&sesskey=9SD9XfnshK&new state=0

Page 303: Asap 2011

D. Osteoprotegerin enhances bone loss in the presence of RANKL in menopause

E. Osteoprotegerin interacts with undifferentiated marrow stem cells to activate osteoblast formation

Educational Objective: To review the pathways that determine the activity of osteoclasts and osteoblasts and the balance between them as it affects bone loss and gain

Rationale: The drop in estrogen at menopause triggers an alteration in the local microenvironment of the skeleton, with the levels of a number of cytokines (e.g.,

interleukin-1 [IL-1], IL-6) that affect the balance between bone resorption and formation all shifting in favor of bone resorption. Recruitment and activation of bone-resorbing

osteoclasts is triggered by a hormonal and cytokine-mediated increased production by osteoblasts of receptor activator of nuclear factor kappa β ligand (RANKL). RANKL attaches

to RANK receptors on undifferentiated marrow stem cells to promote their maturation into osteoclasts, which then become active on the bone surface to increase bone resorption. The

counterbalance to RANKL is another product of the osteoblast under control of both hormones and cytokines, known as osteoprotegerin (OPG). OPG is a decoy molecule for

RANKL, and it limits the interaction between RANKL and undifferentiated marrow stem cells and the activation of mature osteoclasts, thereby limiting bone resorption. Thus, the balance between bone resorption and formation is essentially the balance between RANKL and OPG production by the osteoblast. In brief, menopause forces the balance in favor of RANKL, and

appropriate therapy at menopause forces the balance in favor of OPG.

References:

Riggs BL, Khosla S, Melton LJ III. Sex steroids and the construction and conservation of the adult skeleton. Endocr Rev. 2002;23:279-302.

Wada T, Nakashima T, Hiroshi N, et al. RANKL-RANK signaling in osteoclastogenesis and bone disease. Trends Mol Med. 2006;12:17-25.

Rogers A, Eastell R. Circulating osteoprotegerin and receptor activator for nuclear factor kappa B ligand: clinical utility in metabolic bone disease assessment. J Clin Endocrinol Metab.

2005; 90:6323-6331.

Incorrect

Marks for this submission: 0.00/1.00.

Question 29

Marks: 1.00

Page 304: Asap 2011

Which of the following statements regarding postmenopausal osteoporosis is correct?

Choose one answer .

A. Peak adult bone mass is achieved at the time of epiphyseal closure

B. A woman who has a T-score at the lumbar spine of -1.9 within 1 year of menopause has definitely lost bone mass since she reached her peak bone mass

C. Interleukin cytokine changes during menopause contribute to bone loss in menopause

D. Calcitonin is as effective as a bisphosphonate at increasing BMD

E. Calcitonin is as effective as a bisphosphonate at increasing BMD

Educational Objective: Understand the evaluation and management of bone loss in postmenopausal women

Rationale: During childhood and adolescence the skeleton is formed by a process known as bone modeling, in which bones undergo changes in composition, size, and shape until

epiphyseal closure at the end of puberty. Modeling is governed by osteoblasts (cells responsible for bone formation) and osteoclasts (cells responsible for bone resorption).

During this phase of life, formation exceeds resorption, due to the influence of hormones responsible for growth and development and for the pubertal growth spurt. Genetics plays a

major role in skeletal modeling, and it appears that physical activity does as well. For a number of years (perhaps 5-10) after epiphyseal closure, there is a period of skeletal

consolidation where no change in size or shape is seen, but mineral deposition continues until peak adult bone mass is achieved. From this time to the menopause, healthy women

maintain a stable bone mass with episodes of reversible bone loss during pregnancy and lactation. However, a number of diseases and medications accelerate bone loss, meaning

many adult women reach menopause transition with less than peak adult bone mass. Therefore, it is difficult to determine by DXA T-score at menopause or 1 year after

menopause whether the T-score between < -1 to >2.5 definitely represents bone loss since peak bone mass.

qsid=148304&aid=3887&qid=186&checksum=61a8d3cd5e0fdbfac13a2f15f6fa9560&sesskey=9SD9XfnshK&new state=0

Page 305: Asap 2011

The drop in estrogen at menopause triggers an alteration in the local microenvironment of the skeleton, with the levels of a number of cytokines (e.g., interleukin-1 [IL-1], IL-6) that

affect the balance between bone resorption and formation all shifting in favor of bone resorption.

Calcitonin has been available as an approved therapy (initially for Paget's disease of bone and hypercalcemia of malignancy) for osteoporosis for over a decade. It has been difficult to

complete controlled clinical trials with calcitonin with fracture prevention as the primary outcome, and the data are not as robust as data available for the approved

bisphosphonates. Calcitonin is also not as effective as bisphosphonates in increasing BMD at any skeletal site. There is some suggestion that calcitonin has analgesic properties when

initiated within the first 2 weeks of a vertebral fracture. The data are not very strong, but there are many clinicians who report a large anecdotal experience to support this notion.

The FDA-mandated package insert for teriparatide includes a black box warning label that the drug has been demonstrated to cause osteosarcoma in Fischer 344 rats. This warning is based on an unusual experiment, and there are no matching human data. The clinical trials

with teriparatide were discontinued early by the FDA because of this laboratory animal finding. As a result, the therapy is only approved for 2 years of use. It is contraindicated in

patients who might conceivably be at increased risk for osteosarcoma, and these are listed in the black box warning label. The list includes any patient who has had a previous

osteosarcoma, Paget's disease of bone, or has had any irradiation to the skeleton. There is no recommended limit to the use of bisphosphonates and there are published reports of

safety with alendronate after 10 years of continuous use. There are also isolated reports in small numbers of subjects that the skeleton may become "frozen" after several years of

bisphosphonate therapy. The term "frozen" in this context indicates that the normal cycle of resorption and formation is not functioning, and that the skeleton may be subject to an

increased risk of fatigue damage as a result of this. This theory has not been well substantiated, but does have many proponents. Perhaps more troubling is the observation

that an increasing number of patients on long-term oral or intravenous bisphosphonate therapy have developed osteonecrosis of the jaw that has been very resistant to treatment.

This was initially thought to be only a problem for patients with cancer being treated with intravenous bisphosphonates in doses higher than those used for osteoporosis. While

intravenous use has resulted in the vast majority of cases, it has been reported in a number of cases following bisphosphonate therapy for osteoporosis. The FDA has added a warning regarding the risk of jaw osteonecrosis to all bisphosphonate therapies. Despite all of this, there is no "official" statement from any group or organization suggesting that a time limit

be placed on the duration of bisphosphonate therapy for osteoporosis yet, and this still remains an individual doctor/patient decision.

References:

Schousboe JT, Taylor BC, Ensrud KE. Assessing fracture risk: who should be screened? In: Favus MJ, ed. Primer on the Metabolic Bone Diseases and Disorders of Mineral Metabolism.

6th ed. Washington, DC: American Society for Bone and Mineral Research, 2006: 262-267.

Page 306: Asap 2011

Neer RM, Arnaud CD, Zanchetta JR, et al. Effect of parathyroid hormone (1-34) on fractures and bone mineral density in postmenopausal women with osteoporosis. N Engl J Med.

2001;344:1434-1441.

Bone HG, Hosking D, Devogelaer JP, et al. for the Alendronate Phase III Osteoporosis Treatment Study Group. Ten years' experience with alendronate for osteoporosis in

postmenopausal women. N Engl J Med. 2004;350:1189-1199.

Incorrect

Marks for this submission: 0.00/1.00.

Question 30

Marks: 1.00

A 49-year old Caucasian woman was seen in the clinic for osteoporosis. Her T score in the spine was -2.5, and femoral T score was -2.4. She has not had any fractures and she denied

smoking or alcohol intake. She had a maternal history of osteoporosis and she denied exposure to glucocorticoids .

Evaluation for secondary causes of bone loss revealed the following data:

Serum calcium10.5 mg/dL)normal, 8.9-10.1 mg/dL (

Intact parathyroid hormone89 pg/mL)normal, 15-65 pg/mL (

24-hour urine calcium268 mg/24 hours)normal, 20-275 mg/24 hours

Serum phosphorus3.2 mg/dL)normal, 2.5-4.5 mg/dL (

25-hydroxy vitamin D33 ng/mL)normal. >30 ng/mL(

What is the most likely diagnosis?

Choose one answer .

A. Celiac sprue

qsid=148305&aid=3887&qid=189&checksum=72d502af4f262855a2fad909540e5d1d&sesskey=9SD9XfnshK&new state=0

Page 307: Asap 2011

B. Idiopathic hypercalciuria

C. Vitamin D deficiency

D. Primary hyperparathyroidism

E. Cushing's disease

Educational Objective: To learn the biochemical presentations of common secondary causes of bone loss

Rationale: Most patients with secondary causes of osteoporosis present with asymptomatic disease. Thus, clinicians need to examine biochemical results carefully to guide them as to

the diagnosis .

Primary hyperparathyroidism classically presents with elevated serum calcium, intact parathyroid hormone (PTH), and urinary calcium, and low phosphorus levels. However,

increasingly, the disease is being diagnosed during its early stages with much milder biochemical abnormalities. PTH levels are usually high, serum calcium levels may be just

mildly increased or intermittently elevated, urinary calcium levels may be normal or high normal, and phosphorus levels are more commonly in the normal range .

Idiopathic hypercalciuria is often an occult cause of bone loss that may not have clinical symptoms. Some patients have a history of nephrolithiasis, but more commonly they do not. They usually have normal serum calcium, phosphorus, and PTH levels, but urinary calcium is high (usually >300 mg/24 hours or 3 mg/kg/day). A trial of hydrochlorothiazide given over 6 to 8 weeks usually leads to significant declines in urinary calcium excretion. When excessive

urinary calcium is detected, it is important to distinguish this condition from excessive calcium supplementation, in which case PTH levels are usually low normal or suppressed .

Vitamin D insufficiency (25-hydroxy vitamin D level [25-OHD] between 20-30 ng/mL), and deficiency (25-OHD, <20 ng/mL) are extremely common, and are seen in more than 50% of

patients who are evaluated for low bone mass and osteoporosis. In mild to moderate deficiencies, serum calcium and phosphorus levels are usually normal. Intact PTH levels are

high normal or high, and urinary calcium levels may be normal or low. Screening for vitamin D deficiency is done by measuring the 25-OHD level. The current optimal level is considered

to be >30 ng/mL. The patient does not have vitamin D insufficiency or deficiency by laboratory data criteria.

There is nothing in the clinical and laboratory data provided on this patient in the vignette to suggest Cushings disease. The parathyroid hormone level is normal in Cushing's disease .

Celiac sprue is associated with low serum calcium and low vitamin D levels.

Page 308: Asap 2011

Reference:

Bilezikian JP, Potts JT Jr., Fuleihan G, et al. Summary statement from a workshop on asymptomatic primary hyperparathyroidism: a perspective for the 21st century. J Clin

Endocrinol Metab 2002;87:5353-61.

Incorrect

Marks for this submission: 0.00/1.00.

Question 31

Marks: 1.00

A 55-year old Caucasian man had a vertebral compression fracture lifting 50 lbs. Subsequent dual-energy X-ray absorptiometry (DXA) scan showed a T score of -3.0 in the spine and -2.8

in the femoral neck. He denied other fractures, excessive alcohol intake or smoking .

His laboratory results are as follows:

Serum calcium9.2 mg/dL)normal, 8.9-10.1 mg/dL(

Intact parathyroid hormone64 pg/mL)normal, 15-65 pg/mL(

24-hour urine calcium485 mg/24 hours)normal, 25-300 mg/24 hours(

Serum phosphorus3.6 mg/dL)normal, 2.5-4.5 mg/dL(

25-hydroxy vitamin D28 ng/mL)normal, >30 ng/mL(

What is the most likely contributory factor to his osteoporosis?

Choose one answer .

A. Celiac sprue

B. Idiopathic hypercalciuria

qsid=148306&aid=3887&qid=190&checksum=bdef929d13b82caf46ec7e4f87a7e951&sesskey=9SD9XfnshK&new state=0

Page 309: Asap 2011

C. Vitamin D deficiency

D. Primary hyperparathyroidism

E. Cushing's disease

Educational Objective: To learn the biochemical presentations of common secondary causes of bone loss

Rationale: Idiopathic hypercalciuria is often an occult cause of bone loss that may not have clinical symptoms. Some patients have a history of nephrolithiasis, but more commonly they

do not. They usually have normal serum calcium, phosphorus, and parathyroid hormone (PTH) levels, but urinary calcium is high (usually >300 mg/24 hours or 3 mg/kg/day). A trial of

hydrochlorothiazide given over 6 to 8 weeks usually leads to significant declines in urinary calcium excretion. When excessive urinary calcium is detected, it is important to distinguish this condition from excessive calcium supplementation, in which case PTH levels are usually

low normal or suppressed.

Primary hyperparathyroidism classically presents with elevated serum calcium, intact PTH, and urinary calcium, and low phosphorus levels. However, increasingly, the disease is being

diagnosed during its early stages, with much milder biochemical abnormalities. PTH levels are usually high, serum calcium levels may be just mildly increased or intermittently

elevated, urinary calcium levels may be normal or high normal, and phosphorus levels are more commonly in the normal range .

Vitamin D insufficiency and deficiency is extremely common, and is seen in more than 50% of patients who are evaluated for low bone mass and osteoporosis. In mild to moderate

deficiencies, serum calcium and phosphorus levels are usually normal. Intact PTH levels are high normal or high, and urinary calcium levels may be normal or low. Screening for vitamin D deficiency is done by measuring the 25-hydroxy vitamin D level. The current optimal level

is considered to be >30 ng/mL. This patient had vitamin D insufficiency which is unlikely to be the major contributory factor to his bone loss.

Celiac sprue is associated with low serum calcium, low vitamin D level, and elevated PTH level.

Cushing's disease is a possibility but there are no clinical or laboratory data (serum or urinary cortisol) provided to support the diagnosis .

References:

Bilezikian JP, Potts JT, Jr, Fuleihan G H, et al. Summary statement from a workshop on asymptomatic primary hyperparathyroidism: a perspective for the 21st century. J Clin

Endocrinol Metab. 2002;87:5353-61.

Page 310: Asap 2011

Chapuy M, Preziosi P, Maamer M, et al. Prevalence of vitamin D insufficiency in an adult normal population. Osteoporos Int. 1997;7:439-443.

Lips P. Vitamin D deficiency and secondary hyperparathyroidism in the elderly: consequences for bone loss and fractures and therapeutic implications. Endocr Rev.

2001;22:477-501.

Misael da Silva A, dos Reis L, Pereira R, et al. Bone involvement in idiopathic hypercalciuria. Clin Nephrol. 2002;57:183-191.

Incorrect

Marks for this submission: 0.00/1.00.

Question 32

Marks: 1.00

Which of the following statements about vitamin D deficiency is correct?

Choose one answer .

A. Decreased parathyroid hormone levels occur in patients with prolonged and neglected vitamin D deficiency

B. Vitamin D deficiency is seen in less than 30% of patients with osteoporosis

C. It is safe to initiate bisphosphonate therapy for osteoporosis in vitamin D deficient individuals

D. Optimal treatment of vitamin D deficiency is achieved when the levels of 25-hydroxy vitamin D are corrected to the normal range (>30ng/mL)

E. Levels of 25-hydroxy vitamin D should be obtained prior to organ transplantation to manage post-transplant bone loss

Educational Objective: To understand the prevalence, treatment, and follow-up of vitamin D deficiency

qsid=148307&aid=3887&qid=192&checksum=358e4ec46a35394f1d4f2be0ee778362&sesskey=9SD9XfnshK&new state=0

Page 311: Asap 2011

Rationale: Vitamin D deficiency is very common in patients prior to organ transplantation. Appropriate screening for this condition will help to prevent or reduce the rapid bone loss

that occurs in the first 6 to 12 months after transplantation.

Osteomalacia can occur in adults due to prolonged, uncorrected vitamin D deficiency. Conditions such as celiac sprue, inflammatory bowel disease, gastric surgery, or simply severe nutritional deficiency can cause this. It is reasonable to suspect the presence of osteomalacia in a patient with low bone mass when there is elevation in total or bone-

specific alkaline phosphatase, low vitamin D levels, secondary hyperparathyroidism with elevated parathyroid hormone (PTH) levels, hypocalcemia, hypocalciuria, and

hypophosphatemia. Bone biopsy can be done, but it is usually not necessary to confirm the diagnosis. Clinical symptoms and signs that may be encountered include deep aching bone

pains, muscle weakness, Chvostek's sign and brisk, deep tendon reflexes, and the classic "waddling gait ".

Vitamin D deficiency can be seen in approximately 50% of patients with low bone mass. The optimal 25-hydroxy vitamin D (25-OHD) level is >30 ng/mL; however, in treating vitamin D deficient individuals, it is important to aim to correct secondary hyperparathyroidism and

hypocalciuria as well. Among older individuals, renal insufficiency can lead to poor activation of common vitamin D precursors and may require the use of calcitriol or other active vitamin D analogues to correct secondary hyperparathyroidism. It is important that patients who are

vitamin D deficient be followed with repeat 25-OHD and intact PTH levels; in patients who are receiving calcitriol, urinary calcium levels should be checked as well. Due to current

assay variabilities, it is not sufficient to aim to normalize 25-OHD levels. PTH levels should be brought down to target levels based on the patient's renal status.

Bisphosphonates, particularly when given intravenously, can cause hypocalcemia. In the treatment of patients with osteoporosis, vitamin D deficiency and resulting secondary

hyperparathyroidism should be corrected before initiation of bisphosphonates therapy to avoid hypocalcemia.

References:

Cohen A and Shane E. Osteoporosis after solid organ and bone marrow transplantation. Osteoporos Int. 2003;14:617-630.

Holick M, Siris E, Binkley N, et al. Prevalence of vitamin D inadequacy among postmenopausal North American women receiving osteoporosis therapy. J Clin Endocrinol

Metab. 2005;90:3215-3224.

Lips P. Vitamin D deficiency and secondary hyperparathyroidism in the elderly: consequences for bone loss and fractures and therapeutic implications. Endocr Rev.

2001;22:477-501.

Mosekilde L. Vitamin D and the elderly. Clin Endocrinol. 2005;62:265-281.

Incorrect

Page 312: Asap 2011

Marks for this submission: 0.00/1.00.

Question 33

Marks: 1.00

A 53-year old postmenopausal woman with a history of breast cancer status post- chemotherapy and radiation therapy on aromatase inhibitor therapy was found to have low bone mass or osteopenia in her screening dual-energy X-ray absorptiometry (DXA) scan. Her

T score in the spine was -1.4 and femoral neck T score was -1.2. She denies any fracture history.

Which of the following options would be most helpful in the assessment and management of her osteopenia?

Choose one answer .

A. Discontinue aromatase inhibitor and switch to tamoxifen

B. Obtain a 25 hydroxy vitamin D level and initiate oral supplementation with calcium and vitamin D

C. Obtain tissue transglutaminase to assess for celiac sprue

D. Repeat DXA scan

E. Reduce physical activities to activities of daily living and avoid exercise programs

Educational Objective: To understand the adverse effects of aromatase inhibitor (AI) therapy on bone

Rationale: Aromatase inhibitors inhibit aromatization by > 98%, markedly reducing circulating estradiol levels. Two large studies, "Arimidex, Tamoxifen, Alone or in

Combination" (ATAC) and "Breast International Group 1-98 Collaborative Group" (BIG 1-98) trials showed the time to progression, time to distant metastasis, event-free survival and progression-free survival were significantly better in the anastrazole and letrozole groups

versus tamoxifen. However, fracture rates are significantly higher in patients treated with AI, and bone loss of roughly 2-3 % annually in the spine and 1-2% in the hip were seen, most

qsid=148308&aid=3887&qid=194&checksum=0deb4586ad9a6fcdf5c3501ab657834b&sesskey=9SD9XfnshK&new state=0

Page 313: Asap 2011

pronounced in the first three years of therapy. Bone turnover markers increase significantly when AIs are used. In a study published in 2008, 38% of patients with breast cancer had

vitamin D insufficiency or deficiency. 5

AI therapy is significantly better than tamoxifen in the management of breast cancer patients. The risk/benefit ratio favors continuing the AI. Patients on AIs or androgen

deprivation therapy who have established osteoporosis and those who are at high risk for fractures based on their FRAX score, should be treated with appropriate antiresorptive or

anabolic therapy. The benefits of these agents usually outweigh their risks and it is rare that they are discontinued for their ability to cause bone loss.

Obtaining a 25 hydroxy vitamin D (25-OHD) level would be helpful to identify vitamin D insufficiency or deficiency for treatment. Supplementation with adequate calcium (1600-

1800 mg/day) and vitamin D (1200-1600 international units per day) is indicated as the standard baseline treatment for osteopenia to prevent or reduce bone loss. The

bisphosphonate, risedronate, has been reported to decrease bone loss in postmenopausal breast cancer patients on AI therapy. Postmenopausal breast cancer patients not treated

with AIs achieved a better beneficial effect from risedronate on decreasing bone loss than those on AIs.

Dual-energy X-ray absorptiometry (DXA) scan soon after the screening DXA would add little or any more information to assist with management of the patient.

There is nothing to suggest celiac sprue from the data provided.

Low intensity muscle strengthening exercises to strengthen bone and maintain stability of gait is recommended for the prevention of falls and fractures. Broad limitation or extreme restriction of physical activities may aggravate bone loss. A balance between low intensity

exercises and limitation of activities that may precipitate fractures is important so that bone loss and fracture risk are not increased.

References:

Howell A, Cuzick J, Baum M, et al. Results of the ATAC (Arimidex, Tamoxifen, Alone or in Combination) trial after completion of 5 years' adjuvant treatment for breast cancer. Lancet.

2005;365:60-2.

Thurlimann B, Keshaviah A, Coates AS, et al. A comparison of letrozole and tamoxifen in postmenopausal women with early breast cancer. N Engl J Med. 2005;353:2747-57.

Eastell R, Adams JE, Coleman RE, et al. Effect of anastrozole on bone mineral density: 5-year results from the anastrozole, tamoxifen, alone or in combination trial. J Clin Oncol.

2008;26:1051-7.

Greenspan SL, Brufsky A, Lembersky BC, et al. Risedronate prevents bone loss in breast cancer survivors: a 2-year, randomized, double-blind, placebo-controlled clinical trial. J Clin

Oncol. 2008;26:2644-52.

Page 314: Asap 2011

Camacho PM, Dayal AS, Diaz JL, et al. Prevalence of secondary causes of bone loss among breast cancer patients with osteopenia and osteoporosis. J Clin Oncol. 2008;26:5380-5

Incorrect

Marks for this submission: 0.00/1.00.

Question 34

Marks: 1.00

A 65-year old African American man was seen in consultation for osteoporosis. He denied fractures, smoking history and alcohol intake .

What biochemical evaluation should be ordered?

Choose one answer .

A. Angiotensin-converting enzyme

B. Tissue transglutaminase

C. Free testosterone

D. Insulin antibodies

E. Serum lipid levels

Educational Objective: To recognize hypogonadism as the most common cause of osteoporosis in men

Rationale: Men comprise 20% of diagnosed cases of osteoporosis in the United States. Idiopathic cases comprise a smaller proportion of male osteoporosis. The vast majority

(>60%) have secondary causes. The most commonly found secondary cause among males is hypogonadism. With the increasing use of luteinizing hormone-releasing hormone agonists for prostate cancer, this is becoming even a greater concern. Vitamin D deficiency, primary

hyperparathyroidism and idiopathic hypercalciuria are also common causes. Less commonly

qsid=148309&aid=3887&qid=196&checksum=3083941c7c4f34f2bacab34c6f83bf5e&sesskey=9SD9XfnshK&new state=0

Page 315: Asap 2011

found secondary causes include multiple myeloma, and endocrine conditions such as hyperthyroidism and Cushing's syndrome.

Angiotensin converting enzyme level to assess for sarcoidosis, tissue transglutaminase for celiac sprue, insulin antibodies for latent autoimmune diabetes mellitus of adults, and serum lipid levels for assessment of hyperlipidemia would not be helpful in addressing the cause of

his osteoporosis.

References:

Bilezikian J. Osteoporosis in men. J Clin Endocrinol Metab. 1999;84:3431-3434.

Stein E, Shane E. Secondary osteoporosis. Endocrinol Metab Clin North Am. 2003;32:115-134.

Incorrect

Marks for this submission: 0.00/1.00.

Reminder:

You have three (3) attempts to successfully complete this assessment with a 75% score or higher.

You have 30 days to complete each attempt, once an attempt is started.

If you need to save your answers and come back to the assessment, you must click the 'Next' button at the bottom-left of the assessement prior to logging

out of ASAP.

Only click 'Submit all and finish' once you are satisfied with all your assessment answers.

Bottom of Form

Finish review

Skip Quiz navigation

Quiz navigation

i (Closed ) 1 (Incorrect ) 2 (Incorrect ) 3 (Incorrect ) 4 (Incorrect ) 5 (Incorrect ) 6 (Incorrect ) 7 (Incorrect ) 8 (Incorrect ) 9 (Incorrect ) 10 (Incorrect ) 11 (Incorrect ) 12 (Incorrect ) 13

(Incorrect ) 14 (Incorrect ) 15 (Incorrect ) 16 (Incorrect ) 17 (Incorrect ) 18 (Incorrect ) 19 (Incorrect ) 20 (Incorrect ) 21 (Incorrect ) 22 (Incorrect ) 23 (Incorrect ) 24 (Incorrect ) 25 (Incorrect ) 26 (Incorrect ) 27 (Incorrect ) 28 (Incorrect ) 29 (Incorrect ) 30 (Incorrect ) 31

(Incorrect ) 32 (Incorrect ) 33 (Incorrect ) 34 (Incorrect ) i (Closed )

Finish review

You are logged in as Mohammed Aldawish (Logout)

Page 316: Asap 2011

PBMD

 

Copyright: 2011, American Association of Clinical Endocrinologists (AACE).All material published on the Web site is the property of AACE and may not be reproduced in

any form or by any electronic meansincluding information, storage and retrieval systems without the written consent of AACE.

This activity is sponsored by the American Association of Clinical Endocrinologists.

Page 317: Asap 2011